Sunteți pe pagina 1din 611

Universidad de Santiago de Chile.

Facultad de Ciencia
Departamento de Matemática y C. C.

Cálculo Avanzado

Miguel Martinez - Carlos Silva - Emilio Villalobos


Derechos de Autor

Autor: c Universidad de Santiago de Chile


Se autoriza la reproducción parcial o
total de esta obra, con fines académicos,
por cualquier forma, medio o procedimiento,
siempre y cuando se incluya la cita
bibliográfica del documento.
Agradecimientos
Este texto fue financiado en el marco de los proyectos concursables de
innovación docente que promueve anualmente la Universidad de Santiago de
Chile a través de la Vicerrectorı́a Académica por intermedio de la Dirección
de Docencia.
El centro motor que motivó a los autores a emprender tan significativo
desafı́o fue su compromiso con el proceso de enseñanza aprendizaje que se
lleva semestre a semestre en la Universidad de Santiago, con los estudiantes
de ingenierı́a quienes tienen el imperativo de mejorar sus aprendizajes y elevar
sus estándares de competencias con la finalidad de que puedan asumir con
propiedad el desafı́o de sus asignaturas profesionales y de especialidad con
un mayor empoderamiento en el contexto de: teorı́a, práctica y aplicaciones
a problemas en las áreas de sus distintas especialidades.
En general, cada capı́tulo comienza con una presentación de definiciones,
principios y teoremas, junto con material ilustrativo. Los problemas resueltos
sirven para ilustrar la teorı́a y suministrar herramientas de análisis de los
principios básicos tan importantes en el aprendizaje activo de los estudiantes.
El gran número de problemas resueltos y aplicaciones sirven para encauzar el
aprendizaje del material, ası́ como las autoevaluaciones propuestas al fin de
cada unidad. Hemos escogido un enfoque y nivel de profundidad de acuerdo
con lo que se espera del curso de Cálculo Avanzado, asignatura que se imparte
durante el tercer semestre del Plan Común de la Carrera de Ingenierı́a Civil
de la Facultad de Ingenierı́a de Universidad de Santiago de Chile.
El objetivo del primera parte de este texto es presentar los conceptos
básicos y las aplicaciones de las series de Fourier, y las funciones integrales,
como asimismo, ilustrar su utilización en la resolución de problemas de ecua-
ciones en derivadas parciales y aplicaciones en el campo de la fı́sica e inge-
nierı́a.
En la segunda parte se abordan los temas de funciones vectoriales y cálcu-
lo diferencial de funciones de dos o más variables y sus aplicaciones, incluyen-
do aplicaciones e interpretaciones geométricas y fı́sicas que contribuyan a la
comprensión de los estudiantes.
Unido a lo anterior, en la tercera parte se incluyen los temas de integración
múltiple, integral de lı́nea , superficie y los teoremas de Green, Gauss y Stokes
por sus múltiples aplicaciones en los campos de la fı́sica y ciencias de la
ingeniera
Finalmente, queremos aprovechar la ocasión para expresar nuestro es-
pecial agradecimiento hacia nuestros colegas de la Coordinación de Cálculo
Avanzado que con sus crı́ticas constructivas y opiniones han ayudado a el

i
enriquecimiento del material que conforma este texto. Deseamos también
agradecer la participación directa e indirecta de nuestros estudiantes con los
cuales pusimos a prueba el material que se estaba generando incluyéndolos
en la página web de la asignatura de Cálculo Avanzado.
Agradecemos también muy especialmente la colaboración del profesor
Omar Ramos por la confección de diagramas, figuras e imágenes de fun-
ciones bi y tridimensionales que ilustran conceptos y problemas. También
se encargó de la versión Latex de los distintos archivos que conforman el
manuscrito del texto.
No obstante lo anterior, la responsabilidad por los eventuales errores o
inexactitudes que se puedan encontrar en el texto corresponde a los autores,
quienes estarán atentos para recibir cualquier comentario o sugerencia que
permita mejorar su contenido en las siguientes direcciones:
miguel.martinez@usach.cl, carlos.silva.c@usach.cl,emilio.villalobos@usach.cl.

Los Autores:
Miguel Martı́nez
Concha Carlos Silva Cornejo
Emilio Villalobos Marı́n

ii
Prefacio
El material presentado en el texto contiene los temas tratados en el curso
de Cálculo Avanzado, asignatura semestral para las carreras de Ingenierı́a
de la Universidad de Santiago de Chile, correspondiente al área de Ciencias
Básicas, tiene por prerrequisitos las asignaturas de Cálculo I y Cálculo II de
primer año. Proporciona los conceptos, habilidades y técnicas que permiten
adquirir las competencias matemáticas alineadas con el perfil de competen-
cias profesionales, necesarias para cursar con éxito las asignaturas de ciencias
básicas de la ingenierı́a e ingenierı́a aplicada. Los temas tratados por el texto
y en el orden de aparición son los siguientes: Series e Integrales de Fourier,
que forma parte de este temario porque por razones de tiempo no se incluye
en el Capı́tulo de Series de primer año, este tema resulta necesario en la
formación básica de alumnos de ingenierı́a sobre todo cuando necesiten re-
solver ecuaciones diferenciales parciales usando el método de separación de
variables o bien en aplicaciones en el campo de la ingenierı́a. Este tema bien
podrı́a formar parte de un texto de ecuaciones diferenciales. El tema de fun-
ciones vectoriales de una variable real trata la importancia de la derivada de
este tipo de funciones, interpretación geométrica y analı́tica, y su aplicación
a problemas de movimiento, comportamiento de curvas, especialmente en lo
que tiene que ver con caracterı́sticas geométricas de ellas. Las funciones es-
calares son tratadas en detalle, se analiza el concepto de lı́mite y continuidad
considerando funciones de dos variables, generalizando en aquellos casos que
lo amerita, se ve el tema de la diferenciación con todas sus potencialidades
que garantizan la derivación tanto la derivación parcial, como la derivación
direccional y derivación implı́cita, este tema termina con máximos y mı́ni-
mos y problemas aplicados de optimización. Este último capı́tulo tratan los
temas de integración, integrales dobles y triples en coordenadas cartesianas
y generalizando con cambios de coordenadas, integral de lı́nea para funciones
escalares y vectoriales, propiedades de los campos gradientes y el teorema de
Green; integral de superficie para funciones escalares y vectoriales finalizan-
do con el estudio de los teoremas de Gauss y Stokes. Los temas tratados de
acuerdo con los objetivos generales los podemos describir como sigue: Series
e integrales de Fourier
i) Analizar los conceptos asociados a la definición de la Serie de Fourier, sus
propiedades y procedimientos de cálculo, y aplicarlos a la resolución de
problemas de ingenierı́a.
ii) Formular el concepto de Integral de Fourier, sus propiedades y métodos
de calculo y aplicar esta información en la solución de problemas de
ingenierı́a.

iii
Funciones vectoriales

i) Analizar el concepto de diferenciación de funciones vectoriales, sus propiedades,


procedimientos para realizar cálculos y aplicarlos a la resolución de
problemas de Ciencia e Ingenierı́a

ii) Utilizar los conceptos de vector tangente, normal , binormal, curvatura


y torsión e identidades de Frenet sus propiedades y procedimientos de
cálculos para emplearlos en la resolución de problemas.

Diferenciación parcial

i) Definir los conceptos de lı́mite, continuidad y describir las caracterı́sticas


gráficas de las funciones de varias variables en IR2 en IR.

ii) Analizar criterios para reconocer y evaluar la diferenciabilidad de una


función escalar de varias variables, usar su propiedades, métodos de
cálculos para su aplicación a la resolución de problemas de Ingenierı́a.

iii) Generalizar el concepto de diferenciación para funciones compuestas e


implı́citas de varias variables, sus propiedades, métodos de cálculos y
su aplicación a la resolución de problemas de Ingenierı́a

iv) Aplicar diferentes métodos para determinar máximo y mı́nimos de una


función de varias variables y utilizarlos en la resolución de problemas
de optimización

Integración

i) Analizar el concepto de integral doble sus propiedades y procedimientos


de cálculo , y su aplicación a problemas de fı́sica e ingenierı́a

ii) Analizar el concepto de integral triple sus propiedades y procedimientos


de cálculo, y su aplicación en problemas de fı́sica e ingenierı́a.

iii) Analizar los conceptos de integral de trayectoria e integral de lı́nea, y


utilizar sus propiedades en la resolución de problemas matemáticos, de
fı́sica e ingenierı́a

iv) Analizar los conceptos de integral de superficie, y utilizar sus propiedades


en la resolución de problemas matemáticos, de fı́sica e ingenierı́a
Estructura de cada unidad Cada unidad en su desarrollo teórico y
fundamentación matemática enfatiza los conceptos, los teoremas que
avalan los procedimientos y las técnicas de resolución de problemas.

iv
Unido a lo anterior, en cada tema hay una unidad de problemas resuel-
tos, otra de problemas propuestos, algunos con soluciones y una unidad
de aplicaciones a los temas de ingenierı́a. Finalmente, se incluye tam-
bién un instrumento de autoevaluación que consiste en un test con
problemas de desarrollo que mide el nivel de las competencias cogniti-
vas alcanzado por el estudiante. Este material pretende ser una fuente
de motivación que haga que los estudiantes perseveren en sus estudios
y puedan vencer las dificultades de aprendizaje, y alcanzar niveles que
le permitan rápidamente conectarse con los temas en que está inmerso
un problema y estructurar un a respuesta al problema usando diversas
herramientas matemáticas, esto le dará desde luego un plus durante
toda su vida profesional.

v
Índice general

1. Serie de Fourier 1
1.1. Introducción . . . . . . . . . . . . . . . . . . . . . . . . 1
1.2. Propiedades Generales . . . . . . . . . . . . . . . . . . 2

1.2.1. Lema Elemental . . . . . . . . . . . . . . . . . . 2


1.3. La serie de Fourier de una función . . . . . . . . . . . . 4
1.3.1. Coeficientes de Fourier . . . . . . . . . . . . . . 6
1.3.2. Atributos de la función . . . . . . . . . . . . . . 7
1.3.3. Convergencia de las series de Fourier . . . . . . 9
1.3.4. La integral de funciones pares e impares . . . . 15

1.3.5. Teorema de las funciones pares y de las impares 16


1.4. Desarrollos llamados de medio rango . . . . . . . . . . 17
1.4.1. Extensión impar: . . . . . . . . . . . . . . . . . 17
1.4.2. Extensión par . . . . . . . . . . . . . . . . . . . 19
1.5. Diferenciacion e Integración de la series de Fourier . . . 21
1.5.1. Derivación . . . . . . . . . . . . . . . . . . . . . 21
1.5.2. Integración . . . . . . . . . . . . . . . . . . . . 23
1.5.3. Identidad de Parseval . . . . . . . . . . . . . . . 25
1.6. Integral de Fourier . . . . . . . . . . . . . . . . . . . . 27
1.6.1. Criterio de convergencia de la integral de
Fourier . . . . . . . . . . . . . . . . . . . . . . . 29
1.6.2. Integrales de Fourier de cosenos y senos . . . . . 31
1.7. Aplicaciones de Series de Fourier . . . . . . . . . . . . 38

vi
1.7.1. Onda cuadrada alta frecuencia . . . . . . . . . 38
1.7.2. Rectificador de onda completa. . . . . . . . . . 39
1.7.3. Ecuación de calor unidimensional . . . . . . . . 40
1.7.4. Ecuación de calor: barra aislada . . . . . . . . 41
1.7.5. Ecuación de Onda . . . . . . . . . . . . . . . . 42
1.7.6. Deflexión de una viga . . . . . . . . . . . . . . 44
1.8. Problemas Propuestos . . . . . . . . . . . . . . . . . . 45
1.9. Ejercicios Resueltos . . . . . . . . . . . . . . . . . . . . 50
1.9.1. Serie de Fourier . . . . . . . . . . . . . . . . . . 50
1.9.2. Integral de Fourier . . . . . . . . . . . . . . . . 63
1.10. Ejercicios propuestos . . . . . . . . . . . . . . . . . . . 70
1.10.1. Respuestas . . . . . . . . . . . . . . . . . . . . . 74
1.11. Auto evaluaciones . . . . . . . . . . . . . . . . . . . . 77

2. Funciones Vectoriales de una variable real 92


2.1. Introducción . . . . . . . . . . . . . . . . . . . . . . . . 92
2.2. Funciones Vectoriales . . . . . . . . . . . . . . . . . . 93
2.3. Lı́mite de una función vectorial. . . . . . . . . . . . . . 95
2.3.1. Teorema del lı́mite . . . . . . . . . . . . . . . . 95
2.3.2. Operaciones con funciones vectoriales . . . . . 98
2.3.3. Teoremas del algebra de lı́mites . . . . . . . . . 98
2.3.4. Teorema: producto de función escalar por vec-
torial . . . . . . . . . . . . . . . . . . . . . . . 99
2.4. Continuidad . . . . . . . . . . . . . . . . . . . . . . . 99
2.5. La Derivada . . . . . . . . . . . . . . . . . . . . . . . 100
2.6. Regularidad de una curva . . . . . . . . . . . . . . . . 102
2.6.1. Camino regular . . . . . . . . . . . . . . . . . . 103
2.6.2. Propiedades de la Derivada . . . . . . . . . . . 103
2.7. Parametrización . . . . . . . . . . . . . . . . . . . . . 104
2.7.1. Ejemplos de reparametrizaciones . . . . . . . . 106
2.8. Longitud de Arco . . . . . . . . . . . . . . . . . . . . . 107

vii
2.8.1. La Longitud de Arco como Parámetro . . . . . 109
2.8.2. Parametrización por Longitud de Arco . . . . . 111
2.9. Trayectorias y curvas . . . . . . . . . . . . . . . . . . . 113
2.10. Vectores Unitarios . . . . . . . . . . . . . . . . . . . . 115
2.10.1. Vector Tangente unitario . . . . . . . . . . . . 115
2.10.2. Vector Normal . . . . . . . . . . . . . . . . . . 116
2.10.3. Vector Binormal . . . . . . . . . . . . . . . . . 116
2.11. Curvatura . . . . . . . . . . . . . . . . . . . . . . . . . 118
2.11.1. Cálculo de curvatura usando parámetro t cualquiera
en R3 . . . . . . . . . . . . . . . . . . . . . . . 120
2.12. Planos por un punto de la curva . . . . . . . . . . . . . 123
2.12.1. Plano Osculador . . . . . . . . . . . . . . . . . 124
2.12.2. Plano Normal . . . . . . . . . . . . . . . . . . . 124
2.12.3. Plano Rectificante . . . . . . . . . . . . . . . . 124
2.12.4. Recta Tangente . . . . . . . . . . . . . . . . . . 125
2.12.5. Recta Normal . . . . . . . . . . . . . . . . . . . 125
2.12.6. Recta Binormal . . . . . . . . . . . . . . . . . . 125
2.13. Torsión . . . . . . . . . . . . . . . . . . . . . . . . . . . 127
2.13.1. Cálculo de la torsión usando parámetro t cualquiera
(en R3 ) . . . . . . . . . . . . . . . . . . . . . . 128
2.14. Formulas de Frenet . . . . . . . . . . . . . . . . . . . . 130
2.15. Aplicaciones de Funciones Vectoriales y Curvas . . . . 131
2.15.1. Problemas . . . . . . . . . . . . . . . . . . . . . 132
2.16. Ejercicios resueltos . . . . . . . . . . . . . . . . . . . . 140
2.17. Ejercicios propuestos . . . . . . . . . . . . . . . . . . . 169
2.17.1. Respuestas . . . . . . . . . . . . . . . . . . . . . 173
2.18. Auto Evaluaciones . . . . . . . . . . . . . . . . . . . . 175

viii
3. Funciones de varias variables 189
3.1. Introducción . . . . . . . . . . . . . . . . . . . . . . . . 189
3.2. Funciones Escalares de Variable Vectorial . . . . . . . . 193
3.2.1. Conceptos Topológicos . . . . . . . . . . . . . . 193
3.2.2. Aspectos Geométrico de las Funciones Escalares 197
3.2.3. Gráfica de una Función . . . . . . . . . . . . . . 197
3.2.4. Curvas y Superficies de Nivel . . . . . . . . . . 198
3.2.5. Lı́mite . . . . . . . . . . . . . . . . . . . . . . . 199
3.2.6. Continuidad . . . . . . . . . . . . . . . . . . . . 205
3.2.7. Derivadas Parciales . . . . . . . . . . . . . . . . 207
3.3. Diferenciabilidad en dos variables . . . . . . . . . . . . 209
3.3.1. Derivada Direccional . . . . . . . . . . . . . . . 211
3.3.2. Plano tangente y recta normal . . . . . . . . . . 215
3.3.3. Función Compuesta. La Regla de la Cadena. . . 218
3.3.4. Función Implı́cita . . . . . . . . . . . . . . . . . 222
3.3.5. Jacobiano . . . . . . . . . . . . . . . . . . . . . 226
3.3.6. Máximos y Mı́nimos . . . . . . . . . . . . . . . 228
3.3.7. Extremos Restringidos . . . . . . . . . . . . . . 233
3.4. Problemas Resueltos . . . . . . . . . . . . . . . . . . . 245
3.4.1. Continuidad y diferenciabilidad . . . . . . . . . 245
3.4.2. Regla de la cadena . . . . . . . . . . . . . . . . 248
3.4.3. Derivación Implı́cita . . . . . . . . . . . . . . . 251
3.4.4. Plano Tangente a una Superficie . . . . . . . . . 256
3.4.5. Derivadas Direccionales . . . . . . . . . . . . . . 258
3.4.6. Valores extremos . . . . . . . . . . . . . . . . . 260
3.4.7. Multimplicadores de Lagrange para extremos re-
stringidos . . . . . . . . . . . . . . . . . . . . . 264
3.4.8. Aplicación al cálculo de errores . . . . . . . . . 275
3.5. Ejercicios Propuestos . . . . . . . . . . . . . . . . . . 276
3.5.1. Lı́mites . . . . . . . . . . . . . . . . . . . . . . . 276
3.5.2. Diferenciabilidad, continuidad . . . . . . . . . . 277

ix
3.5.3. Derivadas parciales . . . . . . . . . . . . . . . . 278
3.5.4. Derivadas Direccionales . . . . . . . . . . . . . . 280
3.5.5. Puntos crı́ticos máximos y mı́nimos . . . . . . . 284
3.6. Aplicaciones Derivada Direccional . . . . . . . . . . . . 286
3.7. Aplicaciones de Máximos y Mı́nimos . . . . . . . . . . 289
3.7.1. Aplicación al campo de la mecánica . . . . . . . 289
3.7.2. Aplicaciones a la geometrı́a . . . . . . . . . . . 292
3.7.3. Aplicación al campo de la economı́a . . . . . . 297
3.7.4. Problemas Propuestos de Aplicaciones . . . . . 302
3.8. Auto evaluaciones . . . . . . . . . . . . . . . . . . . . . 304

4. Integración Multiple 315


4.1. Integrales dobles y triples . . . . . . . . . . . . . . . . 315
4.1.1. Integrales Dobles . . . . . . . . . . . . . . . . . 315
4.1.2. Integrales sobre conjuntos acotados de R2 . . . 320
4.1.3. Teorema de Fubini . . . . . . . . . . . . . . . . 322
4.1.4. Áreas y Volumenes . . . . . . . . . . . . . . . . 327
4.1.5. Cambio de variable . . . . . . . . . . . . . . . . 329
4.2. Aplicaciones de la integral doble . . . . . . . . . . . . 333
4.2.1. Masa de una región plana de densidad variable. 333
4.2.2. Momentos y centroide de una región plana . . . 334
4.3. Integrales triples . . . . . . . . . . . . . . . . . . . . . 337
4.3.1. Ideas preliminares . . . . . . . . . . . . . . . . . 337
4.3.2. Teorema de Fubini . . . . . . . . . . . . . . . . 337
4.3.3. Teorema de la integral triple (Para dominios más
generales) . . . . . . . . . . . . . . . . . . . . . 338
4.3.4. Cambio de variable para integrales triples . . . 342
4.3.5. Formula del cambio de variable . . . . . . . . . 345
4.3.6. Masa, Momentos, y Centroide de una Región del
Espacio . . . . . . . . . . . . . . . . . . . . . . 348
4.4. Ejercicios resueltos integrales triples y dobles . . . . . . 350

x
4.4.1. Cálculo de integrales dobles en coordenadas rectángu-
lares cartesianas . . . . . . . . . . . . . . . . . . 350
4.4.2. Cambios de orden de Integración . . . . . . . . 361
4.4.3. Cambios de variables: Coordenadas polares . . . 363
4.4.4. Cambios de variables. Coordenadas curvilı́neas . 367
4.4.5. Cálculo de integrales triples en coordenadas rectángu-
lares cartesianas . . . . . . . . . . . . . . . . . . 373
4.4.6. Coordenadas esféricas . . . . . . . . . . . . . . 377
4.4.7. Coordenadas Cilı́ndricas . . . . . . . . . . . . . 380
4.5. Ejercicios propuestos integrales dobles y triples . . . . . 389
4.5.1. Integrales dobles . . . . . . . . . . . . . . . . . 389
4.5.2. Cálculo de Integrales dobles usando transforma-
ción de coordenadas . . . . . . . . . . . . . . . 392
4.5.3. Integrales triples . . . . . . . . . . . . . . . . . 394
4.5.4. Integrales triples iteradas . . . . . . . . . . . . . 394
4.5.5. Integrales triples en coordenadas rectángulares
cartesianas. . . . . . . . . . . . . . . . . . . . . 396
4.5.6. Calcular las integrales dadas usando las coorde-
nadas adecuadas: . . . . . . . . . . . . . . . . . 397
4.5.7. Resolver las integrales usando coordenadas esféri-
cas: . . . . . . . . . . . . . . . . . . . . . . . . . 399
4.6. Aplicaciones integrales dobles y triples . . . . . . . . . 401
4.6.1. Volumenes de cuerpos en el espacio . . . . . . . 401
4.6.2. Área de figuras planas. . . . . . . . . . . . . . . 404
4.6.3. Momentos y centros de masa para placas planas
delgadas . . . . . . . . . . . . . . . . . . . . . . 406
4.6.4. Centroide de figuras geométricas . . . . . . . . . 407
4.6.5. Momentos y Centros de masa de un sólido . . . 412
4.6.6. Masa de un sólido . . . . . . . . . . . . . . . . . 413
4.6.7. Determinación del centroide dee un sólido . . . 424
4.7. Autoevaluación Integrales dobles y triples . . . . . . . 426

xi
5. Integral de Linea 436
5.1. Campos vectoriales . . . . . . . . . . . . . . . . . . . . 441
5.2. Cambio de parametrización . . . . . . . . . . . . . . . 446
5.2.1. Reparametrización . . . . . . . . . . . . . . . . 447
5.3. Independencia de trayectoria . . . . . . . . . . . . . . . 449
5.4. Campos Conservativos . . . . . . . . . . . . . . . . . . 451
5.4.1. Campo gradiente . . . . . . . . . . . . . . . . . 451
5.4.2. Teorema de Green . . . . . . . . . . . . . . . . 456
5.5. Aplicaciones de la integral de trayectoria . . . . . . . . 461
5.5.1. Área de una pared . . . . . . . . . . . . . . . . 465
5.6. Aplicaciones de la integral de lı́nea . . . . . . . . . . . 467
5.7. Problemas Resueltos . . . . . . . . . . . . . . . . . . . 477
5.7.1. Campo conservativo . . . . . . . . . . . . . . . 481
5.7.2. Teorema de Green . . . . . . . . . . . . . . . . 485
5.8. Problemas propuestos . . . . . . . . . . . . . . . . . . . 492
5.8.1. Integral de trayectoria . . . . . . . . . . . . . . 492
5.8.2. Integral de lı́nea . . . . . . . . . . . . . . . . . . 493
5.8.3. Campos conservativos . . . . . . . . . . . . . . 494
5.8.4. Teorema de Green . . . . . . . . . . . . . . . . 495
5.9. Autoevaluaciones . . . . . . . . . . . . . . . . . . . . . 496

6. Integrales de superficie 504


6.1. Superficie orientada . . . . . . . . . . . . . . . . . . . . 507
6.1.1. Integral de flujo. . . . . . . . . . . . . . . . . . 508
6.1.2. Superficies Parametrizadas. . . . . . . . . . . . 510
6.1.3. Vector normal a S : . . . . . . . . . . . . . . . . 510
6.1.4. Área de una superficie parametrizada . . . . . . 513
6.1.5. Integral de una función escalar sobre una super-
ficie. . . . . . . . . . . . . . . . . . . . . . . . . 516
6.1.6. Integral de Superficie de campos vectoriales . . 517
6.1.7. Aplicación al campo de la fı́sica: . . . . . . . . . 519

xii
6.2. Teoremas de Gauss y de Stokes . . . . . . . . . . . . . 519
6.2.1. Divergencia . . . . . . . . . . . . . . . . . . . . 519
6.2.2. Teorema de la divergencia de Gauss. . . . . . . 520
6.2.3. Teorema de Stokes. . . . . . . . . . . . . . . . . 524
6.3. Problemas Resueltos . . . . . . . . . . . . . . . . . . . 525
6.3.1. Integrales de superficie . . . . . . . . . . . . . . 529
6.3.2. Integral de Flujo de un campo vectorial . . . . . 532
6.3.3. Teorema de la divergencia de Gauss . . . . . . . 537
6.3.4. Teorema de Stokes . . . . . . . . . . . . . . . . 543
6.4. Ejercicios Propuestos . . . . . . . . . . . . . . . . . . . 550
6.4.1. Área de una superficie . . . . . . . . . . . . . . 550
6.4.2. Integrales de funciones escalares sobre superficie 553
6.4.3. Integral de Flujo . . . . . . . . . . . . . . . . . 555
6.4.4. Teorema de la divergencia de Gauss . . . . . . . 557
6.4.5. Teorema de Stokes . . . . . . . . . . . . . . . . 561
6.5. Aplicaciones . . . . . . . . . . . . . . . . . . . . . . . . 564
6.5.1. Aplicaciones Integral de Flujo . . . . . . . . . . 564
6.5.2. Aplicación del teorema de Gauss . . . . . . . . 568
6.5.3. Aplicación teorema de Stokes . . . . . . . . . . 573
6.5.4. Aplicacion teorema de Green . . . . . . . . . . 576
6.5.5. Aplicaciones al electromagnetismo . . . . . . . 580
6.6. Auto evaluaciones . . . . . . . . . . . . . . . . . . . . . 587

xiii
Capı́tulo 1

Serie de Fourier

En el presente capı́tulo se estudiarán los conceptos básicos , métodos


de cálculo de los coeficientes y condiciones de convergencia para repre-
sentar funciones mediante series e integrales de Fourier .

1.1. Introducción
Las funciones periódicas se presentan frecuentemente en una gran var-
iedad de problemas de fı́sica e ingenierı́a, tales como propagación de
ondas en un medio, conducción del calor a lo largo de una varilla ,
resonancia nuclear magnética ,en consecuencia, abordar la solución de
tales problemas, requiere del estudio de la serie de Fourier.
La serie de Fourier es la representación de una función en términos
de una serie trigonométrica infinita cuyas bases son las funciones seno
y coseno. Algunas de las ventajas de ésta representación sobre otras
representaciones, tales como, las series de Taylor, son:
a) primero, se puede representar funciones periódicas en términos de
las bases seno y coseno que tienen diferentes frecuencias;
b) segundo, se puede representar funciones discontinuas en un punto o
seccionalmente continuas en un número finito de puntos;
c) tercero, permite encontrar la respuesta de un sistema que es pertur-
bado por una función periódica, en términos de una frecuencia funda-
mental y cada una de las frecuencias armónicas.

1
1.2. Propiedades Generales

Para problemas con condiciones de frontera periódicas en el intervalo


L  x  L, nos preguntamos si es posible expresar una función como
una combinación lineal de funciones seno y coseno de frecuencias cada
vez mayores, como la siguiente serie infinita (conocida como serie de
Fourier de f (x)):

1
X ◆ ✓ ✓ ◆
n⇡x n⇡x
f (x) = a0 + (an cos + bn sin ) (1,1,1)
n=1
L L

Obviando la igualdad, vale preguntarse ¿converge esta serie infinita?,¿qué condi-


ciones debe cumplir f para que se dé la convergencia?,¿cuándo converge
a f (x)?
Estas preguntas no tienen una respuesta sencilla. Sin embargo, las series
de Fourier normalmente funcionan bastante bien.
Supongamos que f admite desarrollo en serie de Fourier, ¿cómo se ob-
tienen los coeficientes a0 , an y bn en términos de f (x) ?. Para responder
esta última pregunta necesitaremos del siguiente lema.

1.2.1. Lema Elemental

Lema 1.2.1. i) Si m y n son números enteros no negativos distintos,


entonces:
Z L ⇣ n⇡x ⌘ ⇣ m⇡x ⌘ Z L ⇣ n⇡x ⌘ ⇣ m⇡x ⌘
cos cos dx = sin sin dx = 0
L L L L L L
(1.2.1)
ii) Para cualquier par de enteros no negativos m y n,entonces:
Z L ⇣ n⇡x ⌘ ⇣ m⇡x ⌘
cos sin dx = 0 (1.2.2)
L L L

iii)Para cualquier entero positivo n, entonces:


Z L ⇣ n⇡x ⌘ Z L ⇣ n⇡x ⌘
2
cos dx = sin2 dx = L (1.2.3)
L L L L

2
Demostración:
Se prueba integrando directamente: usando la identidad cos ↵ cos =
cos(↵ ) + cos(↵ + )
2
Z L ⇣ n⇡x ⌘ ⇣ m⇡x ⌘ Z ✓ ◆ Z ✓ ◆
1 L (n m)⇡x 1 L (n + m)⇡x
i) cos cos dx = cos dx+ cos d
L L L 2 L L 2 L L

✓ ◆L
1 L m) ⇡x(n
= sin
2 (n m) ⇡ L L
✓ ◆L
1 L (n + m) ⇡x
+ sin
2 (n + m) ⇡ L L
=0

Además, si m = 0 y n 6= 0 es facilmente verificable que la integral

es cero.

Z L ⇣ n⇡x ⌘ ⇣ m⇡x ⌘
En forma similar se prueba que sin sin dx = 0
L L L
sin(↵ ) + sin(↵ + )
ii) Usando la identidad trigonométrica sin ↵ cos =
2
Z L ⇣ n⇡x ⌘ ⇣ m⇡x ⌘ ZL ✓ ◆ ZL ✓ ◆
1 (n
m) ⇡x 1 (n + m) ⇡x
cos sin dx = sin dx + sin dx
L L L 2 L 2 L
L L
✓ ◆
1 L (n m) ⇡x L
= cos | L
2 (n m) ⇡ L
✓ ◆
1 L (n + m) ⇡x L
sin | L
2 (n + m) ⇡ L
= 0

A estas fórmulas integrales se les llama relaciones de ortogonalidad


n ⇣ n⇡x ⌘ ⇣ m⇡x ⌘o
y diremos que en tal caso el conjunto de las funciones cos , sin
L L
8 n = 0, 1, 2, ..., y 8 m = 1, 2, ..., son ortogonales en [ L, L]
iii) La demostración queda como ejercicio para el lector, se prueba

3
integrando directamente.En sı́ntesis, se puede puntualizar que:
Z L ⇣ n⇡x ⌘ ⇣ m⇡x ⌘ ⇢
1 0, si m 6= n
cos cos dx = = m,n
L L L L 1, si m = n
Z L ⇣ n⇡x ⌘ ⇣ m⇡x ⌘ ⇢
1 0, si m 6= n
sin sin dx = = m,n
L L L L 1, si m = n
donde m,n se define como el delta de Kroneker.
Z L ⇣ n⇡x ⌘ ⇣ m⇡x ⌘
1
cos sin dx = 0 8m, n
L L L L
Z L ⇣ n⇡x ⌘
cos dx = 0 8m, n ; y
L L
Z L ⇣ m⇡x ⌘
sin dx = 0 8m, n
L L

1.3. La serie de Fourier de una función


Se debe distinguir entre f (x) y su serie de Fourier en el intervalo L
x  L:
Serie de Fourier de f (x)

1 ⇣
X ⇣ n⇡x ⌘ ⇣ n⇡x ⌘⌘
a0 + an cos + bn sin
n=1
L L

La serie trigonométrica puede incluso no converger y si converge, puede


que no lo haga a f (x). Partiendo del supuesto que la serie converge
podrı́amos determinar los coeficientes de Fourier a0 , an y bn , usando
las relaciones de ortogonalidad.
Sea f (x) definida en el intervalo L  x  L:
1 ⇣
X ⇣ n⇡x ⌘ ⇣ n⇡x ⌘⌘
f (x) = a0 + an cos + bn sin (1.3.4)
n=1
L L

Integrando la identidad ( 1.3.4) se tiene:


Z L Z L X1 ✓ Z L ⇣ n⇡x ⌘ Z L ⇣ n⇡x ⌘ ◆
f (x)dx = a0 dx+ an cos dx + bn sin dx
L L n=1 L
L L L

4
Como todas las integrales de la derecha valen cero, excepto la primera,
ZL
se deduce de aquı́ el valor de a0 , suponiendo que la f (x)dx existe, ası́.
L

ZL
1
a0 = f (x)dx
2L
L
⇣ m⇡x ⌘
Para el cálculo de an multiplicamos la identidad ( 1.3.4) por cos
L
e integramos la serie término a término, queda
Z L ⇣ m⇡x ⌘ L Z⇣ m⇡x ⌘
f (x) cos dx = a0 cos dx+
L L L L
X1 ✓ Z L ⇣ n⇡x ⌘ ⇣ m⇡x ⌘ Z L ⇣ n⇡x ⌘ ⇣ m⇡x ⌘ ◆
an cos cos dx + bn sin cos dx =
n=1 L
L L L L L
X1
=0+ an · L n,m + 0 = Lam
n=1

Por lo tanto, al evaluar n,m , queda un sólo término:


Z L ⇣ m⇡x ⌘
f (x) cos dx = am L,
L L

ası́ el valor de am es
Z L ⇣ m⇡x ⌘
1
am = f (x) cos dx, 8 m 1
L L L

Cambiando el ı́ndice libre m por n , en ambos lados de la ecuación,


queda Z
1 L ⇣ n⇡x ⌘
an = f (x) cos dx, 8 n 1
L L L
⇣ m⇡x ⌘
Ahora, multiplicando ( 1.3.4) por sin e integrando de manera
L
similar y por el lema se tiene
Z ⇣ n⇡x ⌘
1 L
bn = f (x) sin dx, 8 n 1
L L L

Hemos determinado los coeficientes a0 , an y bn ,claro que, bajo muchos


supuestos. Estos cálculos sugieren la siguiente definición.

5
1.3.1. Coeficientes de Fourier
Definición 1.-
i) Sea f una función Riemann integrable en [ L, L], las constantes
9
ZL >
>
1 >
>
a0 = f (x)dx >
>
2L >
>
>
=
Z LL ⇣ n⇡x ⌘
1 2,1,1
an = f (x) cos dx para n = 1, 2, 3, ... >
>
LZ L L >
>
⇣ n⇡x ⌘ >
>
1 L >
bn = f (x) sin dx para n = 1, 2, 3, ... >
>
;
L L L
se denominan los coeficientes de Fourier de f en [ L, L].
ii) La serie:
1 h
X ⇣ n⇡x ⌘ ⇣ n⇡x ⌘i
f (x) ⇠ a0 + an cos + bn sin
n=1
L L

es la serie de Fourier de f en el intervalo [ L, L] , cuando los coefi-


cientes están dados por (2,1,1). Para no hablar de convergencia todavı́a,
escribimos el signo”⇠”que significa que a la derecha se tiene la serie de
Fourier de f en L  x  L.

Observese que, la serie de Fourier de f , se puede interpretar como una


generalización de una combinación lineal en una base ortogonal seno,
coseno, que es aplicada a una función en lugar de un vector estándar
en Rn .
El siguiente ejemplo ilustra como dada una función periódica f (x), de
perı́odo 2⇡, se calculan los coeficientes de Fourier y expresa la serie
trigonométrica de Fourier correspondiente.

Ejemplo 1: Determinar la serie de Fourier de f (x) = x si x 2 [ ⇡, ⇡]

6
Solución: La gráfica de la función es:

Los coeficientes de Fourier de f en [ ⇡, ⇡] , son:


1
R⇡
a0 = 2⇡ xdx = 0

1
R⇡ ⇥ 1 x
⇤⇡
an = ⇡
x cos (nx) dx = n2 ⇡
cos(nx) + n⇡
sin(nx) ⇡
=0

1
R⇡ ⇥ 1 x
⇤⇡
bn = ⇡
x sin(nx)dx = n2 ⇡
sin(nx) n⇡
cos(nx) ⇡

) bn = n2 ( 1)n+1 8n 1
Por tanto, la serie de Fourier de f en[ ⇡, ⇡] es:
1
X 2
( 1)n+1 sin(nx)
n=1
n

1.3.2. Atributos de la función


Lo anteriormente expuesto es válido para cierto tipo de funciones, nos
referimos a las funciones f (x) que son seccionalmente continuas.

Definición 2.- Sea f (x) definida en [a, b]. Entonces f es seccional-


mente continua en [a, b] si:
a) f es continua en [a, b] ,excepto quizás en un número finito de puntos.
b) Ambos lı́mx!a+ f (x) y lı́mx!b f (x) existen y son finitos.
c) f no es continua en x0 , x0 2 (a, b) y los lı́mites lı́mx!x+0 f (x) y lı́mx!x0 f (x)
existen y son finitos .

7
Definición 3.- f (x) es seccionalmente suave en [a, b] si f y f 0 son

seccionalmente continuas en [a, b] .

1
Ejemplo 2: Muestre que f (x) = x 3 no es seccionalmente suave en
ningún intervalo cerrado que contenga en su interior al cero.

Solución: En efecto, se tiene que


1 2 1 2
f 0 (x) = x 3 =) lı́mf 0 (x) = lı́m x 3 = 1,
3 x!0 x!0 3

no existe. Por tanto, la función no es seccionalmente suave.


Observación:
Las funciones seno y coseno, que aparecen como bases en la serie de
2L
Fourier, tienen perı́odos diferentes los que son iguales a para n 1.
n
Por otra parte, un múltiplo entero del perı́odo de una función perı́odica
es también un perı́odo , podemos afirmar entonces, que 2L es el perı́odo
común para las funciones seno y coseno del desarrollo de la serie. Por
lo anterior, la serie de Fourier no sólo representa a f en el intervalo
L  x  L , sino que, proporciona una extensión perı́odica de f en
todos los reales.

8
Ejemplo 3: Encontrar el perı́odo de la función f (x) = 100 cos2 x.

Solución: Utilizando la identidad trigonométrica cos2 ✓ = 12 (1+cos 2✓)


se tiene
1
f (x) = 100 cos2 x = 100 (1 + cos 2x)
2
luego queda

f (x) = 50 + 50 cos 2x

como el perı́odo de cos 2x es ⇡ y una función constante tiene cualquier


perı́odo, entonces f (x) es de perı́odo ⇡.

1.3.3. Convergencia de las series de Fourier


A continuación vamos a establecer las condiciones de suficiencia que
debe cumplir una función f (x) para que pueda ser representada por
medio de una serie de Fourier.

Teorema 1.3.1. Si f (x) es seccionalmente suave en el intervalo


[ L, L], entonces la serie de Fourier de f (x) converge. i) A la ex-
tensión perı́odica de f (x), en los puntos que la extensión perı́odica sea
continua. ii) Al promedio de los lı́mites laterales 12 (f (x+ ) + f (x )) en
los puntos donde la extensión perı́odica tenga una discontinuidad de
salto.

En el siguiente ejemplo, se evalúa si la serie de Fourier resultante de


una función f (x) en un punto x0 dado converge o no en ese punto

9

0 si 3x0
Ejemplo 4: Sea f (x) = .Construir la serie de
x si 0  x  3
Fourier y analizar la convergencia en todo R

Solución: Representemos la gráfica de la función

Los coeficientes de la serie de Fourier de f (x),son:


Z3 Z3
1 1 3
a0 = f (x)dx = xdx =
6 6 4
3 0
Z3 ⇣ n⇡x ⌘
1
an = f (x) cos dx
3 3
3
Z ⇣ n⇡x ⌘ 3
1 3
1 9 cos n⇡x
3
3x sin n⇡x
3
= x cos dx = 2 2
+
3 0 3 3 n⇡ n⇡
0
3 3
= (cos(n⇡) 1) = 2 2 (( 1)n 1)
n2 ⇡ 2 n⇡

Z3 ⇣ n⇡x ⌘
1
bn = f (x) sin dx
3 3
3
Z ⇣ n⇡x ⌘ 3
1 3
1 9 sin n⇡x 3
3x cos n⇡x
3
= x sin dx = +
3 0 3 3 n2 ⇡ 2 n⇡
0
3 3
= cos(n⇡) = ( 1)n
n⇡ n⇡
Por consiguiente, la serie de Fourier la podemos escribir
1 ✓ ⇣ n⇡x ⌘ ⇣ n⇡x ⌘◆
3 X 3 n 3 n
+ (( 1) 1) cos ( 1) sin
4 n=1 n2 ⇡ 2 3 n⇡ 3

10
Tenemos que f es continua en [ 3, 3] ,por lo tanto su extensión perı́od-
ica es seccionalmente continua en R , con discontinuidad de salto en
los puntos x = 3 ± 6n, n 2 Z
Por lo tanto,
⇢ de acuerdo al teorema la serie converge a
f (x) si x 6= 3 ± 6n
fE (x) = 3 n2Z
2
si x = 3 ± 6n
entonces
1 ✓ ⇣ n⇡x ⌘ ⇡ ⇣ n⇡x ⌘◆
3 3 X 1 n n
fE (x) = + 2 (( 1) 1) cos ( 1) sin
4 ⇡ n=1 n2 3 n 3

los coeficientes (( 1)n 1) son nulos, si n es número par e iguales a


2, si n es número impar. Entonces

1 ✓ ✓ ◆ ⇣ n⇡x ⌘◆
3 6 X 1 (2n 1)⇡x ⇡ n
f (x) = cos + ( 1) sin
4 ⇡ 2 n=1 (2n 1)2 3 6n 3

Al evaluar la convergencia en x0 = 3, punto de discontinuidad de la

función, se obtiene

1  1
3 3 6X 1 X 1 ⇡2
= ( 1)2n 1
=) =
2 4 ⇡ n=1 (2n 1)2
2
n=1
(2n 1)2 8

Obsérvese que a partir de la convergencia de la serie de Fourier en un


punto se puede inferir la convergencia de la suma de términos de la
serie resultante.

Definición 4.- Una suma parcial de la serie de Fourier es una suma


de la forma:
N
X ⇣ n⇡ ⌘ ⇣ n⇡ ⌘
S n = a0 + an cos x + bn sin x
n=1
L L

Observación. Al truncar la serie infinita se obtiene un polinomio de


grado n.

11
Ejemplo 5 Sea f (x) = x + ⇡, x 2 [ ⇡, ⇡] . Determine la serie de
Fourier y obtener la gráfica de sumas parciales S1 (x), S3 (x), S1 0(x).

Solución : La gráfica de la función es

Los coeficientes de Fourier de f en [ ⇡, ⇡]


Z⇡  ⇡
1 1 x2 1
a0 = (x + ⇡)dx = + ⇡x = 2⇡ 2 = ⇡
2⇡ 2⇡ 2 ⇡ 2⇡

) a0 = ⇡
Z⇡  ⇡
1 1 1 x
an = (x + ⇡) cos (nx) dx = cos(nx) + sin(nx)
⇡ ⇡ n2 n ⇡

) an = 0
Z⇡  ⇡
1 1 1 x
bn = (x + ⇡) sin(nx)dx = sen(nx) cos s(nx)
⇡ ⇡ n2 n ⇡

) bn = n2 ( 1)n+1

Ası́ la serie de Fourier de f (x) es

X1 ✓ ◆
( 1)n+1 sin 2x sin3x
⇡+2 sin(nx) = ⇡ + 2 sin x + ..
n=1
n 2 3

Para visualizar la convergencia de está serie gráfiquemos algunas de sus


sumas parciales

Xn
( 1)n
Sn (x) = ⇡ + 2 sin(kx)
k=1
k

12
Obtengamos S1 :
X1
( 1)n
S1 (x) = ⇡ + 2 sin(kx)
k=1
k

13
Obtengamos S3

X3
( 1)n
S3 (x) = ⇡ + 2 sin(kx)
k=1
k

Finalmente Obtengamos S10


X10
( 1)n
S10 = ⇡ + 2 sin(kx)
k=1
k

A partir de este ejemplo, podemos inferir que para las series de Fourier
las gráficas de las sumas parciales son curvas aproximadas de la gráfica
de la función perı́odica representada por la serie. Se puede visualizar
además, que en la medida que es mayor el número de términos de
las sumas parciales estas convergen de mejor forma a la gráfica de la
función f .

14
1.3.4. La integral de funciones pares e impares

Lema 1.3.1. (de funciones pares e impares) Sea f una función in-
tegrable en [ L, L] . a) Si f una función par en [ L, L], entonces
RL RL
f (x)dx = 2 f (x)dx. b) Si f es función impar en [ L, L], entonces
L 0
RL
f (x)dx = 0.
L

Demostración
a) f función par, entonces f ( x) = f (x) 8x 2 R. Considerando que
f
es par y el cambio de variable t = x se tiene

Z0 Z0 ZL ZL
f (x)dx = f ( x)dx = f (t)dt = f (x)dx
L L 0 0

entonces

ZL Z0 ZL ZL
f (x)dx = f (x)dx + f (x)dx = 2 f (x)dx
L L 0 0

b) f función impar, entonces f ( x) = f (x) 8x 2 R. Usando este


hecho y el cambio de variable t = x se tiene

Z0 Z0 Z0 LZ

f (x)dx = f ( x)dx = f (t)dt = f (x)dx


L L L 0

entonces
ZL Z0 ZL ZL ZL
f (x)dx = f (x)dx + f (x)dx = f (x)dx f (x)dx = 0
L L 0 0 0

lo que demuestra el lema.


A continuación, vamos a determinar los coeficientes y la serie de Fourier
coseno (o seno) según corresponda, dada una función f par (o impar)
de periodo 2L.

15
1.3.5. Teorema de las funciones pares y de las im-
pares
Teorema 1.3.2. Sea f una función integrable en [ L, L], a) Si f es
par, la serie de Fourier de f en [ L, L] es
1
X ⇣ n⇡x ⌘
a0 + an cos
n=1
L

ZL ZL ⇣ n⇡x ⌘
1 2
con coeficientes a0 = f (x)dx y an = f (x) cos dx,
L L L
0 0
se denomina serie de cosenos. b) Si f es impar, la serie de Fourier de
f en [ L, L] es
X1 ⇣ n⇡x ⌘
bn sin
n=1
L
Z ⇣ n⇡x ⌘
2 L
con coeficiente bn = f (x) sin dx, se denomina serie de
L 0 L
senos.

Demostración: Se deja al lector, debe aplicar el Lema 1.3.1 en el cálculo


de los coeficientes de Fourier.

Ejemplo 6: Calcule la serie de Fourier de f (x) = 1 |x| en 2


x  2.

Solución: A partir de la gráfica de la función podemos inferir que la


función es par.

Es decir f ( x) = 1 | x| = 1 |x| = f (x) 8x 2 R, luego se tiene


que f es par.
Los coeficientes del desarrollo de Fourier, son:

16
2
Z2 ✓ ◆
1 1 x2
a0 = (1 x)dx = x =0
2 2 2
0 0
Z 2 ⇣ n⇡x ⌘ Z 2 ⇣ n⇡x ⌘ Z 2 ⇣ n⇡x ⌘
2
an = (1 x) cos dx = cos dx x cos dx
2 0 2 0 2 0 2
" #2
4 cos n⇡x2
2x sin n⇡x
2
= 0 +
n2 ⇡ 2 n⇡
0

por consiguiente

0 si n es par
an = 8
(2n 1)2 ⇡ 2
si n es impar
Ası́ la serie de Fourier de f (x) = 1 |x| es:
1 ✓ ◆
8X 1 (2n 1)⇡x
cos
⇡ n=1 (2n 1)2
2 2

En muchos problemas se tiene la posibilidad de trabajar con series de


senos o series de cosenos. Por ejemplo , al resolver ecuaciones diferen-
ciales parciales de segundo orden aplicando el método de separación de
variables.

1.4. Desarrollos llamados de medio rango


Sea una función f seccionalmente continua que está definida sólo en
el semi-intervalo [0, L], queremos obtener el desarrollo de f en serie de
Fourier 8x 2 [0, L] . Una forma de hacer lo anterior es extender f al
intervalo [ L, L] y por supuesto , puede ser hecho de muchas maneras,
sin embargo, dos extensiones son las más convenientes e importantes.
Construir una extensión impar lo que origina una serie de senos o
construir un extensión par lo que determina una serie de cosenos.
Estas se denominan desarrollos de medio rango.

1.4.1. Extensión impar:


Supongamos que conocemos f (x) solamente para 0  x  L, entonces
podemos extenderla como una función impar, obteniendo otra función

17
denotada fi (x) definida por:

f (x), 0  x  L
fi (x) =
f ( x), L  x  0

como se muestra en la figura adjunta.

Si f (x) es seccionalmente suave en 0  x  L, entonces fi (x) es también


seccionamente suave y se puede aplicar el teorema de convergencia de
series de Fourier.

La serie de Fourier de fi (x) es

1
X ⇣ n⇡x ⌘
fi (x) = bn sin , LxL
n=1
L

Como estamos interesados solamente en lo que ocurre entre 0  x  L.


En esa región f (x) es idéntica a fi (x) y la serie de Fourier es

1
X ⇣ n⇡x ⌘
f (x) = bn sin , 0xL
n=1
L
Z L ⇣ n⇡x ⌘
2
con coeficiente bn = f (x) sin
L 0 L

Ejemplo 7. Sea la función f (x) = x en el interior 0  x  L.


Obtener el desarrollo de medio rango considerando una extensión im-
par.

18
Solución. Consideremos la extensión impar de f (x) en 0  x  L,
la gráfica de f muestra que la serie de fourier de senos converge a f (x)
en 0  x  L. Sin embargo, en x = L hay una discontinuidad de salto,
luego la serie converge a cero aunque f (L) 6= 0.

Z L ⇣ n⇡x ⌘ Z L ⇣ n⇡x ⌘
2 2 2L
bn = f (x) sin dx = x sin dx = ( 1)n+1
L 0 L L 0 L n⇡

Por lo tanto, la serie resultante es:


1
2L X ( 1)n+1 ⇣ n⇡x ⌘
x= sin , 0xL
⇡ n=1 n L

1.4.2. Extensión par


Supongamos ahora que conocemos f (x) solamente para 0  x  L
, entonces la extendemos como función par, obteniendo otra función
denotada fp (x) definida por:

f (x), 0  x  L
fp (x) =
f ( x), L  x  0
como muestra la figura adjunta:
Si f (x) es seccionalmente continua en 0  x  L, entonces su extensión
par fp (x) lo será también por lo que se puede aplicar el teorema de
convergencia de series de Fourier.
En el intervalo 0  x  L, la función f (x) es idéntica a su extensión
par. La serie que se obtiene se denomina serie de Fourier de cosenos de
f (x).

1
X ⇣ n⇡x ⌘
a0 + an cos , 0  x  L, con coeficientes
n=1
L

19
Z L Z L ⇣ n⇡x ⌘
1 2
a0 = f (x)dx y an = f (x) cos dx
L 0 L 0 L

Ejemplo 8: Construir la serie de Fourier de Cosenos de f (x) = x en

0  x  L.

Solución: Por las caracterı́sticas de la extensión en lo que concierne


a la continuidad de la función tenemos:
1
X ⇣ n⇡x ⌘
x = a0 + an cos , 0xL
n=1
L
Z L Z L
1 1 L 1 x2 L
a0 = f (x)dx = xdx = =
L 0 L 0 L 2 0 2
Z L ⇣ ⌘ Z L ⇣ n⇡x ⌘
2 n⇡x 2
an = f (x) cos dx = f (x) cos dx
L 0 L L 0 L

0 si n par.
an = 4L
n2 ⇡ 2
si n impar.

20
Finalmente, la serie de Fourier coseno de f (x) = x en 0  x  L es:
1 ✓ ◆
L 4L X 1 (2n 1)⇡x
cos
2 ⇡ 2 n=1 (2n 1)2 L

1.5. Diferenciacion e Integración de la se-


ries de Fourier
1.5.1. Derivación
Las series infinitas, aún las convergentes no siempre se pueden derivar
término a término. Un caso ilustrativo, es el de la función f (x) = x
definida para ⇡  x  ⇡, cuya serie de Fourier es
X1
2( 1)n+1
sin(nx)
n=1
n

que converge para ⇡ < x < ⇡, es decir


X1
2( 1)n+1
x= sin(nx), x 2 ] ⇡, ⇡[
n=1
n

Si diferenciamos, esta serie término a término tenemos:


1
X
2( 1)n+1 cos(nx)
n=1

la cual es una serie que no converge en ] ⇡, ⇡[ , ya que si an =


2( 1)n+1 cos(nx) para cada x 2] ⇡, ⇡[, lı́m an no existe, como no
n!1
P1
ocurre que an ! 0 ,concluimos que 2( 1)n+1 cos(nx) no converge
n=1
para cada x 2] ⇡, ⇡[.
Por otro lado, f 0 (x) = 1 8x 2] ⇡, ⇡[. Esto muestra en este caso que
la derivada término a término de la serie, no converge a la derivada de
la función que representa.
La dificultad se nos presenta cada vez que la serie de Fourier de f (x)
tiene una discontinuidad de salto, la derivación término a término no
está justificada en estos casos. Sin embargo, podemos aquı́ considerar el
siguiente teorema que precisa las condiciones para permitir la derivación
término a término.

21
Teorema 1.5.1. Sea f una función continua en [ L, L] con f ( L) =
f (L), si f 0 es seccionalmente suave en [ L, L] donde f 00 (x) existe se
tiene.
n⇡ h ⇣ n⇡x ⌘ ⇣ n⇡x ⌘i
X1
0
f (x) = an sin + bn cos
n=1
L L L

Demostración.-
Se deja al lector, se sugiere escribir la serie de Fourier de f 0 (x), con-
siderando que esta serie converge a f 0 (x) siempre que f 00 (x) exista. Use
integración por partes para relacionar los coeficientes de f 0 (x) con los
correspondientes de f (x).

Ejemplo 9. Dada la función f (x) = x2 en ⇡  x  ⇡ , verifique


si la derivada de esta serie existe.

Solución Claramente se satisface las hipótesis de la proposición an-


terior. La serie de Fourier de la función f (x) en [ ⇡, ⇡] es:
(Ver Problema 2 en problemas resueltos)
X1
⇡2 ( 1)n
f (x) = +4 cos(nx)
3 n=1
n2

Como f 0 (x) = 2x es continua, y existe f 00 (x) = 2 en todo el intervalo,


entonces para ⇡ < x < ⇡
X1
0 ( 1)n+1
f (x) = 2x = 4 sin(nx)
n=1
n

Note que este resultado concuerda con lo establecido en el ejemplo 1


del inciso 2.1.

22
1.5.2. Integración

La precaución que se tiene para la derivación término a término de la


serie de Fourier no se requiere para el caso de la integración .

Teorema 1.5.2. Sea f una función seccionalmente suave en [ L, L]


con serie de Fourier

1 h
X ⇣ n⇡x ⌘ ⇣ n⇡x ⌘i
f (x) = a0 + an cos + bn sin
n=1
L L

Entonces para cada x 2 [ L, L] .

Z
LX1h ⇣ n⇡x ⌘ ⇣ ⇣ n⇡x ⌘ ⌘i
x 1
f (t)dt = a0 (x+L)+ an sin bn cos ( 1)n
L ⇡ n=1 n L L

Demostración;
Zx
Sea F (x) = f (t)dt a0 x 8x 2 [ L, L] , ası́ definida F es continua
L
en [ L, L] , además

ZL ZL
F ( L) = f (t)dt a0 ( L) = a0 L y F (L) = f (t)dt a0 L = 2a0 L a0 L = a0 L
L L

Por lo cual F ( L) = F (L), asimismo F 0 (x) = f (x) a0 8x 2 [ L, L]


donde f es continua. Entonces podemos asegurar que F 0 es seccional-
mente continua en [ L, L] y por el teorema de convergencia tenemos
que

1 h
X ⇣ n⇡x ⌘ ⇣ n⇡x ⌘i
F (x) = A0 + An cos + Bn sin (1.5.5)
n=1
L L

donde para n 1.

23
Z L ✓ ◆
1 n⇡t
An = F (t) cos dt integrando por partes
L L L
✓ ◆L Z L ✓ ◆
1 L n⇡t L 0 n⇡t
= F (t) sin F (t) sin dt
L n⇡ L L
n⇡ L L
Z L ✓ ◆
L n⇡t
= 0 (f (t) a0 ) sin dt
n⇡ L L
Z L ✓ ◆ Z L ✓ ◆
L n⇡t L n⇡t
= f (t) sin dt + a0 sin dt
n⇡ L L n⇡ L L
L
An = bn
n⇡
donde bn es el coeficiente correspondiente de la serie de Fourier de f en
[ L, L].

De manera analoga se tiene que:


Z L ✓ ◆
1 n⇡t L
Bn = F (t) sin dt = an
L L L n⇡

donde an es también el correspondiente coeficiente de la serie de Fourier


de f en [ L, L].
Por lo tanto, reemplazando en 1.5.5

LX1h ⇣ n⇡x ⌘ ⇣ n⇡x ⌘i


1
F (x) = A0 + bn cos + an sin , x 2 [ L, L]
⇡ n=1 n L L

para A0 tenemos:

1
X 1
X
F (L) = a0 L = A0 + An cos(n⇡) =) A0 = a0 L An cos(n⇡)
n=1 n=1

finalmente

1
LX 1
A0 = a0 L + bn cos(n⇡)
⇡ n=1 n

24
ahora sustituyendo A0 se tiene

LX1h ⇣ n⇡x ⌘ ⇣ n⇡x ⌘i


1 1
L1X
F (x) = a0 L+ bn cos(n⇡)+ bn cos + an sin
⇡ n n=1 ⇡ n=1 n L L

y reemplazando en la igualdad inicial obtenemos lo que afirma el teo-


rema.

Z
LX1h ⇣ n⇡x ⌘ ⇣ ⇣ n⇡x ⌘ ⌘i
x 1
f (t)dt = a0 (x+L)+ an sin bn cos ( 1)n
L ⇡ n=1 n L L

1.5.3. Identidad de Parseval


Sea f una función seccionalmente continua en [ L, L] y tal que f 0
es también seccionalmente continua en [ L, L].
Si 1 h
X ⇣ n⇡x ⌘ ⇣ n⇡x ⌘i
f (x) = a0 + an cos bn sin
n=1
L L

es la serie de Fourier de f , entonces


ZL 1
X
1 2 2 ⇥ ⇤
[f (x)] dx = 2 (a0 ) + (an )2 + (bn )2
L n=1
L

que se conoce como identidad de Parseval

Prueba: La serie de Fourier de f converge a f (x) para cada x del


intervalo [ L, L].
1 h
X ⇣ n⇡x ⌘ ⇣ n⇡x ⌘i
f (x) = a0 + an cos bn sin
n=1
L L

Multiplicando por f (x) se tiene


1 h
X ⇣ n⇡x ⌘ ⇣ n⇡x ⌘i
2
f (x) = a0 f (x) + an f (x) cos bn f (x) sin
n=1
L L

25
podemos integrar término a término.

Z L Z L 1 
X Z L ⇣ n⇡x ⌘ Z L ⇣ n⇡x ⌘
2
[f (x)] dx = a0 f (x)dx+ an f (x) cos bn f (x) sin
L L n=1 L L L L

de aquı́ recordando lo que son los coeficientes de una serie de Fourier


se tiene.

ZL 1
X
2 2
[f (x)] dx = 2 (a0 ) L + L [an · an + bn · bn ] =)
n=1
L

ZL 1
X
1 2 2 ⇥ ⇤
[f (x)] dx = 2 (a0 ) + (an )2 + (bn )2
L n=1
L

Obsérvese que la identidad de Parseval, permite inferir la suma de


una serie infinita, dada una función f que tiene una representación de
Fourier para cada x del intervalo [ L, L].


x ⇡<x<⇡
Ejemplo 10. Sea f (x) = , perı́odica de perı́odo
0 x = ⇡, ⇡
P1 1 ⇡2
2⇡. Pruebe que 2
= .
n=1 n 6

Figura 1.1: gráfica función perı́odo 2⇡

Solución: Como f (x) en es una función impar se tiene que :


an = 0 para n = 0, 1, 2, ...y

26
Z⇡ Z⇡  ⇡
1 2 2x cos(nx)
bn = x sin (n⇡) dx = x sin (n⇡) dx =
⇡ ⇡ n⇡ 0
⇡ 0
⇢ 2
n
n = 1, 3, 5, ...
=) bn = 2
n
n = 2, 4, 6, ...

Por tanto

1
X 
n+1 sin(nx) sin x sin 2x sin 3x
f (x) ⇠ 2 ( 1) =2 + ...
n=1
n 1 2 3

Aplicando la identidad de Parseval

Z⇡ 
1 2 1 1 1 1
x dx = 4 2
+ 2 + 2 + 2 + ... =)
⇡ 1 2 3 4

X1 Z⇡  ⇡
1 1 2 1 x3 ⇡2
2
= x dx = =
n=1
n 4⇡ 4⇡ 3 ⇡ 6

X1
1 ⇡2
=
n=1
n2 6

1.6. Integral de Fourier


Las series de Fourier nos proporcionan una herramienta poderosa para
representar funciones perı́odicas. Luego, es conveniente generalizar este
método para incluir funciones no perı́odicas.
A continuación en esta sección vamos a representar una función f no
perı́odica por medio de la integral de Fourier

Definición.- Si f (x) definida en ( 1, 1) es seccionalmente continua


en cada intervalo finito y tiene derivadas por la derecha e izquierda en

27
R
1
todo punto y tal que |f (x)| dx converge, entonces la integral de
1
Fourier de f se define como:
Z1
[A(w) cos wx + B(w) sin wx] dw
0

donde:

Z 1
1
A(w) = f (t) cos wtdt
⇡ 1
Z 1
1
B(w) = f (t) sin wtdt
⇡ 1

A(w) y B(w) se llaman los coeficientes de la integral de Fourier de f (x).

Ejemplo 11. Encontrar la representación por medio de la integral de


Fourier de la función:

1 , |x| < 1
f (x) =
0 , |x| > 1

Solución: Primeramente, determinemos la gráfica de la función

Ahora, calculemos los coeficientes de la Integral de Fourier


1Z Z1 
1 sin wu 1 1 sin w
A(w) = f (u) cos wudu = cos wudu = | 1=2
⇡ w ⇡ w
1 1

Z
1 Z1
1
B(w) = f (u) sin wudu = sin wudu = 0

1 1

28
Por lo tanto, la integral de Fourier de f (x) es:
Z1
1 2
sin w cos wxdw
⇡ w
0

1.6.1. Criterio de convergencia de la integral de


Fourier
Si f (x) es seccionalmente continua en [ L, L] 8 L > 0 y tal que
R
1
|f (t)| dt existe, entonces la integral de Fourier converge a 12 [f (x+ ) +
1
f (x )] (Promedio de los lı́mites izquierdo y derecho de f (x)), 8 x donde
0 0
f (x+ ) y f (x ) existen.

Ejemplo 12. Estudie la convergencia de la Integral de Fourier del


ejemplo 11

Solución Sea f (x) definida en ejemplo 11, debido a que f (x) es sec-
cionalmente suave, la integral de Fourier de f (x) converge a 12 [f (x+ ) +
f (x )] 8 x. De acuerdo con el criterio de convergencia se tiene:
8
Z1 < 1 si 1<x<1
2 sin w 1
cos wxdw = si x = ±1
⇡ w : 2
0 0 si |x| > 1

En particular, una situación interesante se da cuando x = 0.

Z1
2 sin w
cos 0dw = 1 =)
⇡ w
0
Z1
sin w ⇡
dw =
w 2
0

Aunque integrales de este tipo no pueden expresarse en términos de


funciones elementales, suelen presentarse en matemáticas aplicadas con
tal frecuencia , que han recibido un nombre especial y se encuentran

29
tabuladas.
En particular sabemos que:

sin w
lı́m =1
w!0 w

y que

Z1
sin w ⇡
dw =
w 2
0

En el caso de la integral de Fourier, la gráfica de la función f se obtiene


mediante aproximaciones sucesivas sustituyendo el lı́mite superior de
la integral 1 por los números x. De aquı́ que la integral

Zz
sin w
cos wxdw
w
0

es una aproximación de la integral encontrada anteriormente, y por lo


tanto de f (x) .
Supongamos que sólo consideramos las frecuencias w < w0 .En este
caso, nos da como aproximación de f (x)

Zw0
2 cos wx sin w
f (x) ⇡ dw
⇡ w
0

30
Ahora bien,
sin (wx + w) sin(wx w)
cos wx sin w =
2

y, por consiguiente, podemos escribir la última integral en la forma

Zw0 Zw0
1 sin w(x + 1) 1 sin w (x 1)
f (x) ⇡ dw dw
⇡ w ⇡ w
0 0

Consideremos el cambio de variable u = w (x ± 1) =) du = wdx para


la primera y la segunda de estas integrales. Entonces tenemos
w0 Z
(x+1) w0 Z
(x 1)
1 sin u 1 sin u
f (x) ⇡ du du
⇡ u ⇡ u
0 0
1 1
f (x) ⇡ Si [w0 (x + 1)] Si [w0 (x 1)]
⇡ ⇡

En términos fı́sicos, estas curvas describen la salida de un filtro ideal


de pasa baja, que elimina todas las frecuencias superiores w0 cuando
la señal de entrada es un impulso aislado rectangular.

1.6.2. Integrales de Fourier de cosenos y senos


Sea f (x) una función definida en [0, 1), podemos extender esta función
a una función par o impar en ( 1, 1) y calcular la integral de Fourier
de esta última, que resulta ser de coseno y seno respectivamente, lo
cual es completamente análoga a los desarrollos en cosenos y senos de
una función definida en un intervalo [0, L] para el caso de las series de
Fourier.

R
1
Definición: Sea f definida en [0, 1) y sea |f (u)| du convergente,
0
la integral de Fourier en cosenos de f es
Z1
A(w) cos(wx)dw
0

donde el coeficiente es:

31
Z1
2
A(w) = f (u) cos(wu)du

0

A su vez, la integral de Fourier en senos de f es

Z1
B(w) sin(wx)dw
0

donde el coeficiente es:

Z1
2
B(w) = f (u) sin(wu)du

0

En cuanto a la convergencia de la integral de Fourier, en este caso,


si f es seccionalmente suave en todo el intervalo [0, 1], entonces esta
integral converge a 12 [f (x+ ) + f (x )] en (0, 1).


x2 si 0  x  10
Ejemplo 13: Encontrar la integral de Fourier de f (x) = ,
0 si x > 10

si:

a) se considera una extension par de f (x)

b) se considera una extension impar de f (x); y luego

c) en ambos casos, determinar las convergencias de estas integrales .

Solución: Consideremos la gráfica de la función

a) Para obtener la integral de Fourier de cosenos, extendemos f como


una función par fP definida en toda la recta real, luego:

32
Z1 Z10
2 2
A(w) = f (u) cos(wu)du = u2 cos(wu)du
⇡ ⇡
0 0
2 3
2 Z10
2 4u 2
= sin(wu)|10
0 u sin(wu)du5
⇡ w w
0
 2 ✓ ◆ 10
2 u 2 1 u
= sin(wu) sin(wu) cos(wu)
⇡ w w w2 w 0
✓ ◆
1 200 4 40
= 3
sin 10w + cos 10w
⇡ w w ⇡w2

Por tanto, la integral de Fourier de cosenos es:

Z1 ✓ ◆
1 200 4 40
sin 10w + cos 10w cos wxdw
⇡ w w3 w2
0

Al aplicar el criterio de convergencia obtenemos:

Z1 ✓ ◆
1 200 4 40
sin 10w + cos 10w cos wxdw
⇡ w w3 w2
0
8 2
>
> x si 0 < x < 10
<
0 si x > 10
=
>
> 0 si x=0
:
50 si x = 10

33
b) Para obtener la integral de Fourier de senos, extendemos f como
una función impar fI definida en toda la recta real.

Z1 Z10
1 2
B(w) = f (t) sin wtdt = u2 sin wudu
⇡ ⇡
1 0
0 1
 2 10 Z10
2@ u 2
= cos wu + u cos wuduA
⇡ w 0 w
0
 ✓ ◆ 10
2 u2 2 1 u
= cos wu + cos wu + sin wu
⇡ w w w2 w 0
 2
2 10 2 20 2
= cos 10w + 3 cos 10w + 2 sin 10w
⇡ w w w w3

entonces la integral de Fourier de senos es:

Z1 ✓ ◆
1 200 4 40 4
+ 3 cos 10w + sin 10w sin wxdw
⇡ w w w2 w3
0

Finalmente, al aplicar el criterio de convergencia obtenemos:

Z1 ✓ ◆
1 200 4 40 4
+ 3 cos 10w + sin 10w sin wxdw
⇡ w w w2 w3
0
8 2
>
> x si 0 < x < 10
<
0 si x > 10
=
>
> 0 si x=0
:
50 si x = 10

ax
Ejemplo 14: Encontrar la integral de Fourier de f (x) = e si

x > 0 y a es una constante tal que a > 0, considerando una extensión


a) par de f.
b) impar de f.
c) en ambos casos, determinar las convergencias de estas integrales.
Solución
Extensión Par

34
Extensión impar

a) Puesto que f es par , es decir f (x) = f ( x) 8x 2 R se tiene

Z1
f (x) = A(w) cos(wx)dw
0

donde el coeficiente es:

Z1
2 au
A(w) = e cos(wu)du

0

Integrando por partes se tiene

Z1 h ⇣ w ⌘iR
au a au
e cos(wu)du = lı́m e senwu + cos wu
a2 + w2 R!1 a 0
0
a
=
a2 + w2

35
Por consiguiente,

2 a
A (w) =
⇡ a + w2
2

Sustituyendo esta expresión se obtiene:

Z1
2a cos(wx)
dw
⇡ a2 + w 2
0

para x > 0, a > 0.

Finalmente, como la función es continua 8x > 0, la integral converge


a f (x) , entonces
Z1
ax 2a cos(wx)
f (x) = e = dw
⇡ a2 + w 2
0
Z1
cos(wx) ⇡e ax
=) dw =
a2 + w 2 2a
0

b) Puesto que f es impar , es decir f (x) = f ( x) 8x 2 R se tiene

Z1
f (x) = B(w) sin(wx)dw
0

donde el coeficiente es:

Z1
2 au
B(w) = e sin(wu)du

0

Integrando por partes se tiene

Z1
au 1 ⇥ au
⇤R
e sin(wu)du = lı́m e (asenwu w cos wu) 0
a2 + w2 R!1
0
w
=
a2 + w2

36
Por consiguiente,

2 w
B (w) =
⇡ a + w2
2

Sustituyendo esta expresión se obtiene:

Z1
ax 2 w sin(wx)
f (x) = e = dw
⇡ a2 + w 2
0

para x > 0, a > 0.


Estos ejemplos ilustran como puede aplicarse la representación de la
integral de Fourier para evaluar integrales.

37
1.7. Aplicaciones de Series de Fourier
Para dar una visión del uso de las series e integrales de Fourier, se for-
mularán, analizarán y resolverán problemas de sistemas fı́sicos sujetos
a perturbaciones periódicas y no periódicas.

1.7.1. Onda cuadrada alta frecuencia


Una aplicación simple de la Serie de Fourier la podemos encontrar
en el análisis de circuitos electrónicos que son diseñados para manejar
pulsos variables agudos, tales como, una onda cuadrada o un ”diente
de sierra”. Supongamos que una onda cuadrada está definida por la
función:

0, ⇡<x<0
f (x) =
h, 0<x<⇡

Encuentre la serie de Fourier que representa esta señal.


Solución
Los coeficientes de Fourier son:
Z ⇡
1 h
a0 = hdt =
2⇡ 0 2
Z ⇡
1
an = h cos ntdt = 0, n 1
⇡ 0
Z ⇡
1 h
bn = h sin ntdt = (1 cos n⇡)
⇡ 0 n⇡
⇢ 2h
n⇡
, n impar =) bn = (2n2h1)⇡
bn =
0; n par
Ası́ la serie resultante es:
1
X ✓ ◆
2h h sin x sin 3x sin 5x
f (x) = sin (2n 1) x = + + + + ...
n=1
(2n 1)⇡ 2 1 3 5

38
Es importante decir que el primer término representa el promedio de
f (x) sobre el intervalo [ ⇡, ⇡] y que todos los términos en base coseno
se anulan. Además f (x) h2 es una función impar, luego ,tenemos
una serie de fourier sólo con base seno. Por otra parte, los coficientes
bn decrecen inversamente proporcional con n. Fisicamente esto signifi-
ca que la onda cuadrada debe contener muchos componentes de alta
frecuencia. Si el aparato electrónico no deja pasar estos componentes,
la onda cuadrada resultante emerge más o menos redondeada.

1.7.2. Rectificador de onda completa.


Consideremos ahora la salida de un rectificador de onda completa, que
produce corriente continua pulsante como muestra la figura. El rectifi-
cador se puede modelar como un dispositivo que se alimenta con una
onda senoidal ,que deja pasar los los pulsos positivos, e invierte los
pulsos negativos. Esto produce:


sin !x, 0 < !x < ⇡
f (x) =
sin !x, ⇡ < !x < 0
Encuentre la serie de Fourier que respresenta esta señal
Solución
Puesto que f (x) es una función par, es decir f (x) = f ( x), la serie
de fourier será cosenoidal

✓Z 0 Z ⇡ ◆ Z ⇡
1 2 2
a0 = sin !td(!t) + sin !td(!t) = sin !td(!t) =
2⇡ 2⇡ 0 ⇡
Z ⇡ ⇡ 0
2
an = sin !t cos n!td(!t), n 1
⇡ 0
⇢ 2 2
⇡ n2 1
, n par =) an = ⇡1 4n24 1
an =
0, n impar
bn = 0, 8 n

39
Por lo tanto, la serie resultante es:

1
2 4X 1
f (x) = 2
cos (2n!x)
⇡ ⇡ n=1 (4n 1)

La frecuencia de oscilacion más baja es 2!.Las componentes de alta


frecuencia decaen inversamente con n2 , lo que muestra que el rectifi-
cador de onda completa hace un buen trabajo para producir un modelo
aproximado de la corriente continua.

1.7.3. Ecuación de calor unidimensional

El flujo unidimensional de calor en un cuerpo material homogéneo


2 u(x,t)
está modelado por la ecuación c2 @ @x 2 = @u(x,t)
@t
donde u(x, t) es
2
la temperatura del cuerpo y c = 2 la constante de difusión del calor.
Si se considera que 0 < x < 3 y t > 0, y que las temperaturas en
la fronteras son u(0, t) = u(3, t) = 0, limu(x, t) < 1 , entonces la
x!0
solución general de este problema esta dado por:

1
X
2n2 ⇡ 2 t n⇡x
u (x, t) = Cn e sin , 0<x<3y t>0
n=1
3

Encontrar la temperatura de la barra , si la temperatura inicial es


u(x, 0) = 25o C , 0 < x < 3 .
Solución:
Evaluemos la solución general para t = 0, lo que produce:

1
X n⇡x
u (x, 0) = 25 = Cn sin , 0<x<3
n=1
3

Se obtiene una serie de Fourier seno. Ası́, para determinar


⇢ la constante
25 0<x<3
Cn se debe construir una extensión impar f (x) = .
25 3<x<0

40
Podemos encontrar entonces:
Z Z
2 L n⇡x 2 3 n⇡x
Cn = f (x) sin dx = 25 sin dx
L 0 L 3 0 3
 3
50 3 n⇡x 50 (1 cos n⇡)
Cn = cos =
3 n⇡ 3 0 n⇡

De modo, que la temperatura en la barra queda

1
X 50 (1 cos n⇡) 2n2 ⇡ 2 t n⇡x
u (x, t) = e sin , 0<x<3y t>0
n=1
n⇡ 3

Este problema ilustra la importancia de la serie de Fourier para re-


solver problemas de aplicación modelados por ecuaciones diferenciales
parciales de segundo orden.

1.7.4. Ecuación de calor: barra aislada


El flujo unidimensional de calor en un cuerpo material homogéneo
2 u(x,t)
está modelado por la ecuación c2 @ @x 2 = @u(x,t)
@t
donde u(x, t) es
2
la temperatura del cuerpo y c la constante de difusión del calor.
En el caso de una barra aislada, que se prolonga hacia el infinito en
ambos sentidos, la solución general está dada por
Z 1
2 2
u(x, t) = (A (w) cos(wx)+B (w) sin(wx) ) e c w t dw. Si se aplica la
0
condicion inicial u(x, 0) = f (x) , 1 < x < 1 ,donde
R 1 f (x) es la tem-
peratura inicial, se obtiene que u(x, 0) = f (x) = 0 (A (w) cos(wx) +
BZ (w) sin(wx) ) dw es una integral de Fourier
Z con coeficientes A (w) =
1 1 1 1
f (v) cos(wv) dv y B (w) = f (v) sin(wv) dv
⇡ 1 ⇡ 1
Determine la integral de Fourier, si la función temperatura inicial es
2
f (x) = e x /2 ; 1 < x < 1, y la solución general de está ecuación.

Solución:
Z 1
Como f es una función par se tiene Ip = A (w) cos(wx) dw, con
0
, y B (w) = 0 luego

41
Z 1 Z 1
2 v 2 /2 0 2 v 2 /2
A (w) = e cos(wv) dv =) A (w) = ve sin(wv) dv
⇡ 0 ⇡ 0

Integrando por partes se tiene


 Z 1 1
0 2 v 2 /2 v 2 /2
A (w) = e sin(wv) + w e sin(wv) dv
⇡ 0 0

Evaluando la integral y resolviendo EDO(1)

0 2h ⇡ i1 0
A (w) = 0 + w( A (w) =) A (w) = wA (w)
⇡ 2 0
w2 /2
A (w) = Ce , C constante

Luego la integral de Fourier es:


Z ⇡
x2 /2 w2 /2
e =C e cos(wx) dw
0

Por tanto, la solución general queda:


Z 1
w2 /2 c2 w 2 t
u(x, t) = C (e cos(wx)) e dw
0

Este problema ilustra la importancia de la Integral de Fourier para


resolver problemas de aplicación modelados por ecuaciones de difusion
del calor.

1.7.5. Ecuación de Onda


Una onda unidimensional que se desplaza en una cuerda elástica ho-
2 u(x,t) 2
mogénea, está modelado por la ecuación c2 @ @x 2 = @ u(x,t)
@t2
donde
u(x, t) es el desplazamiento de la cuerda desde el eje x en el tiempo t
y c2 la constante la rapidez de la onda en el medio.
Si los extremos de la cuerda están fijos en x = 0, x = L , t > 0, es
decir que las condiciones de frontera son u(0, t) = u(L, t) = 0 , entonces
la solución general de este problema está dado por:
1
X n⇡ct n⇡ct n⇡x
u (x, t) = (An cos + Bn sin ) sin , 0<x<Ly t>0
n=1
L L L

42
Considere que la forma inicial de la cuerda está dado por f (x) , es
decir u (x, 0) = f (x) , y que la velocidad inicial de la cuerda es cero,
@u (x, t)
es decir = 0. Encontrar el desplazamiento u (x, t) de la cuerda
@t
en un tiempo posterior.
Solución.
Determinemos las constantes An y Bn de la solución general aplicando
las condiciones iniciales.
@u (x, t)
Para satisfacer la condición = 0 , será necesario derivar la
@t
solución general, entonces
1
X n⇡c n⇡ct n⇡ct n⇡x
ut (x, t) = ( An sin + Bn cos ) sin
n=1
L L L L
X1
n⇡c n⇡x
ut (x, t) = Bn sin () Bn = 0 8n
n=1
L L

De manera que la solución general se reduce a

1
X n⇡ct n⇡x
u (x, t) = An cos sin ,0 < x < L y t > 0
n=1
L L

Ahora, apliquemos la condición u (x, 0) = f (x) , para determinar la


constante An . Esto da como resultado

1
X n⇡x
u (x, 0) = f (x) = An sen ,0 < x < L y t > 0
n=1
L

que corresponde a una serie de Fourier senoidal. Ası́, es⇢ necesario con-
f (x) si 0 < x < L
siderar una extension impar de la función dada fi (x) = ,
f ( x) si L<x<0
de este modo el coeficiente queda
Z L
2 n⇡x
An = f (x) sin dx
L 0 L
El resultado final es
X1 ✓ Z ◆
2 L n⇡x n⇡t n⇡x
u (x, t) = f (x) sin dx cos sin ,0 < x < L y t > 0
n=1
L 0 L L L

43
1.7.6. Deflexión de una viga

Una viga de longitud L , esta soportada desde sus extremos como mues-
tra la figura adjunta . Sobre la viga actúa una carga uniformemente dis-
tribuida q por unidad de longitud y su deflexión está dada por y (x) .
Si se escoge la dirección del eje y apuntando hacia abajo, como indica
la figura, se sabe que la función y (x) satisface la ecuación:

d4 y 1
4
= q (x)
dx EI
donde q (x) es la carga por unidad de longitud en el punto x, I es
el momento de inercia y E el módulo de elasticidad de la viga. Si en
nuestro caso estas tres cantidades son constantes encuente la deflexión
y (x) de la viga.
Solución.
Puesto que la función y (x) debe se nula en los extremos x = 0 y x = L,
la podemos representar mediante una serie de Fourier de senos.

1
X n⇡x
y (x) = bn sin , x 2 [0, L]
n=1
L

Si suponemos que y (x) es una función continua , con derivadas contin-


uas hasta el cuarto orden en [0, L] , entonces
1 ⇣
X n⇡x ⌘4 n⇡x
y (4) (x) = bn sin , x 2 [0, L]
n=1
L L

A su vez la carga distrribuida por unidad de longitud q (x) = q, también


puede ser desarrollada en serie de Fourier de senos
1
X n⇡x
q= qn sin , x 2 [0, L]
n=1
L

44

1
RL n⇡x
4
n = impar
de donde qn = q sin dx = n⇡
L 0 L 0 n = par
Sustituyendo ambas series de Fourier en la ecuación diferencial

1 ⇣
n⇡x ⌘2
X 1
n⇡x X n⇡x
bn sin = qn sin , x 2 [0, L]
n=1
L L n=1
L

Comparando los coeficientes de ambas serie queda


⇢ 4 ⇢ 4qL4 1
n⇡x 2 n⇡
n = impar n = impar
L
bn = =) bn EI⇡5 n5
0 n = par 0 n = par
Por tanto la deflexión queda determinada por
1
4qL4 X 1 (2n 1)⇡x
y (x) = 5 5
sin , x 2 [0, L]
EI⇡ n=1 (2n 1) L

1.8. Problemas Propuestos


Rectificador media onda
La función adjunta sirve para modelar la salida de un rectificador de
media onda:

sin !x, 0  !x  ⇡
f (x) =
0, ⇡  !x  0
a) Represente graficamente la señal de salida si ésta se extiende peri-
odicamente con periodo 2⇡.
b) Determine la serie de Fourier que la representa.

Solución:

1
1 1 2X 1
f (x) = + sin !x cos (2n!x)
⇡ 2 ⇡ n=1 (4n2 1)
Onda triangular
Una onda triángular se representa por la función:

x, ⇡ < x < 0
f (x) =
x, 0 < x < h

45
a) Represente graficamente la función.
b) Represente f(x) mediante una serie de Fourier.
c) Estudie la convergencia de la serie en x = ⇡, x = 0, y x = ⇡
X1
1 ⇡2
d) Muestre que: =
n=1
(2n 1)2 8

Solución:

b)
1
⇡ 4 X cos (2n 1) x
f (x) =
2 ⇡ n=1 (2n 1)2
Conducción del calor.
Consideremos una varilla delgada, aislada, situada a lo largo del eje
x, desde x = 0 hasta x = a,y supongamos que la conducción de calor
desde la varilla hacia el exterior se da solamente por los extremos de
ella, los cuales se mantienen a temperatura cero. En fı́sica se muestra
que si en tiempo t = 0 la temperatura u a lo largo de la varilla es igual
a u(x, 0) = bn sin nx, donde bn = cte y n 2 Z+ , entonces para el tiempo
2
t > 0 la temperatura es igual a u(x, t) = bn (sin nx) e n t , donde  > 0
es una constante positiva. Asimismo, hay un principo de superposición
que nos permite añadir los efectos de diferentes distribuciones iniciales
de temperatura. Por lo tanto, si la temperatura inicial es:

1
X
u(x, 0) = f (x) = bn sin nx
n=1

entonces en tiempo t > 0, se tiene:


1
X
n2 t
u(x, t) = bn (sin nx) e para 0  x  a
n=1

De acuerdo con todo esto, hallar la temperatura para t > 0 para las
siguientes temperaturas iniciales dadas.
a) u(x, 0) = f (x) = 3 sin x + 5 sin 2x. ¿Que tipo de extensión de f (x)se
requiere en este caso?
b) u(x, 0) = f (x) = ex sin x.¿Que tipo de extensión de f (x)se requiere
en este caso?

46
Soluciones.

a)
t 4t
u(x, t) = f (x) = 3 sin xe + 5 sin 2xe

b)
1
4 X n ⇥ ⇤ n2 t
u(x, t) = 2
( 1)n 1
e⇡ 1 sin nx e
⇡ n=1 n + 4

Valor de la raı́z media cuadrática

Las series de fourier se constituyen en una herramienta poderosa en el


análisis del comportamiento de los sistemas fı́sicos sujetos a pertuba-
ciones periódicas f (t).
El valor de la raı́z media cuadrática ó RMC de una función f (t), sobre
un intervalo (a, b) ,se define como:

sR
b
a
f 2 (t) dt
hf (t)i =
b a

a) Sea f (t) una función definida x 2 [a, b] , con un perı́odo funda-

mental T = b a. Pruebe que aplicando la identidad de Parseval el


valor RMC se reduce a la formula:

v
u 1
u
t 2 1X 2
hf (t)i = a0 + [a + b2n ]
2 n=1 n

b) Determine RMC de f (t) = E sin !t, con E y ! constantes positi-


vas.
Solución:

47
2⇡
b) El perı́odo fundamental de la función f (t) = E sin !t, es !
.
Entonces el valor RMC de f (t) es:

s
Z 2⇡
1 ! E
hf (t)i = E 2 sin2 (!t) dt = p
(2⇡/!) 0 2

Cuerda vibrante. Extremos fijos


Un cuerda vibra libremente con ambos estremos fijos en x = 0 y x = L.
a) Si su movimiento esta descrito por la ecuación de onda:

@ 2 u (x, t) 2
2 @ u (x, t)
= v
@t2 @x2

con las condiciones iniciales:

@u (x, 0)
u (x, t) = f (x) y = g(x)
@t

Suponga que la solución de esta ecuación es una serie de Fourier de la


forma:
X 1
n⇡x
u(x, t) = bn (t) sin( )
n=1
L

sustituya esta solución en la ecuacion anterior y determine los coefi-


cientes b (t) .
b) Considere la presencia de un medio resistivo que amortigua las vi-
braciones de acuerdo con la ecuación

@ 2 u (x, t) 2
2 @ u (x, t) @u (x, t)
=v k
@t2 @x2 @t

Suponga que rige la solución anterior con las mismas condiciones ini-
ciales y nuevamente determine el coeficiente b (t) , suponiendo que el
2 k 2
amortiguamiento es pequeño, es decir n⇡L 2
>0
c) Repita los calculos pero suponiendo que el amortiguamiento es grande
n⇡ 2 k 2
es decir L 2
< 0.
Soluciones:

48
a)
n⇡⌫t n⇡ t
bn (t) = An cos( ) + Bn sin( )
L L
Z ⇣ n⇡x ⌘ Z L ⇣ n⇡x ⌘
2 L 2
An = f (x) sin dx, Bn = g (x) sin dx
L 0 L n⇡ 0 L

b)
k
bn (t) = e 2 t (An cos(!n t) + Bn sin(!n t))
Z ⇣ n⇡x ⌘ Z L ⇣ n⇡x ⌘
2 L 2 k
An = f (x) sin dx, Bn = g (x) sin dx + An ,
L 0 L L!n 0 L 2!n
⇣ n⇡ ⌘2 ✓ k ◆2
2
!n = >0
L 2

c)
k
bn (t) = e 2 t (An cosh( n t) + Bn sinh( n t))
Z ⇣ n⇡x ⌘ Z L ⇣ n⇡x ⌘
2 L 2 k
An = f (x) sin dx, Bn = g (x) sin dx + An
L 0 L L n 0 L 2 n
⇣ n⇡ ⌘2 ✓ k ◆2
2
donde, n = <0
L 2

Distribución de temperatura en un disco

En una placa circular de radio ⇢ = 1, cuyas secciones superior e inferior


están aisladas, se mantiene la mitad de su periferia superior a una
temperatura constante T1 y la otra mitad a una temperatura constante
T2 .Encontrar la temperatura de la placa en condiciones estacionarias.
a) La ecuación de difusión del calor, en coordenadas polares (⇢, ✓) ,en
2 2
condiciones estacionarias esta dada por @@⇢2 + ⇢1 @@⇢ + ⇢12 @@✓2 = 0,donde
(⇢, ✓) es la funcion temperatura. Suponga que (⇢, ✓) ,se puede sepa-
rar como (⇢, ✓) = M (⇢) N (✓)y pruebe que la ecuación se transforma
2 00 0 00
en ⇢ MM + ⇢ MM = NN .
b) A partir del resultado anterior , haga cada lado de la ecuación igual
a 2 y encuentre las EDO(2)
2
N ” (✓) + N (✓) = 0

49
⇢2 M ” (⇢) + ⇢M (⇢) + M (⇢) = 0

b) Pruebe que N (✓) = A1 cos ✓ + A2 sin ✓ y M (⇢) = B1 ⇢ +


B2 ⇢ son soluciones de las correspondientes ecuaciones anteriores.
c) Pruebe que la solución general es

X1
T1 + T2 (T T ) (1 cos n⇡) n
(⇢, ✓) = M (⇢) N (✓) = ⇢ senn✓
n=1
2 n⇡

1.9. Ejercicios Resueltos


Mediante la inclusión de ejercicios resueltos se espera que
los estudiantes tengan oportunidad de movilizar sus capaci-
dades para buscar, analizar, procesar, representar y comu-
nicar diferentes tipos de información, decodificando y tra-
duciendo la información contenida en las funciones, gráficos,
series de Fourier, integrales de Fourier y sus propiedades.

1.9.1. Serie de Fourier


Problema 1

Sea f (x) periódica de perı́odo 2 dada por


⇢ 1
2
x, 0  x  1
f (x) = 3
x 2
, 1x2

a) determinar su serie de Fourier


b) estudie la convergencia de la serie en x0 = ⇡
Solución
La serie de Fourier en este intervalo es
1
X
a0 + (an cos (nx) + bn sin n (nx)), x 2 [0, 2]
n=1

con coeficientes

50
1
R2 1
R1 R2
a0 = 2
f (x) dx = 2
( 12 x)dx + 1
2
(x 3
2
)dx
0 0 1
 1  2
1 1 x2 1 x2 3
a0 = 2
x + 2
x =0
2 2 0 2 2 1
R2
an = f (x) cos nxdx
0

R1 1
R2 3
an = 2
x cos nxdx + (x 2
) cos nxdx
0 1

Integrando por partes, se tiene


 1  1
1 cos n⇡x x sin n⇡x
an = sin n⇡x + +
2n⇡ 0 (n⇡)2 n⇡ 0
 2  2
cos n⇡x x sin n⇡x 3
+ + sin n⇡x
(n⇡)2 n⇡ 1
2n⇡ 0

2 (1 ( 1)n )
) an =
(n⇡)2
R2
bn = f (x) sin nxdx
0

R1 1
R2 3
bn = 2
x sin nxdx + (x 2
) sin nxdx
0 1

Integrando por partes, se tiene


 1  1
1 sin n⇡x x cos n⇡x
bn = cos n⇡x +
2n⇡ 0 (n⇡)2 n⇡ 0
 2  2
sin n⇡x x cos n⇡x 3
+ + cos n⇡x =0
(n⇡)2 n⇡ 1
2n⇡ 1

La serie de Fourier de f en [0, 2] es


1
X 1
(1 ( 1)n ) 4X 1
2 cos nx = 2 cos ((2n 1) x)
n=1
(n⇡) 2 ⇡ n=1 (2n 1)2

b) Como f es continua ,entonces la serie en x = ⇡ converge a


1 7
f ( ⇡) = f (4 ⇡) = (4 ⇡) = ⇡
2 2
51
Problema 2

a) Desarrollar en serie de Fourier la función periódica de perı́odo 2⇡,


definida por:

f (x) = x2 , ⇡x⇡

b) A partir del resultado obtenido calcular la suma de:

X1
1
n=1
n2

X1
1
c) Determine la convergencia de la serie
n=1
n4
Solución:
a) La función f es par por lo cual obtendremos una serie de cosenos,
que tiene la forma:
X1
a0 + an cos (nx)
n=1

R⇡ R⇡ h 3 i⇡
1 1 1 x ⇡2
a0 = ⇡
f (x)dx = ⇡
x2 dx = ⇡ 3
= 3
0 0 0

2
R⇡ 2
R⇡
an = ⇡
f (x) cos(nx)dx = ⇡
x2 cos(nx)dx
0 0
h i⇡
x2 sin(nx) 2x cos(nx) 4 cos(n⇡) 4( 1)n
an = n
+ n2
= n2
= n2
0
Luego, la serie de Fourier de f en [ ⇡, ⇡]es:

X1
⇡2 ( 1)n
+4 cos (nx)
3 n=1
n2

Como la función es continua en R ,tenemos:

X1
⇡2 ( 1)n
x2 = +4 2
cos (nx) , 8 x 2 R
3 n=1
n

52
b) La serie numérica se puede obtener poniendo x = ⇡ y f (⇡) = ⇡ 2 ,
✓ ◆
2 ⇡2 1 1 1
⇡ = 4 ...
3 12 22 32

de donde 1 ✓ ◆
X 1 1 ⇡2 ⇡2
= ⇡ =
n=1
n2 4 3 6

c) Como la función f es seccionalmente suave para ⇡  x  ⇡ y


f ( ⇡) = f (⇡) se cumplen las condiciones de suficiencia de la identidad
de Parseval, entonces:
Z⇡  2 2 X1  2
1 ⇥ 2 ⇤2 ⇡ 4 ( 1)n
x dx = 2 + =)
⇡ 3 n=1
n2

 1
2 4 X 16

1 x5
= ⇡ + =)
⇡ 5 ⇡ 9 n=1
n4
1
X 1 ⇡4
=
n=1
n2 90

Problema 3

Sea f (x) = |x| + 1, 1  x  1, la función periódica de perı́odo 2,


determinar:
a) Su serie de Fourier
b) La convergencia de la serie:

1
X 1
n=1
(2n 1)2
1
X 1
c) La convergencia de la serie
n=1
(2n 1)4
Solución
a) f (x) = |x| + 1 es función par, con semiperı́odo L = 1, entonces
tenemos una serie coseno, que tiene la forma:
1
X
S (x) = a0 + an cos (n⇡x)
n=1

53
Con coeficientes
R1 R1
a0 = 12 f (x)dx = 12 (x + 1) dx = 3
2
0 0
R1 R1
an = 2 f (x) cos(n⇡x)dx = 2 (x + 1) cos(nx)dx
0 0
h i1 h i1
an = 2 sin(n⇡x)
n⇡
+ 2 x sin(n⇡x)
n⇡
+ 2 cos(n⇡x)
(n⇡)2
0 0
h i1
2 cos(n⇡x) 2(( 1)n 1)
an = (n⇡) 2 = (n⇡)2
0

0; si n par
an = 4
; si n impar
(n⇡)2

Por consiguiente, la serie de Fourier de f en [ 1, 1] es:


1
3 4X 1
S (x) = cos ((2n 1) ⇡x)
2 ⇡ 2 n=1 (2n 1)2

b) Como la función es continua en R ,considerando el valor x = 0,se


obtiene por el teorema de la convergencia puntual:
1
3 4 X 1
+ 2 = f (0) = 1
2 ⇡ n=1 (2n 1)2
1
X 1 ⇡2
=
n=1
(2n 1)2 8

c) Como la función f es seccionalmente suave para 1  x  1 y


f ( 1) = f (1) se cumplen las condiciones de suficiencia de la identidad
de Parseval, entonces:
Z1 2 X1   2
2 3 4
2 [x + 1] dx = 2 + 2 =)
2 n=1
(2n 1)
0
" #1 1
(x + 1)3 9 X 16
2 = + =)
3 2 n=1 ⇡ (2n 1)4
2
0
1
X 1 ⇡4
=
n=1
(2n 1)2 96

54
Problema 4
[x]
a) Para f (x) = e , 0  x  2 ,obtener su serie de Fourier en cosenos,
de perı́odo 4.
b) Del resultado determinar la convergencia de:

X1
( 1)n 1
n=1
2n 1

Solución
a) Evaluando la función parte entera tenemos
8
< 1 si 0  x < 1
f (x) = e 1 si 1  x < 2
: 2
e si x=2
Con extensión par fp (x) de f (x) se obtiene la serie:
1
X n⇡x
a0 + an cos
n=1
2

1
1
R R2
a0 = 2
1dx + e 1 dx = 12 [1 + e 1 ]
0 1
1
R R2 sinn⇡x
1 sin
n⇡x
an = cos n⇡x
2
dx + e 1
cos n⇡x
2
dx = n⇡
2
|10 + e n⇡
2
|21
2 2
0 1
sin n⇡ 1 sin n⇡ sin
n⇡
sin n⇡
= 2 n⇡2 + 2e n⇡
2
=2 n⇡
2
[1 e 1]

Finalmente, la serie es:

1
X
1+e 1 sin n⇡ n⇡x
+ 2(1 e 1) 2
cos
2 n=1
n⇡ 2

b) Convergencia de x0 = 2 punto de discontinuidad con lı́mites laterales


e 1 se tiene convergencia:

1
X
1 1+e 1
1 sin n⇡ 2
e = + 2(1 e ) cos n⇡
2 n=1
n⇡

55
1
X
e 1
1 1 sin n⇡ 2
= 2(1 e ) cos n⇡
2 n=1
n⇡

X1
( 1)n 1 ⇡
=
n=1
2n 1 4

Problema 5

Sea f (x) = x2 [x] , para x 2 [0, 2] .


a) Obtener la serie de Fourier coseno de f (x) .
b) Obtener a qué valores converge la serie para cada x 2 [0, 2] .

Solución
a) Si se evalúa la función parte entera de x tenemos
[x] = 0, 8x 2 (0, 1) y [x] = 1, 8x 2 (1, 2) .

x2 , 0x<1
Entonces la función queda f (x) = 2
x 1, 1  x < 2

Consideremos ahora una extensión par de la función f , entonces la serie


coseno de f (x) es
1
X ⇣ n⇡x ⌘
S (x) = a0 + an cos
n=1
2

con coeficientes
R2 R1 R2
a0 = 12 f (x) dx = 12 x2 dx + 12 (x2 1)dx
0 0 1
 1  2
1 x3 1 x3 5
a0 = 2
+ 2
x = 6
3 0 3 1
R2
an = f (x) cos n⇡x
2
dx
0
R1 R2 2
an = x2 cos n⇡x
2
dx + (x 1) cos n⇡x
2
dx
0 1
Integrando por partes, se tiene
 ✓ ◆ 1
8x n⇡x 2 2 16 n⇡x
an = 2 cos + x sin
(n⇡) 2 n⇡ (n⇡)3 2 0

56
 ✓ ◆ 2  1
8x n⇡x 2 2 16 n⇡x 2 n⇡x
+ 2 cos + x 3 sin sin
(n⇡) 2 n⇡ (n⇡) 2 1 n⇡ 2 0

16 2
) an = 2 cos n⇡ + sin n⇡
(n⇡) n⇡
Sustituyendo estos resultados, se obtiene la serie de Fourier
1 
5 X 16 2 n⇡x
S (x) = + 2 cos n⇡ + sin n⇡ cos
6 n=1 (n⇡) n⇡ 2

Tenemos que f es seccionamente continua en [0, 2] , por lo tanto su ex-


tensión perı́odica es seccionalmente continua en R , con discontinuidad
de salto en los puntos x = 1 y x = 2
Por lo tanto, de acuerdo con el teorema de convergencia, la serie con-
verge a 8
>
> f (x) si 0  x < 1
< 1
2
si x=1
S(x) =
>
> f (x) si 1  x < 2
: 3
2
si x=2

Problema 6

Utilice la serie de Fourier para demostrar la identidad trigonométrica


3 1
sin3 (x) = sin(x) sin(3x)
4 4

Solución
Se calcula la serie de Fourier de f (x) = sin3 (x) en [ ⇡, ⇡] . Como f (x)
es impar la serie será:
X1
bn sin n⇡
n=1

con coeficientes:

Z⇡
2
bn = sin3 (x) sin(nx)dx

0

57
En primer lugar, calculemos la integral para n 6= 1
Z⇡ h i Z⇡
3 3 cos nx ⇡ 3
sin x sin nxdx = sin x |0 + sin2 x cos x cos nxdx
n n
0 0
cos(n 1)x cos(n + 1)x
Usando la identidad trigométrica: cos x cos nx =
2
La última integral se puede expesar como
Z⇡
3
= sin2 x [cos(n 1)x cos(n + 1)x] dx (1)
2n
0

En segundo lugar, calculemos el valor del coeficiente b1 para n = 1 en


(1)
Z⇡ Z⇡ Z⇡
13 3 3 1 cos 4x
b1 = sin2 x cos 2xdx = (1 cos 2x) cos 2xdx = dx
⇡2 4⇡ 4⇡ 2
0 0 0
2·3⇡ 3
b1 = =
4⇡ 2 4
En tercer lugar, para n > 1 en (1)
2
✓ ◆ Z⇡ ✓ ◆
3 4 2 sin(n + 1)x sin(n 1)x ⇡ sin(n + 1)x sin(n 1)x
bn = sin x + |0 + sin 2xdx
2n n+1 n 1 n+1 n 1
0
Z⇡ ✓ ◆
3 sin(n + 1)x sin(n 1)x
bn = + sin 2xdx
2n n+1 n 1
0
Usando la identidad trigonométrica
Z⇡
3 1 1
bn = (cos(n + 1)x cos(n + 3)x⇡) dx
2n n + 1 2
0
Z⇡
3 1 1
(cos(n 3)x cos(n + 1)x)dx = 0, 8 n 6= 3
2n n 1 2
0

Para n = 3 el cálculo directo, produce:


3 ⇡2 1
b3 = =
2·3·22⇡ 4
Por tanto, la serie de Fourier de f en [ ⇡, ⇡]es:
3 1
sin(x) sin(3x)
4 4

58
Problema 7

Sea f (x) = x(sin x), para ⇡  x  ⇡, entonces:


a) Determine la serie de esta función.

b) Pruebe la convergencia de la serie:


X1
( 1)n 1
=
n=1
n2 1 4

c) Pruebe que esta serie se puede diferenciar término a término y utilice


este hecho para obtener el desarrollo de Fourier de sin (x) + x cos (x) .
Solución
a) La función f (x) es par, es decir f (x) = f ( x) 8 x 2 ( ⇡, ⇡),
entonces:
bn = 0
Z⇡ Z⇡
1 1
a0 = f (x)dx = x sin xdx =
⇡ ⇡
0 0
2 3
Z⇡
14
) a0 = [x ( cos x)]⇡0 + cos xdx5 = 1

0
Z⇡ Z⇡
2 2
an = f (x) cos(nx)dx = x sin x cos(nx)dx
⇡ ⇡
0 0
Para n 6= 1
Z⇡
1
an = x [sin ((n + 1) x) sin ((n 1) x)] dx

0
Integrando por partes, queda
 ✓ ◆ ⇡
1 cos ((n + 1) x) cos ((n 1) x)
an = x +
⇡ (n + 1) (n 1) 0

1 sin ((n + 1) x) sin ((n 1) x)
+
⇡ (n + 1)2 (n 1)2 0
Evaluando los lı́mites de la integral produce

59
2 ( 1)n+1
) an =
n2 1

Para n = 1

Z⇡ Z⇡
2 1 1
a1 = x sin x cos xdx = x sin(2x)dx =
⇡ ⇡ 2
0 0

Por tanto, la serie de Fourier de f para x 2 [ ⇡, ⇡]es:

X1
1 ( 1)n+1
f (x) = 1 cos x + 2 2
cos (nx)
2 n=2
n 1

b) En x = 0 hay un punto de continuidad de la función, entonces la


serie converge a f (0)

X1
1 ( 1)n+1
f (0) = 0 = 1 cos 0 + 2 cos (0)
2 n=2
n2 1

Finalmente

X1
( 1)n+1 1
=
n=2
n2 1 4

c) Sea f (x) = x(sin x), para ⇡  x  ⇡.


i) Como f (x) = x sin x ,es producto de funciones continuas, es continua
en [ ⇡, ⇡] .
ii) f 0 (x) = sin x + x cos x ,es producto y suma de funciones continuas,
es continua en [ ⇡, ⇡] .
00
iii) Existe f (x) = 2 cos x x sin x, y también es continua en [ ⇡, ⇡] .
Además f ( ⇡) = ( ⇡) (sin ( ⇡)) = ( ⇡) ( sin (⇡)) = ⇡ sin ⇡ = f (⇡)
Por tanto, se satisfacen las hipótesis del teorema de diferenciación de
la serie de Fourier, entonces para ⇡ < x < ⇡

60
X1
1 ( 1)n+1
f (x) = 1 cos x + 2 2
cos (nx)
2 n=2
n 1
X1
0 1 ( 1)n
=) f (x) = sin x + x cos x = sin x + 2 sin (nx)
2 n=2
n2 1

Problema 8

a) Desarrollar en serie de Fourier la función perı́odica de perı́odo 2⇡.


Representar graficamente y estudiar la convergencia de la serie en R.

0, si ⇡x0
f (x) =
x, si 0 < x  ⇡

b) A partir del resultado anterior obtenga la suma de la serie:

1
X 1
n=1
(2n 1)2

c) Pruebe que esta serie se puede integrar término


Z x a término y obtener
un desarrollo en serie trigonométrica para f (u) du en [ ⇡, ⇡] .

Solución
a) Calculemos los coeficientes de Fourier.
2 0 3
Z⇡ Z Z⇡ Z⇡
1 1 4 1
a0 = f (x)dx = f (x)dx + f (x)dx5 = xdx
2⇡ 2⇡ 2⇡
⇡ ⇡ 0 0
 2 ⇡
1 x ⇡
) a0 = =
2⇡ 2 0 4
Z⇡ Z⇡
1 1
an = f (x) cos(nx)dx = x cos(nx)dx
⇡ ⇡
⇡ 0

Usando el método de integración por partes se tiene:


 ⇡ 
1 x cos(nx) cos(nx) 1 ( 1)n 1
an = + = 0 0 +
⇡ n n2 0 ⇡ n2 n2

( 1)n 1 0 si n par
an = = 2
2
n⇡ n2 ⇡
si n impar

61
Ası́:

a2n = 0 8n
2
a2n 1 = 8 n.
(2n 1)2 ⇡
Z⇡ Z⇡
1 1
bn = f (x) sin(nx)dx = x sin(nx)dx
⇡ ⇡
⇡ 0
 ⇡
1 x cos(nx) sin(nx) cos(n⇡)
= + =
⇡ n n2 0 n
luego estos coeficientes son:
( 1)n+1
) bn =
n
Por lo tanto, la serie de Fourier de f para x 2 [ ⇡, ⇡] ,es:
1
" #
⇡ X ( 1)n 1 ( 1)n+1
+ cos nx + sin(nx)
4 n=1 n2 ⇡ n

Esta serie converge a:


i) f (x) = 0 para ⇡ < x  0, puesto que, son puntos de continuidad
de f.
ii) f (x) = x para 0 < x < ⇡, son puntos de continuidad de f.
f (⇡+) + f (⇡ ) ⇡
iii) = en los puntos de discontinuidad del tipo
2 2
x = ⇡ + 2n⇡ con n 2 Z.

b) Aplicando el criterio de convergencia en x = 0, f (0) = 0 se tiene


✓ ◆
⇡ 2 1 1 1
0= 2
+ 2 + 2 + ...
4 ⇡ 1 3 5

de donde ✓ ◆
⇡ 2 1 1 1
= 2
+ 2 + 2 + ...
4 ⇡ 1 3 5

y de aquı́
1
X 1 ⇡2
=
n=1
(2n 1)2 8

62

0, si ⇡x0
c) Como f (x) = es una función seccionalmente
x, si 0 < x  ⇡
continua en [ ⇡, ⇡] , con serie de Fourier

1
" #
⇡ X 2 ( 1)n+1
+ cos ((2n 1) x) + sin(nx)
4 n=1 ⇡ (2n 1)2 n

se satisface las hipótesis del teorema de integración de series de Fourier,


luego puede integrase término a término. Entonces, para cualquier x 2
[ ⇡, ⇡] , se tiene:
Z x ⇢
0, si ⇡x0
Primero, f (u) du = x2 en [ ⇡, ⇡]
⇡ 2
si 0<x⇡
Segundo, integrando la serie produce
Z x" 1
!#
⇡ X 2 cos ((2n 1) u) ( 1)n+1 sin(nu)
+ + du
⇡ 4 n=1
⇡ (2n 1)2 n
" 1
!#x
⇡ X 2 sin ((2n 1) u) ( 1)n+1 cos(nu)
= u+
4 n=1
⇡ (2n 1)3 n2

1
!
⇡ X 2 sin (2n 1) x ( 1)n+1 cos(nx) 1
= x + ⇡2 + +
4 n=1
⇡ (2n 1)3 n2 n2

Por tanto, la función f queda representada por la serie anteriormente


obtenida.

1.9.2. Integral de Fourier


Problema 9

a)
⇢ Halle la representación de la integral de Fourier de la función f (x) =
x, |x| < ⇡
0, |x| ⇡
b) De esta representacion deducir que:
Z 1 Z 1
sin(w⇡) cos(w⇡)
2
sin(wx)dx = ⇡ sin(wx)dx
0 w 0 w

63
Solución
a) Como f es una función impar, entonces

Z1
f (x) = B(w) sin(wu)dw
0

con coeficiente

Z⇡
2
B (w) = u sin(wu)du

0 ⇡
2 cos(wu) sin(wu)
= u +
⇡ w w2 0

2 cos(w⇡) sin(w⇡)
B (w) = ⇡ +
⇡ w w2

Por consiguiente

Z1 
2 cos(w⇡) sin(w⇡)
f (x) = ⇡ + sin(wx)dw
⇡ w w2
0

Es la integral de Fourier de f para |x| =


6 0
b) En particular cuando |x| > ⇡, se tiene f (x) = 0. Entonces la
integral converge a cero

Z1 
2 cos(w⇡) sin(w⇡)
0= ⇡ + sin(wx)dw
⇡ w w2
0

Por tanto,

Z1 Z1
sin(w⇡) cos(w⇡)
sin(wx)dw = ⇡ sin(wx)dw
w2 w
0 0

64
Problema 10

Halle la representación de la integral de Fourier de la función f (x) =


xe |x| si x 2 ( 1, 1) y estudie su convergencia en R.

Solución
Se tiene que f (x) es una función impar. Examinemos, si se cumplen las
condiciones de existencia de integral de Fourier.
En primer lugar

Z1 Z1
|x|
xe dx = 2 xe x dx
1 0
2 3
Z1
= 2 4 xe x |1
0 + e x dx5
0
=2·1=2

la integral es convergente
Además, f es continua y diferenciable 8x.
Los coeficientes de Fourier de f son:

A(w) = 0 ya que f es una función impar


Z1
4w
B(w) = ue |u| sin(wu)du =
(1 + w2 )2
1

Entonces, para todo x la integral de Fourier converge a:

Z1
x 4 w
xe = sin(wx)dw
⇡ (1 + w2 )2
0

65
Problema 11

Sea f la⇢función pulso rectangular unitario de perı́odo 2 definida por


1
2
si <x<
f (x) =
0 si 1  x < ó < x  1
a) Representar graficamente f (x)
b) Obtener la serie de Fourier de f (x) .
c) Si an ( ) es el coeficiente n-ésimo de la serie anterior, calcular los
lı́mites:
lı́m ( lı́m+ (an ( )) , lı́m+ ( lı́m (an ( )))
n!1 !0 !0 n!1

Solución
b) Como f es una función par de perı́odo 2 ,entonces :

Z1 Z
1 1
a0 = f (x) dx = dx =
2 2
0 0
Z1 Z
1 1 sen(n⇡ )
an = 2 f (x) cos(n⇡x)dx = 2 cos (n⇡x) dx = = an ( )
2 n⇡
0 0

1 sin(n⇡ )
donde en este caso definimos an ( ) =
n⇡
bn = 0 8n

Luego, se tiene que:


1
1 1 X sen(n⇡ )
f (x) ⇠ + cos (n⇡x) , x 2 [ 1, 1]
2 n=1
n⇡

c) En primer lugar calculemos:

1 sen(n⇡ )
lı́m+ ( lı́m (an ( ))) = lı́m+ ( lı́m ) = lı́m+ (0) = 0
!0 n!1 !0 n!1 n⇡ !0
n!1

En segundo lugar
✓ ◆
1 sen(n⇡ )
lı́m ( lı́m+ (ak ( )) = lı́m ( lı́m+ ) = lı́m (1) = 1
n!1 !0 n!1 !0 n⇡ n!1

66
Problema 12

Dada la función f (x) = xe x


con x > 0,
a) Verifique que considerando las extensiones par e impar de la función
f:

Z 1  Z 1 
1 w2 2w
cos wx dw = senwx dw
0 (1 + w2 )2 0 (1 + w2 )2

b) Estudiar la convergencia de la IF para deducir que:

Z 1  Z 1 
1 w2
dw = dw
0 (1 + w2 )2 0 (1 + w2 )2

Solución
Consideremos para f (x) = xex con x > 0 su extensión par


xe x si x > 0
fp (x) = =)
xex si x < 0
Z1 Z1
1 x
fp (x) ⇠ A (w) cos wxdw con A (w) = 2 xe cos wx dx

0 0

Ahora, consideremos la extensión impar de f


xe x si x > 0
fi (x) = =)
xex si x < 0
Z1 Z1
1
fi (x) ⇠ B (w) sin wxdw con B (w) = 2 xe x senwx dx

0 0

Podemos calcular los coeficientes A (w) y B (w) integrando por partes:

67
Z1 Z 1
x
A (w) = 2 xe cos wx dx = 2 e x (x cos wx)dx =)
0
0
 x 1
xe ( cos wx + wsenwx) e x ((1 w2 ) cos wx 2wsenwx)
A (w) = 2
(1 + w2 ) (1 + w2 )2 0
 2
1 w
A(w) = 2
(1 + w2 )2

Z1 Z 1
x
B (w) = 2 xe sin wx dx = 2 e x (x sin wx)dx =)
0
0
 x 1
xe ( sin wx w cos wx) e x ((1 w2 ) sin wx + 2w cos wx)
B (w) = 2
(1 + w2 ) (1 + w2 )2 0

2w
B(w) = 2
(1 + w2 )2
Construyendo las respectivas integrales de Fourier y aplicando el teo-
rema de la convergencia , puesto que f es una función seccionalmente
suave 8x > 0 ,se tiene que :

Z1 
x 2 1 w2
xe = cos wxdw
⇡ (1 + w2 )2
0
Z1 
x 2 2w
xe = senwxdw
⇡ (1 + w2 )2
0

Por lo tanto, las extensiones son iguales:

Z1  Z1 
1 w2 2w
cos wx dw = sin wx dw
(1 + w2 )2 (1 + w2 )2
0 0

b) En x = 0 se tiene un punto en que estas extensiones son continuas,


luego ambas integrales convergen a f (0) = 0

Z1 Z1 Z1
1 w2 1 w2
dw = 0 =) dw = dw
(1 + w2 )2 (1 + w2 )2 (1 + w2 )2
0 0 0

68
Problema 13
Z1
Si f (x) es una función par ,con integral de Fourier f (x) = ⇡1 A (w) cos(wx)dw,
0
demuestre que:
Z1
dA(w)
a) xf (x) = ⇡1 A⇤ (w) cos(wx)dw, donde A⇤ (w) = dw
0
Z1
d2 A(w)
b) x2 f (x) = ⇡1 A⇤ (w) cos(wx)dw, donde A⇤ (w) = dw2
0
Solución
Z1
1
a) Se tiene que xf (x) = A⇤ (w) sin(wx)dw, es una función impar,

0
Z1

entonces A (w) = 2 v f (v) sin(wv)dv (1).
0
Z1 Z1
1
Como f (x) = ⇡ A (w) cos(wx)dw con A (w) = 2 f (v) cos(wv)dv.
0 0
Z1
dA(w)
Entonces, derivando el coeficiente queda dw
= 2 vf (v) sin(wv)dv (2)
0

Por lo tanto, comparando (1) y (2) se = A⇤ (w) tiene dA(w)


dw
Z1
b) Como x2 f (x) = ⇡1 A⇤ (w) cos(wx)dw, es una función par,
0
Z1
2
entonces A⇤ (w) = ⇡
v 2 f (v) cos(wv)dv (1)
0
Z1 Z1
1
Como, f (x) = ⇡
A (w) cos(wx)dw con A (w) = 2 f (v) cos(wv)dv.
0 0
Z1
dA(w)
Por consiguiente dw
= 2 vf (v) sin(wv)dv =)
0

69
Z1
d2 A(w)
dw2
= 2 v 2 f (v) cos(wv)dv (2)
0
d2 A(w)
Por lo tanto, comparando (1) y (2)se tiene dw2
= A⇤ (w) .

1.10. Ejercicios propuestos


1.- Sea f una función de perı́odo ⇡ dada por

sin 2x si 0  x  ⇡/2
f (x) =
0 si ⇡/2  x  ⇡

a) Obtener la serie de Fourier de f (x).


P
1
1
b) Deducir la convergencia de la serie: 4n2 1
n=1

x (⇡ x) si 0<x<⇡
2.- Sea f una función dada por f (x) =
x (⇡ + x) si ⇡<x<0
a) Represente graficamente la función f usando Maple
b) Obtener la serie de Fourier de f (x).
P
1
( 1)(n 1)
c) Deducir la convergencia de la serie: (2n 1)3
n=1
3.- Un pulso triángular simétrico de altura y ancho ajustables es

a 1 xb si 0  x  b
descrito por: f (x) =
0 si b  x  ⇡
ab 2ab (1 cos nb)
a) Muestre que los coefientes de Fourier son: a0 = 2⇡
, an = ⇡ (nb)2

b) Tome a = 1 y b = 2
calcule y represente las cinco primeras
sumas parciales.

70
4. Sea f una función dada por f (x) = 1 + |x| x 2 [ 1, 1]
a) Obtener la serie de Fourier de f (x).
P
1
1
b) Deducir la convergencia de la serie: (2n 1)2
n=1

5. Encontrar la serie de coseno de Fourier de la función perı́odica de


perı́odo 4, dada por f (x) = e [x] 0  x  2.
P
1
1
b) Deducir la convergencia de la serie: 2n 1
n=1

P
1
1
c) Usando la identidad de Parseval deducir la convergencia de la serie: (2n 1)2
n=1

6. Sea f (x) = x sin x, ⇡x⇡


a) Obtener la serie de Fourier de f.
b) Pruebe que esta serie se puede diferenciar término a término.
c) Use el resultado anterior para obtener el desarrollo de Fourier.
de sin x + x cos x, ⇡  x  ⇡.
P
1
1
b) Deducir la convergencia de la serie (2n 1)6
n=1

0 si ⇡x0
7. Sea f (x) =
x si 0<x⇡
a) Obtener la serie de Fourier de f.
b) Pruebe que esta serie se puede integrar término a término.
c) Use los resultados anteriores para obtener el desarrollo en
Z x
serie trigonométrica para f (u) du

8. a) Establecer que si f (x) = x, ⇡ < x < ⇡ entonces

X1
( 1)n+1
x=2 sin nx
n=1
n

P
1
1 ⇡2
b) Con la identidad de Parseval deducir la convergencia n2
= 6
n=1

c) Muestre que la integración de la serie de Fourier de f (x) = x, ⇡<


x<⇡

71
conduce a.

X1
( 1)n+1 ⇡2
=
n=1
n2 12

d) Sea f (x) una función continua definida en ⇡ < x < ⇡ con serie de
P1
Fourier a0 + (an cos nx + bn sin nx). Si g(x) = f (x ⇡) pruebe que
n=1
la serie
de Fourier de g(x) es

1
X
a0 + (( 1)n an cos nx + ( 1)n bn sin nx)
n=1

e) Aplicando los resultados de a) y d), obtener la serie de Fourier de


perı́odo
2⇡ de la función definida por g(x) = x ⇡ , 0 < x < ⇡.
9. Sea f (x) una función seccionalmente contı́nua, impar de perı́odo 2⇡,
con
P1
serie de Fourier bn sin(nx)
n=1
Rx
a) Verificar que g(x) = f (t)dt, x2R
0

es función par de perı́odo 2⇡


P
1
bn
b) Deducir que n
(1 cos(nx)) es la serie de Fourier de g(x) y
n=1
P
1
bn 1
R⇡ Rx
que n
= ⇡
( f (t)dt)
n=1 0 0

10. Sea f (x) , x 2 R función impar con integral de Fourier


R⇡
Ii = ⇡1 B (w) sin wx dw. Pruebe que la integral de Fourier de
0
R⇡
g (x) = f (x) sin x es: Ip = A (w) cos(wx) dw, con
0
⇢ 1

[B (w + 1) B (w 1)] w>1
A(w) = 1

[B (w + 1) + B (w 1)] 0  w < 1

72

1 x2 si |x|  1
11. Sea f (x) = , obtener la integral de Fourier
0 si |x| > 1
y estudie su convergencia en x0 = 0.

cos x si |x|  ⇡
12. a) Obtener la integral de Fourier de f (x) =
0 si |x| > ⇡
b) Estudiar la convergencia de la IF en x0 = 0 , x1 = ⇡.

Z1
2 w x
13. Establecer la igualdad sin w⇡dw = e si x > 0 y de
⇡ 1 + w2
0
R1 w2
esto deducir el valor al cual converge (1+w2 )2
dw.
0

14. Obtener la Integral de Fourier de f (x) = e |x|


, x 2 R.
R1 cos(wx)
Del resultado, deducir el valor de (1+w2 )
dw.
0
15. Aplicando la la representación de la integral de Fourier
demostrar que:
⇢ ⇡
R1 cos(⇡w/2) cos wx cos x si |x| < ⇡2
a) dw = 2
0 1 w2 0 si |x| > ⇡2
⇢ ⇡
R1 1 cos ⇡w si 0 < x < ⇡
b) sin(wx)dw = 2
0 w 0 si x>⇡
16. Si f (x)es una función par con integral de Fourier
R1
f (x) = A (w) cos(wx)dw, demuestre que:
0

1
R1 w
f (ax) = ⇡a
A a
cos(wx)dw, a > 0
0
17. Pruebe que la integral de Fourier de f puede escribirse como
1 R1 sin(w(t x))
lı́m f (t) dt
w!1 ⇡ 1 t x

73
1.10.1. Respuestas

1) a)
1
1 1 2X 1
f (x) ⇠ + sin 2x 2
cos 4nx
⇡ 2 ⇡ n=1 4n 1


b) Estudie la convergencia en f 2

1
X 1 1
=
n=1
4n2 1 2


x (⇡ x) si 0<x<⇡
2) a) f (x) =
x (⇡ + x) si ⇡<x<0

b)

1
8X 1
f (x) ⇠ sin(2n 1)x
⇡ n=1 (2n 1)3

f (⇡ + ) + f (⇡ )
c) Como f (x) es discontinua en x = ⇡ la serie converge a 2
luego

X1
( 1)(n 1) ⇡3
=
n=1
(2n 1)2 32


3) Si a = 1 y b = 2
el gráfico de la función es
4)
a)

74
1
3 4 X 1
f (x) ⇠ cos(2n 1)⇡x
2 ⇡ n=1 (2n 1)2
2

b) Como f (x) es continua 8x la serie converge a f (0), luego

1
X 1 ⇡2
=
n=1
(2n 1)2 8

c) Aplicando la identidad de Parseval, se tiene:


1
X 1 ⇡4
=
n=1
(2n 1)4 96

5. a)
1
X
1+e 1
2 1 sin( n⇡ ) 2 n⇡
f (x) ⇠ + 1 e cos( x)
2 ⇡ n=1
n 2

b) Como f(x) es discontinua en x0 = 2 converge a los lı́mites


laterales en ese punto, entonces se tiene la convergencia

1
X 1 ⇡
=
n=1
2n 1 4

6. a)
X1
1 ( 1)n+1
x sin x = ⇡ ⇡ cos x + 2⇡ 2
cos (nx) , -⇡  x  ⇡
2 n=1
n 1

0
b) f y f son seccionalmente suave en ⇡  x  ⇡ y f ( ⇡) = f (⇡) ,
luego se satisfacen las condiciones del teorema de diferenciación.
c)Tenemos que

X1
1 ( 1)n
x cos x + senx = ⇡ sin x + 2⇡ n sin (nx) , ⇡x⇡
2 n=1
n2 1

7. a)

75
1 
1 X ( 1)n 1 ( 1)n+1
f (x) ⇠ ⇡+ cos (nx) + sin (nx) , ⇡x⇡
4 n=1
⇡n2 n

b) f es seccionalmente continua en ⇡  x  ⇡ ,entonces se


satisfacen las condiciones del teorema de integracion. Luego,
su serie puede integrarse término a término
Z x ⇢
0 ⇡x0
c) Tenemos que f (u) du = x2
⇡ 2
0<x⇡
Esta función esta representada por la serie obtenida al integrar
la serie de Fourier anterior

1 
1 1 2 X ( 1)n 1 sin (nx) ( 1)n+1 ( cos nx + ( 1)n )
⇡x + ⇡ + +
4 4 n=1
⇡n2 n n n

8)e)

1
X sin(nx)
x ⇡= 2
n=1
n

11)a) Z 
1
1 4 sin w 4 cos w
cos wx dw
⇡ 0 w3 w2

b) La función es continua en x0 = 0 luego la IF converge a f (0)


Z 1
4 sin w 4 cos w
dw = ⇡
0 w3 w2

12)
a) Comof (x) es una función par se tiene que:
Z⇡
2 2 w sin w⇡
A(w) = cos v cos(wv) dv =
⇡ ⇡ 1 w2
0
B (w) = 0

76
Por lo tanto,
Z⇡
2 w sin w⇡
IF = cos wxdw.
⇡ 1 w2
0

b) En x0 = 0 hay un punto de continuidad de f (x), entonces

Z⇡
2 w sin w⇡
dw = f (0) = 1
⇡ 1 w2
0

y x1 = ⇡ es un punto de discontinuidad de f (x), entonces:


Z⇡
2 w sin 2w⇡ f (⇡ + ) + f (⇡ ) 0+1
2
dw = =
⇡ 1 w 2 2
0

13) Considere una extensión impar de f (x), entonces


2w
A (w) = 0 y B (w) = 1+w2

2
R1 w
La Integral de Fourier de f (x)es ⇡ 1+w2
sin wx dw.
0
R1 w2 ⇡
Usando la identidad de Parseval (1+w2 )2
dw = 4
0

2
R1 cos(wx)
14) Como f es par tiene Integral de Fourier f (x) = ⇡ (1+w2 )
dw.
0

Al estudiar la continuidad en x0 = 1 se obtiene la convergencia


R1 cos(w) ⇡
(1+w2 )
dw = 2e
0

1.11. Auto evaluaciones


En el aprendizaje de Cálculo Avanzado como parte de la matemática
se requiere el dominio de dos tipos básicos de conocimientos:
a) el conocimiento conceptual y
b) el conocimiento procedimental.
El primero está vinculado al razonamiento y reflexión, se caracteriza
por ser un conocimiento teórico, producido por la actividad cognitiva,

77
que permite establecer relaciones entre sus componentes; su carácter es
declarativo y se asocia con el saber qué y el saber por qué.
El segundo está vinculado a la acción y se relaciona con métodos y las
estrategias para representar conceptos y transformar estas representa-
ciones; usando habilidades y destrezas para analizar comprensivamente,
elaborar, comparar, resolver algoritmos, evaluar y argumentar. De este
modo el conocimiento procedimental permite depurar y refinar la con-
strucción del conocimiento conceptual, como asimismo, usar de manera
eficaz y flexible, en un contexto aplicado, los conceptos, proposiciones,
teorı́as y modelos matemáticos. En consecuencia, está asociado con el
saber hacer.

Evaluación formativa
Al término de cada unidad , se incluyen una serie de evaluaciones for-
mativas que deben realizar los propios alumnos en situaciones de auto-
evaluación de acuerdo a la dinámica de trabajo individual, por equipos
o grupal si se utiliza en el trabajo en el aula o en trabajo personal
realizado en casa.
Por consiguiente, esta modalidad de evaluación tiene por función gener-
ar evidencias de los resultados de aprendizaje logrados por los propios
estudiantes para retroalimentar su avances de aprendizaje. Asimismo,
debe proporcionar información a los estudiantes sobre sus aciertos, difi-
cultades, falencias y omisiones en el proceso de aprendizaje del Cálcu-
lo Avanzado de modo de introducir cambios y ajustes en los méto-
dos de estudios. De este modo, las autoevaluaciones abarcan tanto los
conocimientos conceptuales como procedimentalales luego son una in-
stancia para que los estudiantes perfeccionen su capacidad para: re-
conocer los conceptos, principios, reglas y propiedades que se sustenta
el Cálculo Avanzado; identificar y aplicar métodos matemáticos en la
resolución de problemas; analizar y evaluar información matemática
proveniente de otras ciencias ; y analizar y evaluar las soluciones de un
problema para fundamentar su pertinencia.

Estrategia de aplicación
Como se ha señalado el objetivo fundamental de estos instrumentos
de autoevaluación es que el alumno verifique, constate y descubra las
falencias que aún persisten en el ambito cognitivo y procedimental de

78
su aprendizaje del tema objeto del estudio, ya sea que este sea el total
o una parcialidad de un tema. Se pretende que con esta constatación el
alumno se motive para persistir en su estudio, si aún no ha logrado el
nivel necesario que asegure su éxito en las evaluaciones regulares que se
harán del tema. Para que este ejercicio tenga el éxito propuesto, se debe
trabajar el tiempo estipulado, y con las condiciones medioambientales
necesarias de privacidad y silencio , que aseguren la efectividad de la
actividad, o que reproduzcan condiciones requeridas de una prueba o
examen..

Series e Integrales de Fourier

Autoevaluación No1
El estudiante:

1) Representará una función f (x) definida en el intervalo [0, L] por una


serie de Fourier coseno o seno, aplicando extensiones pares o impares
según sea el caso, y evaluar si la serie converge o no en un punto x0
dado del dominio.
2) Obtendrá la suma de ciertas series infinitas empleando el teorema
de la convergencia de las series de Fourier.
3) Verificará si una serie de Fourier se puede diferenciar término a térmi-
no y aplicar el teorema de la diferenciación para obtener la derivada de
la serie de fourier

4) Representará una función f (x) no perı́odica por una Integral de


Fourier y emplear el teorema de la convergencia para probar si la Inte-
gral Fourier converge o no en un punto x0 del dominio.
5) Probará el teorema de Parseval para una función f que se representa
por una integral de Fourier y deducirá la convergencia de ciertas sumas
de series.
6) Representará una función periódica f(x) de perı́odo 2L por una serie
de Fourier, y aplicará el teorema de la convergencia para determinar si
la serie converge o no en un punto x0 del dominio.
Tiempo : 1 hora 20 minutos
Problema 1

79
a) Desarrollar en serie de Fourier de cosenos la función
⇣x⌘
f (x) = sin
2

definida en el intervalo [0, ⇡].


b) Deducir la convergencia de la serie
1
X 1 1
=
n=1
4n2 1 2

b) Pruebe que la serie de f se puede diferenciar término a término


⇣x⌘ y
utilice este hecho para obtener el desarrollo de Fourier de cos .
2
Problema 2
R1
a) Probar que si f (x)R= 0 (A (w) cos (wx)+B (w) sin (wx))dw,
R1 con co-
1
eficientes A (w) = ⇡1 1 f (u) cos (wu) du y B (w) = ⇡1 1 f (u) sin (wu) du.
R1 R1
Entonces ⇡1 1 [f (x)]2 dx = 0 (A2 (w) + B 2 (w))dw

⇡, |x|  a R1 2
b) Si f (x) = ,pruebe que 0 sinw2wa dw = a⇡
0, |x| > a 2

Problema 3

0 si 5<x<0
Sea f (x) = tal que f (x + 10) = f (x)
3 si 0<x<5
a) Obtener la serie de Fourier de f (x)
b) Defina f (x) en los puntos 5, 0 y 5 de manera que la serie de Fourier
obtenida anteriormente converja a f (x) 8x con |x|  5.

Pauta de Corrección
Problema 1
Para obtener el desarrollo en serie de Fourier de cosenos de la función
dada, construimos una extensión par de la función f tal que
8 ⇣ ⌘
< sin x , 0x⇡
fp (x) = ⇣2x ⌘
: sin , ⇡x<0
2
Como esta función es par, de perı́odo 2⇡ ,tenemos que

80
bn = 0
1
R⇡ 1
R⇡ x 1
⇥ x
⇤⇡
a0 = ⇡
f (x)dx = ⇡
sin 2
dx = ⇡
2 cos 2 0
0 0
2
⇥ ⇤
a0 = ⇡
1 cos ⇡2 = 2

2
R⇡ 2
R⇡
an = ⇡
f (x) cos(nx)dx = ⇡
sin x2 cos(nx)dx
0 0

1
R⇡ ⇥ 1 1

an = ⇡
sin 2
+n x sin 2
n x dx
0
" #⇡
1 1
1
cos +n x
2
cos +n x
2
an = ⇡ 1 1
2
+n 2
+n
0

1 2 2 4
an = ⇡
+ =
1 + 2n 1 2n ⇡ (1 4n2 )
Por lo tanto, la serie de fp en [0, ⇡] ,es:

1
2 1X 4
+ cos (nx)
⇡ ⇡ n=1 (1 4n2 )

La función fp seccionalmente continua, de perı́odo 2⇡, aplicando el


teorema de convergencia de Fourier , tenemos que:

1
2 1X 4
f (x) = + cos (nx) , 8x 2 (0, 2⇡)
⇡ ⇡ n=1 (1 4n2 )

En particular para x = 0,

1
2 1X 4
+ = 0
⇡ ⇡ n=1 (1 4n2 )
1
X 4 1
=
n=1
(4n2 1) 2

x
c) Sea f (x) = sin 2
, para 0  x  ⇡.
x
i) Como f (x) = sin 2
, es continua en (0, ⇡) .

81
ii) f 0 (x) = 12 cos x
2
, es continua en (0, ⇡) .
00 1 1
iii) Existe f (x) = 4
sin 2
, y también es continua en (0, ⇡) .
Además f (0) = (sin (0)) = (sin (⇡)) = sin 2⇡ = f (⇡)
Luego, se satisfacen las hipótesis del teorema de diferenciación de la
serie de Fourier, entonces para ⇡ < x < ⇡,derivando la serie de Fourier
de f , obtenemos:

⇣x⌘ 2
1
1X 4
sin = + cos (nx)
2 ⇡ ⇡ n=1 (1 4n2 )
1 ⇣x⌘ 1X
1
4n
cos = 0 sin (nx)
2 2 ⇡ n=1 (1 4n2 )
⇣x⌘ 1X
1
8n
cos = sin (nx)
2 ⇡ n=1 (4n2 1)

Problema 2
a) Mutiplicando la integral de Fourier por f y realizando el producto
interno en ( 1, 1) , se tiene

Z 1 Z 1 Z 1
2
[f (x)] dx = [(A (w) f (x) cos (wx) + B (w) f (x) sin (wx))dw]dx
1 0 1
Z 1 Z 1 Z 1 Z 1
2
[f (x)] dx = (A (w) f (x) cos (wx) dx + B (w) f (x) sin (wx) dx)dw
1 0 1 1

Z 1 Z 1
2
[f (x)] dx = ⇡ (A2 (w) + B 2 (w))dw =)
1
Z 1 Z 10
1
[f (x)]2 dx = (A2 (w) + B 2 (w))dw
⇡ 1 0


⇡, |x|  a
b) Como f (x) = , es función par, entonces la integral
0, |x| > a
de Fourier de f es
Z 1
A (w) cos (wx) dw
0

82
con coeficiente
Z 1 Z a
2 2
A (w) = f (u) cos (wu) du = ⇡ cos (wu) du
⇡ ⇡
0 a
0
sin wu sin wa
= 2 =2
w 0 w
2
4 sin wa
) A2 (w) =
w2

Por otra parte:


Z 1 Z a
1 2 1
[f (x)] dx = ⇡ 2 dx = 2a⇡
⇡ 1 ⇡ a

Como la función f es par, se tiene


Z Z 1
1 1 2
[f (x)] dx = A2 (w) dw
⇡ 1 0

Entonces, al reemplazar los términos


Z 1
sin2 wa
2a⇡ = 4 dw
0 w2
Z 1
sin2 wa a⇡
) 2
dw =
0 w 2

Problema 3
La serie de Fourier buscada es de la forma:
P1 n⇡x n⇡x
f (x) ⇠ a0 + (an cos + bn sin )
n=1 5 5
con coeficientes
 5
1 R5 3x 3
a0 = 0
3dx = =
10 10 0 2
h
1 0 R n⇡x R5 n⇡x i
an = 5
0 cos dx + 0
3 cos dx
5 5 5
 5
3 5 n⇡x
an = sin =0
5 n⇡ 5 0
1 hR 0 n⇡x R5 n⇡x i
bn = 5
0 sin dx + 0 3 sin dx
5 5 5
83
 5
3 5 n⇡x
bn = cos
5 n⇡ 5 0
3
bn = [1 cos n⇡]
n⇡
Sustituyendo los coeficientes en la serie obtenemos
3 3P 1 [1 cos n⇡] n⇡x
+ sin( )
2 ⇡ n=1 n 5
b) Como f (x) es una función seccionalmente continua entonces en los
f (x+
o ) + f (xo )
puntos de discotinuidad converge a . Luego en los pun-
2
3
tos 5, 0 y 5 converge a .
2
Por tanto la función debe asumir estos valores en los puntos menciona-
dos para que se cumpla la covergencia propuesta.

Autoevaluación No 2

Tiempo : 2 horas
Problema 1
Dada la función f (x) = x [2x] , 0 < x < 1
a) Obtener la serie de Fourier de cosenos.
1
b) Establecer la convergencia de la serie en x0 = 0 y x1 = .
2

Problema 2
Deducir la igualdad de la integral de Fourier
Z ✓ ◆
2 ⇡ cos ⇡w sin ⇡w
x= + sin wxdw
⇡ w w2

para 0 < x < ⇡.


Problema 3
Sea f (x) continua por tramos, impar de perı́odo 2⇡ con serie de Fourier
P1
bn sin (nx) .
n=1
Rx
a) Verificar que g (x) = 0 f (t) dt , x 2 R es función par de perı́odo
2⇡.

84
P1 b
n P1 b
n
b) Deducir cos (nx) es la serie de Fourier de g (x) y que
n=1 n n=1 n

P1 b
n 1 R⇡ Rx
= f (t) dt dx
n=1 n ⇡ 0 0

Pauta de Corrección
Problema 1
a) Consideremos una extensión par de f (x) de perı́odo P = 2, es decir
f (x + 2) = f (x)
1
Como f (x) = x [2x] , 0 < x < 1, se tiene que 0 < x < , [2x] = 0 y
2
1
< x < 1, [2x] = 1.
2 8
>
< x 1
si 0 < x <
Ası́ f (x) = 2 , luego los coeficientes de Fourier
> 1
: x 1 si <x<1
2
son
 2 1/2  1
R 1/2 R1 x (x 1)2
a0 = 0 xdx + 1/2 (x 1) dx = + =0
2 0 2 1/2
R 1/2 R1
an = 2 0 x cos n⇡xdx + 2 1/2 (x 1) cos n⇡xdx
 1/2  1  1
sin n⇡x cos n⇡x sin n⇡x cos n⇡x sin n⇡x
an = 2 x + 2 +2 x + 2 2
n⇡ (n⇡) 0 n⇡ (n⇡) 1/2 n⇡ 1/2
n⇡
2 2 sin 2
an = [cos n⇡ 1] +
(n⇡)2 n⇡
2 n 2 ( 1)n 1
an = 2 [( 1) 1] +
(n⇡) (2n 1)⇡
bn = 0
De este modo la serie de Fourier queda
1
" #
X 2 n 2 ( 1)n 1
[( 1) 1] + cos n⇡x
n=1
(n⇡)2 (2n 1)⇡

b) En x0 = 0 hay un punto de continuidad de f, entonces la serie


converge a

85
" #
P
1 2 n 2 ( 1)n 1
f (0) = 0 = 2 [( 1) 1] +
n=1 (n⇡) (2n 1)⇡
1
Para x1 = se tiene un punto de discontinuidad de f , entonces la serie
2
converge a
⇣ ⌘
+
f 12 + f 12 1
+ 1
= 2 2
= 0.
2 2

Problema 2

x si ⇡ < |x| < ⇡
Consideremos la función f (x) = , la cual es
0 si R |x| > ⇡
1
impar, entonces la integral de Fourier de f es 0 B (w) sin (wx) dx, con
coeficiente:
2 R1 2 R⇡
B (w) = 0
f (x) sin (wx) dx = x sin (wx) dx
⇡ ⇡ 0
 ⇡
2 cos wx sin (wx)
B (w) = x +
⇡ w w2 0
✓ ◆
2 cos w⇡ sin (w⇡)
B (w) = ⇡ +
⇡ w w2
Por tanto la integral de Fourier , queda
✓ ◆
2 R1 cos w⇡ sin (w⇡)
⇡ + sin (wx) dx
⇡ 0 w w2
Como f es continua en (0, ⇡) , la integral de Fourier converge a
✓ ◆
2 R1 cos w⇡ sin (w⇡)
f (x) = x = ⇡ + sin wxdw
⇡ 0 w w2
Problema 3
a) En primer lugar, verifiquemos que g (x) es una función par. En efecto
sea

R x Rx
g ( x) = 0
f (t) dt = 0
f ( u) ( du), si cambiamos la variable u =
t
Pero la función del integrando f (u) es una impar es decir f ( u) =
f (u) 8u
Rx
entonces g ( x) = 0 f (u) (du) = g (x)

86
En segundo lugar, estudiemos el perı́odo de la función
Z x+2⇡
g (x + 2⇡) = f (t) dt
0
Z x Z x+2⇡
= f (t) dt + f (t) dt
0 x
Z x Z ⇡
= f (t) dt + f (t) dt
0 ⇡
Z x
= f (t) dt = g (x)
0

Por tanto g (x) tiene perı́odo 2⇡

b) Integrando la serie de Fourier de f , se tiene

Z x Z 1
xX
g (x) = f (t) dt = bn sin (nx) dt
0 0
" 1
#n=1
x
X cos (nx)
= bn
n=1
n
0
1
X 1
X
bn bn
= cos (nx)
n=1
n n=1
n

Como g (x) es par su serie de Fourier es de la forma


P
1
g(x) = a0 + an cos nx donde ⇡  x  ⇡ y coeficientes
n=1

Z ⇡ Z
1 1 ⇡
a0 = g (x) dx = g (x) dx
2⇡ ⇡ ⇡ 0
Z ✓Z x ◆ X1
1 ⇡ bn
= f (t) dt dx =
⇡ 0 0 n=1
n

Autovaluación No 3

Tiempo : 1 hora 20 minutos

87
Problema 1
Sea f (x) = cos bx, ⇡  x  ⇡, con f (x + 2⇡) = f (x) , b constante
no entera.
a) Obtener la serie de Fourier de f.
b) A partir de la convergencia de la serie, deducir la igualdad
✓ ◆
⇡ 1 1 1
= 2b + ⌥ ...
senb⇡ 2b2 b2 12 b2 22

Problema 2
8 ✓ ◆
< |x|
1 si |x| < a
Sea la función f (x) = a
:
0 si |x| > a
a) Obtener la integral de Fourier de f.
2 R 1 1 cos w
b) Deducir que 1 x = cos wxdw, si 0  x  1; y
⇡ 0 w2
R 1 1 cos w
0
cos wx dw = 0, si x > 1.
w2

Problema 3
a) Utilice la integral de Fourier para obtener la convergencia
Z 1
sin w ⇡
dw =
0 w 2
.
R 1 sin wx
b) Probar que la función definida por g (x) = x
0
dw, x > 0
w

g (0) = es continua en cero.
2

Pauta de Autocorrección
Problema 1
a) Como f (x) = cos bx, en ⇡  x  ⇡ es función par, entonces la
serie de Fourier de f es cosenoidal,

P
1
es decir f (x) ⇠ a0 + an cos nx con coeficientes
n=1

88
 ⇡
1 R⇡ 1 sin b⇡
a0 = 0
cos bxdx = sin b⇡ =
⇡ b⇡ 0 b⇡
 ⇡
2 R⇡ 2 sin (b n) x sin (b + n) x
an = 0
cos bx cos nxdx = +
⇡ ⇡ 2(b n) 2 (b + n) 0

2 sin (b n) ⇡ sin (b + n) ⇡
an = +
⇡ 2(b n) 2 (b + n)
2b ( 1)n sin b⇡
an =
⇡(b2 n2 )
Por tanto la serie de fourier queda

sin b⇡ 2b sin b⇡ P
1 ( 1)n
f (x) ⇠ + 2
cos nx
b⇡ ⇡ n=1 (b n2 )
b) En x0 = 0 hay un punto de continuidad de f entonces por el teorema
de convergencia de la serie, esta converge a
sin b⇡ 2b sin b⇡ P 1 ( 1)n
f (0) = 1 = + 2
b⇡ ⇡ n=1 (b n2 )
Entonces
⇡ 1 P
1 ( 1)n
= + 2b 2
()
sin b⇡ b n=1 (b n2 )
✓ ◆
⇡ 1 P1 ( 1)n
= 2b + ()
sin b⇡ 2b2 n=1 (b2 n2 )
✓ ◆
⇡ 1 1 1
= 2b + ⌥ ...
sin b⇡ 2b2 b2 12 b2 22

Problema 2
a) Como:
✓ ◆ ✓ ◆
| x| |x|
f ( x) = 1 = 1 = f (x) 8x 2 [ a, a]
a a
la función f es par, entonces la integral de Fourier de f es
R1
0
A (w) cos (wx) dx, con coeficiente:
2 hR a ⇣ x⌘ R1 i
A (w) = 1 cos (wx) dx + 0 cos (wx) dx
⇡ 0 a a

2 Ra ⇣ x ⌘
A (w) = 0
1 cos (wx) dx
⇡ a
✓ ◆
2 1 cos aw
A (w) =
⇡ aw2

89
Por tanto la integral de Fourier queda
Z ✓ ◆
2 1 1 cos aw
cos (wx) dx,
⇡ 0 aw2

b) Como la función f es continua para 8x 2 [0, 1],aplicando el teorema


de convergencia con a = 1, se deduce que la integral de Fourier converge
a
2 R 1 1 cos w
f (x) = 1 x = cos wxdw, y del mismo modo
⇡ 0 w2
2 R 1 1 cos w
f (x) = 0 = cos wxdw, 8x > 1
⇡ 0 w2

Problema 3

1 si |x|  1
a)Sea la función f (x) = , como la función f es par,
0 si |x| > 1
entonces la integral de Fourier de f es
R1
0
A (w) cos (wx) dx, con coeficiente:
2 hR 1 R1 i
A (w) = 1 cos (wx) dx + 0 cos (wx) dx
⇡ 0 1

2 R1
A (w) = cos (wx) dx
⇡ 0
 1
2 sin wx 2 sin w
A (w) = =
⇡ w 0 ⇡ w
Por tanto, la integral de Fourier queda
Z ✓ ◆
2 1 sin w
f (x) = cos (wx) dx
⇡ 0 w

Como la función es continua en x = 0,entonces la integral converge a

Z 1
2 sin w
f (0) = 1 = dx =)
⇡ 0 w
Z 1
sin w ⇡
dx =
0 w 2

b) Sea la integral

Z 1
x sin wx ⇡
g (x) = dw, x > 0 g (0) =
0 w 2

90
w
aplicando el cambio de variables u = x
=) xdu = dw, se tiene

Z 1 Z 1
x2 sin u x2 sin u
g (x) = xdu = du
0 xu 0 u
Calculemos ahora el lı́mite de ésta función cuando x ! 0

Z 1
sin u x2
limg (x) = lı́m du
x!0 x!0 0 u
Z 1
sin u ⇡
= du = = g (0)
0 u 2

Lo que implica que g(x) es continua en cero.

91
Capı́tulo 2

Funciones Vectoriales de una


variable real

2.1. Introducción
!
La recta de R3 que pasa por el punto P 0 = (x0 , y0n, z0 ) y es paralela aoun
!
vector !a = (a1, a2, a3 ) se define como el conjunto P 0 + t! a | t 2 R .En
esta definición de recta a cada número real t corresponde el punto
!
P 0 + t!a de R3 es decir a cada valor t de R le asocia el punto
(x0 + ta1, y0 + ta2, z0 + ta3 ) de R3 . Tal correspondencia o asociación
genera lo que llamaremos una función vectorial de una variable real
!
que en este caso es de R en R3 . Si denotamos por f a tal función
entonces su regla de correspondencia es
!
f (t) = (x0 + ta1, y0 + ta2, z0 + ta3 )

!
El dominio de f es el conjunto de todos los números reales y el rango
! !
de f es la recta que pasa por el punto P o y es paralela al vector
!a . Este es un ejemplo del tipo de funciones que estudiaremos en este
módulo; para tales funciones consideraremos los conceptos de lı́mite,
continuidad, derivada e integral. Desde el punto de vista conceptual no
hallaremos ideas nuevas y en la mayor parte de los casos las técnicas
usadas son las mismas desarrolladas en el cálculo de funciones real de
una variable real.

92
2.2. Funciones Vectoriales

Definición 2.2.1. Una función vectorial de una variable real es una


función cuyo dominio es un conjunto de números reales y el rango es
un conjunto de vectores o puntos de Rn
! !
Notación f : D ✓ R ! Rn tal que 8 t 2 D, f (t) = (f1 (t), f2 (t), . . . , fn (t)),
donde fk : D ✓ R ! R para cada k = 1, 2, .., n es una función real de
!
variable real. Cada fk es la k-ésima componente del vector f (t).
!
Si la función f describe el movimiento de una partı́cula, el vector
!
f (t) = (f1 (t), f2 (t), . . . , fn (t)) señala la posición en el instante t, es
decir en estos casos t representa la variable tiempo.
Ejemplo 1
! !
Sea f : I ✓ R ! R3 tal que f (t) = (cos t, sin t, t), I = [0, 2⇡]
!
Hacer un esquema del rango de f
Solución:
!
Pongamos f (t) = (x(t), y(t), z(t)) donde x = cos t, y = sin t, z = t.
En este caso para cualquier valor de t se cumple x2 + y 2 = 1 que
!
es la proyección en el plano XY de cualquier punto f (t) de la curva
que está sobre el manto de un cilindro de radio unitario x2 + y 2 = 1,
!
y z = t señala la distancia de f (t) al plano XY.
!
El rango de f es entonces una curva que partiendo de (1, 0, 0)
describe un arco completo de una helicoidal en el manto del cilindro
x2 + y 2 = 1 de R3 .

93
Ejemplo 2
! ! !
Sea f : I ✓ R ! R3 tal que f (t) = (t, t, t), describa el rango de f .
Solución:
!
Las imagenes f (t) = (t, t, t) las podemos escribir vectorialmente de
!
la forma f (t) = (0, 0, 0) + t(1, 1, 1) lo que nos permite reconocer que
se trata de una recta que pasa por el origen (0, 0, 0) en la dirección del
vector !v = (1, 1, 1).

Ejemplo 3
! !
Sea f : I ✓ R ! R3 tal que f = (t, t, 2t2 ), I = [ 3, 3] describa el
!
rango de f.
Solución:
!
Ponemos f (t) = t(1, 1, 0) + t2 (0, 0, 2), de esta expresión se puede afir-
!
mar que f (t) es la suma de un vector a lo largo de la recta y = x
en el plano XY y un vector perpendicular al plano XY. Quiere decir
!
entonces que el rango de f se encuentra en el plano que contiene los
vectores (1, 1, 0), (0, 0, 2) perpendicular al plano XY.
Si se considera
p en un punto
p (t, t, 0) en el plano XY y u distancia al
origen, u = t + t = 2t, resulta que z = 2t2 = u2 . Por lo tanto, el
2 2
!
rango de f es una porción de la parábola z = u2 que esta en el plano
y = x perpendicular al plano XY y que contiene al eje z.

94
2.3. Lı́mite de una función vectorial.

Previamente aclaremos o recordemos algunos conceptos en cuanto a la


métrica que usaremos.

Definición 2.3.1. (distancia)


!
Si !
a y b son una par de elementos (puntos) de Rn dados por
! !
a = (a1 , a2 , a3 , . . . , an ), b = (b1 , b2 , b3 , . . . , bn ) la distancia desde !
a
! ! !
hasta b es b a se define por

" n
# 12
! X
b !
a = (bi ai ) 2
i=1

que en R2 y R3 viene a corresponder a lo que definimos como distancias


entre dos puntos.
!
Definición 2.3.2. Se dice que el vector l = (l1 , l2 , l3 , . . . , ln ) es el
!
lı́mite de la función vectorial f : I ✓ R ! Rn en t0 2 I , si para cada
✏ > 0 existe un número > 0 tal que siempre que t esta en el dominio
! ! !
de f y 0 < |t t0 | < entonces f (t) l <✏

2.3.1. Teorema del lı́mite


!
Teorema 2.3.1. Sea f : I ✓ R ! Rn función vectorial. Entonces

95
! !
lı́m f (t) = l = (l1 , l2 , l3 , . . . , ln ) () lı́m fk (t) = lk , k = 1, 2, ..., n
t!t0 t!t0

!
donde f (t) = (f1 (t), f2 (t), . . . , fn (t))

Demostración.-
i) (() Suponemos que lı́m fk (t) = lk , k = 1, 2, ..., n
t!t0

Sea ✏ > 0 dado

! !
f (t) l = k(f1 (t) l1 , f2 (t) l2 , ..., fn (t) ln )k
= k(f1 (t) l1 , 0, 0, ..., 0) + (0, f2 (t) l2 , 0, ..., 0) + (0, 0, 0, ..., fn (t) ln )k
 k(f1 (t) l, 0, ..., 0)k + k(0, f2 (t) l2 , 0, ..,0)k + ... + k(0, ..., 0, fn (t) ln )k
 |f1 (t) l| + |f2 (t) l2 | + ... + |fn (t) ln |


Si lı́m fk (t) = lk =)Si n
> 0 existe k > 0 tal que 0 < |t t0 | <
t!t0
k =) |fk (t) lk | < n✏ , k = 1, 2, ..., n
tomando = mı́n { k } , k = 1, 2, ..., n se tiene que 0 < |t t0 | < =)
|fk (t) lk | < n✏ para todo k = 1, 2, .., n.

De la desigualdad anterior, mayorando por .
n
X✏ n
! ! ✏ ✏ ✏
f (t) l < + ... + = = n=✏
n n i=1
n n

8 t 2 (t0 , t0 + ), lo que prueba esta parte del teorema.


! !
ii) ()) Suponemos que lı́m f (t) = l
t!t0

Sea ✏ > 0 dado

! ! !
lı́m f (t) = l = (l1 , l2 , l3 , . . . , ln ) y f (t) = (f1 (t), f2 (t), . . . , fn (t))
t!t0

implica que existe > 0 tal que

96
! ! !
f (t) l < ✏ todo t en el dominio de f tal que 0 < |t t0 | <

pero |fk (t) lk | < kf (t) lk todo k, entonces |fk (t) lk | < ✏ todo t
!
en el dominio de f tal que 0 < |t t0 | < .
Por lo tanto
lı́m fk (t) = lk todo k = 1, 2, ..., n.
t!t0

Se ha establecido entonces que:


! !
lim f (t) = l = (l1 , l2 , l3 , ...., ln ) , lim fk (t) = lk , k = 1, 2, 3, ...., n.
t!to t!to

Ejemplo 4

! !
Si f (t) = (cos t, sin t) , calcule lı́m⇡ f (t)
t! 2

Solución:
✓ ◆
!
lı́m⇡ f (t) = lı́m⇡ cos t, lı́m⇡ sin t = (0, 1)
t! 2 t! 2 t! 2

Ejemplo 5

La trayectoria de una partı́cula en el espacio R3 está dada por la


función vectorial !
c (t) = (cos 2t, sin t, 2t

). Calcule: lim⇡ !
c (t), lim!c (t)
t! 2 t!⇡
y lim ! c (t) .

t! 2

Solución.-
lı́m (cos 2t, sin t, 2t

) = (lı́m⇡ cos 2t, lı́m⇡ sin t, lı́m⇡ 2t

) =( 1, 1, 1)
t! ⇡2 t! 2 t! 2 t! 2

lı́mt!⇡ (cos 2t, sin t, 2t



) = (lı́m cos 2t, lı́m sin t, lı́m 2t

) = (1, 0, 2)
t!⇡ t!⇡ t!⇡

lı́m⇡ (cos 2t, sin t, 2t



) = ( lı́m⇡ cos 2t, lı́m⇡ sin t, lı́m⇡ 2t

) = ( 1, 1, 1)
t! 2
t! 2
t! 2
t! 2

97
2.3.2. Operaciones con funciones vectoriales
!
Definición 2.3.3. Sean f , !g : I ✓ R ! Rn funciones vectoriales,
! ! ! ! ! ! !
entonces, para cada t 2 I se define f + !
g, f g, f · g, f ⇥ g

de la forma siguiente:
! !
a) ( f + ! g )(t) = f (t) + ! g (t) = ((f1 + g1 )(t), (f2 + g2 )(t), ..., (fn + gn )(t))
! ! !
b) ( f g )(t) = f (t) ! g (t) = ((f1 g1 )(t), (f2 g2 )(t), ..., (fn gn )(t))
! ! !
c) ( f · g )(t) = f (t) · ! g (t) = f1 (t)g1 (t) + f2 (t)g2 (t) + ... + fn (t)gn (t)
Xn
= fi (t) · gi (t)
i=1

! !
d) (f ⇥!
g )(t) = f (t) ⇥ ! g (t)
= ((f2 g3 )(t) (f3 g2 )(t)), (f3 g1 )(t) (f1 g3 )(t), (f1 g2 )(t) (f2 g1 )(t)).

en este caso para n = 3.


Definición 2.3.4. Producto por escalar
!
Si : I ! R y f : I ! Rn es función vectorial, definimos:
! ! !
f : I ! Rn tal que ( f )(t) = (t) f (t) = ( (t)f1 (t), (t)f2 (t), ..., (t)fn (t))

Estas definiciones nos llevan al siguiente teorema

2.3.3. Teoremas del algebra de lı́mites

!
Teorema 2.3.2. Sean f , ! g : I ✓ R ! Rn funciones vectoriales,
! ! !
to 2 I , si lı́m f (t) = !
a y lı́m ! g (t) = b ; ! a , b 2 Rn ,entonces:
t!to t!to
h! i ! !
a) lı́m f + ! g (t) = lim f (t) + lı́m ! g (t) = !
a + b
t!to t!to t!to
h! i ! !
b) lı́m f !
g (t) = lı́m f (t) lı́m ! g (t) = !
a b
t!to t!to t!to
h! i ! !
c) lı́m f · g (t) = lı́m f (t) · lı́m !
! g (t) = !a · b
t!to t!to t!to
h! i ! !
d) lı́m f ⇥ ! g (t) = lı́m f (t) ⇥ lı́m ! g (t) = !
a ⇥ b , con n = 3 en
t!to t!to t!to
este caso.

98
Demostración:
Es consecuencia directa de aplicación del Teorema del Lı́mite y la
definición
de las operaciones. Se deja como ejercicio al lector.

2.3.4. Teorema: producto de función escalar por


vectorial
!
Teorema 2.3.3. Sean : I ✓ R ! R y f : I ✓ R ! Rn , to 2 I tal
que:
!
lı́m (t) = ↵ ; y lı́m f (t) = !
a ,↵ 2 R y !a 2 Rn , entonces:
t!to t!to
h !i !
lim f (t) = lim (t)lim f (t) = ↵! a
t!to t!to t!to

Demostración: Se deja como ejercicio al lector.

2.4. Continuidad
! !
Definición 2.4.1. Sea f : I ! Rn y to 2 I , diremos que f es con-
tinua en to si para cada " > 0,existe > 0 tal que 8 t 2 I : |t to | <
! !
,entonces f (t) f (to ) < "

! !
Note que: si f = (f1 , f2 , f3 , ..., fn ) tal que f : I ! Rn y fi : I ! R,
i = 1, 2, 3, ..., n.
!
Entonces f es continua en to , ssi, fi es contı́nua en to para todo
i = 1, 2, 3, ..., n.
n 1
P 2
2
En efecto, vea que |fi (t) fi (to )|  ((fi (t) fi (to ))
i=1
!
=) si f es continua, entonces fi es continua. Reciprocamente co-
! ! Pn
mo f (t) f (to )  |fi (t) fi (to | podemos inferir que si fi es
i=1
!
continua, i = 1, 2, 3, ..., n,entonces f es continua.
Por lo tanto, decimos que una función vectorial es continua ,si y sólo
si, lo son cada una de sus funciones componentes

99
Observación: Los teoremas de continuidad que obviaremos en este
caso y que son una réplica de los teoremas de lı́mites se pueden probar
fácilmente.

2.5. La Derivada
!
Definición 2.5.1. Sea f : I ✓ R ! Rn función vectorial y to 2 I. Se
! ! !
define la derivada de f en to , denotada ddtf (t0 ) o f´(to ) por el lı́mite:

! ! !
f´(to ) = lı́m f (to +h)h f (to )
, cuando este lı́mite existe.
h!0

Geométricamente ! c 0 (to ) es un vector tangente a la curva C descrita


por la trayectoria !
c (t) en el punto ! c (t0 ) orientado.

De la definición del lı́mite se deduce que:


! !
Teorema 2.5.1. Sea f : I ✓ R ! Rn , to 2 I y f = (f1 , f2 , f3 , ..., fn )
!
entonces f es derivable en to si y solo si cada fi es derivable en to .

Demostración
! !
! f (to + h) f (to )
f ‘(to ) = lı́m
h!0 h

100
1
= lim [f1 (to + h) f1 (to ), f2 (to + h) f2 (to ), ..., fn (to + h) fn (to )]
h!0 h
h i
f1 (to +h) f1 (to ) f2 (to +h) f2 (to ) fn (to +h) fn (to )
= lim h
, lı́m h
, ..., lı́m h
h!0 h!0 h!0
⇥ 0 ⇤
= f1 (to ), f20 (to ), ..., fn0 (to )

!
Por lo tanto f 0 (to ) existe si y solo si fk0 (to ) existe para k = 1, 2, 3, ..., n
Definición Si suponemos que ! c (t) describe una trayectoria seguida por

una partı́cula podemos definir al vector !c´(t) como el vector velocidad


! !
en la trayectoria c en el punto c (t). Asimismo , definiremos la rapidez
en ese punto como ! c ´(t)

Ejemplo 6
Sea !c (t) = (r cos(t), r sin(t)), trayectoria cuyo camino corresponde a
una circunferencia de radio r. Muestre que ! c (t) y !
c 0 (t) son ortogonales

Muestre que !
c (t) y !
c 0 (t) son ortogonales

Solución.
Tenemos ! c (t) es derivable lo que implica ! c 0 (t) = ( r sin t, r cos t)
Por consiguiente
!
c (t) · !
c 0 (t) = (r cos(t), r sin(t)) · ( r sin t, r cos t)

= r (sin t) r cos (t) r (cos t) r sin (t) = 0

)!
c (t) y !
c 0 (t) son ortogonales 8 t.

101
2.6. Regularidad de una curva
Una diferencia significativa de la derivada de funciones vectoriales re-
specto de la suavidad o regularidad de la curva en el aspecto geométrico
es que en este caso la derivabilidad no detecta picos en la curva. Para
describir este hecho consideramos el siguiente ejemplo:

Ejemplo 7
! !
Sea f : R ! R2 definida por f (t) = (t3 , t2 |t|). Su gráfico como
muestra la figura es similar al gráfico de y = |x| en R2 y sabemos que
la función f (x) = |x| no es derivable en x = 0.

Muestre que la función dada es derivable para t = 0.


Solución
!
f (t) = (t3 , t2 |t|) ) x(t) = t3 , y(t) = t2 |t|

x(t) = t3 ) x0(t) = 3t2 , 8 t,


y(t) = t2 |t| ) y0(t) = 3t2 , t > 0 3t2 , t < 0

y(h) y(0) h2 |h|


Además si t = 0 ) y0(0) = lim = lim = limh |h| =
h!0 h h!0 h h!0
0
!
) f´(0) = (0, 0)
Lo que prueba que esta función es derivable en todo R y que hay puntos
en los cuales la derivada es cero, esto geometricamente significa que
la curva no es suave en ese punto, cambia rapidamente de dirección,
presenta un peak.

Con el objeto de advertir este comportamiento geométrico en una curva


y su relación con la derivada demos la siguiente definición.

102
2.6.1. Camino regular
!
Definición 2.6.1. Sea f : I ✓ R ! Rn una función de clase C 1 (I).Se
! ! !
dice que f (t) describe un camino regular si f 0 (t) 6= 0 8 t 2 I.

Consideremos el siguiente ejemplo para aclarar aún más este concepto

Ejemplo 8
Sea la curva C es descrita por !
c (t) = (cos3 t, sin3 t), 0  t  2⇡, que

corresponde a un Astroide

Muestre que no es una curva regular en 0  t  2⇡,

Solución
Esta trayectoria regular no es regular, pues en t = 0, ⇡2 , ⇡, y 3⇡ 2
la
derivada se anula como podemos verificar en el siguiente cálculo.
!c´(t) = (3 cos2 t( sin t); 3 sin2 t cos3 t) = ( 3 cos2 t sin t; 3 sin2 t cos3 t)
!
c´(0) = (0, 0) ; !
c´( ⇡2 ) = (0, 0) , !
c´(⇡) = (0, 0) , y !
c´( 3⇡ ) = (0, 0)
2

2.6.2. Propiedades de la Derivada


!
Teorema 2.6.1. Sean f , ! g : I ✓ R ! Rn funciones derivables y
↵ : I ✓ R! R una función derivable, entonces:
! !
i) (k f )0 (t) = k f´(t)

103
! !
ii) ( f + ! g )0 (t) = f 0 (t) + !g 0 (t)
! !0 !
iii) ( f g ) (t) = f 0 (t) ! g 0 (t)
! ! !
iv) (↵ f )0 (t) = ↵0 (t) f (t) + ↵(t) f 0 (t)
! ! !
v) ( f · !
g ) (t) = f´(t)· ! g (t)+ f (t)· !
0
g´(t) Producto Punto o Producto
interno
! !
vi) ( f ↵)´(t) = f´(↵(t))↵´(t)
! ! !
vii) ( f ⇥ !g )0 (t) = f´(t) ⇥ ! g (t) + f (t) ⇥ !g 0 (t) para n = 3
Demostración: i), ii), iii) se dejan al lector

! !
(↵ f )0 (t) = (↵(t) f (t))´= g ((↵f1 )´(t), (↵f2 )´(t), (↵f3 )´(t), ..., (↵fn )´(t))
= (↵0 (t)f1 (t) + ↵(t)f´01 (t), ↵0 (t)f2 (t) + ↵(t)f20 (t), ..., ↵0 (t)fn (t) + ↵(t)fn0 (t))
= (↵0 (t)f1 (t), ↵0 (t)f2 (t), ..., ↵0 (t)fn (t) + ↵(t)f10 (t), ↵(t)f20 (t), ..., ↵(t)fn0 (t))
= ↵0 (t)(f1 (t), f2 (t), ..., fn (t)) + ↵(t)(f10 (t), f20 (t), ..., fn0 (t))
! !
= ↵0 (t) f (t) + ↵(t) f 0 (t)

! !0 ⇣! ⌘0
v) ( f · g ) (t) = !
f (t) · g (t)
" n # n
d X X d
= fi (t)gi (t) = (fk (t)gk (t))
dt k=1 k=1
dt
n
X
= [fk0 (t)gk (t) + fk (t)gk0 (t)]
k=1
n
X n
X
= fk0 (t)gk (t) + fk (t)gk0 (t)
k=1 k=1
! !
= f 0 (t) · !
g (t) + f (t) · !
g 0 (t)

v), vi) se dejan al lector.

2.7. Parametrización

Si !c (t) define una trayectoria donde t es el parámetro, podemos mod-


ificar la expresión que define !c (t) por !
c (s) de tal modo de tener el
mismo conjunto de imagenes, esto lo llamaremos reparametrización.

104
! !
Definición 2.7.1. Sean f : I ✓ R ! Rn tal que f = (f1 , f2 , f3 , ..., fn )
define un camino regular en el espacio Rn y ' : I1 ! I una fun-
ción de clase C 1 (I) biyectiva y tal que '0 (s) 6= 0, 8 s 2 I1 , entonces
! ! !
f o : I1 ! Rn tal que f o (s) = ( f ')(s) se llama reparametrización
!
de la trayectoria) f .

Observación:
De esta definición se tiene que:

! ! !0 !
1) f o (s) = ( f ')(s) ) f (s) = f´('(s)) · '´(s)
!0 ! !
si '(s) = t ) f o (s) = f´(t) · '´(s) = '´(s) · f´(t)

! !
) f 0o (s) = '´(s) · f 0 (t)

!0
como '(s) es un escalar, esto significa que f o (s) es '´(s) veces la
!
velocidad que llevarı́a una partı́cula parametrizada por f (t).

2) '0 (s) 6= 0 ) '´(s) > 0, 8s 2 I1 v '´(s) < 0, 8s 2 I,


!0 !
de la expresión f o (s) = '´(s) · f 0 (t) se puede inferir que:
! !
a.- Si '´(s) > 0 , 8 s 2 I1 ) fo (s) conserva la orientación de f (t)
! !
b.- Si '´(s) < 0 , 8 s 2 I1 ) fo (s) invierte la orientación de f
! ! !
Si fo (s) es una reparametrización de f (t) y del hecho que fo 0 (s) =
! ! !
'´(s) · f´0 (t) en cada punto fo (s) = f (t) si '(s) = t, se deduce el

105
siguiente teorema.
!
Teorema 2.7.1. Sea f : I ✓ R ! R2 ( o R3 ) un camino regular
! !
y fo = f ' : J ✓ R ! R2 ( o R3 ) una reparametrización de él (
donde ' tiene las condiciones pedidas en la definición). Entonces la
! !
recta tangente a la curva C (traza de f ) en f (to ) con t0 2 I , es
!
la misma que la recta tangente a C en fo (s0 ) si t0 = '(s0 ).
! ! !
Demostración.- t0 = '(s0 ) ) P 0 = f (to ) = f ('(s0 ) ) =
! ! ! !0
fo (s0 ) además fo´(s) = '´(s)· f´(s) implica que los vectores f (to )
!0
y fo (s0 ) son paralelos,entonces las rectas tangentes a la curva en
!
f (to ) coinciden.

2.7.1. Ejemplos de reparametrizaciones


!
Sea f : [a, b] ! R3 ,una trayectoria regular . Entonces:
! !
1) La trayectoria fo : [a, b] ! R3 tal que t 7 ! f (a + b t) es la
!
reparametrizacion de f que corresponde a la aplicacion ' : [a, b] 7 !
!
[a, b] dada por t 7 ! a + b t, llamamos a f o trayectoria opuesta a
!
f.
!
2) La trayectoria ! g : [0, 1] ! R3 tal que t 7 ! f (a + (b a)t) es una
!
reparametrizacion de f que corresponde a la aplicación ' : [0, 1] 7 !
!
[a, b] dada por t 7 ! a + (b a)t, y que conserva la trayectoria de f .

Ejemplo 9
! !
Sea f : [ 5, 10] ! R3 definida por t 7 ! f (t) = (t, t2 , t3 ) . Reparametrizar
!
como trayectoria opuesta a f .
Solución
!
Aplicando el apartado 1) tenemos: fo : [ 5, 10] ! R3 tal que t 7 !
!
f (5 t) = ((5 t), (5 t)2 , (5 t)3 ) es la reparametrización opuesta
!
a f que corresponde a la aplicación ' : [ 5, 10] 7 ! [ 5, 10] dada por
t 7 ! 5 t.
Ejercicio
!
Sea f : [a, b] ! R3 , una⇥ trayectoria
⇤ regular y k una contante positiva
. Sea la aplicación ' : 0, b k a 7 ! [a, b] dada por '(t) 7 ! kt + a.

106
! ⇥ ⇤
Muestre que la trayectoria reparametizada f ' : 0, b k a ! R3
!
mantiene la trayectoria de f pero la recorre k veces más rápido.

2.8. Longitud de Arco


!
Sea C una curva descrita por f (t) = (f1 (t), f2 (t), f3 (t), ..., fn (t)), defini-
da en un intervalo I = [a, b] en R.Sea P una partición de [a, b] y Lp
la longitud de la poligonal originada por P.

n
X ! !
Lp = f (ti ) f (ti 1 )
i=1

Para cada partición P se tiene una correspondiente Lp .

!
Definición 2.8.1. La curva C descrita por f (t) = (f1 (t), f2 (t), f3 (t), ..., fn (t))
definida en [a, b] se dice que es rectificable si

{Lp /P es partición de [a, b]}

tiene una cota superior. Si C es rectificable entonces la longitud ` de C


es el supremo del conjunto de los Lp , es decir

` = sup {Lp /P es partición de [a, b]}


!
Definición 2.8.2. Sea f : I ✓ R ! Rn una trayectoria regular de
! !
clase C 1 . La longitud de f entre t = a, t = b denotada por `( f ) o
simplemente ` se define por:

Z b
!
`= f´(t) dt
a

107
Ejemplo 10.

Si !
c : [0, 2⇡] ! R2 tal que !c (t) = (r cos t, r sin t). ¿ Es una trayectoria
regular? ¿Cuál es la longitud de la Curva asociada?
Solución.
Como ! c´(t) = ( r sin t, r cos t) 6= (0, 0) 8t 2 [0, 2⇡] la trayectoria es
regular, luego
p
k!c´(t)k = ( r cos t)2 + (r sin t)2 ) = r
Z 2⇡ Z 2⇡
)`= k!
c´(t)k dt = rdt = 2⇡r
0 0

Ejemplo 11.
¿Cuál es la longitud del astroide dado por la ecuación: !
c (t) = (a cos3 t, a sin3 t) con
t 2 [0, 2⇡]?
Solución:

Verifiquemos primeramente si el astroide es una curva regular:


!
c´(t) = ( 3a cos2 t sin t, 3a cos t sin2 t)

Podemos inferir que el astroide no es una curva regular, pues


!c´(t) = ( 3a cos2 t sin t, 3a cos t sin2 t) = (0, 0) para t = 0, ⇡2 , ⇡, 3⇡ , 2⇡
2
pero dado que es simétrica respecto de⇤ ambos
⇥ ejes podemos calcular su
longitud en el segmento del dominio 0, ⇡2 y luego multiplicamos por

108
cuatro.
⇡ ⇡
Z2 Z2 p
` !
= k c´(t)k dt = 9a2 cos4 t sin2 t + 9a2 sin4 t cos2 tdt
4
0 0

Z 2  ⇡
sin2 t 2
3a
= 3a sin t cos tdt = 3a =
2 0 2
0
=) ) ` = 6a

2.8.1. La Longitud de Arco como Parámetro


!
Teorema 2.8.1. Sea f : I ✓ R ! Rn función de clase C 1 tal que
! !
f´(t) 6= 0 8 t 2 I . Entonces la longitud s definida por
Zt
!0
s(t) = f (u) du, con t 2 I , de una curva puede introducirse como
0
un parámetro de la curva y :

!
df
ds
=1

Demostración:

Zt
!0 ds !
s(t) = f (u) du ) s´(t) = = f 0 (t) 0
dt
0

Claramente s = s(t) es monótona, estrictamente creciente y contı́nua,


por lo tanto s(t) tiene función inversa, que denominaremos por t(s) tal
!
que t´(s) existe y es positiva para todo s con 0  s  l( f ).

!
f (t(s)) = (f1 (t(s)), f2 (t(s)), ..., fn (t(s))

Derivando usando la regla de la cadena tenemos:

109
!
df
= (f1´(t(s)) · t´(s), f2´(t(s)) · t´(s), ..., fn´(t(s)) · t´(s))
ds
! !
df
d f dt dt
= · = ds
dt ds dt

De donde se obtiene que:

! ! !
df f´(t) f´(t)
= =) =1
ds ks´(t)k s´(t)

Observación:

!
1. Del teorema anterior se desprende, la rapidez con que recorre f
es constante e igual a 1 si está paramétrizada con el parámetro
longitud de arco.
2. Si !r (s) = (x(s), y(s), z(s)) describe una curva de R3 , y s es
parámetro longitud de arco, entonces
d!
Tb = r
ds
es vector tangente unitario, ya que: Tb = db
r
ds
=1

Propiedades 2.8.1. La longitud de un arco es independiente de la


parametrización

Demostración:
! !
Sean f : [a, b] ! Rn un camino regular y f ⇤ : [c, d] ! Rn una
!
reparametrización de f ; por lo cual existe ' de [c, d] en [a, b] de C1
que es sobreyectiva y '´(s) 6= 0 8 s 2 [c, d]

Entonces:

110
⇣ !⌘ Zd Zd
!⇤ !
s fo = f ´(u) du = f´('(u))'´(u) du
c c
Zd
!
= f´('(u)) '´(u)du si '´(u) > 0
c

⇣ !⌘ Z b ! !
t = '(u) ) s fo = f´(t) dt = s( f )
a
⇣ !⌘ !
) s fo = s( f )

De la misma forma si '´(u) < 0, la parametrización invierte el


sentido y '(c) = b, '(d) = a

⇣ ! ⌘ Zd ! Zd
!
) s f ⇤
= f´('(u))'´(u) du = f´('(u)) ( '´(u)) du
c c
Zd Zd
! !
= f´('(u)) '´(u)du = f´(t)) dt =)
c c
⇣ !⌘ !
s fo = s( f )

2.8.2. Parametrización por Longitud de Arco


!
Definición 2.8.3. Sea f : I ✓ R ! Rn , función de clase C 1 y tal
!
que f´(t) 6= 0 en I = [a, b] , se define s por

Zt
!0
s= f (u) du = '(t)
a

De la definición de s se pueden hacer las siguientes precisiones:

Zt
! ds !
1. s = '(t) = f´(u) du ) = '´(t) = f´(t)
dt
a

111
ds
2. Si t 2 (a, b] ) dt
> 0 ) s = '(t) es función estrictamente
creciente.
3. Por (2) s = '(t) es biyectiva y por lo tanto invertible, sea t =
' 1 (s).

!
Por tanto, la parametrización de f en términos s es:
! !
f (s) = f (' 1 (s))

Ejemplo 12
Considere la trayectoria: !r (t) = (a cos t, asent) ,y reparametrice en
función de la longitud de arco s.
Solución
La ecuación cartesiana de la curva C es x2 + y 2 = a2 ,que corresponde
a una circunsferencia centrada en el origen de radio a.
Verifiquemos si es una curva regular, su derivada es:

!r´(t) = ( a sin t, a cos t) 6= (0, 0) 8t )


p
!
k r´(t)k = a2 sin2 t + a2 cos2 t
) k!
r´(t)k = a

Determinemos su longitud de arco,

Zt Zt
s = k!
r´(u)k du = adu = [au]t0 = at
0 0
s
) s = at o t=
a

Por lo tanto, !
r (s) = (a cos as , a sin as ) es una parametrización por lon-
gitud de arco de la circunferencia de radio a.

Observación: No siempre es sencillo parametrizar por longitud de


arco, a modo de ejemplo veamos el siguiente caso.
Ejemplo 13

112
Considere la curva C curva descrita por la trayectoria! r (t) = (t, t2 +1),
t 2 [0, 3] , y parametrice en función de la longitud de arco s.
Solución.
La curva corresponde a una parabóla que tiene por ecuación cartesiana:
y = x2 + 1, x 2 [0, 3]

Determinemos la longitud de la curva:


! p
r´(t) = (1, 2t) ) kr´(t)k = 1 + 4t2
Zt p " p p #t
ln 1 + 4u 2 + 2u 2
u 1 + 4u
s = 1 + 4u2 du = +
4 2
0 0
1 p tp
s = ln 1 + 4t2 + 2t + 1 + 4t2
4 2

En este caso no es posible obtener t = ' 1 (s) es decir despejar t en


función de s a partir de

1 p tp
s = '(t) = ln 1 + 4t2 + 2t + 1 + 4t2
4 2

Luego no es posible hacer explicita la función t = g 1 (s).

2.9. Trayectorias y curvas


La aplicación ! c : [a, b] ! R3 (o R2 ) continua , definida de un inter-
valo I al espacio R3 o al plano R2 , la llamaremos trayectoria. La imagen
C en R3 (o R2 ) de la trayectoria corresponde a lo que es una curva.

Definición 2.9.1. Se llama curva de la trayectoria ! c : I ⇢ R !


R3 (R2 ) dada por !
c (t) = (x(t), y(t), z(t)) al conjunto de imagenes de
!
c ;es decir

C= !
c (t) 2 R3 / t 2 I = (x(t), y(t), z(t)) 2 R3 : t 2 I

113
Ejemplo 1: Sea ! c : R ! R3 una función definida por ! c (t) =
(x0, y0, z0 )+t(v1, v2, v3 ) es una recta L en el espacio que pasa por (x0, y0, z0 )
y tiene la dirección ! v = (v1, v2, v3 ). A partir de esta función se deducen
las ecuaciones: x(t) = x0 + tv1, y(t) = y0 + tv2, z(t) = z0 + tv3 , que
se conocen como ecuaciones paramétricas de una recta en el espa-
cio.Luego, la recta es la imagen de la trayectoria: ! c (t) = (x0, y0, z0 ) +
t(v1, v2, v3 ), t2R
Generalmente, usamos t como variable independiente y ! c (t) señala la
posición de una partı́cula en el espacio y t en este caso es la variable
tiempo.

Ejemplo 2: Sea ! c : [0, 2⇡] ! R2 una función definida por !


c (t) =
(cos t, sin t) ,¿cuál es la curva asociada a esta trayectoria?
Solución
A partir de las funciones paramétricas tenemos , x (t) = cos t y (t) = sin t
que cumplen con
(x (t))2 + (y (t))2 = (cos t)2 + (sin t)2 = 1 8t 2 [0, 2⇡]
Por lo tanto, es una trayectoria cuya imagen corresponde a una cir-
cunsferencia centrada en el origen y radio unitario.
⇥ ⇤
Observe que ! c (t) = (cos 3t, sin 3t) con t 2 0, 2⇡ describe la misma
3
curva. En este caso tenemos que trayectorias parametrizadas en forma
diferente describen una misma curva. La variable t se designa usual-
mente con el nombre de parámetro. De ahora en adelante diremos que
!c (t) = (x(t), y(t), z(t)) es una representación paramétrica de la curva
C, donde t es el parámetro.
Definición 2.9.2. Sea ! c (t):[a, b] ! R3 continua, una trayectoria que
describe la curva C.diremos que C en una curva cerrada si y solo si
!c (a) = !
c (b).

114
Una curva cerrada simple, o curva de Jordan es una curva cerrada que
tiene la propiedad :si ! c (t1 ) = !
c (t2 ) =) (t1 = t2 ) _ (t1 = a y
t2 = b).Como vemos esto ocurrre si la función ! c es inyectiva en I es
! !
decir 8t1 , t2 2 I, t1 6= t2 =) c (t1 ) 6= c (t2 )

Ejercicios 3
A partir de la representación paramétrica dada, describa las curvas y
encuentre las ecuaciones cartesianas de las mismas, si ello es posible:
1) !c (t) = (t, t) t 0
2) ! 2
c (t) = (4 t , t) t 2 [ 2, 3]
3) !c (t) = (cos t, sin t, 2t) 0  t  1
4) !c (t) = (a cos t, b sin t, 1) 0  t  2⇡

2.10. Vectores Unitarios


2.10.1. Vector Tangente unitario
Sea !c (t) : [a, b] ! R3 una trayectoria y C la curva descrita por !c (t),
! ! !
supongamos que c tiene derivada de tercer orden y que c´(t) 6= 0 8
t 2 [a, b] . Definimos el Vector Tangente unitario en un punto ! c (t) de
la trayectoria, como sigue:
!c´(t)
b
T(t) = !
c´(t)

b
En el caso que el parámetro sea el parámetro longitud de arco, entonces: T(s) =!
c´(s)

Propiedad
Sea !c (s) : [0, L] ! R3 una trayectoria parametrizada por longitud de
arco. Pruebe que los vectores T b y TT
b ´ son ortogonales.
En efecto:
2
b ·T
Como: T b = T
b =1 =) d bc
(T·T) b ·T
=0)T b´+ T
b´· T
b =0)
ds
b ·T
2T b´= 0

b ·T
)T b´= 0 ; es decir que T
b es ortogonal a T

115
2.10.2. Vector Normal

b 0 (s)
T
b
b´ 6= 0 definimos : N =
En todos los puntos donde T ,Vector
b 0 (s)
T
normal principal a la curva C en el punto !c (s).

2.10.3. Vector Binormal


Hay un tercer vector unitario que es perpendicular tanto a T b como a
b . Se define por el producto cruz de estos vectores, y es denominado
N
vector Binormal, denotado por: B b=T b ⇥N b
n o
El conjunto de vectores b b b
T, N, B forma un sistema de vectores
unitarios, ortogonales entre sı́, orientados positivamente en este orden
en cada punto ! c (s) de la curva, es decir se cumple que:
b ·N
T b = 0, b ·B
N b = 0, b ·T
B b =0
b ·T
T b = 1, b ·N
N b = 1, b ·B
B b =1
b ⇥N
T b = B,b N b ⇥B b = T,
b b ⇥T
B b =N b
n o
En la medida que varı́a el conjunto de vectores Tb, N b, B
b ,este se de-
splaza a lo largo de la curva y se llama triedro movil.

Ejemplo 4
Considere la hélice circular definida por: !
c (t) = (3 cos t, 3 sin t, 4t), t >
b b b
0 . Hallar los vectores T , N , y B.

116
Solución.

Sea !c ´(t) = ( 3 sin t, 3 cos t, 4) =)


!
c ´(t) = 5

Zt
s
) s= kc(u)k du = 5t ) t =
5
0
! s s s
) c (s) = (3 cos , 3 sin , 4 )
5 5 5

Por lo tanto: Tb = !
c 0 (s) = ( 3
5
sin 5s , 3
5
cos 5s , 4
5
)

Del resultado anterior podemos inferir que Tb = 1

b =
Ahora determinemos el vector normal a partir de su definición: N
b´(s)
T
b´(s)
T

b´(s) = (
T 3
cos 5s , 3
sin 5s , 0)
25 25
q q
b´(s) = 3 s 2 3 s 2 3 2 3
T 25
cos 5
+ 25
sin 5
= 25
= 25

117
3
b=( cos 5s , 3
sin 5s , 0)
N 25
3
25
= ( cos 5s , sin 5s , 0)
25

b =1
Del resultado anterior podemos inferir que: N

A partir de los vectores Tb y N


b se tiene B
b = Tb ⇥ N
b)
bi b
j b
k
b=
B 3
sin 5s 3
cos 5 45 =
s 4
sin 5s i 4
cos 5s j + 35 j y podemos com-
5 5 5 5
cos 5s sin 5s 0
b =1
probar que: B

De los calculos anteriores podemos resumir que tenemos:


En cada punto ! c (s) de la curva
Tb = ( 3
5
b = ( cos s ,
sin 5s , 35 cos 5s , 45 ), N 5
b = ( 4 sin s ,
sin 5s , 0), B 5 5
4
5
cos 5s + 35 , 0)
s
como t = podemos reparametrizar la trayectoria en funcion de t,
5
como sigue
Tb(t) = ( 3
5
b (t) = ( cos t,
sin t, 35 cos t, 45 ), N b = ( 4 sin t,
sin t, 0), B(t) 5
4
5
cos t + 35 , 0)

Como ejercicio dejamos que verifique que: Tb ⇥ N


b =B
b N
b ⇥B
b = Tb B
b ⇥ Tb = N
b

2.11. Curvatura

!
Sea f : I ! Rn función vectorial dos veces diferenciable, parametriza-
!
da por el parámetro longitud de arco. Al número k(s) = f ”(s) se
! !
le llama curvatura de f en el punto f (s).
Intuitivamente, de la definición se infiere que la curvatura es una medida
de cuanto se ”doblaüna curva, como una medida del alejamiento de la
curva de la recta tangente.
Ejemplo 5
Calcular la curvatura de la hélice !
c (t) = (3 cos t, 3 sin t, 4t), t 0
Solución.

118
Sabemos que al parametrizar en función del arco se tiene
! s s s
c (s) = (3 cos , 3 sin , 4 )
5 5 5
3 s 3 s 3
k(s) = k! c ”(s)k = ( cos , sin , 0) =
25 5 25 5 25

En este caso, la curvatura es constante y como k(s) = k! c ”(s)k =


Tb´(s) ,significa que el vector tangente unitario T b (s) tiene la misma
rapidez de variación de su dirección, en todos los puntos.

Ejemplo 6
Mostrar que la curvatura de una recta es cero.
Solución

Sea P0 = (xo , yo , zo ) punto de la recta y !


v = (a, b, c) su vector dirección,
!
entonces podemos escribir c (t) = (xo + ta, yo + bt, zo + ct)

! Rt p p
c´(t) = (a, b, c) =) s = a2 + b2 + c2 dt = t a2 + b2 + c2
0
p s
) s = t a2 + b 2 + c 2 o t= p
a2 + b2 + c 2

Reparametrizando en función del arco se tiene


! sa sb sc
c (s) = (xo + p , yo + p , zo + p )
2 2
a +b +c 2 2 2
a +b +c 2 a + b2 + c 2
2

Derivando la expresion anterior:

! 1
c 0 (s) = p (a, b, c) ) !
c ”(s) = (0, 0, 0)
2 2
a +b +c 2

) k(s) = 0

Por lo tanto, la curvatura en cualquier punto es cero.

Cı́rculo, circunferencia de curvatura

119
Si !c (s) es un punto de la curva C y k la curvatura. La circunferencia
que es tangente la curva C en el punto ! c (s) de radio R = k1 se llama
circunferencia de curvatura y R radio de curvatura.
El centro de esta circunferencia se encuentra en la dirección del vector
!
c ”(s).

2.11.1. Cálculo de curvatura usando parámetro t


cualquiera en R3
!
Teorema 2.11.1. Sea f : I ✓ R ! R3 función vectorial, al menos
!
dos veces diferenciable ,tal que: f´(t) 6= 0 8t 2 I. Entonces:
! !
f´(t) ⇥ f ”(t)
k(t) =
! 3
f´(t)

Demostración:
Sea s parámetro longitud de arco tal que t = '(s)
!
! b (s) = f ‘(t) ) ! 1 !
f´(s) = T ! f´(s) = ! f ‘(t)
f ‘(t) f ‘(t)

!00
Debemos calcular f (s). Usando la regla de la cadena se tiene:

120
0 1
! d @ 1 ! A dt dt 1 1
f ”(s) = ! f ‘(t) con = = !
dt f ‘(t) ds ds ds f ‘(t)
dt
! 2 ! !
Por otra parte, como f ‘(t) = f ‘(t) · f ‘(t)

Se tiene:
! d ⇣ ! ⌘ ! ! ! !
2 f ‘(t) f ‘(t) = f ‘(t) · f ”(t) + f ”(t) · f ‘(t)
dt

d ⇣ ! ⌘ ! !
f ‘(t) · f ”(t)
) f ‘(t) = !
dt f ‘(t)

!
Volviendo a la derivada de f´(s) :

! ! ! d ⇣ ! ⌘
! f ”(t) f ‘(t) f ‘(t) f ‘(t) 1
f ”(s) = dt ·
2 ds
kf ‘(t)k
dt
! !
! ! ! f ‘(t) · f ”(t)
f ”(t) f ‘(t) f ‘(t) !
! f ‘(t) 1
f ”(s) = · !
! 2
f ‘(t) f ‘(t)
! ! 2 ! ⇣! ! ⌘
! f ”(t) f ‘(t) f ‘(t) f ‘(t) · f ”(t)
f ”(s) =
! 4
f ‘(t)

! 2 ! !
Como f ”(t) = f ”(t) · f ”(t) )

0 ⇣! ⌘ 12
! ! 2 ! !
! 2
B f ”(t) f ‘(t) f ‘(t) f ‘(t) · f ”(t) C
f ”(s) =@ A
! 4
f ‘(t)

! 2 ! 4 ! 2 ⇣! ! ⌘2 ! 2 ⇣! ! ⌘
! 2 f ”(t) f ‘(t) 2 f ‘(t) f ‘(t) · f ”(t) + f ‘(t) f ‘(t) · f ”(t)
f ”(s) =
kf ‘(t)k8

121
! 2 ! 2 ⇣! ! ⌘2
! 2 f´(t) f ”(t) f ‘(t) · f ”(t)
f ”(s) =
! 6
f ‘(t)

! ! 2
! 2 f ‘(t) ⇥ f ”(t)
f ”(s) =
! 6
f ‘(t)
Por lo tanto:
! !
f ‘(t) ⇥ f ”(t)
k(t) =
! 3
f ‘(t)
Caso particular es el de curvas en el plano.
Ejemplo 7
Sea !r (t) = (x(t), y(t)) una trayectoria de R2 , en este caso

|x´(t)y”(t) x”(t)y´(t)|
k(t) =
[(x´(t))2 + (y´(t))2 ]3/2

Solución
Se deja al alumno, como indicación se sugiere poner
! !
r (t) ⌘ f (t) = (x(t), y(t), 0)

y aplicar la formula del teorema precedente

Ejemplo 8 Calcule la curvatura de la espiral de Arquı́mides ⇢ = a✓.

Solución.-
x = ⇢ cos ✓
=) r(✓) = (a✓ cos ✓, a✓sen✓)
y = ⇢sin✓

se considera que x(✓) = a✓ cos ✓ y y(✓) = a✓sen✓, derivando estas


funciones se tiene

x´(✓) = a cos ✓ a✓sen✓ y y´(✓) = asen✓ + a✓ cos ✓


x´(✓) = 2asen✓ a✓ cos ✓ y y´(✓) = 2a cos ✓ a✓sen✓

122
Figura 2.1: Espiral de Arquı́mides

efectuando los productos y simplificando se tiene


x´(✓) y”(✓) x”(✓)y´(✓) = 2a2 + a2 ✓2 = a2 (2 + ✓2 )

por otro lado también


(x´(t))2 + (y´(t))2 = a2 + a2 ✓2 = (1 + ✓2 )a2

reemplazando en la formula se tiene


|a2 (2 + ✓2 )| 2 + ✓2
k(✓) = y simplificando k(✓) =
[(1 + ✓2 )a2 ]3/2 a(1 + ✓2 )3/2

para cualquier ✓ positivo.


2 + ✓2
Obsérvese que lı́m = 0, esto significa que la curvatura
✓!1 a(1 + ✓ 2 )3/2
de la espiral de Arquı́mides muy lejos del origen tiende a ser casi una
recta.

2.12. Planos por un punto de la curva


Sea !c : I ✓ IR ! R3 un camino regular dos veces diferenciable
!
y P0 = ! c (t0 ) (donde t puede ser parámetro longitud de arco).
!! !
Si T , N y B son los vectores tangente, normal y binormal de la
curva en !
c (t0 ) = (x0 , y0 , z0 ).Podemos definir los siguientes planos por
! !
P0 = c (t0 )

123
2.12.1. Plano Osculador
! ! !
Plano determinado por T y N en el P0 cuya ecuación es:

!
(x x0 , y y0 , z z0 ) · B = 0

2.12.2. Plano Normal


! ! !
Plano determinado por N y B en el punto P0 cuya ecuación es:
!
(x x0 , y y0 , z z0 ) · T = 0

2.12.3. Plano Rectificante


! ! !
Plano determinado por B y T en el punto P0 cuya ecuación es:

!
(x x0 , y y0 , z z0 ) · N = 0

!
Con estos mismos vectores en el punto P0 se definen las rectas:

124
2.12.4. Recta Tangente
Cuya ecuación vectorial es:

! ! !
P (t) = P0 + t T

Es decir (x, y, z) = (x0 , y0 , z0 ) + t (T1 , T2 , T2 ) , t 2 IR

2.12.5. Recta Normal


Cuya ecuación vectorial es:

! ! !
P (t) = P 0 + t N

Es decir (x, y, z) = (x0 , y0 , z0 ) + t (N1 , N2 , N2 ) , t 2 IR

2.12.6. Recta Binormal


Cuya ecuación vectorial es:

! ! !
P (t) = P 0 + t B

Es decir (x, y, z) = (x0 , y0 , z0 ) + t (B1 , B2 , B2 ) , t 2 IR

Ejemplo 9

Considere el camino regular definido por !


c (t) = (t3 , t2 , t). Obtener:
a) las ecuaciones de los planos Osculador, Normal, Rectificante y
b) las rectas tangente, normal y binormal a esta curva en el punto
(1, 1, 1).

Solución
Previo a responder este problema observemos que para determinar los
planos

125
y rectas pedidas no es obligatorio trabajar con los vectores unitarios
por lo cual útil observar que:

! ! !
c´(t) es vector paralelo a T (t) , es decir: !
c´(t) = ↵ T
! ! !
c´(t) ⇥ !c ”(t) es vector paralelo a B (t), es decir :!
c´(t) ⇥ !
c ”(t) = B
!
(!c´(t) ⇥ !c ”(t)) ⇥ !
c´(t) es vector paralelo a N (t) es decir :

!
(!
c´(t) ⇥ !
c ”(t)) ⇥ !
c´(t) = N

De este modo, en nuestro problema particular.


!c´(t) = (3t2 , 2t, 1) y !
c´´(t) = (6t, 2, 0) Lo que implica al evualar en t =
1 queda:
!c´(1) = (3, 2, 1)
i j k
!
yc´´(1) = (6, 2, 0) por consiguiente !
c´(1) ⇥ !
c ”(1) = 3 2 1 =
6 2 0
2i + 6j 6k = 2(i 3j + 3k)

i j k
(!
c´(1) ⇥ !
c ”(1) ⇥ !
c´(1) = 2 6 6 = 18i 16j 22k = 2(9i
3 2 1
8j 11k)

Ası́ estamos en condiciones de calcular en el punto (1, 1, 1):


Plano Rectificante
(x 1, y 1, z 1) · (9, 8, 11) = 0
) 9x 8y 11z + 10 = 0 ecuación del plano
Plano Osculador
(x 1, y 1, z 1) · (1, 3, 3) = 0
) x 3y + 2z 1 = 0 ecuación del plano
Plano Normal

(x 1, y 1, z 1) · (3, 2, 1) = 0

126
) 3 x + 2y + z 6 = 0 ecuación del plano

x 1 y 1 z 1
Recta tangente: = =
3 2 1
x 1 y 1 z 1
Recta Normal: = =
9 8 11

x 1 y 1 z 1
Recta binormal: = =
1 3 3

2.13. Torsión
Si una partı́cula se mueve siguiendo un camino C, el plano osculador
en un punto P de la curva es un buen referente para observar el giro
o torsimiento de la curva, que no es otra cosa que la medida del ale-
jamiento de la curva del plano osculador en una vecindad del punto
P.
b
El comportamiento de la derivada del vector Binormal B(s) respecto
!
del parámetro longitud de arco da la razón de cambio del vector B
respecto del plano osculador
Por otro lado se puede obsevar que:
b (s)·B
B b (s) = 1 =) B´(s)·
b b (s)+B
B b (s)·B´(s)
b b
= 0 =)) B´(s)· b (s) =
B
b
0 por lo que afirmamos que B´(s) b (s) .
es perpendicular a B

b (s) · Tb (s) = 0 =) B
Además como B b (s) · T´(s)
b b
+ B´(s) · Tb (s) = 0
b b (s) por lo que B
b (s) · T´(s)
b b
T´(s) = ↵N = 0 =) ) ddsB · Tb = 0 por lo
b
que se deduce ddsB es perpendicular a Tb. Estas dos últimas conclusiones
b b y a Tb, esto significa
señalan que ddsB es un vector perpendicular a B
b b ,es decir normal al plano rectificante
entonces que ddsB es paralelo a N
en el punto P. Este razonamiento nos permite formular la siguiente
definición.
Sea ! c : I ✓ IR ! R3 un camino regular tres veces diferenciable
!
parametrizado por longitud de arco y tal que f´´(s) 6= 0 8s 2 I ( es
decir con curvatura no nula). Al número real ⌧ (s) tal que

127
b
dB
= b (s)
⌧ (s)N
ds

se llama Torsión de la curva en el punto ! c (s). Entonces, se tiene que


b
dB b
⌧ (s) = · N (s)
ds
Observaciones:
b
dB
1) El signo menos tiene el propósito de que si ⌧ (s) > 0,entonces
ds
tiene la dirección de N b (s). Ası́ cuando P se mueve sobre la curva
en una dirección positiva Tb, B(s)
b gira alrededor de Tb en el mismo
sentido que un tirabuzón diestro que avanza en la dirección de Tb, como
se muestra en la figura adjunta.

b
2) La k⌧ (s)k = ddsB es una medida de la rapidez con que la curva se
despega del plano osculador
b
3) La primera consecuencia importante, de lo anterior, es que si B(s)
b constante) significa que la curva se mantiene en el plano
no varı́a ( B
b
dB
osculador y = 0.
ds
Afirmamos, entonces que: ! c (s) describe una curva plana si y
solo si su torsión es cero

2.13.1. Cálculo de la torsión usando parámetro t


cualquiera (en R3 )
!
Teorema 2.13.1. Sea f : I ✓ R ! R3 función vectorial al menos
! !
tres veces diferenciable tal que: f ´( t) 6= 0 y f 00 ( t) 6= 0 8t 2 I.
Entonces:

128
h! ! i !
f 0 (t) ⇥ f 00 (t) · f 000 (t)
⌧ (t) =
!0 ! 2
f (t) ⇥ f 00 (t)

La demostración de esta fórmula la dejamos como interesante aunque


largo ejercicio para el lector.
Ejemplo 10
La curva C resulta de la intersección de las superficies z = 2x2 y y
z = x + y. Verifique usando la fórmula que ⌧ (1) = 0.
Solución.-
Sea z = 2x2 y y z = x + y =) x + y = 2x2 y de aquı́ despejando
x
y se tiene y = 2
2x 1
Haciendo x = t se tiene una parametrización para la curva intersección
de las superficies:
✓ ◆
! t 2t3
c (t) = t, 2 ,
2t 1 2t2 1
Las derivadas son:
✓ ◆
! (2t2 + 1) 2t2 (2t2 3)
c’(t) = 1, ,
(2t2 1)2 (2t2 1)2
✓ ◆
! 4t(2t2 + 3) 4t(2t2 + 3)
c ”(t) = 0, ,
(2t2 1)3 (2t2 1)3
✓ ◆
! 12(4t4 + 12t2 + 1) 12(4t4 + 12t2 + 1)
c ”’(t) = 0, ,
(2t2 1)4 (2t2 1)4
En P = (1, 1, 2), t = 1 y evaluando las derivadas en t = 1 se tiene:
!
c ’(1) = (1, 3, 2) , !
c ”(1) = (0, 20, 20) , !
c ”’(1) = (0, 204, 204)

⇥! ⇤ 1 3 2
c ’(t) ⇥ !
c ”(t) · !
c ”’(t) = 0 20 20 = 0. =) ⌧ (1) = 0.
0 204 204
Comentario.- El resultado no podı́a ser otro ya que la curva C está en
un plano, el plano z = x + y.

129
2.14. Formulas de Frenet

Fueron obtenidas por el matemático Frances Jean Frédéric Frenet en


1847 en su tesis doctoral, hoy se les conoce como las formulas de Frenet.
dTb
1) = kNb
ds
dNb
2) = k Tb + ⌧ Bb
ds

b
dB
3) = b
⌧ N
ds

Demostración.-
1) Igualdad establecida en la fundamentación de la definición de la
torsión
2) De las respectivas definiciones se tiene:
b
dT
b =
N ds b
, T(t) =!
c´(s) y k(s) = !
c ”(s)
dTb
ds

b
dT b
dT
b = ds dTb b
N =) = ds
N
dTb ds
ds

b
dT
b
T(t) =! c´(s) y k(s) = ! c ”(s) =) = ! b
c ”(s) N
ds
b
= k(s) N
b
dT
) c
= k N.
ds
2) A partir de Nb =Bb ⇥ Tb y diferenciando se tiene:

b b b
dN
ds
= dB
ds
⇥ Tb + B
b⇥ dT
ds
b ) ⇥ Tb + B
=( ⌧ N b ⇥ (k N
b)
b ⇥ Tb) + k(B⇥
= ⌧ (N b Nb) = ⌧ B
b k Tb.

b
dN
) b
= ⌧B k Tb
ds
130
Finalmente, las fórmulas de Frenet se pueden resumir en una repre-
sentación matricial, donde la matriz de transformación es antisimétri-
ca.
0 1 0 10 b 1
Tb0 (s) 0 k 0 T
B b0 C @ A B b C
@ N (s) A = k 0 ⌧ @ N A
b 0
B (s) 0 ⌧ 0 b
B

2.15. Aplicaciones de Funciones Vectori-


ales y Curvas
Cuando una partı́cula se mueve en el espacio y describe una curva en
tres dimensiones, es conveniente fijar a la partı́cula un sistema de coor-
denadas rectangular, de tal manera que una coordenada sea tangente
a la trayectoria, otra sea normal a ella y una tercera que sea ortogonal
a las dos primeras. Es decir , las coordenadas tangencial, normal y bi-
normal constituyenn un sistema
o de referencia cuya base esta constituida
por los vectores Tb, N b, B
b .

Las ecuaciones de la velocidad y la aceleracion pueden obtenerse como


sigue:
!v =!
!
r 0 = d r ds = s0 Tb
ds dt

! 1
a =!
v0=!
r 00 = s Tb + s0 Tb0 = s Tb + (s0 )2 b
00 00

N donde ⇢ =
k
Puede concluirse que:

1. la velocidad es tangente a la trayectoria y su magnitud es s0

131
2. la aceleración tiene dos componentes.
a) Una es la componente tangencial , de magnitud (s00 ),debida
al cambio de magnitud de la velocidad.
b) La otra es la componente normal, de magnitud (s0 )2 /⇢, hacia
el centro de curvatura. Esta componente se debe al cambio de
dirección de la velocidad.

Tanto la velocidad como la aceleración siempre están sobre el plano


osculador.

2.15.1. Problemas
Problema 1 Una partı́cula se mueve a lo largo de la curva: y = 2x x2
con una componente horizontal de la velocidad de 4m/s, es decir vx =
4m/s. Hallar las componentes tangencial y normal de la aceleración en
el punto (1, 1), si en el instante t = 0, x(0) = 0
Solución:
La función de posición de la partı́cula es:
!
r (t) = (x(t), 2x(t) x2 (t)), t 2 I

y derivando obtenemos su velocidad !


v (t) = (x0(t), 2x0(t) 2x(t)x0(t)).
Como la componente horizontal de la velocidad es 4m/s entonces
x0(t) = 4 , integrando x(t) = 4t+C =) x(0) = 0+C = 0 =) C = 0
Luego !r (t) = (4t, 8t 16t2 ) =) !v (t) = !
r 0 (t) = (4, 8 32t) =)
!
a =! r 00 (t) = (0, 32)
! !r 0 (1/4)
Como ! r 0 (1/4) = (4, 0), entonces T (1/4) = !0 = (1, 0) y
k r (1/4)k
!
N (1/4) = (0, 1) .
Luego, las componentes tangencial y normal de la aceleración se
obtienen proyectando el vector aceleración en esas direcciones.
!
a =!
T a (1/4) · T (1/4) = (0, 32) · (1, 0) = 0 y
!
aN = !
a (1/4) · N (1/4) = (0, 32) · (0, 1) = 32

132
Problema 2
a) Una partı́cula se mueve en el espacio de tal forma que en el in-
stante t = t0 ,el vector velocidad es (1, 1, 1) ,y el vector aceleración es
( 2, 1, 0) .Hallar las componentes tangencial y normal de la aceleración
en el instante t = t0 .
b) Si la partı́cula describe una curva en el espacio de tal manera que
los vectores velocidad y aceleración tienen siempre magnitud constante,
pruebe que la curvatura es constante en cada punto de la curva.

Solución:

a) Puesto que , se conoce la velocidad tenemos:


! p
v (t0 ) = k!v (t0 )k Tb = (1, 1, 1) =) k! v (t0 )k = 3
!a (t0 ) = aT (t0 )Tb + aN (t0 )N b =)
!
v (t0 ) · !a (t0 ) = aT (t0 ) k!v (t0 )k Tb · Tb + aN (t0 ) k!v (t0 )k Tb · N
b
! !
v (t0 ) · a (t0 ) (1, 1, 1) · ( 2, 1, 0)
aT (t0 ) = ! = p
k v (t0 )k 3

Por otra parte:

!
v (t0 ) ⇥ !
a (t0 ) = aT (t0 ) k!v (t0 )k Tb ⇥ Tb + aN (t0 ) k!v (t0 )k Tb ⇥ N
b
p
k!v (t0 ) ⇥ ! a (t0 )k k(1, 1, 1) ⇥ ( 2, 1, 0)k 3 2
aN (t0 ) = = p =
k! v (t0 )k 3 2

k!
v (t) ⇥ !a (t)k
b) La curvatura esta dada por: k (t) = ! 3
k v (t)k

Supongamos que k! v (t)k = c1 yk!a (t)k = c2


Dado que k!
v (t)k = c ,al derivar está expresión , se tiene !
1 v (t)· !
a (t) =
0,
entonces !
v (t) es ortogonal con !
a (t).
Ası́
k!
v (t) ⇥ !a (t)k k!
v (t)k k!a (t)k k!
a (t)k c2
k (t) = ! 3 = ! 3 = ! 2 = 2
k v (t)k k v (t)k k v (t)k c1

Problema 3

133
La aceleración de una partı́cula en función del tiempo viene dada por
!
a (t) = (2t, 3t2 , 4t3 ) . Si en el instante t = 0 está en el origen de coor-
denadas con velocidad inicial ! v (0) = (1, 0, 1).

a) Hallar la velocidad y la posición en cualquier instante de tiempo.


b) Hallar el valor de t en el que la partı́cula pasa por el plano xy.

Solución:

a) Sea !r (t) la posición de la partı́cula en el instante t. Entonces


!
r (t) = !
0
v (t) y ! r (t) = !
00
a (t).

Por lo tanto:
Zt Zt Zt
d!
v = !
a (u)du = 2u, 3u2 , 4u3 du
0 0 0
2 3 4 t
= (u , u , u ) 0
= (t2 , t3 , t4 )

Entonces:
!
v (t) !
v (0) = (t2 , t3 , t4 ) =) !
v (t) = (t2 + 1, t3 , t4 1)

Ahora:
Zt Zt Zt
d!
r (t) = !
v (u)du = (u2 + 1, u3 , u4 1) du
0 0 0
3 4
! ! t t t5
r t) r (0) = ( + t, , t) =)
3 4 5
3 4
! t t t5
r t) = ( + t, , t)
3 4 5

t5
b) La partı́cula pasa por el plano xy cuando z = 0 =) t =
5
0 =) t = 0 y t = 5.

Problema 4

1 d2 y/dx2
Usando la ecuación = .
⇢ (1 + (dy/dx)2 )3/2

134
a) Hallar el radio de la curvatura de la curva representada por y =
x2 + 2x en el punto x = 2.
x2
b) Hallar los radios de la curvatura de la curva representada por +
16
y2
= 1 en los puntos x = 0 y x = 4 respectivamente.
9
Solución:
a) La curva representada por y = x2 + 2x corresponde a una parábola
que se abre hacia arriba cuyo vértice está en el punto ( 1, 1).

Usando la ecuación calculemos el radio de curvatura: y = x2 + 2x =)

dy d2 y
dx
= 2x + 2 =) dx2
=2
1 2
= .Ası́, el radio de curvatura en x = 0 es:
⇢ (1 + 4(x + 1)2 )3/2
1 2
=
⇢ (37)3/2
x2 y2
b) La curva representada por + = 1 corresponde a una elipse
16 9
con semiejes mayor a = 4 y menor b = 3 respectivamente.
r dy 3 v x
x2 = 16 u
Derivando la ecuación y = 3 1 se tiene dx u
t x2 =)
16 1
16
d2 y 3 1
= 16 0 13/2
dx2 x2
@1 A
16

135
1 3 1
Por lo tanto , el radio de curvatura queda =
⇢ 16 7x2 3/2
(1 )
256

1 3 16
Ası́, el radio de curvatura en x = 0 es: = =) ⇢ =
⇢ 16 3

Problema 5

El vector posición de una partı́cula móvil a lo largo de una curva,


está dado por: !r = atbi + b cos tb b
j + b sin tk en donde a y b son
2!
1 d r
constante usando la ecuación: = donde ⇢ es el radio de
⇢ ds2
curvatura. Hallar la curvatura de la curva.

Solución:

Parametricemos , el vector posición en función de la longitud de arco


s.

136
Zt Zt p t
s = k!
r 0 (⌧ )k d⌧ = k(a, b sin t, b cos t)k d⌧ = a2 + b 2 ⌧
0
0 0
s
=) t= p
a2 + b 2
✓ ◆
! s s s
r (s) = ap , b cos p , b sin p
a2 + b 2 a2 + b 2 a2 + b 2
✓ ◆
! a b s b s
r 0 (s) = p , p sin p ,p cos p
a2 + b2 a2 + b2 a2 + b 2 a2 + b 2 a2 + b 2
✓ ◆
! b s b s
r 00 (s) = 0, 2 2
sin p , 2 2
cos p
a +b a +b a +b
2 2 a + b2
2

Por consiguiente, el radio de curvatura es:

1 d2 !
r b a2 + b 2
= = =) ⇢ = = constante
⇢ ds2 a2 + b 2 b

Problema 6
En un cierto instante, las magnitudes de la velocidad y la aceleración
de un proyectil son 20 m/s y 80 m/s respectivamente, y el ángulo de
inclinación entre ellas es de 30o .Determinar el radio de curvatura de la
trayectoria en ese instante

Solución:
Sean Tb y N b los vectores unitarios en las direcciones tangencial y
normal respectivamente. Como la velocidad es tangente a la trayectoria
tenemos que
!v = k!v k Tb = 20 (m/s) Tb
Las componentes tangencial y normal de la aceleración son:

137
p
aT = a cos ✓ = 80 cos 30o = 40 3 m/s2
aN = asen✓ = 80 sen30o = 40 m/s2
! p
a = 40 3 m/s2 Tb + 40 m/s2 N b

El radio de curvatura puede calcularse a partir de:

k! k!
2 2
vk vk 400
aN = =) ⇢ = = = 10m
⇢ aN 40
Problema 7
La aceleración de una partı́cula, está dada por: ! j + 2b
a = 2tbi + 3t2b k en
! 2
donde a está m/s y t está en s. Si en el instante t = 0, v = 0. !
A partir de la componente normal de la aceleración hallar el radio de
curvatura de la trayectoria de la partı́cula en el instante t = 1 s.
Solución:
En primera instancia determinemos la velocidad de la partı́cula
Zt Zt Zt ⇣ ⌘
d!
v = !
a (u)du = j + 2b
2⌧bi + 3⌧ 2b k j + 2tb
d⌧ = t2bi + t3b k
0 0 0
!
v (t) !
v (0) = t2bi + t3b k =) !
j + 2tb j + 2tb
v (t) = t2bi + t3b k

Ahora evaluemos la velocidad y la aceleración respectivamente, en el


instante t = 1s :
! j + 2b
v (1) = bi + b k
! j + 2b
a (1) = 2bi + 3b k
Ası́ la componente normal de la aceleración en el instante t = 1 es
k! k!v (1) ⇥ ! k!
2 3
v (1)k a (1)k v (1)k
aN (1) = = =) ⇢ = ! =
⇢ k!
v (1)k k v (1) ⇥ !a (1)k
p 3
6
k( 4, 2, 1)k
p 3 p 3
6 6
Por lo tanto el radio de curvatura es:⇢ = = p m
k( 4, 2, 1)k 21
Problema 8

138
El vector posición de una partı́cula se describe por la función vectorial
!
r (t) = a cos(wt)bi + a sin (wt) bj + bt2 b
k ,donde a y b son constantes
positivas.
Expresar la velocidad y la aceleración en sus componentes tangencial
y normal.
Solución:
Calculemos la velocidad y la aceleración en coordenadas rectangulares.
!
v =! r 0 (t) = awsen(wt)bi + aw cos (wt) b j + 2btb
k
!
a =!
r 00 (t) = aw2 cos(wt)bi j + 2bb
aw2 sin (wt) b k
A continuación , podemos calcular la rapidez y la magnitud de la acel-
eración
p p
k!vk= ! v ·!
v = a2 w2 + 4b2 t2
p p
k!ak= ! a ·!
a = a2 w4 + 4b2
Ahora, estamos en condiciones de expresar la velocidad y la aceleración
en sus componentes tangencial y normal.
! p
v = k! v k Tb = a2 w2 + 4b2 t2 Tb
!
v (t) · ! a (t) 4b2 t
aT (t) = = p
k!v (t0 )k a2 w2 + 4b2 t2
p
k!v (t) ⇥ ! a (t)k a4 w6 + 4a2 b2 w4 t2 + 4a2 b2 w2
aN (t) = = p
k! v (t0 )k a2 w2 + 4b2 t2
Por tanto, la aceleración es:
2
p
! 4b t a4 w6 + 4a2 b2 w4 t2 + 4a2 b2 w2 b
a =p Tb + p N
a2 w2 + 4b2 t2 a2 w2 + 4b2 t2

Problema 9
La posición de una partı́cula sobre la periferia de una rueda de 8 cm
de diámetro se especifica como s(t) = t3 4t2 + 8t en donde s es la
longitud del arco, medida en cm, a lo largo de la periferia circular, a
partir de un origen conveniente, y t es el tiempo, en segundos.
Hallar la magnitud de la aceleración k!a ken el instante en que la mag-
nitud de la aceleración tangencial es de 4cm/s2 .
Solución:
A partir de la ecuación dada s(t) = t3 4t2 + 8t, obtenemos
s0 (t) = k!
v k = 3t2 8t + 8

139
00
s (t) = aN (t) = 6t 8
El radio de curvatura ⇢ en este caso es el radio de la rueda. Ası́ que:
⇢ = 82 = 4cm.
La aceleración es

2
! 00 (s0 (t))2 b (3t2 8t + 8) b
a (t) = s (t)Tb + N = (6t 8)Tb + N
⇢ 4

En el instante que aT = 4, 6t 8 = 4, de donde t = 2s. Luego, en el


instante t = 2s.
!a (2) = 4Tb + 4Nb
Por lo tanto, la magnitud de la aceleración es:
p p
k!
ak= ! a ·! a = 4 2cm/s2

2.16. Ejercicios resueltos


Ejercicio 1
Un par de trayectorias de [0, 1) en R3 se definen por !
c (t) = (cos t, sin t, bt)
y!r (t) = (1, 0, t). Responda las siguientes preguntas:
a) ¿Se intersectan las curvas generadas por !
c (t) y !
r (t)?
b) Si estas trayectorias representan el desplazamiento de un par de
partı́culas.
¿En qué puntos ,si los hay, estas partı́culas se encuentran?

Solución:
a) !
c (t) es la ecuación de la hélice ascendente sobre el manto del cilindro
x2 + y 2 = 1 y cada vuelta demora 2⇡ unidades de tiempo. Asimismo,
!
r (t) = (1, 0, t) es una recta vertical paralela al eje axial del cilindro ,
que
está sobre el manto de x2 + y 2 = 1 y pasa por (1, 0, 0).
Igualando las primeras componentes cost = 1 ,obtenemos que las curvas
se
intersectan para t = 0, 2⇡, 4⇡, . . .

140
b) Igualando las terceras componentes bt = t =) Si b = 1,entonces las
partı́culas se encuentran en los puntos (1, 0, 0), (1, 0, 2⇡), ..., (1, 0, 2n⇡)
con n 2 Z+
0.

Ejercicio 2
La curva C es definida a partir de la trayectoria !
c (t) = (2 cos(t), 2 sin(t), t)
con 0  t  2⇡ . Describa la representación gráfica de C y pruebe que
si se usa como parámetro la longitud de arco s , el vector tangente a
la
curva es un vector unitario.
Solución:
Por la continuidad de las funciones x(t) = 2cos(t), y(t) = 2sin(t) y
z(t) = t
podemos inferir que C parte del punto ! c (0) = (x(0), y(0), z(0)) =
(2, 0, 0)
y termina en !
c (2⇡) = (x(2⇡), y(2⇡), z(2⇡)) = (2, 0, 2⇡); además que
la
curva asciende a través del manto del cilindro x2 + y 2 = 4 porque
[x(t)]2 + [y(t)]2 = [2 cos(t)]2 + [2 sin(t)]2 = 4 como se ilustra en la figura

El vector posición de esta curva es !c (t) = (2cos(t), 2sin(t), t). El vector


!
tangente es c 0 (t) = ( 2 sin(t), 2 cos(t), 1) ⌅(a) y la longitud del vector
tangente es
p p
k! c 0 (t)k = [ 2 sin(t)]2 + [2 cos(t)]2 + 1 = 5 (b)

141
La longitud total de esta curva es
Z 2⇡ Z 2⇡ p p
Longitud = ! 0
k c (t)k dt = 5dt = 2⇡ 5
0 0

Rt
Definimos s(t) = 0
kc0 (u)k du para t 2 [0, 2⇡] =) s(t) es la longitud
de
curva C desde (2, 0, 0) hasta (x(t), y(t), z(t)).
Claramente s(t) es continua y estrictamente creciente en [0, 2⇡] la ecuación
s = s(t) puede resolverse para t como una función de s, es decir t =
t(s) (c)
En este caso t = ps ası́ es que
5
✓ ✓ ◆ ✓ ◆ ◆
! s s s
c (s) = !
c (t(s)) = 2 cos p , 2 sin p ,p
5 5 5

es vector posición en términos de s, derivando


✓ ✓ ◆ ✓ ◆ ◆
! 0 ! 0 s 1 s 1 1
c (s) = c (t(s)) = 2 sin p p , 2 cos p p ,p
5 5 5 5 5
✓ ✓ ◆ ✓ ◆ ◆
2 s s 1
= p sin p , cos p , (2.16.1)
5 5 5 2

Calculando su módulo
s ✓ ◆  ✓ ◆
2 2
2 s s 1
k!
c 0 (s)k = p sin p + cos p +
5 5 5 4
r
2 1
= p 1+ =1
5 4

Por lo tanto, !
c 0 (s) es vector unitario.

Especificaciones:
a) Si ! c (t) describe la trayectoria de una partı́cula en el espacio, el
vector
!c 0 (t) = ( 2 sin(t), 2 cos(t), 1) es la velocidad con que se desplaza la
partı́cula
por la curva C en el punto ! c (t), en el instante “t”.

142
p
b)k!c 0 (t)k = 5 es la rapidez con que se desplaza la partı́cula 8t, lo
que
significa que la partı́cula se mueve con rapidez constante 8t.
c) Asimismo, la longitud del arco es
Z t
s(t) = k!
c 0 (t)k du
Z0 t p p
= 5du = 5t
0

p s
s= 5t =) t = p
5

En general y en teorı́a la ecuación s = s(t) siempre se puede resolver


para t
en términos de s, es decir tener t = t(s). En la práctica existen casos
en
los que por razones algebraicas no se puede tener t = t(s) ¿Conoces
algún
caso?

Ejercicio 3
Una partı́cula se mueve en el espacio con vector posición
✓ ◆ 32
! ! ! 2 ! !
r (t) = t A + t2 B + 2 t A⇥B
3

! !
donde A y B son dos vectores unitarios fijos que forman ángulo
de ⇡3
radianes.
Calcular la velocidad de la partı́cula en el instante t y determinar en
cuanto
tiempo recorre una distancia de 12 unidades de longitud de arco desde
la
posición en t = 0.
Solución:

143
La velocidad es el vector !
v (t) = !
r 0 (t) donde
✓ ◆ 12
! ! ! 2 2! !
r 0 (t) = A + 2t B + 3 t A⇥B
3 3

Por lo tanto
✓ ◆ 12
! ! ! 2 ! !
v (t) = A + 2t B + 2 t A⇥B
3

Para la segunda parte del problema usaremos


Z t q
s(t) = k r (t)k du con k r (t)k = !
! 0 ! 0
r 0 (t) · !
r 0 (t)
0

Calculando
!
r 0 (t) · !
r 0 (t) =
✓ ◆ 12 ! ✓ ◆ 12 !
! ! 2 ! ! ! ! 2 ! !
= A + 2t B + 2 t A⇥B · A + 2t B + 2 t A⇥B
3 3
= 1 + 4t + 4t2 (a)

por lo tanto
!r 0 (t) · !
r 0 (t) = (1 + 2t)2
q
!
=) k r (t)k = (1 + 2t)2 = 1 + 2t
0

ası́ que
Z t
s(t) = (1 + 2u) du
0
⇥ ⇤t
= u + u2 0 = t + t 2

De acuerdo a lo que se pide en el problema s(t) = 12 =) t2 + t 12 = 0


Las soluciones de esta ecuación son t1 = 3 y t2 = 4 por la naturaleza
del
problema debe ser t 0 luego desconsideramos t2 = 4. En conse-
cuencia, 3
unidades de tiempo son necesarios

144
Especificaciones:
8 ⇣
> ! ! 2
1 ! ! ⌘ ⇣! ! 2
1 ! !⌘
>
> A + 2t B + 2 t 2
A ⇥ B · A + 2t B + 2 t 2
A ⇥ B =
>
> 3 3
>
< ! ! ! ! 1 ! ! ! ! ! ! !
A · A + 2t B · A + 2 23 t 2 A ⇥ B · A + 2t A⇣· B + 4t⌘2 B · B
a) 1 ! ! ! 1 ! ! !
>
> +2(2t) 2
t 2
A ⇥ B · B + 2 2
t 2
A · A⇥B
>
> 3
⇣! ! ⌘ 3
⇣! ! ⌘ ⇣! ! ⌘
>
> 1 !
: +2(2t) 32 t 2 B · A ⇥ B + 4 23 t A ⇥ B · A ⇥ B
Como:
! ! ! ! ! !
A · A = 1, B · B = 1, A · B = cos ⇡3 = 12
! ! ! ! ! !
A · A ⇥ B = 0, B · A ⇥ B = 0, y
⇣! ! ⌘ ⇣! ! ⌘ ! ! 2 ⇣ p ⌘2
A ⇥ B · A ⇥ B = A ⇥ B = 23 = 3
4

Se concluye que:

! 8 3
r 0 (t) · !
r 0 (t) = 1 + t + 0 + t + 4t2 + 0 + 0 + 0 + +
3 4
= 1 + 4t + 4t2 = (1 + 2t)2

Ejercicio 4
!
Sea F : I ! Rn una función velocidad dos veces diferenciable.
a) Pruebe que la aceleración se puede escribir como combinación lineal
de los vectores unitarios Tb y N
b , especı́ficamente se pide establecer que:
✓ ◆ ✓ ◆2
! d2 s b ds b
a (t) = 2
T + k(t) N
dt dt
= aT Tb + aN N
b

b) Aplicando a) calcule las componentes tangencial y normal del vector


aceleración correspondiente a !
r (t) = (t, cos2 t, sin2 t).
Solución:
!
a) Sea !
g una reparametrización de F por longitud de arco entonces
!
F (t) = !
g (s) si y sólo si t = t(s) () s = s(t).
Aplicando la regla de la cadena, queda
! ! ds
F (t) = !
g (s(t)) =) F 0 (t) = !
g 0 (s) s0 (t) = Tb
dt
145
derivando nuevamente respecto de t

!00
F (t) = !
g 00 (s) [s0 (t)] + !
2
g 0 (s) s00 (t)

Por definición k = k(s) = k!


g 00 (s)k es la, curvatura de C en g(s).
por otro lado

dTb
b=
N ds
=) !
g 00 (s) = k! b
g 00 (s)k N
dTb
ds

Relacionando las igualdades anteriores

!00 ⇣ ⌘ ✓ ds ◆2 d2 s
! !
a (t) = F (t) = k g (s)k N00 b + Tb 2
dt dt
✓ ◆ 2
d2 s b ds
= 2
T + k!g 00 (s)k Nb
dt dt
✓ ◆2
d2 s b ds b
= 2
T +k N
dt dt

lo que significa que


✓ ◆2
! d2 s ds
a (t) = 2 Tb + k b
N
dt dt

b) En la aplicación de a) en ! r (t) = t, cos2 t, sin2 t


tenemos ! r 0 (t) = (1, sin(2t), sin(2t)) .
Usando la expresion anterior, se tiene

q
4 sin(2t) cos(2t)
s (t) = k!
0
r 0 (t)k = 1 + 2 sin2 (2t) =) s00 (t) = p
1 + 2 sin2 (2t)

Luego, la componente tangencial aT es

2 sin(2t)
aT = s00 = p
1 + 2 sin2 (2t)

146
Además, ! r 00 (t) = (0, 2 cos(2t), 2 cos(2t)) y haciendo el producto cruz
entre
!r 0 (t) y !
r 00 (t) se tiene
!
r 0 (t) ⇥ !
r 00 (t) = 2 cos(2t)b
j 2 cos(2t)b
k (e)

p p
k!
r 0 (t) ⇥ !
r 00 (t)k = 4 cos2 (2t) + 4 cos2 (2t) = 8 cos2 (2t)
p
= 2 2 |cos(2t)|

Como la curvatura k en función de t es

k!
r 0 (t) ⇥ ! r 00 (t)k
k=
k!
3
r 0 (t)k
reemplazando p
2 2 |cos(2t)|
k= 3
1 + 2 sin2 (2t) 2

de ✓ ◆2 p
ds 2 2 |cos(2t)|
aN = k(t) = 3 1 + 2 sin2 (2t)
dt 2
1 + 2 sin (2t) 2

Por lo tanto, la componente normal de la aceleración en este caso es


p
2 2 |cos(2t)|
aN = p
1 + 2 sin2 (2t)

Ejercicio 5
Sea la trayectoria regular !
r : R ! R3 , definida por:

! 4au2 2au(1 u2 ) a(1 u2 )


r (u) = ( , , ), a > 0
(1 + u2 )2 (1 + u2 )2 1 + u2

a) Pruebe que la función ' : R ! ] ⇡, ⇡[ tal que t = '(u) =


2 arctan u

147
define la reparametrización de !
r,

!
r ⇤ : ] ⇡, ⇡[ ! R tal que r̄(t) = (a sin2 t, a sin(t) cos(t), a cos t)

b) Verifique que !
r ⇤ (t) está contenida en una superficie esférica.
c) Probar que los planos normales a la curva descrita por r̄(t) pasan
por el
centro de la esfera.

Solución

De '(u) = 2 arctan u se tiene que:


2
'0 (u) = 1+u2
> 0 8u 2 R, por lo tanto, es estrictamente creciente en R
y como ' es continua, ' es biyectiva, y por tanto, invertible. Además
t
' 1 (t) = tan( )
2

1
' : ] ⇡, ⇡[ ! R es de clase C 1 en ] ⇡, ⇡[ .
Por lo anterior, se puede definir r̄(t) :

! t
r ⇤ (t) = (!
r ' 1 )(t) = !
r (' 1 (t)) = !
r (tan( ))
2

4a tan2 ( 2t ) 2a tan( 2t )(1 tan2 ( 2t ) a(1 tan2 ( 2t ))


=) !
r ⇤ (t) = ( , ,
(1 + tan2 ( 2t ))2 (1 + tan2 ( 2t ))2 (1 + tan2 ( 2t ))

Sustituyendo
q
1 cos t 2 cos t
4a( 11+cos
cos t
) 2a ( ) 2 cos t
a( 1+cos )
!
r ⇤ (t) = ( t
,
1+cos t 1+cos t
, t
)
2 2 2
( 1+cos t
)2 ( 1+cos t
)2 ( 1+cos t
)
p
= (a(1 cos2 t), a cos t 1 cos2 t, a cos t)
Por lo tanto, la reparametrización la podemos definir por:

!
r ⇤ (t) = (a sin2 t, a sin(t) cos(t), a cos t)

148
b) Consideremos ahora

x(t) = a sin2 t, y(t) = a sin t cos t, z(t) = a cos t =)

x2 + y 2 + z 2 = a2 sin4 t + a2 sin2 t cos2 t + a2 cos2 t

= a2 sin2 t(sin2 t + cos2 t) + a2 cos2 t

= a2 (sin2 t + cos2 t) = a2

) la curva está en la esfera x2 + y 2 + z 2 = a2

c) Se !
r ⇤ (t0 ) un punto cualquiera de la curva.

La ecuación del plano normal es:

⇡N : ((x, y, z) r̄(t0 )) · r̄0 (t0 ) = 0

y (0, 0, 0) 2 ⇡N si sólo si !
r ⇤ (t0 ) · !
r ⇤0 (t0 ) = 0

!
r ⇤ (t0 )·!
r ⇤0 (t0 ) = (a sin2 t0 , a sin(t0 ) cos(t0 ), a cos t0 )·(2a sin t0 cos t0 , a(cos2 t0 sin2 t0 ), a sin

= 2a2 sin3 t0 cos t0 + a2 (sin t0 cos3 t0 sin3 t0 cos t0 ) a2 sin t0 cos t0


= a2 sin3 t0 cos t0 + a2 sin t0 cos3 t0 a2 sin t0 cos t0
= a2 sin t0 cos t0 a2 sin t0 cos t0
=0

) (0, 0, 0) 2 ⇡N

Ejercicio 6
Sea !
r : R+ ! R3 , definida por: !
r = (t2 , 23 t3 , t),la trayectoria regular
que
describe una partı́cula que se mueve a lo largo de una curva C. Para el
instante t = 1, determine:

149
a) Su velocidad ,rapidez y aceleración.
b) Los versores tangente, normal y binormal a trayectoria en ese in-
stante
c) La curvatura y torsión de la curva en ese punto.
d) Las componentes tangencial y normal de la aceleración en el punto.
e) La hodógrafa del movimiento o trayectoria que se determina en el
espacio
de velocidad.

Solución:
a) A partir de la definición tenemos que la velocidad ,rapidez y acel-
eración
en función del tiempo están dadas por:
!v (t) = !r 0 (t) = (2t, 2t2 , 1) =) !
v (1) = !r 0 (1) = (2, 2, 1)
p p
|!v (t)| = ! r 0·!
r 0 = (4t2 + 4t4 + 1) =) |! v (1)| = 3
!a (t) = !r (t) = (2, 4t, 0) =) !
00
a (1) = (2, 4, 0)

b) Los versores se pueden calcular usando las identidades


!
r 0 (t) !
r 0 (1)
Tb (t) = =) Tb (1) = = (2,2,1)
|!
r 0 (t)| |!
r 0 (1)| 3

! 00 00
b (t) = r 0 (t)⇥!
r (t) b (1) =
!
r 0 (1)⇥!
r (1) ( 4,2,4)
B ! !
r (t)⇥ r 00 (t)
=) B ! !
r (1)⇥ r 00 (1)
=
| 0
| | 0
| 6

b (t) = B
N b (t) ⇥ Tb (t) =) N
b (1) = B
b (1) ⇥ Tb (1) = ( 1,2. 2)
3
c) La curvatura y torsión en el punto se pueden determinar usando
las identidades:
! 00 00
r 0 (t)⇥!
r (t) !
r 0 (1)⇥!
r (1)
6 2
 (t) = 3 =)  (1) = 3 = 33
= >0
|!
r 0 (t)| |!
r 0 (1)| 9

00 000 00 000
!
r 0 (t)⇥! r (t) · ! r (t) !
r 0 (1)⇥! r (1) · ! r (1) 8 2
⌧ (t) = ! 2 =) ⌧ (1) = 2 = =
|r 0 (t)⇥! r 00 (t) | |!
r 0 (1)⇥! r 00 (1) | 36 9

d) Derivando la velocidad tenemos


!
a (t) = !
r (t) = |!
00
v (t)| Tb (t) +  (t) |!
0 2 b
v (t)| N (t) ,

entonces las componentes tangencial y normal de la aceleración son:

150
aT = | ! aT = | !
0 0
v (t)| = p4t 2+ 8t3) v (1)| = p4 + 8 = 12 3
=4
(4t +4t4 +1) =) (4+4+1)
! a =  (1) |!
2 2
a =  (t) | v (t)|
N N v (1)| = 2 (3)2 = 2 9
e) A partir del vector velocidad !
v (t) = !
r 0 (t) = (2t, 2t2 , 1) podemos
inferir
sus componentes
9
x0 (t) = 2t =
2
y 0 (t) = 2t2 =) y 0 (x) = x2
;
z 0 (t) = 1 z0 = 1
Por lo tanto, la hodógrafa es una parabóla en el plano z 0 = 1 del espacio
de velocidades.
Ejercicio 7
Sea !
r : R+ ! R3 , definida por: !
r = (a cos t, asent, (t)), trayectoria
regular que describe una partı́cula que se mueve a lo largo de una
curva C. ¿Cuál debe ser la función (t) para que la trayectoria esté
contenida en un plano para todo t?.

Solución:
La trayectoria de la partı́cula es plana si y solo sı́ la torsión es nula
para
todo t. Es decir:
00 000
!
r 0 (t)⇥! r (t) · !
= 0 8t =) !
r 0 (t) ⇥ !
r (t) · !
00 000
r (t)
⌧ (t) = ! 2 r (t) = 0 8t,
| r 0 (t)⇥!
| r 00 (t)

luego derivemos y calculemos el producto mixto


! 9
r 0 = ( asent, a cos t, 0 (t)) =
!r 00 = ( a cos t, asent, 00 (t)) =)
! ;
r 000 = (asent, a cos t, 000 (t))
000
!
r 0 (t) ⇥ !
r (t) · !
00
r (t) = a2 ( 000
(t) + 0
(t)) = 0 =)

000 0
( (t) + (t)) = 0

La ecuación diferencial homógenea de tercer orden , tiene solución de


la
forma (t) = e t ; entonces
0 t 00 2 t 000 3 t
(t) = e =) (t) = e =) (t) = e

151
3
reemplazando términos en la ecuación anterior: ( + )e t = 0, e t =
6 0
3
8t conduce a la ecuación caracterı́stica ( + ) = 0, luego
(t) = C1 + C2 cos t + C3 sent, donde C1 , C2 , C3 son constantes reales.

Por lo tanto, si la trayectoria está dada por


!
r (t) = (a cos t, asent, C1 + C2 cos t + C3 sent)

la partı́cula siempre se moverá sobre un mismo plano.

Ejercicio 8

a) Sea ! r : I ✓ R ! R3 una trayectoria regular de modo que


!r 0 (t) 6= 0 8t. supongamos que hay un t0 2 I para el que la distancia
del
origen al punto! r (t ) 2 R3 alcanza un valor mı́nimo. Pruebe que en
0
ese
punto !
r 0 (t0 ) es perpendicular a !
r (t0 ) .
b) Usando lo anterior , hallar los puntos en que la recta dada por
!
r (t) = (t + 1, 3t 2, 2t 1) está más cerca del origen.

Solución:
a) En efecto consideremos la función distancia d(t)= |!
r (t)|de un punto
cualquiera al origen y supongamos que hay un t0 2 I, tal que se
alcanza un mı́nimo, entonces se tiene que
!
r (t0 ) · !
r 0 (t0 )
d0 (t0 ) = |!
0
r (t0 )| = = 0,
|!r (t )| 0

de donde se tiene que !


r (t0 )· !
r 0 (t0 ) = 0 , por lo tanto, !
r (t0 ) y !
r 0 (t0 )
son ortogonales en ese punto.
b) Basta encontrar un t0 2 I para el que la distancia del punto !
r (t0 ) 2
R3
alcanza su valor mı́nimo que satisface !r (t ) · !
0r 0 (t ) = 0 0

r (t0 ) !
! ·r 0 (t0 ) = (t0 + 1, 3t0
2, 2t0 1) · (1, 3, 2) = 0 =)
1
(t0 + 1) + 3(3t0 2) + 2(2t0 1) = 0 =) t0 =
2
152
Por lo tanto, el punto buscado es !
r 1
2
= 3
2
, 1
2
,0 .
Ejercicio 9
Se llama evoluta de una curva ! r : I ✓ R+ ! R2 a la curva que
describen
los centros de curvatura de !
r . Determine la evoluta de la curva
! 2
r (t) = (t, t ).
Solución:
La curva que describe el centro de curvatura está dada por la ecuación:
!
c (t) = ! 1 b
r (t)+ (t) N (t) , determinemos la curvatura y el versor normal.

!
r 0 (t) = (1, 2t, 0) =) !
00
r (t) = (0, 2, 0) =)
!
r 0 (t) ⇥ !
r (t) = (0, 0, 2) =) !
00
r 0 (t) ⇥ !
r (t) ⇥ !
00
r 0 (t) = ( 4t, 2, 0)

Además: !
r 0 (t) ⇥ !
r (t) = 2 y !
r 0 (t) ⇥ !
r (t) ⇥ !
00 00 1/2
r 0 (t) = 2 (1 + 4t2 )
! 00
r 0 (t)⇥!
r (t)
2
 (t) = 3 =
|!
r 0 (t)| (1 +4t2 )3/2

! 00
b (t) = r 0 (t)⇥!
r (t)⇥!
r 0 (t) ( 4t,2)
N ! ! 00 ! = , reemplazando términos en la
| r (t)⇥ r (t)⇥ r 0 (t)
0
| 2(1 +4t2 )1/2
ecuación tenemos
3/2
! (1 +4t2 ) ( 4t,2) 1
c (t) = (t, t2 )+ 2 2(1 +4t2 )1/2
= 4t3 , 3t2 + 2
,que corresponde
a
la ecuación paramétrica de la evoluta.

1.10 Ejercicio 10

Sea !r : I ✓ R ! R3 una trayectoria regular, dada por !


r = (x(t), y(t), z(t)),
t 2 I. Pruebe que la ecuación del plano osculador en !
r (t ), t 2 I,es: 0 0

x x (t0 ) y y (t0 ) z z (t0 )


x0 (t0 ) y 0 (t0 ) z 0 (t0 ) =0
x00 (t0 ) y 00 (t0 ) z 00 (t0 )
Solución:
! ! !
La ecuación del plano osculador es : ( f r (t0 )) · B (t0 ) = 0, donde
bi b
j b
k
!
B (t0 ) = !
r 0 (t0 ) ⇥ !
r 00 (t0 ) = 0 0
x (t0 ) y (t0 ) z (t0 )0

x00 (t0 ) y 00 (t0 ) z 00 (t0 )

153
! 00 00 00 00
B (t0 ) = (y 0 (t0 ) z (t0 ) y (t0 ) z 0 (t0 ) , x (t0 ) z 0 (t0 ) x0 (t0 ) z (t0 ) ,
x0 (t0 ) y 00 (t0 ) x00 (t0 ) y 0 (t0 ))

pongamos: x0 = x (t0 ) , y0 = y (t0 ) y z0 = z (t0 )

! ! !
(f r (t0 )) · B (t0 ) = 0 =)

00 00 00 00
(x x0 , y y0 , z z0 ) · (y00 z0 y0 z00 , x0 z00 x00 z0 , x00 y000 x000 y00 ) = 0

desarrollando
00 00 00 00
(y00 z0 y0 z00 )(x x0 ) + (x0 z00 x00 z0 )(y y0 ) + (x00 y000 x000 y00 )(z z0 ) = 0

esto último se puede escribir

x x (t0 ) y y (t0 ) z z (t0 )


0
x (t0 ) y 0 (t0 ) z 0 (t0 ) =0
00 00
x (t0 ) y (t0 ) z 00 (t0 )

lo que prueba la hipótesis.


Ejercicio 11
Un automovilista se desplaza por una carretera recta. En el instante
t=0
llega a una rotonda la que recorre con un trayectoria
!
f (t) = (acost, asent, bt(2 t)), t 2 [0, 2]

En el instante t = 2 sale de la rotonda y vuelve a continuar por una


carretera recta.
a) Cálcule la curvatura máxima de la rotonda para t 2 [0, 2]¿en qué pun-
to
ocurre?
b) Determine la torsión de la rotonda para t 2 [0, 2] .Haga un gráfico
de la
torsión en función del tiempo.

154
Solución:
a) Calculemos la curvatura de la trayectoria mediante la identidad:
!0 !00
f (t)⇥ f (t)
 (t) = ! 3
| f 0 (t)|
!0 ! ⇣p ⌘
f (t) = ( asent, a cos t, 2b(1 t)) =) f 0 (t) = a2 + 4b2 (1 t)2
!00
f (t) = ( acost, asent, 2b) =)
!0 !
f (t) ⇥ f 00 (t) = ( 2ab cos t + 2ab(1 t)sent, 2absent 2ab(1
t) cos t, a2 )
!0 ! p
f (t) ⇥ f 00 (t) = a a2 + 4b2 (1 + (1 t)2 ,reemplazando en la identi-
dad,
tenemos p
a a2 + 4b2 (1 + (1 t)2
 (t) =
(a2 + 4b2 (1 t)2 )3/2
0
Sea (t) = a2 + 4b2 (1 t)2 =) (t) = 8b2 (1 t) = 0 =) En
t=1
hay un punto crı́tico de (t)
00 2
Como (1) = 8b > 0, en t = 1 hay un mı́nimo de (t) y un máximo
de
 (t) pues son inversamente proporcionales
p
a2 +4b2
!
Luego,  (t) = a2
y se alcanza en el punto f (1) = (a cos 1, asen1, b).
!0 !00 !000
f (t)⇥ f (t) · f (t)
b) Calculemos ahora la torsión; ⌧ (t) = ! ! 2 8t
| f 0 (t)⇥ f 00 (t)|
!000 ! ! !
f (t) = (asent, a cos t, 0) =) f 0 (t) ⇥ f 00 (t) · f 00 (t) = 2a2 b(1 t)
2a2 b(1 t)
⌧ (t) = a2 [a2 +4b2 (1+(1 t)2 ]
Se tiene que
2b 2b
⌧ (0) = [a2 +8b2 ]
,⌧ (1) = 0, ⌧ (2) = [a2 +8b2 ]
a2 +4b2
⌧ 0 (t) = 0 =) 4b2 (1 t)2 = a2 + 4b2 =) (1 t)2 = 4b2
a2
(1 t)2 = 1 + 4b2
> 1 =) (1 t)2 > 1 lo cual es imposible porque
(1 t)  1
para 0  t  2.
Por lo tanto, ⌧ no tiene punto crı́tico en[0, 2]

155
El gráfico es del tipo
Ejercicio 12
a) Demuestre que la curva descrita por !
r (t) = (t cos t, t sin t, t) se
encuentra
sobre la superficie de un cono. Dibuje la curva.
b) Si una partı́cula parte del origen siguiendo la trayectoria anterior-
mente
descrita, determine en qué punto intersecta la esfera:

x2 + y 2 + z 2 = 2.

c) Calcule la longitud de la curva desde el origen al punto de itersección.

Solución:
a) A partir de las ecuaciones paramétricas tenemos:
9
x (t) = t cos t =
y (t) = t sin t =) x2 + y 2 = t2 (cos2 t + sin2 t) =) x2 + y 2 = t2
;
z (t) = t
) z 2 = x2 + y 2
corresponde a la ecuación de un cono, cuyo sector superior ,se dibuja
en el gráfico adjunto 8t 0
b) Calculemos para que valor del parámetro t la partı́cula intersecta la
esfera, sustityendo las ecuaciones paramétricas en la
esfera x2 + y 2 + z 2 = 2t2 = 2 =) t = 1.Luego, la posición del punto
de impacto es ! r (1) = (cos 1, sin 1, 1)
c) La longitud de la curva es:
Z 1 q
l= k!
r 0 (t)k du con k!
r 0 (t)k = !r 0 (t) · !
r 0 (t)
0

156
con !r 0 (t) = (cos t t sin t, sin t + t cos t, 0)
q p
! 0
k r (t)k = !
r 0 (t) · !
r 0 (t) = 2 + t2 =)
" #
Z 1p ⇣p ⌘ x p2 + x 2 1
l = 2 + t2 dt = ln 2 + x2 + x +
0 2
0
p p
ln( 3 + 2) 3
l = +
2 2

Ejercicio 13
Un proyectil es lanzado desde el nivel del suelo con una velocidad inicial
de 100 m/seg con un ángulo de elevación de 30o .Determine:
a) la función vectorial y las ecuaciones paramétricas de la trayectoria
del proyectil.
b) la altura máxima alcanzada.
c) el alcance del proyectil
d) la velocidad y rapidez en el impacto contra el suelo.
e) la curvatura en el punto de impacto.
Solución:
a) Inicialmente tenemos t = 0 , !
r 0 = (0, 0) ,y !
v 0 = (100 cos 30, 100 sin 30)
Integrando:
!a (t) = !
r ” (t) = (0, 10) =)
!
v (t) = !
r ´(t) = (100 cos 30, 10t + 100 sin 30)

Integrando por segunda vez se obtiene:!


r (t) = ((100 cos 30) t, 5t2 +
(100 sin 30)t)

157
p
Evaluando las funciones trigonométricas queda: !
r (t) = (50 3 t, 5t2 +
50t)
Por lo tanto, las ecuaciones paramétricas de la trayectoria del proyectil
son
p
x(t) = 50 3 t, y(t) = 5t2 + 50t
b) Determinemos el tiempo que demora en llegar a la altura máxima
dy
dt
=0 =) 10t + 50 = 0 =) t = 5s
Asi la altura máxima h alcanzada por el proyectil es
h = y(5) = 125 + 250 = 125m
c) El alcance máximo se logra cuando y(t) = 0, es decir si: 5t2 +50t =
0
t( 5t + 50) = 0 ) t = 0, t = 10.
p
Entonces el alcance es: x (10) = 500 3
p
d) La velocidad del proyectil en el impacto es:!
v (10) = (50 3, 50)
! 00
r 0 (10)⇥!
r (10)
e) La curvatura en el punto de impacto es:K (10) = 3 =
p k!
r 0 (10)k
5 3
104

Ejercicio 14
Sea C una curva determinada por la intersección de los cilindros:
x2 = 1 y, z 2 = y
a) Parametrizar C de forma !
r (t) = (x (t) , y (t) , z (t)), t 2 I.
Indicación: x2 + z 2 = 1
b) Obtener Tb, Nb , B,
b K y ⌧ en P = (0, 1, 1)
Solución:
a) Se puede parametrizar como !
r (t) = (cos t, sin2 t, sin t), t 2 [0, 2⇡]
,
Calculemos el valor del parámetro para
!
r (t1 ) = (cos t1 , sin2 t1 , sin t1 ) = (0, 1, 1) =) t1 = ⇡2 .
Ası́, ! r ⇡ = (0, 1, 1).
2
b)!
r 0 (t) = ( sin t, sin t cos t, cos t) =) !
r0 ⇡
2
= ( 1, 0, 0) =)

158
Tb ⇡2 = ( 1, 0, 0)
!r 00 (t) = ( cos t, 2 cos 2t, sin t); !
00
r ⇡2 = (0, 2, 1)
! p
r 0 ⇡2 ⇥ ! r ⇡2 = (0, 1, 2) =) ! r 0 ⇡2 ⇥ !
00 00
r ⇡2 = 5
⇣ ⌘ ⇣ ⌘
=) B b ⇡ = 0, p1 , p2 y N b ⇡ = 0, p2 , p1
2 5 5 2 5 5

Derivando por tercera vez tenemos:


!
r 000 (t) = (sin t, 4 sin 2t, cos t) =) ! r
000 ⇡
= (1, 0, 0)
2
!0 ⇡ ! ⇡ ⇥!
00
r 0 ⇡2 ⇣ ⌘
Nb ⇡ = r 2 ⇥ r 2
= 0, p2 , p1
2 !r 0 ⇡ ⇥! r 00 ⇡ ⇥ !r0 ⇡ 5 5
2 2 2
Por otra parte
000

!
r 0 ( ⇡2 )⇥!
00
r ( ⇡2 ) p ⇡
!
r 0 ( ⇡2 )⇥!
00
r ( ⇡2 ) · !r ( ⇡2 )
K = 3 = 5 y⌧ = 2 =0
2 k!
r 0 ( ⇡2 )k 2 !
k (2)
r 0 ⇡ !
⇥r 00 ⇡
( 2 )k

Ejercicio 15
Dada la ecuación paramétrica de la trayectoria !r (t) = (2t3 3t2 , t
2arctan(t)), encontrar todos los valores de t para los cuales la curva
(i) Tiene tangente horizontal.
(ii) Tiene tangente vertical.
(iii) No es regular.
Solución:
El vector tangente a la curva !
r (t) = (2t3 3t2 , t 2arctan(t)) es
! t 2
1 y 0 (t)
r 0 (t) = (6t2 6t, 2 ) cuya pendiente es m (t) = 0 =)
t +1 x (t)
y 0 (t) t+1
m (t) = 0
=
x (t) 6t (t2 + 1)
(i) Para que la tangente sea horizontal, ésta tiene que existir, es decir
! !
r 0 (t) 6= 0 , y además m (t) = 0. Por tanto, t = 1.
ii) Para que la tangente sea vertical, ésta tiene que existir, es decir!
r 0 (t) 6=
!
0 , y además m(t) = 1. Por tanto,t = 0.
!
(iii) Para que la curva sea no regular ! r 0 (t) = 0 =) x0 (t) = y 0 (t) = 0,
es decir, t = 1.

Ejercicio 16

159
Se llama evoluta de una curva parametrizada regular ! r (t) ,con curvatu-
ra no nula, al lugar geométrico de los centros de curvatura. Denotamos
la evoluta de ! r (t) por !
c (t).
(i) Encontrar una parametrización de ! c (t).
✓ 2◆
! t
(ii) Hallar la evoluta de la parábola r (t) = t, .
2
⇣p p p ⌘
!
(iii) Hallar la evoluta de la hélice r (s) = 2 cos(s), 2 sen(s), 22 s ,
2 2

s 2 IR y comprobar que es regular y que s es su parámetro arco.


Solución:
(i) Si !
r (t) es punto de la curva y !
c (t) su correspondiente centro de
curvatura para t entonces la condición que define el lugar geométrico
1 !
se tiene la relación !
c (t) !
r (t) = N (t) de donde se deduce que
k(t)
la
ecuación de la evoluta es

! 1 !
c (t) = !
r (t) + N (t)
k(t)

(ii) Calculamos en primer lugar el vector normal y la curvatura de la


parábola.
✓ 2◆
! t p
r (t) = t, =) ! r 0 (t) = (1, t) =) |!
r 0 (t)| = 1 + t2 =)
2
! ( t, 1)
N (t) = p
1 + t2

Por otra parte


! |!
r 0 (t) ⇥ ! r ” (t)| 1
r ” (t) = (0, 1) =) k(t) = ! 3 = p 3
| r 0 (t)| 1 + t2
Por lo tanto
✓ 2◆ p 3 ✓ ◆
! t 1 + t2 3 3 2
c (t) = t, + p ( t, 1) = t ,1 + t
2 1 + t2 2
iii) Calculamos el vector tangente a la curva a partir de su definición
⇣p p p ⌘ ⇣ p p p ⌘
! 2 2 2 !
r (s) = 2 cos(s), 2 sen(s), 2 s =) r (s) = 0 2
sen(s), 2 cos(s), 22
2
2

160
=) |!
r 0 (s)| = 1
⇣ p p p ⌘
Por tanto Tb (s) = 2
2
sen(s), 2
2
cos(s), 2
2

Por otra parte,


⇣ p p ⌘ p
Tb0 (s) = !
r 00 (s) = 2
2
cos(s), 2
2
sen(s), 0 =) |!
r 00 (s)| = 2
2

de donde
!r 00 (s)
b
N (s) = !00 = ( cos(s), sen(s), 0)
| r (s)|
p
Además k(s) = |! r 00 (s)| = 2 2

Finalmente
1 ! ⇣ p p p ⌘
!c (s) = !
r (s) + N (s) = 2
cos(s), 2
sen(s), 2
s , s 2 IR
2 2 2
k(s)
⇣p p p ⌘
lo que implica !
0
c (s) = 22 sen(s), 22 cos(s), 22 s , y por tanto que

|!
c 0 (s)| = 1 para cada s 2 IR y en definitiva que !
c es regular y
está
parametrizada por arco s.
Ejercicio 17
Encontrar una parametrización de las siguientes curvas.
(i) y = x2 + 3x
(ii) x2 + y 2 + 2y = 0
y2
(iii) x2 =1
4
(iv) x2 + 3y 2 = 1
Calcular en cada caso los vectores tangente unitario Tb, normal N
b , la
curvatura  (t) y la torsión ⌧ (t)
Solución:

(i) Como se trata de una parábola podemos parametrizarla tomando


x = t =) y = t2 + 3t
Ası́ !r (t) = (t, t2 + 3t), y calculamos el vector tangente, !
r 0 (t) =
(1, 2t + 3) p
=) k! r 0 (t)k = 4t2 + 12t + 10

161
!r 0 (t) (1, 2t + 3)
y Tb (t) = !0 =p
k r (t)k 4t2 + 12t + 10

Para calcular el vector normal en IR2 tenemos en cuenta que es un


vector
ortogonal al tangente y unitario.
b (t) = p ( 2t 3, 1)
N
4t2 + 12t + 10
|!
r (t) ⇥ !r ” (t)|
La curvatura de una curva plana es k(t) = ! 3 , con-
0
| r (t)|
siderando
que IR2 es subespacio de IR3
En nuestro caso !r 0 (t) = (1, 2t + 3, 0) =) ! r 00 (t) = (0, 2, 0)
2
Por tanto, k(t) = ⇥p ⇤3 , la curvatura de la parábola varı́a
4t2 + 12t + 10
en
función de t.
Finalmente, la torsión de la curva plana es
!
r 0 (t) ⇥ !r ” (t) · !
r 000 (t)
⌧ (t) = ! 3 = 0, pues !
r 000 (t) = (0, 0, 0)
0
| r (t)|
lo que significa que la curva siempre está en el mismo plano.
(ii) La ecuación dada representa una circunferencia de centro el (0, 1)
y
radio r = 1, ya que x2 + y 2 + 2y = x2 + (y + 1)2 1.
Por tanto, una parametrización es !
r (t) = (cos(t), 1 + sen(t))
Calculamos el vector tangente.
!
r 0 (t) = ( sen(t), cos(t)) =) k! r 0 (t)k = 1 =)
!
r 0 (t)
b
T (t) = !0 = ( sen(t), cos(t))
k r (t)k
Para calcular el vector normal tenemos en cuenta que es un vector
ortogonal
al tangente y unitario.
Nb (t) = ( cos(t), sen(t))

162
|!
r 0 (t) ⇥ ! r ” (t)|
La curvatura de una curva plana es k(t) = ! 3 , con-
| r 0 (t)|
siderando
que IR2 es subespacio de IR3
En nuestro caso !
r 0 (t) = ( sen(t), cos(t), 0) =) !
r 00 (t) = ( cos(t), sent(t), 0)
=) |!r (t) ⇥ !r ” (t)| = 1
Por tanto, k(t) = 1 la curvatura de una circunsferencia es constante
Finalmente, la torsión de la curva plana es
!
r 0 (t) ⇥ !r ” (t) · !
r 000 (t)
⌧ (t) = ! 3 = 0,pues !
r 0 (t) ⇥ !
r ” (t) · !
r 000 (t) =
0
| r (t)|
(0, 0, 0).
(iii) La ecuación dada representa una hipérbola de semieje a = 1 y b
= 2.
Por tanto, una parametrización es !r (t) = (cosh(t); 2senh(t))
Calculamos el vector tangente.! r 0 (t) = (senh(t), 2 cos h(t))
p
=) k! r 0 (t)k = senh2 (t) + 4 cos h2 (t)

!r 0 (t) (senh(t), 2 cos h(t))


y Tb (t) = !0 =p
k r (t)k senh2 (t) + 4 cos h2 (t)
Para calcular el vector normal tenemos en cuenta que es un vector
ortogonal
al tangente y unitario.

Nb (t) = p( 2 cos h(t), senh(t))


senh2 (t) + 4 cos h2 (t)
|!
r 0 (t) ⇥ !r ” (t)|
La curvatura de una curva plana es k(t) = ! 3 , con-
0
| r (t)|
siderando que IR2 es subespacio de IR3
En nuestro caso :! r 0 (t) = (senh(t), 2 cos h(t), 0) =) !
r 00 (t) =
(cosh(t); 2senh(t), 0)
=) |! r 0 (t) ⇥ !
r ” (t)| = 2
2
Por tanto k(t) = hp i3 , la curvatura de la hipérbo-
2 2
senh (t) + 4 cos h (t)
la

163
varı́a en función de t.
Finalmente, la torsión de la curva plana es
!
r 0 (t) ⇥ !r ” (t) · !
r 000 (t)
⌧ (t) = ! 3 = 0, pues !
r 0 (t) ⇥ !
r ” (t) · !
r 000 (t) =
0
| r (t)|
(0, 0, 0)
(iv) La ecuación dada representa una elipse de centro el (0, 0)y semiejes
a=1
1
y b= p
3
1
Entonces, una parametrización es ! r (t) = (cos(t), p sen(t))
3
Calculamos el vector tangente.
p
! 1 ! 3p
r 0 (t) = ( sen(t), p cos(t)) =) k r 0 (t)k = 2sen2 (t) + 1 =)
3 3
! p
b r 0 (t) 3 1
T (t) = !0 =p ( sen(t), p cos(t))
k r (t)k 2sen2 (t) + 1 3
Para calcular el vector normal tenemos en cuenta que es un vector
ortogonal
al tangente y unitario.
p
b (t) = p 3 1
N ( p cos(t), sen(t))
2sen2 (t) + 1 3
|!
r 0 (t) ⇥ ! r ” (t)|
La curvatura de una curva plana es k(t) = ! 3 , con-
| r 0 (t)|
siderando
que IR2 es subespacio de IR3 .
En nuestro caso
! 1 1
r 0 (t) = ( sen(t), p cos(t), 0) =) ! r 00 (t) = ( cos(t), p sent(t), 0)
3 3
1
=) |! r (t) ⇥ !
r ” (t)| = p
3
3
Por tanto, k(t) = hp i3 la curvatura de la elipse varı́a en
2
2sen (t) + 1
función de t.

164
Finalmente, la torsión de la curva plana es
!
r 0 (t) ⇥ !
r ” (t) · !r 000 (t)
⌧ (t) = =0
|!
3
r 0 (t)|

pues !
r 0 (t) ⇥ !
r ” (t) · !
r 000 (t) = (0, 0, 0)
Ejercicio 18
Sean !r : I ! IR3 una curva regular y consideremos ! c la evoluta de !
r
!
. Demostrar que c es regular si y sólo si la torsión y la derivada de la
curvatura de !r no se anulan simultáneamente en ningún punto y en
este caso demostrar que el parámetro arco de ! c , s⇤ satisface que
q
1
(k 0 (t))2 + (k (t))2 (⌧ (t))2 |!
⇤ 0 2
(s (t)) = 2 r 0 (t)| 8t 2 I
k (t)

Solución:
La evoluta de está definida como la curva
! 1 ! ! 1
c (t) = !
r (t) + N (t) = !
r (t) + ⇢ (t) N (t), donde ⇢ (t) = es el
k(t) k(t)
radio
de curvatura de ! r.
Por lo tanto, si s denota el parámetro arco de ! r tenemos que s (t) =
|!r 0 (t)|
! ! ! !
c 0 (t) = ! r 0 (t) + ⇢ (t) N (t) + ⇢ (t) N 0 (t) = !
0 0
r 0 (t) + ⇢ (t) N (t) +
!
⇢ (t) N 0 (s) s0 (t)
Por Frenet sabemos que:
!0 b (s) =) !
N (s) = k (s) Tb (s) ⌧ (s) B N 0 (s) = ⇢ (s) Tb (s)
⌧ (s) B b (s)
! h i
!c 0 (t) = |! r 0 (t)| Tb (t)+⇢ (t) N (t)+⇢ (t) s0 (t) Tb (s) ⇢ (s) ⌧ (s) B
0
b (s)
!
=) ! c 0 (t) = ⇢ (t) N (t) ⇢ (t) ⌧ (t) |! b (t) .
0
r 0 (t)| B
p
Ası́ |! c 0 (t)| = (⇢0 (t))2 + ⇢2 (t) ⌧ 2 (t) |!r 0 (t)|)2 puesto que el Triedro
de
Frenet es ortonormal. Sustituyendo ⇢ y ⇢0 por su expresión en función
de la curvatura, resulta que
s
q
! (k 0 (t))2 ⌧ 2 (t) !0 2 1
(k 0 (t))2 + k 2 (t) ⌧ 2 (t) |!
0 2
| c (t)| = 4
+ 2 | r (t)| = 2 r 0 (t)|
(k (t)) k (t) k (t)

165
!
para cada t 2 I y por tanto, como ⇢ (t) y |!
r 0 (t)| 6= 0 c es regular
en t 2 I
si y sólo si k 0´ (t) = ⌧ (t) = 0.
La expresión anterior muestra también que cuando !
c es regular, la
derivada
de su parámetro arco coincide con el término de la derecha de la igual-
dad
Ejercicio 19
Consideremos la curva C dada por !
r (t) = (et , e2t , t), t 2 R.
Hallar su curvatura y su torsión en el punto (1, 1, 0). ¿Es cierto qué la
curva
tiene torsión negativa en todos sus puntos?
Solución:
Calculamos primero las derivadas:
! p
r 0 (t) = (et , 2e2t , 1) ) k!
r 0 (t)k = 1 + e2t + 4e4t
!
r 00 = (et , 4e2t , 0)
! p
r 0⇥! r 00 = ( 4e2t , et , 2e3t ) ) !
r 0⇥!
00
r = e2t + 16e4t + 4e6t
! 000
r = (et , 8e2t , 0)
Y podemos obtener la curvatura y la torsión en cada punto:
! p
r 0⇥!
00
r e2t + 16e4t + 4e6t
k(t) = = ⇥p ⇤3
k!
3
r 0 (t)k 1 + e2t + 4e4t
!
r 0⇥! r ·!
00
r
000
4e3t
⌧ (t) = ! !00 2 = ⇥p ⇤2
kr0⇥ r k e2t + 16e4t + 4e6t

En el punto (1, 1, 0) = (et0 , e2t


0 , t0 ) =) t = 0

De donde deducimos que la curvatura y la torsión son


! p
r 0 (0) ⇥ !
00
r (0) 21
k(0) = =
k!
3
r 0 (0)k 63/2
!
r 0 (0) ⇥ ! r (0) · !
00 000
r (0) 4
⌧ (0) = ! ! 2 =
0
k r (0) ⇥ r (0)k 00
21
Por último, la función ⌧ (t) es claramente positiva, 8 t 2 IR

Ejercicio 20

166
Considere la curva C dada por !
r (t) = (cosh(t), sinh(t), t), t 2 IR
(a) Dibuje aproximadamente su traza.
(b) Si se recorre la traza partiendo desde punto (1, 0, 0), tras recorrer
una
p
longitud de arco 2 sobre la curva, ¿cuáles son las coordenadas del
punto del espacio en el que nos encontraremos?
Solución:
Consideremos la proyección de la curva sobre el plano XY :
x2 y 2 = cos h2 (t) sinh2 (t) = 1 se trata de una rama de hipérbola,
cuyo
eje de simetria es X, toma valores positivos de X y se recorre en el
sentido
creciente de Y .
La tercera componente es simplemente z = t, para valores t > 0 la
curva
se eleva con respecto al plano z = 0 hacia, mientras que para valores
de
t < 0, “baja” con respecto el plano z = 0.

Determinemos el valor del parámetro en el punto de partida


(1, 0, 0) = (cosh(t), sinh(t), t) =) t = 0
Empezamos en s(0), la longitud de la curva, desde ese punto, se mide
la
longitud de arco.
Calculemos la rapidez, y luego la longitud del arco

167
!r (t) = (cosh(t), sinh(t), t), t 2 IR ) !
r 0 (t) = (senh(t), cos h(t), 1), t 2
IR
p p p
=) k! r 0 (t)k = cosh2 (t) + sinh2 (t) + 1 = 2cosh2 (t) = 2cosh(t)
.
Ası́ que la longitud desde s(0) hasta
Rt p Rt p p
s(t) = 0 k! r 0 (u)k du = 2 0 cosh(u)du = 2 senh(u)|t0 = 2senh (t)
Ahora sólo queda determinar el valor de t que hace que la longitud sea
p
exactamente 2. Esto es, resolver
p p
2 = 2sinh(t) ) 1 = sinh(t) ) t = arcsinh(1).
De manera que estaremos en el punto de coordenadas
!
r (arcsinh(1)) = (cosh(arcsinh(1)), sinh(arcsinh(1)), arcsinh(1)) =)

! p
r (arcsinh(1)) = ( 2, 1, arcsinh(1))
Ejercicio 21
Consideremos la curva C dada por !
r (t) = (t, t2 , t3 ) 2 R.
Hallar su curvatura y su torsión en el punto (0, 0, 0). ¿En qué punto
tiene
la curva una torsión (en valor absoluto) máxima?
Solución:
Calculamos primero las derivadas:
! p
r 0 (t) ⇥ !
00
r (t) 4 + 36t2 + 36t4
k(t) = = ⇥p ⇤3
k!
3
r 0 (t)k 1 + 4t2 + 9t4
!
r 0⇥! r ·!
00
r
000
12
⌧ (t) = ! !00 2 =
0
kr ⇥ r k [4 + 36t2 + 36t4 ]2
De donde deducimos que la curvatura y la torsión son
!r 0 (0) ⇥ !
00
r (0)
k(0) = =2
k!
3
r 0 (0)k
!
r 0 (0) ⇥ ! r (0) · !
00 000
r (0)
⌧ (0) = =3
k! r 0 (0) ⇥ !
2
r 00 (0)k
Como el denominador que aparece en la expresión de ⌧ (t) es siempre
una cantidad positiva que toma su valor mı́nimo en t = 0, concluimos
que el valor absoluto de la torsión alcanza su valor máximo en t = 0.

168
2.17. Ejercicios propuestos

1. Sean f~, ~g : [a, b] ! Rn funciones derivables, probar que:

a) (↵f~ + ~g )(t) = ↵f~ (t) + ~g (t) , ↵ y constantes


b) (↵f~)0 (t) = ↵0 (t) f~ (t) + ↵(t)f 0 (t) si ↵ = ↵(t) es una función escalar.
⇣ ⌘0
c) f · ~g (t) = f~0 (t) · ~g (t) + f~(t) · ~g 0 (t)
~
⇣ ⌘0
~
d) f ⇥ ~g (t) = (f 0 ⇥ g) (t) + (f ⇥ g 0 ) (t)

2. Para f~(t) = (e2t , t2 , e 2t


) calcular:

a) f~0 (t)
b) f~00 (t)
c) f~0 (t)

3. Obtener una función paramétrica vectorial de la curva resultante


al intersectar las superficies:

x2 + y 2 + z 2 = 2
p
yz= 2 (1 + y)

169
4. Parametrizar en la forma ~r0 = ~r(t) = (x(t), y(t), z(t)), t 2 [a, b] las curvas C
definidas por:

a) Recta por A = (1, 2, 3) y B = (2, 2, 2)


b) Recta intersección de los planos x + y + z = 0 y y z=0

c) Intersección del cilindro recto x2 + y 2 = 9 y el plano z = 1

d) El cilindro x2 ax + y 2 = 0 intersección la esfera x2 + y 2 + z 2 = a2

5. Calcular la longitud l de la curva descrita por la trayectoria ~r(t) =


(a cos3 t, a sin3 t), t 2 [0, 2⇡]
6. Una hormiga camina sobre la superficie S : x(u, v) = sin u cos v, y (u, v) =
sin u sin v, z (u, v) = cos u

6
 u  ⇡2 , ⇡
2
 v  ⇡ en el instante t = 0, se mueve de tal
forma
⇣p que en cualquier
⌘ tiempo t > 0 describe la trayectoria !
r (t) =
p
3 1 3
2
sin t, 2 , 2 cos t .
Demuestre que la hormiga se mueve sobre la superficie esferica S y
determine cuando y en donde la hormiga abandona el sector descrito
de la esfera

170
7. Halle el o los valores de t para los cuales el vector tangente a la
curva descrita por r(t) = (2t2 + 1, 3t 2), t 2 R sea paralelo al
vector ~u = (2, 1).
8. Calcule
2 3 4
x2 x3 x4
lı́m( (x+t)t , (x+t)t , (x+t)t )
t!0

9. Discuta la continuidad de la función


(t, sint t ) si t 6= 0
f (t) = {
(0, 1) si t = 0
! ! !
10. Demuestre que: si f (t) es constante, entonces f y f 0 son or-
togonales.
11. Encuentre un vector tangente y la recta tangente a la hélice cónica
de representación paramétrica

! t
f (t) = (t cos t, t sin t, )
2⇡
en los puntos (0, 0, 0) y (0, ⇡2 , 14 )
!
12. Sea la hélice descrita por f (t) = (cos t, sin t, t) t 2 [0, ⇡2 ]. Pruebe
!
que en ningún punto de esta curva f 0 (t) es paralelo a la cuerda
! !
de f (0) a f ( ⇡2 )
13. Una partı́cula se mueve en el plano a lo largo de la espiral ⇢ = e✓
con una rapidez constante de 5 pies por segundo.

171
a) ¿Cuáles son la velocidad y la aceleración de la partı́cula cuando
✓ = ⇡4 ?
b) ¿Cuánto tarda la partı́cula en ir desde el punto correspondiente a
✓ = 0 hasta el punto correspondiente a ✓ = ⇡?
c) Si ✓ = 0 cuando t = 0, proporcione ecuaciones paramétricas para la
trayectoria de la partı́cula.

14. Determine los puntos en que la recta tangente a la curva de R3


dada por

~r(t) = 6t + 2t3 ı̂ + (6t 2t3 )ˆ


| + 3t2 k̂

es paralela al plano x + 3y + 2z = 5
!
15. Demuestre que el camino descrito por f : [ 2, 2] ! R2 definida
!
por f (t) = (t2 1, t3 + 2t2 t 2) no es simple. ¿Es cerrado?¿Es
cerrado simple?
16. Determine las ecuaciones de la recta tangente y del plano nor-
!
mal a la curva descrita por f : I ! R3 definida por f (t) =
!
(et sint, et cost, 5t) en el punto P = f (0)
! !
17. Sea f : [0, 5] ! R3 la trayectoria f (t) = (sin t, cos t, t). Obtenga
! !
una reparametrización f c de f que conserve su orientación y que
!
recorra la trayectoria de f en la quinta parte del tiempo en el que
!
lo hace f .
18. Un objeto ubicado inicialmente en el origen de coordenadas se
desplaza de acuerdo al movimiento

~r(t) = (t2 , t cos t, t sin t)

a) Determine el punto en que el objeto impacta a la esfera x2 +y 2 +z 2 =


2
b) Calcular la longitud recorrida por el objeto hasta el instante del
impacto.
c) Calcular el ángulo entre la trayectoria y la esfera en el instante del
impacto.

172
19. El movimiento de una partı́cula se realiza en el cı́rculo unitario del
plano XY de acuerdo con la fórmula (x, y, z) = (cos t2 , sin t2 , 0) ,
t 0.

a) Como funciones de t. ¿Cuál es el vector velocidad y la rapidez de la


partı́cula?
b) ¿En qué punto del cı́rculo se debe liberar la partı́cula para que
golpee un blanco localizado en (2, 0, 0)?
c) ¿En que tiempo t se debe liberar? (usar t 0 más pregunta b))
d) ¿Cuál es su velocidad y rapidez en el instante en que se libera?
e) ¿En qué instante se golpea el blanco?

20. Si f : I ! IR es una función diferenciable, demostrar que la


gráfica de f , es decir la curva y = f (x), es una curva regular y
calcular sus vectores tangente , normal, su curvatura, su radio de
curvatura, y su centro de curvatura.
21. Sea C una curva de IR3 dada por la ecuación! r (t) = (4 cos(t), 4sen (t) , 4 cos t(t)), t 2
[0, 2⇡]

a) Verificar que es una elipse.


b) Determinar los vectores Tb, Nb, B
b en cualquier punto C
c) Calcular la curvatura y torsión de C en todo punto.
d) Determinar en qué puntos de C la curvtura k es máxima y en cuáles
es mı́nima

2.17.1. Respuestas
2.

a) f~0 (t) = 2 (e2t , t, e 2t )


b) f~00 (t) = 2(2e2t , 1, 2e 2t )
p
c) f~0 (t) = 2 e4t + e 4t + t2
p
3. ~r(t) = (cos t, 1 + sin t, 2 sin t), 0  t  2⇡
4. a) ~r(t) = (1 + t, 2, 3 + 5t) t2R

173
b) ~r(t) = ( 2t, t, t) t2R
c) ~r(t) = (3 cos t, 3 sin t, 1) t 2 [0, 2⇡]
p
d) ~r(t) = ( a2 + a2 cos t, a2 sin t, 2a sin 4t ) t 2 [0, 2⇡]
p
3 1
5. 6a, 6 t = ⇡2 , (x, y, z) = ( , , 0)
2 2
3
7. t = 2
8. (2x, 3x2 , 4x3 )
9. Es continua
! !
10. f · f 0 = 0
11. 1, 0, 2⇡1
, ! 1
r (t) = (t, 0, 2⇡ t)
⇡ 1
(0, 2 , 4 ), !
r (t) = ( 2 t, 2 + t, 14 + t 2⇡
⇡ ⇡ 1
)
13.

a) ~v = (0, 5), ~a = ( p252 e 4 , 0)
p
b) 15 2(e⇡ 1)
!
c) f (t) = [ p52 t + 1](cos ln( p52 t + 1), sin ln( p52 t + 1))

14. ~r(2) = (28, 4, 12)


! !
15. No es simple f (1) = (0, 0) = f ( 1)
16. ~r(t) = (t, 1 + t, 5t), x + y + 5z = 1 plano normal
17. f¯ : [0, 1] ! R3 , f¯ (s) = (sin(5s), cos(5s), 5s)
18.

a) (1, cos1, sin1)


b) 1,57

19.

a) ~v = ( 2t sin t2 , 2t cos t2 , 0); s = 2t


p
3
b) ( 12 , 2
, 0)
q
5⇡
c) 3
q p q
5⇡ 3 1 5⇡
d) v = 2 3
( , , 0);
2 2
s=2 3
p
3 p3+10⇡
e) 2 15⇡

174
20. La gráfica de la función es la curva !r : I ! IR2 determinada por
!r (x) = (x, f (x))
p
Como ! r 0 (x) = (x, f 0 (x)), resulta que |!r 0 (x)| = 1 + (f 0 (x))2 >
0 8x 2 I y por tanto la curva es regular
(1, f 0 (x)) ! ( f 0 (x) , 1)
Tb (x) = p y N (x) = p
1 + (f 0 (x))2 1 + (f 0 (x))2
p
|f ” (x)| ( 1 + (f 0 (x))2 )3
k(x) = p y R(x) =
( 1 + (f 0 (x))2 )3 |f ” (x)|

! (1 + (f 0 (x)2 )
c (x) = (x, f (x)) + ( f 0 (x) , 1)
|f ” (x)|

21.

A partir de las ecuaciones paramétricas, se tiene que:


x (t) = 4 cos(t)
y (t) = 4sen (t) =) x2 + y 2 = 16
z (t) = 4 cos(t) z=x
( 4sen (t) , 4 cos (t) , 4sen (t)) b ( 16, 0, 16)
Tb (t) = p , B (t) = p
16 + 16sen2 (t) 16 2
b (t) ⇥ Tb (t) = (cosp
b (t) = B
N
(t) , 2sen (t) , cos (t))
p
2 1 + sen2 (t)
p
16 2
k(t) = hp i3 , ⌧ (t) = 0
43 1 + sen2 (t)

La curvatura es máxima si t = 0, ⇡ =) P0 = (4, 0, 4)


⇡ 3⇡
La curvatura es mı́nima si t = , =) P1 = (0, 4, 0)
2 2

2.18. Auto Evaluaciones


Autoevaluación No 1
El estudiante:
1) Calculará ecuación cartesiana de la trayectoria, curvatura y torsión
en un punto de la curva, dada la ecuación vectorial de una curva regular,

175
dos veces diferenciable, con ecuaciones paramétricas en función de un
parámetro cualquiera t,
2) Determinará la ecuación de la recta tangente y del plano osculador
!
a la curva descrita por la trayectoria f : I ✓ R ! R3 en un punto
de la curva.
3) Resolverá problemas de dinámica de partı́culas querequiera calcular
longitud de una curva, curvatura máxima , torsión de la curva, ecua-
ciones de rectas ,y planos principales en un punto dado de la curva.

Tiempo : 2 horas

Tiempo : 2 horas

Pregunta 1
Sea la hélice !
r (t) = (a cos t, a sin t, bt) , b 6= 0, determine:
1 2
a) los valores de a y b si su curvatura es  (t) = y su torsión ⌧ (t) = .
5 5
b) La ecuación del cilı́ndro sobre el cuál se encuentra la hélice.

Pregunta 2
✓ ◆ ✓ ◆
t3 2 8
Para la curva C dada por: !
r (t) = , 2t, en P0 = , 4, 1
3 t 3
Obtener las ecuaciones vectoriales y cartesianadas de:
a) Recta tangente.
b) Planos osculador.

Pregunta 3
En el instante t= 0 , una casa es lanzada al espacio por causa de un
tornado siguiendo la trayectoria
!
r (t) = eat cos t, eat sin t, beat b

con a > 0, b > 0, t 0.


a) Calcular la distancia que recorre la casa hasta el instante t = ⇡.
b) En el instante t1 con t1 2 [0, ⇡] , en el cual la trayectoria tiene
curvatura máxima, un residente que dormı́a tranquilo es lanzado

176
hacia el exterior.
i) ¿qué trayectoria sigue el residente?
ii) ¿en qué punto se encuentra la casa en t = ⇡?
iii) Obtener la ecuación de plano osculador a la trayectoria de la casa
en t = ⇡.

Pauta de Autocorrección

Pregunta 1
En primer lugar determinemos la curvatura la torsión en función del
tiempo
! p
r 0 (t) = ( a sin t, a cos t, b) 8t 2 IR =) k!
r 0 (t)k = a2 + b2
! 00
r (t) = ( a cos t, a sin t, 0)
!r 000 (t) = ( a sin t, a cos t, 0)
!rp0 (t) ⇥ ! r 00 (t) = (ab sin t, ab cos t, a2 ) =) k! r 0 (t) ⇥ !
r 00 (t)k =
a a2 + b 2
!r 0 (t) ⇥ !r 00 (t) · !
r 000 (t) = a2 b sin2 t, a2 b cos2 t, 0
Entonces, se tiene
!
r 0 (t) ⇥ !
00
r (t) a
 (t) = = ,y
k!
3/2 a2 + b 2
r 0 (t)k
!
r 0 (t) ⇥ ! r (t) · !
00
r 000 (t) b
⌧ (t) = =
k!
3/2 a + b2
2
r 0 (t)k
Luego, a partir de los valores dados de curvatura y torsión, obtenemos
a 1 b 2
= , = =) a = 1, b = 2
a2 + b 2 5 a2 + b 2 5

Por tanto la ecuación de la hélice es


!
r (t) = (cos t, sin t, 2t)

b) A partir de la ecuaciones paramétricas de la hélice , se tiene


x = cos t, y = sin t, z = 2t 0 =) x2 + y 2 = 1, z 0, corresponde a
un

177
cilindro de radio r = 1, cuyo eje axial coincide con el eje z.

Pregunta 2
a) La ecuación de la recta tangente al punto P0 se define por
!
f ( )=! r (t0 ) + ! r 0 (t0 ) , con 2 IR
✓ 3 ◆ ✓ ◆
! t0 2 8
siendo r (t0 ) = , 2t0 , = , 4, 1 =) 2t0 = 4 () t0 = 2
3 t0 3
Calculemos ahora la dirección de la recta tangente
✓ ◆ ✓ ◆
! 0 2 2 ! 0 1
r (t) = t , 2, 2 =) r (t0 ) = 4, 2,
t 2
Sustituyendo términos en la ecuación de la recta tangente, obtenemos
✓ ◆ ✓ ◆
8 1
(x, y, z) = , 4, 1 + 4, 2, , 2 IR
3 2

Eliminando el parámetro , tenemos la ecuación cartesiana


8
x
3 =y 4
=
z 1
1
4 2 2

b) La ecuación del plano osculador al punto P0 se define por


⇣! ⌘ ! !
f (t) r (t0 ) · B (t0 ) = 0, siendo B (t0 ) = !
! r 0 (t0 ) ⇥ !
r 00 (t0 ) el
vector
normal al plano osculador en el punto P0 .
✓ ◆ ✓ ◆
! 0 2 2 ! 00 4
Como r (t) = t , 2, 2 =) r (t) = 2t, 0, 3 ()
t t
✓ ◆
! 00 1
r (t0 ) = 4, 0,
2
.
De manera que
i j k
!
B (t0 ) = !
r 0 (t0 ) ⇥ !
r 00 (t0 ) = 4 2 1
2
= (1, 4, 8)
1
4 0 2
Por tanto la ecuación de plano, queda

178
 ✓ ◆
8
(x, y, z) , 4, 1 · (1, 4, 8) = 0
3
3x 12y 24z = 64

Pregunta 3
a) Derivando la ecuación de la trayectoria se tiene

!
r 0 (t) = aeat cos t eat sin t, aeat sin t + eat cos t, abeat
= eat (a cos t sin t, a sin t + cos t, ab)

Calculando la norma de este vector obtenemos


p
k!
r 0 (t)k = eat a2 + 1 + a2 b2

Entonces, la longitud de la curva es


Z ⇡ ✓ a⇡ ◆
! ! 0
p e 1
l( r ) = 2 2
k r (t)k dt = a + 1 + a b 2
0 a

!
r 0 (t) ⇥ !
00
r (t)
b) La curvatura está dada por la expresión  (t) =
k!
3/2
r 0 (t)k
Derivando por segunda vez,
! 00
r (t) = aeat (a cos t sin t, a sin t + cos t, ab)
+eat ( a sin t cos t, a cos t sin t, 0)

=) !
r 0 (t)⇥!
00
r (t) = e2at (ab(sin t a cos t), ab(a sin t + cos t, a2 + 1)
p p
=) !r 0 (t) ⇥ !
00
r (t) = e2at a2 + 1 a2 + 1 + a2 b2
Luego, obtenemos
p
a2 + 1
 (t) =
eat (a2 + 1 + a2 b2 )

Por tanto,  (t) es máximo para t1 = 0, cuando se tiene el valor mı́nimo


del
denominador

179
i) El residente sale despedido por la recta tangente a la trayectoria del
tornado, en el instante t1 = 0, luego la ecuación de la recta será
!
R (0) = !
r (0) + !
r 0 (0) = (1, 0, 0) + (a, 1, ab)

ii) En el instante t = ⇡ la casa se encuentra en el punto


!
r (⇡) = ( ea⇡ , 0, bea⇡ b)

iii) El plano osculador a la trayectoria de la casa en t1 = ⇡ es


! ! !
(R r (⇡)) · B (⇡) = 0

donde el vector binormal se determina por


!
B (⇡) = !
r 0 (⇡) ⇥ !
00
r (⇡) = e2⇡a a2 b, ab, a2 + 1

Por tanto, la ecuación que produce es

(x ea⇡ , y, z (bea⇡ b)) · e2⇡a a2 b, ab, a2 + 1 = 0

Autoevaluación No 2

Tiempo : 2 horas
Pregunta 1
Una partı́cula se desplaza a partir del instante t = 0 siguiendo la trayec-
toria:
! ⇡
r (t) = (t, ln (sec t + tagt) , ln sec t) , 0  t <
2
Determinar la velocidad y rapidez en el instante que ha recorido una
p
2
distancia igual .
2
Pregunta 2
Dada la curva C definida por las superficies x2 +y 2 +z 2 = 11, x2 +y 2 = 2,
p
z > 0, determinar en el punto 0, 2, 3 ;
a) la ecuación del plano osculador
b) los versores tangente, normal y binormal

180
Pregunta 3
Para la curva C dada por !
r =!
r (s) ,donde s es el parámetro longitud
de
arco, deducir las fórmulas:
000
a) !
r 0 (s) · !
r (s) = 2 (s)
b) !r 0 (s) · !
r (s) ⇥ !
00 000
r (t) = 2 ⌧

c) Usar las fórmulas anteriores para calcular , ⌧ de la curva C


✓ ◆
! 4 3
r (s) = cos s, 1 sin s, cos s
5 5

d) Identificar la curva C

Pauta de Autocorrección

Pregunta 1
Derivando la ecuación de la trayectoria:

! ⇡
r (t) = (t, ln (sec t + tagt) , ln sec t) , 0  t <
2

obtenemos la velocidad

! ⇡
r 0 (t) = (1, sec t, tagt) , 0  t <
2
En consecuencia la rapidez de la partı́cula, queda

! p p
r 0 (t) = 1 + sec2 t + tag 2 = 2 sec t

Entonces, la distancia recorrida en función del tiempo, es


p ✓ ◆
p Z t 2 1 + sin t
s (t) = 2 sec udu = ln
0 2 1 sin t

A continuación determinemos el tiempo t0 , que corresponde a la dis-


tancia

181
p
2
:
2
p ✓ ◆ p
2 1 + sin t0 2
ln =
2 1 sin t0 2
✓ ◆
1 + sin t0
ln = 1
1 sin t0
✓ ◆
e 1
t0 = arcsin ⇡ 0, 48
e+1

Por lo tanto la velocidad en ese tiempo es


!
v (t0 ) = !
r 0 (t0 ) = (1, sec (0,48) , tag (0, 48)) = (1; 1, 127; 0, 521)

y la rapidez
q
k!
r 0 (t0 )k = 1 + (1, 127)2 + (0, 521)2

Pregunta 2
a) Determinemos la ecuación de la curva C, sustituyendo la segunda
ecuación
en la primera, obtenemos 2 + z 2 = 11 =) z 2 = 9, z = ±3. Como
z > 0, la curva
dada es la circunsferencia x2 + y 2 = 2, z = 3.
p p
La ecuación paramétrica de la curva es ! r (t) = 2 cos t, 2 sin t, 3 ,
y el
p p
valor del parámetro en el punto es ! r (t0 ) = 2 cos t0 , 2 sin t0 , 3 =
p
0, 2, 3
p
=) 2 cos t0 = 0 =) t0 = ⇡2 .
Calculemos ahora la ecuación del plano osculador, derivando, obten-
emos
! p p p
r 0 (t) = 2 sin t, 2 cos t, 0 =) ! r 0 ⇡2 = 2, 0, 0
! 00 p p ! p
r (t) = 2 cos t, 2 sin t, 0 =) r 00 ⇡2 = 0, 2, 0
El vector normal al plano en el punto dado es
i
p j k
!
r0 ⇡
⇥!
r 00 ⇡
= 2 0 0 = (0, 0, 2)
2 2 p
0 2 0

182
La ecuación del plano osculador viene dada por
h! i ! ⇥ p ⇤
f !
r ⇡2 · N ⇡2 = 0 () (x.y.z) 0, 2, 3 · (0, 0, 2) =)
z=3
b) El versor tangente, en el punto es
! p
r 0 ⇡2 0, 2, 0
b
t = !0 ⇡ = p = (0, 1, 0) .
r 2 2
El versor binormal viene dado por
!0 ⇡ !00 ⇡
bb = r 2 ⇥ r 2 =
(0, 0, 2)
= (0, 0, 1) .
!
r 0 ⇡
⇥ r! 00 ⇡
2
2 2
Finalmente el versor normal es
b = bb ⇥ b
n t = (1, 0, 0) .
Pregunta 3
a) A partir de !r =!
r (s) tenemos que
!r (s) = Tb (s)
0

derivando por segunda vez y aplicando la identidad de Frenet, queda


! 00
r (s) = Tb (s) = N
0
b (s) =)
000
! 0
r (s) =  Nb (s) + N
b 0 (s)
Utilizando nuevamente la identidades de Frenet , obtenemos
000 ⇣ ⌘
! 0
b b
r (s) =  N (s) +  ⌧ B T =) b
000
! 0
r (s) =  Nb (s) + ⌧ B
b 2 Tb
Calculando el producto interno y considerando la ortonormalidad de la
base
n o
Tb, N
b, B
b , produce

000 ⇣ 0 ⌘
!
r (s) · !
0
r (s) = Tb (s) ·  Nb (s) + ⌧ B
b 2 Tb = 2

00 000 ⇣ ⌘
!
r (s) · !
0
r (s) ⇥ ! b (s) ⇥ 0 N
r (s) = Tb (s) · N b (s) + ⌧ B
b 2 Tb
= 2 ⌧

b) A partir de !
r (s) = 4
5
cos s, 1 sin s, 3
5
cos s =)

183
! 0
r (s) = 4
sin s, cos s, 35 sin s =)
5
! 00
r (s) = 4
cos s, sin s, 35 cos s =)
5
! 000
r (s) = 4
sin s, cos s, 35 sin s
5
Realizando el producto interno
000
!
r (s) · !
0
r (s) = 1= 2 =)  = 1
000
!
r (s) · !
0
r (s) ⇥ !
00
r (s) = 0
c) Desde  = 1 podemos inferir que la curva es una circunferencia y
por
⌧ = 0 concluimos, que la curva es plana.
d) Eliminando el parámetro s en las ecuaciones paramétricas
✓ ◆
4 3
(x, y, z) = cos s, 1 sin s, cos s
5 5

Produce
x2 + (y 1)2 + z 2 = 1, 3x + 4z = 0

Autoevaluación No 3

Pregunta 1
✓ ◆
1 1
Dada la ecuación paramétrica de la curva !
r (t) = t, + 1, t ,
t t
en el
punto (1, 2, 0) , determine:
a) la curvatura de la curva
b) La torsión de la curva

Pregunta 2
Sea r = r (✓) la ecuación de una curva C en coordenadas polares, con
a  ✓  b ángulo polar y P (✓) punto de C. Demuestre que
Rbp
a) La longitud de C desde P (a) hasta P (b) es L = a r2 + (r0 )2 d✓
0 00
2(r )2 rr + r2
b) La curvatura de C en P (✓) es  = .
[r2 + (r0 )2 ]3/2

184
c) Usando lo anterior,calcular la longitud L y la curvatura  de la
espiral
logarı́tmica r (✓) = e ✓ , donde es constante.
Pregunta 3
Un punto se mueve sobre la parábola y = x2 . Si en el instante t = ↵ ,
P
0 00
se encuentra en (0, 0) y s (↵) = a ,s (↵) = b ; donde la función s = s (t)
es la longitud de arco de la curva y a , b constantes. Determine la
velocidad y la aceleración de P en el instante t = ↵.

Pauta de Autocorrección

Pregunta 1
El valor del parámetro en el punto viene dado por
✓ ◆
! 1 1
r (t0 ) = t0 , + 1, t0 = (1, 2, 0) =) t0 = 1
t0 t0
Derivando sucesivamente
✓ ◆
! 1 1
r (t) = 1, 2 , 2 1 =) !
0
r 0 (1) = (1, 1, 2)
t t
✓ ◆
! 2 2
r (t) = 0, 3 , 3 =) !
00
r 00 (1) = (0, 2, 2)
t t
✓ ◆
! 6 6
r 000 (t) = 0, 4 , 4 =) ! r 000 (1) = (0, 6, 6)
t t
La curvatura se determina mediante la identidad
! p
r 0 (1) ⇥ !
00
r (1) k(2, 2, 2)k 2
 (1) = = p =
k!
3/2 6
r 0 (1)k 63

Mientras que la torsión está dada por la identidad


!r 0 (1) ⇥ !r (1) · !
00
r 000 (1) k(2, 2, 2) · (0, 6, 6)k
⌧ (1) = ! ! 2 = p =0
k r 0 (1) ⇥ r 00 (1)k 63
Obsérvese que la torsión es cero en todos los puntos ya que la curva
está
contenida siempre en el mismo plano.
Pregunta 2

185
a) En este caso podemos parametrizar la ecuación de la curva C de la
forma
!r =! r (✓) = (r (✓) cos ✓, r (✓) sin ✓)
Derivando esta expresión respecto de ✓ obtenemos

!
r =!
0 0 0
r (✓) = (r (✓) cos ✓
0
r (✓) sin ✓, r (✓) sin ✓ + r (✓) cos ✓)
De modo que
! ⇥ !0 ! ⇤1/2 q 0
r sin ✓)2 + (r0 sin ✓ + r cos ✓)2
0 0
r (✓) = r (✓) · r (✓) = (r cos ✓
p
=) !
0
r (✓) = r2 + (r0 )2 .
Entonces la longitud del arco entre ✓ = a y ✓ = b para la curva !
r (✓)
es
R b !0 Rbp
L= r (✓) d✓ =
a
r2 + (r0 )2 d✓.
a
!
r 0 (✓) ⇥ !
00
r (✓)
b) Deduciremos la curvatura usando la fórmula  (✓) =
k!
3/2
r 0 (✓)k
Tenemos que
! 0 0
r = r (✓) (cos ✓, sin ✓) + r (✓) ( sin ✓, cos ✓)
Derivando por segunda vez obtenemos

! 00 00 0
r = r (✓) (cos ✓, sin ✓) + 2r (✓) ( sin ✓, cos ✓) r (✓) (cos ✓, sin ✓)
Determinemos el producto vectorial considerando R2 un subespacio
vectorial
de R3
h i
!
r ⇥!
0
r
00
=
0
r (cos ✓, sin ✓, 0) + r( sin ✓, cos ✓, 0)
h 00 0
i
⇥ r (cos ✓, sin ✓) + 2r ( sin ✓, cos ✓) r(cos ✓, sin ✓)
⇣ 0 00

= 0, 0, 2(r )2 rr + r2

ya que se satisfacen que


(cos ✓, sin ✓, 0) ⇥ (cos ✓, sin ✓, 0) = (0, 0, 0) y
(cos ✓, sin ✓, 0) ⇥ ( sin ✓, cos ✓, 0) = (0, 0, 1)
De esta forma
!
r ⇥!
0
r
00 0
= 2(r )2
00
rr + r2

186
Por lo tanto

!
r 0 (✓) ⇥ !
00
r (✓)
0 00
2(r )2 rr + r2
 (✓) = =
k!
3/2
r 0 (✓)k [r2 + (r0 )2 ]3/2
✓ 0 ✓ 00 2
c) Sea r (✓) = e =) r (✓) = e =) r (✓) = e ✓.
Aplicando directamente las fórmulas obtenidas en a) y b), obtenemos
la longitud de la curva y su curvatura
Z bp Z bp
2 0 2
L = r + (r ) d✓ = e2 ✓ + 2 e2 ✓ d✓
a a
2
1+ b a
= e e

0 00
2(r )2 rr + r2 (1 + 2 ) e2 ✓
 (✓) = = p
[r2 + (r0 )2 ]3/2 (1 + 2 ) 1 + 2 e3 ✓

1
= p
1 + 2e ✓

Pregunta 3
Dada la ecuación cartesiana de la curva y = x2
Sea !
r (t) = (x (t) , y (t)) la función vectorial que describe el movimiento
del
punto tal que y (t) = [x (t)]2 , entonces al derivar esta expresión tenemos
0 0 00 0 00
y (t) = 2x (t) x (t) =) y (t) = 2((x (t))2 + x (t) x (t)).
Sabemos que en el instante t = 0, !
r (↵) = (x (↵) , y (↵)) = (0, 0) =)
x (↵) = 0, y (↵) = 0
0 00 0 2
Al evaluar las derivadas de y produce: y (↵) = 0, y (↵) = 2 x (↵)
Por otra parte , sabemos que:
q
0 !
s (↵) = r (↵) = (x0 (↵))2 + (y 0 (↵))2 = x (↵) = a
0 0

 0 00 0 00
00 ! 0 0 x (t) x (t) + y (t) y (t)
s (↵) = r (t) (↵) = (↵) =
k!
r 0 (t)k
0 00
00 x (↵) x (↵) 00
s (↵) = = b =) x (↵) = b
a
187
Por lo tanto, la velocidad y la aceleración en el punto P son
⇣ 0 ⌘
! 0
r (↵) = x (↵) , y 0 (↵) = (a, 0)
00
⇣ 00 ⌘
!r (↵) = x (↵) , y 00 (↵) = b, 2a2

188
Capı́tulo 3

Funciones de varias variables

3.1. Introducción
El concepto de función, ya establecido, en cursos precedentes puede ser
extendido a situaciones del tipo
!
f : A ⇢ IRm ! B ⇢ IRn , m, n 2 IN

en el sentido que son correspondencias que asignan a cada vector ! x 2


! ! ! !
A un único vector y 2 B anotado y = f ( x ) . Consideradas las
componentes de ! x y de ! y la igualdad anterior, también, se anota de
la forma
!
(y1 , y2 , . . . , yn ) = f (x1 , x2 , . . . , xm );
estas funciones se denominan funciones vectoriales de variable vectorial
, o bien, n funciones dependientes de m variables independientes. Sus
respectivos dominio de definición A y recorrido B se denominan campos
vectoriales.
En este curso se considerán funciones del tipo
!
f : A ⇢ IRm ! B ⇢ IR

esto es con m > 1 y n = 1 , que por las caracterı́stica de los espacios


pasan a llamarse, funciones reales de variable vectorial ( de m variables
independientes). En la unidad anterior fueron consideradas funciones
vectoriales
!
f : A ⇢ IR ! IRn

189
es decir, con m = 1 y n > 1. Estas funciones se denotan como
!
y = f (x1 , x2 , . . . , xm )

alternativamente ! y = f (! x ) con! x = (x1 , x2 , . . . , xm ) y se estudiarán


sus propiedades en cuánto a variaciones, gráficas, aplicaciones, etc, a
partir de los conceptos de lı́mite, continuidad, y derivada. En particular,
para los casos usuales m = 2 y m = 3, se denotan z = f (x, y), (x, y) 2
A ⇢ IR2 ; w = f (x, y, z)(x, y, z) 2 A ⇢ IR3 , respectivamente.

Ejemplos

Caso 1)
a) La correspondencia z = f (x, y) = xy , A = IR2 , B = IR , restringi-
dos x e y a valores positivos f (x, y) = xy corresponde al área del
rectángulo de lados x e y (comparado con f (r) = ⇡r2 , ésta representa
el área del cı́rculo de radio r y es función del tipo f : A ⇢ IR ! B ⇢
IR, o sea con m = 1 y n = 1.
b) La correspondencia w = f (x, y, z) = xyz , tiene A = IR3 , B = IR
, y con x, y, z positivos representa el volumen V del prisma recto de
aristas x, y, z.
Caso 2)
a) La función f : A ⇢ IR2 ! B ⇢ IR dada por
p
z = f (x, y) = x2 + y 2 , A = IR2 , B = IR,

representa la distancia del punto P = (x, y) al origen O = (0, 0) de


IR2 .
b) La función f : A ⇢ IR3 ! B ⇢ IR dada por
p
w = f (x, y, z) = x2 + y 2 + z 2 , A = IR3 , B = IR,

representa la distancia del punto P = (x, y, z) al origen O = (0, 0, 0)


de IR3 .
Caso 3)

1
La función z = f (x, y) = p , en que f : A ⇢ IR2 ! B ⇢ IR
+ x2 y2
tiene dominio A = IR2 ,y recorrido B = IR, representa el potencial

190
electrostático en cada punto P = (x, y) del plano debido a una carga
unitaria colocada en el origen O = (0, 0) de IR2 .
Nota: En estos ejemplos de funciones con m = 2, ó m = 3 se puede
considerar los conceptos de lı́neas de nivel y superficies de nivel como

S = {(x, y) 2 A/ f (x, y) = c}

para c constante dada; y

S = {(x, y, z) 2 A/ f (x, y, z) = c}

para c constante dada.


n p o
2 2 2
a) En el caso 2) se tiene que S = (x, y) 2 IR / x + y = c con
p
c 0 son circunsferencias de radio c (ó el origen si c = 0).
( )
1
b) En el caso 3) S = (x, y) 2 IR2 {(0, 0)} / p = c, c > 0
x2 + y 2
1
las lı́neas que verifican x2 + y 2 = 2 se llaman lı́neas equipotenciales
c
alrededor de la carga, describen circunsferencias centradas el (0, 0) de
1
radio .
c
Unido a lo anterior, en topografı́a las curvas determinadas por ecuación
f (x, y) = c se llaman también contornosp de nivel de una superficie.
Por ejemplo, la función z = f (x, y) = 25 x2 y 2 que tiene

A = (x, y) 2 IR2 / x2 + y 2  25 , B = [0, 5]


representa es
puna ”montaña. férica de radio basal 5 y altura 5; con c = 3
se tiene 25 x2 y 2 = 3, de lo cual se tiene el contorno de nivel
x2 + y 2 = 16.
Caso 4)
La presión P ejercida por un gas ideal encerrado en un cilindro pistón es
T
dada por la función P (T, V ) = k donde k cte, T es la temperatura
V
y V el volumen del cilindro. En este caso las lı́neas de nivel reciben el
nombre de lı́neas isobáricas P = c, ó isotermas cuando T = c con T
función temperatura en cada punto (x, y, z) .
Caso 5)

191
a) El área S de la superficie del cuerpo humano es una función del
peso w y la altura h dada por S (w, h) = 0, 091w0,425 h0,725 (deducción
empı́rica), donde w está en libras y h en pulgadas y S en pie2 .
b) También S es dada por S (h, t) = 2ht donde h es la altura en cm
y t es la longitud de la circunferencia máxima del muslo en cm. Por
ejemplo, una niña de altura h = 156 cm y 50 cm de de circunsferencia
máxima del muslo tienen una superficie corporal de 15,600 cm2 .

Caso 6)
La función f (x, y) = C xa y 1 a con C y a constantes, donde x son las
unidades de trabajo e y las unidades de capital representa un modelo
de unidades de producción (Modelo de Cobb-Douglas), que se utiliza
en economı́a y en la evaluación de proyectos.

Caso 7)
a) La potencia eléctrica para un voltage E y resistencia R es la función
E2
de dos variables P (E, R) = .
R
b) La resistencia total al conectar en paralelo dos resistencias R1 , R2
1 1 1
es regido por la ecuación = + , de donde se deduce R =
R R1 R2
R (R1 , R2 ) .
Caso 8)
a) La aceleración centrı́peta de una partı́cula que se mueve en la cir-
v2
cunferencia de radio r siendo v la rapidez es dada por a (r, v) = .
r
b) El desplazamiento vertical de una cuerda larga sujeta en el origen,
que cae bajo la acción de su propio peso es dada por
8 g
< (2axt x2 ) , si 0  x  at
u (x, t) = 2a2 g
: t2 , si x > at, a cte
2a2
Caso 9)
La concentración molecular C (x, t) de un lı́quido es dada por
x2
1/2
C (x, t) = t e k t

y esta función verifica la ecuación de difusión

192
k @ 2C @C
= .
4 @x2 @t

Caso 10)
Cuando una chimenea de h metros de altura emite humo que contiene
contaminante óxido nı́trico la concentración C (x, z) a x km de distancia
y z metros de altura es dada por
✓ ◆
a b(z h)2 b(z h)2
C (x, z) = 2 e x2 +e x2
x
@C @C
Calcular e interpretar los valores de , ,en el punto P = (2, 5)
@x @t
para a = 200, b = 0, 02; h = 10 m.
Caso 11)
Un ejemplo de una función que depende de 4 variables es la que es-
tablece la ley de Poiseuille en la cual la intensidad del flujo de un fluido
viscoso (como la sangre) través de un conducto (como una arteria), es
R4
Q (R, L, p1 , p2 ) = k (p1 p2 )
L
con k cte, R radio de conducto, L su longitud y p1 , p2 presiones en los
extremos del conducto.

3.2. Funciones Escalares de Variable Vec-


torial
3.2.1. Conceptos Topológicos
Nuestro espacio universo será Rn pensado preferentemente con n = 2
o n = 3.
Si !
x 2 Rn entonces cada vector ! x = (x , x , . . . , x ) esta conformado
1 2 n
por una n- upla ordenada de números reales.
La métrica que se usará, es la métrica usual, es decir si !
x ,!
y 2 Rn tal
que:
!x = (x1 , x2 , . . . , xn )
!y = (y1 , y2 , . . . , yn )

193
entonces

n
! 12
X
k!
x !
yk= (xi yi ) 2
i=1

A kk se le llama norma euclidea y permite calcular la distancia entre


los puntos x e y.
Si y = 0 entonces
n
! 12
X
k!
xk= x2i
i=1

Vecindad

Sea x0 2 Rn , una vecindad de x0 es:

V (x0 ) = {x 2 Rn | kx x0 k < }

En R2 esta vecindad es un disco centrado en P0 . En R3 es una esfera


centrada en P0 y de radio
Caso especial es el de la vecindad despuntada

V ⇤ (x0 ) = V (x0 ) {x0 }

Punto Interior

Sea S ✓ Rn , x 2 S es un punto interior de S si existe > 0 tal que:

V (x) ⇢ S

Conjunto Abierto

Sea A ✓ Rn , diremos que A es un conjunto abierto si y sólo si todos


sus puntos son interiores.

194
Ejemplo: ¿Cuáles de los siguientes conjuntos son abiertos?

A = {(x, y) 2 R2 | x2 + y 2  1} No es conjunto abierto


A = {(x, y) 2 R2 | |x| < 1, |y| < 1} Es conjunto abierto
A = {(x, y) 2 R2 | 0  x < 1} No es conjunto abierto

Conjunto Cerrado

Sea B ✓ Rn , diremos que B es un conjunto cerrado si su complemento


Rn B es abierto.

Punto Frontera

Sea A ✓ Rn , x0 es un punto frontera de A si y solo si 8 > 0 : V (x0 ) \


A 6= ? y V (x0 ) \ (Rn A) 6= ?

Interior de un Conjunto

Sea A ✓ Rn ,el conjunto de todos los puntos interiores de A se llama el


o
interior de A y se denota A o sea:
o
A = {x | x es punto interior de A}

Frontera de un Conjunto

Sea A ✓ Rn , la frontera de A es el conjunto de todos los puntos frontera


de A y se denota F r(A).

Proposición 1 Si S es abierto, no contiene ninguno de sus puntos


fronteras.
Dem. Directamente de la definición de conjunto abierto.

195
Proposición 2 Si S es cerrado, contiene a todos sus puntos fronteras.

Segmento Lineal

Si P1 , P2 2 Rn , el segmento lineal que une P1 con P2 es P = (1 t)P1 +


P2 , 0  t  1. P1 se llama punto inicial y P2 punto terminal.

Linea Poligonal

Una poligonal está formada por un número finito de segmentos lineales


unidos sucesivamente por sus extremos.

Punto Aislado

Sea A ✓ Rn , x0 es punto aislado de A si y solo si x0 2 A y 9 > 0 :

V ⇤ (x0 ) \ A = ?

Punto de Acumulación

Sea A ✓ Rn , x 2 Rn , x es punto de acumulación de A si toda


vecindad despuntada de x contiene puntos de A, es decir:
x es punto de acumulación de A () V ⇤ (x0 ) \ A 6= ?

Región

Sea R ✓ Rn ,diremos que el conjunto R es una región si es un conjunto


abierto y cualquier par de puntos de ella pueden unirse mediante una
lı́nea poligonal, contenida en R.

196
Región Cerrada

Es una región unida con su frontera.

Teorema 3.2.1. Sea C ✓ Rn y C es conjunto cerrado, entonces C


contiene en todos sus puntos de acumulación.

3.2.2. Aspectos Geométrico de las Funciones Es-


calares
Sea f : D ✓ Rn ! R esta función a cada x 2 D, x = (x1 , x2 , . . . , xn )
le asigna una imagen b 2 R, b un escalar.
b = f (x1 , x2 , . . . , xn )
a este tipo de funciones se les llama usualmente campo escalar.
Con el objeto de resaltar sus aspectos geométricos, especialmente para
funciones de dominio en R2 o R3 analizaremos conceptos como, gráfi-
cas, curvas de nivel, superficies de nivel, trazos, etc. de estas funciones.
Sea f : D ✓ Rn ! R si x 2 D =) x = (x1 , x2 , . . . , xn ), D es el
dominio de la función f.

f (D) = Im(f ) = {z 2 R | z = f (x1 , x2 , . . . , xn )}

3.2.3. Gráfica de una Función


Sea f : D ✓ Rn ! R , definimos la gráfica de f como el subconjunto
de Rn+1 formado por todos los puntos (x1 , x2 , . . . , xn , f (x1 , x2 , . . . , xn ))
para cada (x1 , x2 , . . . , xn ) 2 Rn . Simbólicamente

Gf = (x1 , x2 , . . . , xn , f (x1 , x2 , . . . , xn )) 2 Rn+1 | (x1 , x2 , . . . , xn ) 2 D

Ası́, si n = 2 y f es tal que z = f (x, y) entonces su gráfica será:

Gf = (x, y, z) 2 R3 | z = f (x, y)

197
3.2.4. Curvas y Superficies de Nivel

Sea f : D ✓ Rn ! R y C un escalar. Entonces el conjunto de puntos de


nivel de valor C ,se define como el conjunto de puntos (x1 , x2 , . . . , xn ) 2
D para los cuales f (x1 , x2 , . . . , xn ) = C.Simbólicamente, el conjunto
de nivel

S = {(x1 , x2 , . . . , xn ) 2 D | f (x1 , x2 , . . . , xn ) = C}

Si n = 2 el conjunto {(x, y) | f (x, y) = C} es una curva de nivel.


Si n = 3 el conjunto {(x, y, z) | f (x, y, z) = C} es una superficie
de nivel.

Ejemplo:
1.- Trazar la curva de nivel de f (x, y) = x y, C = 0, 1, 2 · ··

2.- Trazar la curva de nivel de f (x, y) = 10 x2 4y 2 , C = 0, 1, 2 · ··


Es familia de elipses centradas para C < 10, cuyo eje mayor se
encuentra sobre el eje x

Observación: Las curvas y las superficies de nivel permiten


dar una imagen de la gráfica de la función.

198
3.2.5. Lı́mite
Sea f : D ✓ Rn ! R una función escalar y x0 un punto de acumulación
del dominio D. Formularemos la definición de lı́mite de una función
escalar de la siguiente forma.

Definición 3.2.1. La función f tiene como lı́mite al número L 2 R


cuando x ! x0 , si para cada ✏ > 0, existe (✏, x0 ) > 0 tal que

x 2 D y 0 < kx x0 k < entonces |f (x) L| < ✏

Simbólicamente:

lı́m f (x) = L () 8✏ > 0, 9 > 0 :


x!x0
x 2 D y 0 < kx x0 k < =) |f (x) L| < ✏

En el caso particular n = 2 esta definición es

lı́m f (x, y) = L () 8✏ > 0, 9 > 0 : (x, y) 2 D y


(x,y)!(x0 ,y0 )

0 < k(x, y) (x0 , y0 )k < =) |f (x, y) L| < ✏

Ejemplo

Sea f (x, y) = x2 + 2xy. Determinar si existe lı́m (x2 + 2xy)


(x,y)!(3, 1)

199
Análisis:
Si (x, y) ! (3, 1) significa que x está cercano a 3 e y está cercano a
1 por lo tanto el valor de x2 + 2xy debe estar próximo a 3 se espera
entonces que si este lı́mite existe debe ocurrir que:

lı́m x2 + 2xy = 3
(x,y)!(3, 1)

Observe que:
q q
2
|x 3|  (x 3)  (x 3)2 + (y + 1)2 = k(x, y) (3, 1)k
q q
2
|y + 1|  (y + 1)  (x 3)2 + (y + 1)2 = k(x, y) (3, 1)k

por lo cual:

k(x, y) (3, 1)k < =) |x 3| < y |y + 1| < x2 + 2xy 3 <

por otro lado , se tiene

x2 + 2xy 3 = (x 3)2 + 2(x 3)(y + 1) + 4(x 3) + 6(y + 1)


2
 |x 3| + 2 |x 3| |y + 1| + 4 |x 3| + 6 |y + 1|

Sin pérdida de generalidad se puede poner la condición < 1 entonces


mayorando término a término, produce

x2 + 2xy 3 < |x 3| + 2 |y + 1| + 4 |x 3| + 6 |y + 1|
< + 2 + 4 + 6 = 13

"
Definiendo = min 1, 13 ,

Todo lo anterior permite afirmar que:

k(x, y) (3, 1)k < ) x2 + 2xy 3 < 13 x2 + 2xy 3


"
< 13 = 13 = "
13
200
Lo cual prueba que lı́m (x2 + 2xy) = 3
(x,y)!(3, 1)

Consideremos otro ejemplo en que usamos también la definición pero


otro procedimiento

Ejercicio:

Probar que

✓ ◆
x2 y 2
lı́m =0
(x,y)!(0,0) x2 + y 2

Solución: Sea ✏ > 0 y

2
x2  x2 + y 2 , y 2  x2 + y 2 =) x2 y 2  x2 + y 2
x2 y 2
=)  x2 + y 2 , (x, y) 6= (0, 0)
x2 + y 2

p p
Sea = ", k(x, y) (0, 0)k = k(x, y)k = x2 + y 2

p p p
x2 + y 2 < =) x2 + y 2 < " =) x2 + y 2 < "

x2 y 2 x2 y 2
) = <"
x2 + y 2 x2 + y 2

Lo que prueba que


✓ ◆
x2 y 2
lı́m =0
(x,y)!(0,0) x2 + y 2

201
Los Teoremas de Lı́mites

Sean f : D1 ✓ Rn ! R , g : D2 ✓ Rn ! R tales que si a =


(a1 , a2 , . . . , an ) es punto de acumulación de D1 y D2 y lı́m f (x) = L,
x!a
lı́m g(x) = M, aquı́ x = (x1 , x2 , . . . , xn ). Entonces:
x!a

a.) lı́m (f + g) (x) = L + M


x!a

b.) lı́m (f g) (x) = L M


x!a

c.) lı́m (f · g) (x) = L · M


x!a
✓ ◆
f L
d.) lı́m (x) = , M 6= 0
x!a g M

La demostración de estas propiedades es idéntica a la correspondiente


de las funciones de R en R.

Teorema 3.2.2. Sea f : D ✓ Rn ! R tal que lı́m f (x) = L, y g : I ✓


x!a
R ! R es una función continua en I. Entonces g f : D ✓ Rn ! R y
⇣ ⌘
lı́m (g f ) (x) = g lı́m f (x) = g(L)
x!a x!a

2
Ejemplo: Evaluar lı́m ex+y
(x,y)!(3,2)

Solución:
Sea g(z) = ez continua en todo R, lı́m x + y 2 = 7, entonces
(x,y)!(3,2)

2
lı́m ex+y = e7
(x,y)!(3,2)

Trayectorias:

Sea (x, y) un punto de R2 . Una trayectoria por (x, y) es cualquier recta


o curva que contenga a (x, y)

202
Regla de las dos trayectorias Una condición necesaria (no sufi-
ciente) para que lı́m f (x, y) exista y sea L, es que si los lı́mites
(x,y)!(x0 ,y0 )
lı́m f (x, ' (x)) y lı́m f (x, (x)) existen , para cualquier trayectoria
x!x0 x!x0
y = '(x) , y = (x) que pase por (x0 , y0 ),deben valer L

x2 y
Ejemplo: Sea f (x, y) = x4 +y 2
¿Existirá lı́m f (x, y)?.
(x,y)!(0,0)

Solución:
Dominio de f es R2 {(0, 0)} y claramente (0, 0) es punto de acumu-
lación
del dominio de f .
Sea T1 = {(x, y) | y = ax} familia de rectas que pasan por el origen si
el
lı́mite existe deberı́a ocurrir que:

x2 y x2 (↵x)
lı́m = lı́m
(x,y)!(0,0) x4 + y 2 x!0 x4 + ↵2 x2

(↵x)
= lı́m 2 =0
x!0 x + ↵2

Esto señala que si el lı́mite existe este debe ser cero, seguimos averiguan-
do:
Sea T2 = {(x, y) | y = x2 } parábola por el origen.

x2 y x4 1
lı́m 4 2
= lı́m 4 4
=
(x,y)!(0,0) x + y x!0 x + x 2

No puede ser, el lı́mite si existe no puede tener dos valores diferentes.


Luego, no existe
x2 y
lı́m
(x,y)!(0,0) x4 + y 2

Lı́mites Iterados

Se llaman lı́mites iterados a los siguientes.


✓ ◆ ✓ ◆
lı́m lı́m f (x, y) ; lı́m lı́m f (x, y)
x!x0 y!y0 y!y0 x!x0

203
x2 y 2
Ejemplo: Sea f (x, y) = x2 +y 2
, (x, y) 6= (0, 0) .Determine los lı́mites
iterados de f.
Solución:
Tenemos que (0, 0) es punto de acumulación del dominio de f .
Evaluemos, los lı́mites iterados
✓ ◆
x2 y 2 x2
lı́m lı́m 2 = lı́m =1
x!0 y!0 x + y 2 x!0 x2
✓ ◆
x2 y 2 y2
lı́m lı́m = lı́m 2 = 1
y!0 x!0 x2 + y 2 y!0 y

Los conceptos de lı́mites y lı́mites iterados se relacionan según los sigu-


ientes
teoremas.
Teorema 3.2.3. Si lı́m f (x, y) existe, y si para cada x en una
(x,y)!(x0 ,y0 )
vecindad reducida de x0 , lı́m f (x, y) existe. Entonces
y!y0
✓ ◆
lı́m lı́m f (x, y) = lı́m f (x, y)
x!x0 y!y0 (x,y)!(x0 ,y0 )

Este teorema nos lleva a formular otro equivalente.


Teorema 3.2.4. Si lı́m f (x, y) existe, y si para cada y en una
(x,y)!(x0 ,y0 )
vecindad reducida de y0 , lı́m f (x, y) existe. Entonces
x!x0
✓ ◆
lı́m lı́m f (x, y) = lı́m f (x, y)
y!y0 x!x0 (x,y)!(x0 ,y0 )

Ahora, se puede combinar ambos teoremas, lo que produce:


Teorema 3.2.5. Si lı́m f (x, y) existe, y si para cada x en una
(x,y)!(x0 ,y0 )
vecindad reducida de x0 , lı́m f (x, y) existe, y si para cada y de una
y!y0
vecindad reducida de y0 , lı́m f (x, y) existe. Entonces
x!x0
✓ ◆ ✓ ◆
lı́m lı́m f (x, y) = lı́m lı́m f (x, y) = lı́m f (x, y)
y!y0 x!x0 x!x0 y!y0 (x,y)!(x0 ,y0 )

x2 y 2
En el ejemplo anterior podemos concluir que no existe lı́m 2 2 ,pues
(x,y)!(0,0) x +y
de existir su lı́mites iterados deben ser iguales.

204
3.2.6. Continuidad

Una función f es continua en a ( a es punto de acumulación de D ✓ Rn )


si para cada " > 0 existe > 0 tal que

8x 2 D : kx ak < =) |f (x) f (a)| < "


donde x = (x1 , x2 , ..., xn ), a = (a1 , a2 , ..., an ). O lo que es lo mismo:
Se tiene que f es continua en un punto a si:
i) f (a) existe, ii)lı́m f (x) existe y ; iii)lı́m f (x) = f (a).
x!a x!a

Es decir, para que una función de varias variables sea continua en un


punto debe estar definida allı́, debe tener lı́mite en él y el valor de la
función en el punto debe ser igual al valor del lı́mite en ese punto. En
R2 podemos enunciar esta propiedad de la siguiente forma.
Una función f en continua en un punto interior (x0 , y0 ) de una región
R si f (x0 , y0 ) está definida y

lı́m f (x, y) = f (x0 , y0 )


(x,y)!(x0 ,y0 )

f será continua en la región R si es continua en cada punto de R. Las


funciones que no son continuas se dicen que son discontinuas.

Teoremas de Continuidad

Son similares a los teoremas para funciones de una variable. Esto sig-
nifica que, si una función es combinación de otras funciones y estas
funciones a su vez son continuas entonces la función es continua excep-
to en aquellos puntos en los que no está definida.

8
x2 y 2 <
si (x, y) 6= (0, 0)
Ejemplo. Sea la función f (x, y) = x2 + y 2 ,estudie
:
0 si (x, y) = (0, 0)
la continuidad de f en R2 .
Solución.

205
Claramente si (x, y) 6= (0, 0), f es un cuociente de funciones con-
tinuas por lo que también es continua. Además, la función es continua
en (0, 0) pues se demostró en 1.4.2, que

x2 y 2
lı́m = 0 = f (0, 0),
(x,y)!(0,0) x2 + y 2

Por lo tanto, esta función es continua en todo R2 . Continuando con


este mismo ejemplo, podemos usar coordenadas polares para mostrar
de que éste último lı́mite vale cero. Decir que (x, y) ! (0, 0) , en
coordenadas polares es equivalente a que r ! 0 (independientemente
del valor de ✓).
x2 y 2
Expresando la función f (x, y) = en coordenadas polares,
x2 + y 2
x = r cos ✓, y = r sin ✓, obtenemos la función
r2 cos 2 ✓ sin 2 ✓ 2
2 cos ✓ sin ✓
2
g(r, ✓) = = r
cos 2 ✓ + sin 2 ✓ cos 2 ✓ + sin 2 ✓

observese que
cos 2 ✓ sin 2 ✓
' (✓) = = cos 2 ✓ sin 2 ✓  1.
cos 2 ✓ + sin 2 ✓

está acotada, y además


(r) = r2

Lo que implica que:

lı́mg(r, ✓) = lı́m (r)'(✓) = 0.


r!0 r!0

Podemos concluir que si '(✓)  M está acotada, en una vecindad con


centro en el origen, y (r) ! 0 cuando r ! 0,entonces lı́m (r)'(;) =
r!0
0.

Continuidad en un conjunto abierto.

Diremos una función es continua en un conjunto abierto U ✓ Rn ,si ella


es continua en cada punto del conjunto U .

206
3.2.7. Derivadas Parciales

Sea f : D ✓ R2 ! R función de dos variables que está definida en una


vecindad del puntos (x0 , y0 ). La derivada parcial de f respecto de x en
(x0 , y0 ) se define por:

@f f (x0 + h, y0 ) f (x0 , y0 )
(x0 , y0 ) = lı́m
@x h!0 h

Similarmente, la derivada parcial de f respecto de y en (x0 , y0 ) se define


por:
@f f (x0 , y0 + h) f (x0 , y0 )
(x0 , y0 ) = lı́m
@y h!0 h
siempre que estos lı́mites existan. Las derivadas de orden superior son
una reiteración de la definición anterior, es decir, derivadas sucesivas,
ası́:

✓ ◆
@ 2f @ @f fx (x0 + h, y0 ) fx (x0 , y0 )
= = lı́m
@x2 @x @x h!0 h
✓ ◆
@ 2f @ @f fy (x0 , y0 + h) fy (x0 , y0 )
2
= = lı́m
@y @y @y h!0 h
2
✓ ◆
@ f @ @f fy (x0 + h, y0 ) fy (x0 , y0 )
= = lı́m
@x@y @x @y h!0 h

Ejemplo:

Ejemplo:

Calcular las derivadas parciales de:

f (x, y) = x4 sin y + cos xy

Solución:
De acuerdo con la definición debemos calcular

(x + h)4 sin y + cos(x + h)y (x4 sin y + cos xy)


lı́m
h!0 h

207
Desarrollando el numerador se tiene:

x4 sin y + 4x3 h sin y + 6x2 h2 sin y + 4xh3 sin y + h4 sin y


+ cos(xy) cos(hy) sin(xy) sin(hy) x4 sin y cos(xy) =
4x3 h sin y + 6x2 h2 sin y + 4xh3 sin y + h4 sin y
+ cos(xy)(cos(hy) 1) sin(xy) sin(hy) =

Calculando lı́mite

(x + h)4 sin y + cos(x + h)y (x4 sin y + cos xy)


lı́m
h!0 h
(cos(hy) 1) sin(hy)
= 4x3 sin y + cos xylı́m sin(xy)lı́m
h!0 h h!0 h

Como
(cos(hy) 1) sin(hy)
lı́m =0 y lı́m =y
h!0 h h!0 h

Tenemos que este lı́mite y por tanto la derivada es


@f
(x, y) = 4x3 sin y y sin(xy)
@x

De igual forma se puede calcular por definición.

@f
(x, y) = x4 cos y x cos xy
@y

Derivadas Parciales Cruzadas

Teorema 3.2.6. (Teorema de Schwarz)


@ 2f
Sea f : D ✓ R2 ! R una función, D abierto. Si las derivadas y
@x@y
@ 2f
existen y son continuas en D, entonces:
@y@x
@ 2f @ 2f
=
@x@y @y@x

208
Demostración queda propuesta.
Como ejemplo ilustrativo considere la función
(
x3 y xy 3
x2 +y 2
si (x, y) 6= (0, 0)
f (x, y) =
0 si (x, y) = (0, 0)

y verifique que:

@ 2f @ 2f
(0, 0) = 1, (0, 0) = 1
@x@y @y@x

Observación: Use la definición y encuentre estos resultados.

3.3. Diferenciabilidad en dos variables


Sea f una función definida en una vecindad de (x0 , y0 ). Diremos que
f es diferenciable en (x0 , y0 ) si existen números A y B tales que:

f (x0 + h, y0 + k) f (x0 , y0 ) = Ah + Bk + ↵ (h, k) k(h, k)k

y si el residuo tiene la propiedad lı́m ↵ (h, k) = 0. Diremos que f


k(h,k)k!0
es diferenciable en una región, si es diferenciable en cada punto de la
región.

Teorema 3.3.1. Si f es diferenciable en (x0 , y0 ), entonces f es con-


tinua en (x0 , y0 ).

Demostración:
Sea (x, y) 2 V ((x0 , y0 )) y

f (x0 +h, y0 +k) f (x0 , y0 ) = A (x x0 )+B (y y0 )+↵ (h, k) k(x, y) (x0 , y0 )k

Además sabemos que

|x x0 |  k(x, y) (x0 , y0 )k , |y y0 |  k(x, y) (x0 , y0 )k

entonces

209
kf (x0 + h, y0 + k) f (x0 , y0 )k  |Ah + Bk + ↵ (h, k)| k(x, y) (x0 , y0 )k

Si (x, y) ! (x0 , y0 ) , tenemos que ↵ (h, k) ! 0 y k(x, y) (x0 , y0 )k !


0
por lo tanto kf (x0 + h, y0 + k) f (x0 , y0 )k ! 0

) lı́m f (x0 + h, y0 + k) = f (x0 , y0 )


(h,k)!(0,0)

La diferenciabilidad es también una condición más fuerte que la exis-


tencia de las derivadas parciales.

Teorema 3.3.2. Si f es diferenciable en (x0 , y0 ), entonces las derivadas


parciales de primer orden existen en (x0 , y0 ) y

fx (x0 , y0 ) = A
fy (x0 , y0 ) = B

Demostración
Sea f diferenciable en (x0 , y0 ) =)

p
f (x0 + h, y0 + k) f (x0 , y0 ) = Ah + Bk + ↵ (h, k) h2 + k 2

dividiendo por h 6= 0 y haciendo k = 0 se tiene:

f (x0 + h, y0 ) f (x0 , y0 ) |h|


=A+↵
h h

f (x0 + h, y0 ) f (x0 , y0 ) |h|


=) A = |↵ (h)| = |↵ (h)|
h h

pero lı́m ↵ (h) = 0


h!0

f (x0 + h, y0 ) f (x0 , y0 )
) lı́m =A
h!0 h
De manera analoga

210
f (x0 , y0 + k) f (x0 , y0 )
) lı́m =B
k!0 k
Se puede tener también el siguiente criterio para la diferenciabilidad de
una
función.

Teorema 3.3.3. Sea f : D ✓ R2 ! R una función. Si f tiene primeras


derivadas parciales continuas en una región D. Entonces f es diferen-
ciable en cada punto de D.

Nota: Todas las ideas dadas para funciones de dos variables se pueden
extender a funciones definidas en un espacio n dimensional.

Diferencial Total

En dos dimensiones:
Si f es diferenciable en (x0 , y0 ) entonces la diferencial total es:

@f @f
df = dx + dy en (x0 , y0 )
@x @y

En tres dimensiones:.
Si f es diferenciable en (x0 , y0 , z0 ) entonces la diferencial total es:

@f @f @f
df = dx + dy + dz en (x0 , y0 , z0 )
@x @y @z

3.3.1. Derivada Direccional


Gradiente

Sea f : D ✓ Rn ! R, P 2 D abierto y f diferenciable en P. Entonces el


vector gradiente de f en P se denota rf (P ) y se define por la fórmula:
✓ ◆
@f @f @f
rf (P ) = (P ) , (P ) , ..., (P )
@x1 @x2 @xn

211
Ejemplo: Sea f (x, y, z) = 3x3 y 2 z. Calcular rf (1, 2, 3).
Solución:
La función tiene derivadas parciales continuas, entonces

rf (x, y, z) = 9x2 y 2 z, 6x3 yz, 3x3 y 2

Evaluando, el gradiente de la función f en el punto (1, 2, 3) queda

rf (1, 2, 3) = (108, 36, 12) = 12 (9, 3, 1)

Propiedades
Si f y g son funciones diferenciables demuestre que:

r (f g) = f rg + grf
✓ ◆
f grf f rg
r = si g no es cero
g g2
Ejemplo
Si f es diferenciable en una variable y g = f (x2 + y 2 + z 2 ) . Calcular
rg · rg.
Solución.
Si ponemos u = x2 + y 2 + z 2 al derivar parcialmente se tiene
@g @g @g
= 2xf 0 (u) ; = 2yf 0 (u) ; = 2zf 0 (u).
@x @y @z

=) rg = 2f 0 (u)(x, y, z) =) rg · rg = 4 [f 0 (u)] 2 [(x, y, z) · (x, y, z)]

= 4 [f´(u)] 2 (x2 + y 2 + z 2 )
(Definición Derivada direccional)
Sea f : D ✓ Rn ! R y P0 un punto interior de D. Sea u b vector
unitario kbuk = 1. La derivada direccional de f en P0 en la dirección de
b se define por:
u

f (P0 + hb
u) f (P0 )
lı́m
h!0 h
Si este lı́mite existe f tiene derivada direccional en P0 en la dirección
@f
b y la denotamos
de u (P0 ) .
@b
u
212
@f
Teorema 3.3.4. Si f es diferenciable en P, entonces (P ) existe
@b
u
b, unitarios y
para todos los vectores u
@f
(P ) = rf (P ) · u
b
@b
u

Demostración:
Considerando la demostración en R3 .Sean P0 = (x, y, z) y u
b = (u1, u2 , u3 )
Por definición de diferenciabilidad se tiene:

f (P0 + hb
u) f (P0 ) = f (x + hu1 , y + hu2 , z + hu3 ) f (x, y, z)
@f @f @f
= h · u1 + h · u2 + h · u3 + ↵ |h|
@x @y @z
✓ ◆
@f @f @f
= h u1 + u1 + u3 + ↵ |h|
@x @y @z

Dividiendo por h y tomando lı́mite cuando h ! 0 se tiene ↵ ! 0 y

f (P0 + hb
u) f (P0 ) @f @f @f
lı́m = u1 + u2 + u3
h!0 h @x @y @z

@f
) (P0 ) = rf (P0 ) · u
b
@b
u
@f
Nota: Para cada u b vector unitario fijo, la derivada direccional de-
@b
u
fine una nueva función, a la cual a su vez se le puede aplicar la definición
de derivada direccional y tenemos ası́ las derivadas direccionales de or-
den superior.
✓ ◆
@ 2f @ @f
2
=
@bu @b
u @b
u

Ejemplo
Sea f (x, y) = x3 y 2 , calcular la derivada direccional de f en el punto
P0 ( 1, 2) en la dirección del vector !
u = (4, 3).
Solución.
Como f (x, y) = x3 y 2 en una función diferenciable 8 (x, y) 2 R2 ,entonces

213
@f
(P ) = rf (P ) · u
b
@b
u
En primer lugar calculemos el vector gradiente
rf (P0 ) = (3x2 y 2 , 2x3 y)
En segundo lugar calculemos el vector unitario
!u (4, 3)
b= ! =
u
kuk 5
Se deduce que
@f (4, 3)
(x, y) = 3x2 y 2 , 2x3 y ·
@b
u 5
Por lo tanto, evaluando en P0 ( 1, 2) queda
@f (4, 3)
( 1, 2) = 3 ( 1)2 22 , 2 ( 1)3 2 ·
@b
u 5
96 12 108
+= =
5 5 5
Teorema 3.3.5. Supongamos que f tiene segundas derivadas parciales
continuas en una vecindad de un punto P. Entonces:
@ 2f
2
u · r)2 f
(P ) = (b
@bu
Demostración:

@f
= f 1 u1 + f 2 u2 + f 3 u3
@b u
@ 2f @f1 @f2 @f3
2
= u1 + u2 + u3
@bu @b
u @b
u @b
u
2
@ f
= u1 (f11 u1 + f12 u2 + f13 u3 ) + u2 (f21 u1 + f22 u2 + f23 u3 )
@bu2
+u3 (f31 u1 + f32 u2 + f33 u3 )

Como las derivadas cruzadas son iguales se tiene

@ 2f
= u21 f11 + 2u1 u2 f12 + 2u1 u3 f13 + u22 f22 + 2u2 u3 f23 + u23 f33
@bu2
✓ ◆2
@ @ @
= u1 + u2 + u3 f = (bu · r)2 f
@x @y @z

214
@f
Consecuencias de (P ) = rf (P ) · u
b
@b
u
@f
1.) Si rf (P ) = 0 =) ! (P ) = 0 8b u
@u
2.) La dirección de máximo crecimiento de la derivada direccional viene
dada
@f
por (P ) = krf (P )k en la dirección del vector gradiente.
@bu
3.) La dirección de mayor decrecimiento (o mı́nimo crecimiento) viene
dada en
@f
la dirección rf (P ) y el valor mı́nimo es (P ) = krf (P )k .
@bu

Demostración:
Basta considerar

@f
(P ) = rf (P ) · u
b = krf (P )k cos ✓
@b
u

3.3.2. Plano tangente y recta normal

Sea z = f (x, y) la ecuación de una superficie cualquiera. En lo que sigue


conviene para mayor comprensión del razonamiento usar la expresión
F (x, y, z) = 0 para denotar la superficie de nivel de la función

F (x, y, z) = f (x, y) z = 0 ó
F (x, y, z) = z f (x, y) = 0

Sea entonces la superficie F (x, y, z) = 0 y P0 = (x0 , y0 , z0 ) un punto


de ella. Sea !r (t) = (x(t), y(t), z(t)) la ecuación paramétrica de una
curva C en dicha superficie y que pasa por el punto (x0 , y0 , z0 ).

Tomando la diferencial de F (x, y, z) = 0 tenemos

@F @F @F
dx + dy + dz = 0
@x @y @z

En P0 = (x0 , y0 , z0 ) de la curva donde !


r (t0 ) = P0 se tiene

215
@F (P0 ) @F (P0 ) @F (P0 )
· x´(t0 )dt + · y´(t0 )dt + · z´(t0 )dt = 0 =)
@x @y @z
@F (P0 ) @F (P0 ) @F (P0 )
· x´(t0 ) + · y´(t0 ) + · z´(t0 ) = 0
@x @y @z

Es decir

@F (P0 ) @F (P0 ) @F (P0 )
, , · (x´(t0 ), y´(t0 ), z´(t0 )) = 0
@x @y @z
) 5F (P ) · ! r´(t ) = 0
0 0

Esto nos dice que 5F (P0 ) es perpendicular a ! r´(t0 ). Esto ocurre para
toda curva C que pase por P0 y como ! r´(t0 ) es vector tangente a
C se tiene que 5F (P0 ) es perpendicular a toda recta tangente a
la superficie en P0 . Por lo tanto, 5F (P0 ) es un vector normal a la
superficie en P0 .

Plano Tangente

La expresión de la forma:

@F (P0 ) @F (P0 ) @F (P0 )


(x x0 ) + (y y0 ) + (z z0 ) = 0
@x @y @z
es la ecuación del plano tangente a la superficie F (x, y, z) = 0 en el
punto P0 = (x0 , y0 , z0 ).
Si la superficie es z = f (x, y) =) F (x, y, z) = z f (x, y) = 0 la
ecuación del plano tangente se puede escribir, también como:

z z0 = fx (x0 , y0 )(x x0 ) + fy (x0 , y0 )(y y0 )

Recta Normal

Sean P0 (x0 , y0 , z0 ) un punto dado y el vector director de esta recta


5F (P0 )
La ecuación vectorial es:

216
@F (P0 ) @F (P0 ) @F (P0 )
(x, y, z) = (x0 , y0 , z0 ) + t( , , ), t2R
@x @y @z
Ecuación paramétrica:

@F (P0 )
x = x0 + t
@x
@F (P0 )
y = y0 + t , t2R
@y
@F (P0 )
x = z0 + t
@z

Eliminando el parámetro t obtenemos la ecuación cartesiana:

x x0 y y0 z z0
= =
Fx (P0 ) Fy (P0 ) Fz (P0 )

Ejemplo: Sea z = ex (cos y + 1) en el punto (0, ⇡2 , 1) ,calcular las

ecuaciones del plano tangente y la recta normal a la superficie.

Solución.
La componentes del vector gradiente son

fx (x, y) = ex (cos y + 1), fy (x, y) = ex (seny)


⇡ ⇡
fx (0, ) = 1, fy (0, ) = 1
2 2

Reemplanzando términos en la ecuación del plano tenemos:



z 1 = 1 · (x 0) + ( 1)(y )
2

=) z 1=x y+
2

Finalmente, la ecuación del plano tangente queda

217

x y z+ +1=0
2

y

(x, y, z) = (0, , 1) + t(1, 1, 1)
2

la ecuación de la recta normal en su forma vectorial.

3.3.3. Función Compuesta. La Regla de la Cadena.

En este módulo abordaremos en forma básica la siguiente problemática.


Si u es una función diferenciable de variables x, y, z, · · ·, y a su vez
estas últimas son funciones de otras variables nuevas t y/o s ¿podemos
encontrar la primera derivada parcial de u con respecto a las nuevas
variables t y/o s expresada en términos de las derivadas parciales de
las funciones dadas?. Ası́, por ejemplo, si un fenómeno fı́sico está ocur-
riendo digamos en una región cilı́ndrica, resultarı́a mejor expresar las
cantidades que interesen en términos de coordenadas cilindricas y no
en cartesianas.

Caso particular:

Sea f : D ✓ R2 ! R tal que u = f (x, y), tiene dos variables


independientes. Supongamos que cada una de estas variables es diferen-
ciable de una simple variable independiente t. Si x = x(t) y y = y(t),
la derivada de la función compuesta (regla de la cadena) con respecto
a t es

du @u dx @u dy
= · + ·
dt @x dt @y dt
du
= ru · !
r 0 (t)
dt

218
Caso particular de dos variable independientes simples.

Sea f : D ✓ R2 ! R tal que u = f (x, y), tiene dos variables


independientes. Estas variables a su vez son funciones diferenciables
de dos variables simples independientes t y s. La expresión de las
derivadas de la función compuesta (o regla de la cadena) es:

@u @u dx @u dy
= · + ·
@t @x dt @y dt

@u @u dx @u dy
= · + ·
@s @x ds @y ds
Las expresiones anteriores las podemos expresar matricialmente como:
0 1
✓ ◆ ✓ ◆ dx dy
@u @u @u @u B dt dt C
, = , @ dx dy A
@t @s @x @y
ds ds

dz x
Ejemplo 1: Hallar , si z = , donde x = et y y = ln t.
dt y
Solución.
Aplicando la derivación compuesta, tenemos
✓ ◆
dz @u dx @u dy dz 1 t x 1
= · + · =) = e +
dt @x dt @y dt dt y y2 t
✓ ◆
dz et et 1 t 1 1
= =e
dt ln t (ln t)2 t ln t t(ln t)2

@u @u
Ejemplo 2: Hallar y si u = f (x, y), donde x = t2 s2 ,
@t @s
y = ets

Solución.

219
En este caso

@u dx dy @u
= fx (x, y) +fy (x, y) =) = fx (x, y)(2t)+fy (x, y)(s ets )
@t dt dt @t

Similarmente

@u dx dy @u
= fx (x, y) +fy (x, y) =) = fx (x, y)( 2s)+fy (x, y)(t ets )
@s ds ds @s

De forma más general si

f : D ✓ Rn ! R tal que z = f (y1, y2, y3, · ··, yn, ), es función de n


variables independientes. Supongamos también que cada una de estas
variables independientes es función diferenciable de otras m variables
simples independientes x1 , x2 , x3 , · · ·, xm . La expresión de la derivada
de la función compuesta (regla de la cadena) es similar, pudiendose
escribir cada una de estas derivadas por

X @z @yi n
@z
= · , j = 1, 2, ..., m
@xj i=1
@y i @x j

Esta ecuación se puede escribir utilizando matrices, como

0 1
@y1 @y1
✓ ◆ ✓ ◆B ... C
B @x1 @xm C
@z @z @z @z B .. .. C
,..., = ,..., B . . C
@x1 @xm @y1 @yn @ A
@yn @yn
···
@x1 @xm

✓ ◆
@yi
La matriz n ⇥ m se denomina matriz jacobiana
@xj i=1...n, j=1...m
de la transformación yi = yi (x1, x2, x3, · ··, xm ).

220
Ejemplo 3: Sea f (x, y, z) = x + x2 y + zey , en donde x, y, y z ,
están relacionadas con u y v a través de la transformación

x = uv, y = u2 v2, z = u sin v

@f @f @ 2f
Calcule , , y , en el punto (u, v) = (2, 2).
@u @v @v@u

Solución.
La matriz jacobiana para esta transformación es
0 1 0 1
xu xv v u
@ yu yv A = @ 2u 2v A
zu zv senv u cos v
0 1 0 1
xu xv 2 2
@ yu yv A (2, 2) = @ 4 4 A
zu zv sin 2 2 cos 2

Para (u, v) = (2, 2), los valores de x, y y z son 4, 0 , y 2sen2


respectivamente.Entonces la matriz reglón (fx fy fz ) es

(fx fy fz ) = 1 + 2xy x2 + zey ey

evaluando en el punto (4, 0, 2sen2) queda

1 + 2xy x2 + zey ey (4, 0, 2sen2) = (1 16 + 2sen2 1)

De donde
0 1
xu xv
(fu fv ) = (fx fy fz ) @ yu yv A
zu zv
= (fx xu + fy yu + fz zu f x xv + f y yv + f z z v )

Evaluando

(fu fv ) (2, 2) = (66 + 9 sin 2 62 8 sin 2 + 2 cos 2)

Por lo tanto
@f @f
(2, 2) = 66 + 9 sin 2 y (2, 2) = 62 8 sin 2 + 2 cos 2
@u @v
221
Ahora bien, como

fu = f x xu + f y yu + f z z u
= (1 + 2xy )v + (x2 + zey )2u + ey sin v

Se tiene
@ 2f
= fuv = 2(xyv + yxv )v + 1 + 2xy + 2(2xxv + zey yv + ey zv )u
@v@u
+ yv ey senv + ey cos v

Evaluando en el punto (2, 2) se tiene

@ 2f
(2, 2) = 1 36sen2 + 9 cos 2
@v@u

3.3.4. Función Implı́cita


Cuando definimos funciones en forma implı́cita decimos, por ejemplo,
sea y = '(x) una función diferenciable definida implı́citamente por
medio de la ecuación F (x, y) = 0, o bien, sea z = f (x, y), definida
implı́citamente por la ecuación F (x, y, z) = 0.
El primer caso lo podemos ejemplificar con una función y = '(x) defini-
da por la ecuación de la hipérbola x2 4xy 3y 2 = 9.
Muchas veces las ecuaciones que relacionan a las variables pueden ser
tan complejas y no lineales, que no es posible esperar encontrar rela-
ciones sencillas, si las hay, explı́citas que expresen a una variable en
términos de las otras.
En términos generales, lo que se pide es que esta relación entre las
variables exista localmente, es decir en alguna vecindad de un punto
donde las ecuaciones se satisfacen. La mayorı́a de las veces no se esperan
resultados globales, es decir, no todos los puntos que satisfacen las
relaciones implı́citas satisfacen también las explı́citas.
Con el objeto de ilustrar estas ideas analicemos el siguiente ejemplo.

Ejemplo: Pruebe que la ecuación z 5 + z + xy = 0 define una función


z = f (x, y) para todos los valores x e y.
Solución.

222
Se debe establecer que para cada x e y se puede resolver en forma
única para z la ecuación de quinto grado. Ya que esto no es obvio,
razonamos utilizando la gráfica de

u = z5 + z + a

gráfica pendiente

Como u´(z) = 5z 4 + 1, 8a =) u es estrictamente creciente, entonces

para cada a existen z1 y z2 tal que u(z1 ) < 0 < u(z2 ) por lo cual
existe un único z tal que u(z) = 0 para cada a .

Si se identifica a con la cantidad xy se ha establecido el hecho


planteado.

Luego, ”z = f (x, y) se define implı́citamente por la ecuación z 5 + z +


xy = 0”

Derivación implı́cita

Sea F : U ✓ R2 ! R definida en un conjunto abierto U . Sea P0 =


(x0 , y0 ) 2 U un punto tal que:
i) F (x0 , y0 ) = 0
@F @F
ii) , son continuas en alguna vecindad V (P0 ) y
@x @y
@F
iii) (x0 , y0 ) 6= 0.
@z
Entonces existe una vecindad (x0 , x0 + ) de x0 , (y0 a, y0 + a)
vecindad de y0 , y una función única f (implı́cita) de clase C 1 tal
que:
a) f (x0 ) = y0 y f (x) 2 (y0 a, y0 + a) 8x 2 (x0 , x0 + )

b) F (x, f (x)) = 0 8x 2 (x0 , x0 + )


c) 8x, 2 V (x0 ) tiene derivadas que pueden calcularse como:

223
dy Fx (x, y)
=
dx Fy (x, y)

En el caso de funciones de dos variables definida implı́citamente, el


siguiente teorema afirma la existencia de funciones implı́citas bajo cier-
tas circunstancias y da fórmulas para obtener las derivadas parciales
de estas funciones.

Teorema 3.3.6. (Teorema de la funció implı́cita)


Sea F : U ✓ R3 ! R definida en un conjunto abierto U . Sea P0 =
(x0 , y0 , z0 ) 2 U un punto tal que:
i) F (x0 , y0 , z0 ) = 0
@F @F @F
ii) , , son continuas en alguna vecindad V (P0 ) y
@x @y @z
@F
iii) (x0 , y0 , z0 ) 6= 0.
@z
Entonces existe una vecindad V = V (x0 , y0 ), y una vecindad
(z0 a, z0 + a) de z0 y una función única f (implı́cita) de clase C 1
sobre V tal que:
a) f (x0 , y0 ) = z0 y f (x, y) 2 (z0 a, z0 + a) 8(x, y) 2 V.

b) F (x, y, f (x, y)) = 0, 8(x, y) 2 V.


c) 8(x, y) 2 V = V (x0 , y0 ) tiene derivadas que pueden calcularse como:

@F
@f (x, y) (x, y, f (x, y))
= @x
@x @F
(x, y, f (x, y))
@z

@F
(x, y, f (x, y))
@f (x, y) @y
=
@y @F
(x, y, f (x, y))
@z

Este teorema se puede generalizar a más variables, de tal modo que


si se dan las condiciones exigidas por el teorema y u = f (x, y, z, · · ·)

224
está definida implı́citamente por F (x, y, z , · · ·, u) = 0 y Fu (x, y, z , · ·
·, u) 6= 0, entonces:

@F @F
@f @x , @f = @y
=
@x @F @y @F
@z @z

,etc.

@z @z
Ejemplo. Calcular y si z se define implı́citamente en la
@x @y
ecuación

x2 y 8xyz = yz + z 3

Solución.
Tenemos que:

F (x, y, z) = x2 y + 8xyz + yz + z 3

Derivando parcialmente con respecto a x , y ,z.

Fx (x, y, z) = 8yz 2xy,


Fy (x, y, z) = 8xz + z x2 ,
Fz (x, y, z) = 8xy + y + 3z 2

Claramente estas derivadas son continuas en R3 , son funciones polinómicas,


Por consiguiente, F pertenece a C1 .
Para todo (x, y, z) en que Fz (x, y, z) 6= 0 se tiene.

@z 8yz 2xy
=
@x 8xy + y + 3z 2
@z 8xz + z x2
=
@y 8xy + y + 3z 2

225
3.3.5. Jacobiano

Si f1 , f2 , f3, . . . , fn son funciones diferenciables de Rn en R y si (x1 ,


x2 , x3 , . . . , xn ) 2 D su dominio tal que

f1 = f1 (x1 , x2 , x3 . . . , xn )
f2 = f2 (x1 , x2 , x3 . . . , xn )
..
.
fn = fn (x1 , x2 , x3 . . . , xn )

El Jacobiano de las funciones f1 , f2 , f3, . . . , fn , respecto de las


variables x1 , x2 , x3 . . . , xn se define por el determinante de las primeras
@(f1 , f2 , f3, . . . , fn )
derivadas parciales y se denota ,
@(x1 , x2 , x3 . . . , xn )
@f1 @f1 @f1
@x1 @x2
... @xn
@f2 @f2 @f2
@(f1 , f2 , f3, . . . , fn ) @x1 @x2
... @xn
= ..
@(x1 , x2 , x3 . . . , xn ) .
@fn @fn @fn
@x1 @x2
... @xn

Dos funciones definidas implı́citamente

Teorema 3.3.7. Sean F , G: U ✓ R4 ! R definidas en un conjunto


abierto F (x, y, u, v) y G(x, y, u, v). Sea P0 = (x0 , y0 , u0 , v0 ) 2 U un
punto tal que:
i) F (x0 , y0 , u0 , v0 ) = 0 y G(x0 , y0 , u0 , v0 ) = 0
ii) F y G tienen derivadas parciales continuas en alguna vecindad
V (P0 ) y
@ (F, G)
iii) (x0 , y0 , u0 , v0 ) 6= 0.
@ (u, v)
Entonces las ecuaciones F (x, y, u, v) = 0 , G(x, y, u, v) = 0 definen
funciones implı́citas u = u(x, y), v = v(x, y) en alguna vecindad V
(x0 , y0 ), las cuales tienen primeras derivadas parciales continuas con
respecto a cada una de las variables, tal que :
a) u0 = u(x0 , y0 ), v0 = v(x0 , y0 ).
b) F (x, y, u (x, y) , v (x, y)) = 0 , G(x, y, u (x, y) , v (x, y)) = 0, V (x0 , y0 )

226
c) Las derivadas parciales de u y v en V (P0 ) están dadas por

@(F, G) @(F, G)
@u @(x, v) @u @(y, v)
= , =
@x @(F, G) @y @(F, G)
@(u, v) @(u, v)
@(F, G) @(F, G)
@v @(u, x) @v @(u, y)
= , =
@x @(F, G) @y @(F, G)
@(u, v) @(u, v)

@u @v
Ejemplo Calcule las derivadas parciales y donde u y v
@x @x
están definidas por las ecuaciones implı́citas

x2 + 2uv = 1, x3 u3 + v 3 = 1

Solución.
En este caso F (x, u, v) = x2 + 2uv 1 = 0 y G(x, u, v) = x3 u3 +
v3 1 = 0

@(F, G) 2v 2u
= = 6(u3 + v 3 )
@(u, v) 3u 3v 2
2

@(F, G)
6= 0 para todo (x, u, v) donde u 6= 0 y v 6= 0.
@(u, v)
@(F, G) 2x 2u
= = 6x(v 2 xu)
@(x, v) 3x2 3v 2

@(F, G) 2v 2x
= = 6x(xv + u2 )
@(u, x) 3u2 3x2

Entonces, derivando u y v con respecto a x, obtenemos

@u 6x(v 2 xu) x(xu v 2 )


= =
@x 6(u3 + v 3 ) u3 + v 3
y

227
@v 6x(xv + u2 ) x(xv + u2 )
= =
@x 6(u3 + v 3 ) u3 + v 3

3.3.6. Máximos y Mı́nimos


Máximos y Mı́nimos (extremos locales o relativos) para funciones de
dos o más variables.
Sea f : U ✓ R2 ! R, U conjunto abierto.
i) f tiene un máximo local en x0 2 U si f (x0 ) f (x) 8x 2 B (x0 , y0 )
ii) f tiene un mı́nimo local en x0 2 U si f (x0 )  f (x) 8x 2 B (x0 , y0 )

Ejemplo 1: La función f (x, y) = 4 x2 y 2 tiene un máximo local


en (0, 0), pues f = f (0,0) f (x, y) = x2 + y 2 0 ya que x2 0,
2
y 0 8(x, y) 2 B (0, 0).

Ejemplo 2: La función z = f (x, y) = x2 + y 2


Tiene un mı́nimo local en (0, 0) : f (0, 0)  x2 + y 2 = f (x, y)
f (0, 0) = 0 es mı́nimo absoluto.

p
Ejemplo 3: La función z = f (x, y) = 1 3
x2 + y 2
f (0, 0) = 1 es mı́nimo local y absoluto.
Observación: Si las desigualdades son válidas en todo U se tendrá ex-
tremo absolutos.

Punto Crı́tico: (Condición necesaria)

Definición. Un punto del dominio de f es un punto crı́tico si todas


las derivadas parciales de f son cero en el punto o f no es diferenciable
en el punto.

Caso particular:
Si f : R2 ! R, (x0 , y0 ) es punto crı́tico de f si y solo si :
i)
@f (x0 , y0 ) @f (x0 , y0 )
= 0, =0
@x @y

228

ii)
@f (x0 , y0 ) @f (x0 , y0 )
@ , y/o @
@x @y

@f @f
Ejemplo : Sea z = f (x, y) = x2 y 2 =) = 2x = 2y
@x @y
=) (0, 0) punto crı́tico.

Punto silla:

Definición : Sea punto (x0 , y0 ) 2 Dom(f ) . Si cualquier B (x0 , y0 ) con-


tiene puntos (x, y) 2 B (x0 , y0 ) tales que f (x, y) f (x0 , y0 ) > 0 y
puntos (x, y) 2 B (x0 , y0 ) tales que f (x, y) f (x0 , y0 ) < 0 se llama
punto ensilladura.
Nota. Un punto crı́tico en que f no es máximo ni mı́nimo se llama
punto silla.

Ejemplo: En la función f (x, y) = x2 y 2 , (0, 0) es punto ensilladura,


pues f = f (0, 0) f (x, y) = x2 + y 2 no se puede decir nada aún del
signo, pero para f = f (0,0) f (x, 0) = x2  0 y f = f (0, 0)
f (0, y) = y 2 0, permite concluir que (0, 0) es punto ensilladura.
Observación: Las definiciones anteriores son extensibles a funciones
de más variables.

Teorema 3.3.8. Sea f una función continua de dos variables definida


en una región cerrada y acotada R del plano XY. Entonces:
a.) Al menos hay un punto (x0 , y0 ) 2 R en que f alcanza su valor
mı́nimo.
b.) Al menos hay un punto (x0 , y0 ) 2 R en que f alcanza su valor
máximo.

Observación: Este Teorema es extensible a más variables.


Definición: Sea f : U ✓ Rn ! R función definida en un conjunto
abierto U 2 Rn y sea x 2 U . Suponga que todas las derivadas parciales
de segundo orden existen en x. A la matriz cuadrada de orden n.

229
@ 2f
A = (aij )i,j=1,...,n donde aij = (x)
@xi @xj
x = (x1 , x2 , ..., xn )

se llama matriz Hessiana (o simplemente Hessiano) de la función f en


x y se denota H(x).

Caso particular: Para f (x, y)


!
@2f @2f
@x2 @y@x
H(x, y) = @2f @2f
@x@y @y 2

Caso particular: Para f (x, y, z)


0 @2f @2f @2f
1
@x2 @y@x @z@x
B @2f @2f @2f C
H(x, y, z) = @ @x@y @y 2 @z@y A
@2f @2f @2f
@x@z @y@z @z 2

Definición. Sea la matriz

0 1
a11 a12 . . . a1n
B . C
B a21 . . C
An = B .. .. C
@ . . A
an1 ann

Consideremos las submatrices Ak de An (k = 1, 2, . . . , n) definidas de


las siguientes maneras:
0 1
✓ ◆ a11 a12 a13
a11 a12
A1 = (a11 ) , A2 = , A3 = @ a21 a22 a23 A , · · · ,
a21 a22
a31 a32 a33
0 1
a11 a12 . . . a1n
B . C
B a21 . . C
An = B . . C
@ .. .. A
an1 ann

230
Teorema 3.3.9. (Consideraciones suficientes para la existencia de ex-
tremos locales)
Sea f : U ✓ Rn ! R una función definida en un conjunto abierto U de
!
Rn y x un punto crı́tico de f (es decir rf (x) = 0 ) y supongamos que
las derivadas parciales de segundo orden son continuas en una vecindad
abierta de x. Entonces:
a.) Si todas las submatrices Ak de la matriz Hessiana H(x) definidas
de la forma anterior tienen determinante positivo, f tiene un mı́nimo
local en x.
b.) Si todas las submatrices Ak de la matriz Hessiana H(x), definidas de
la forma anterior, tienen determinantes de signo alternado comenzando
por A1 < 0, f tiene un máximo local en x.

Ejemplo 1: Sea f : R2 ! R definida por f (x, y) = x2 + 3y 2 2x


12y + 13. Determine los valores extremos de f .
Solución
Aplicando la condición necesaria de punto crı́tico, obtenemos

rf (x, y) = (2x 2, 6y 12) = (0, 0) =) (1, 2) es el único punto crı́tico.


Calculemos la matriz Hessiana
✓ ◆
2 0
H(x, y) =
0 6
Examinemos los determinantes de las submatrices
A1 = (2) =) det A1 = 2 > 0
✓ ◆
2 0
A2 = =) det A2 = 12 > 0
0 6
) f tiene mı́nimo local en (1, 2).

Ejemplo 2: Sea f : R2 ! R definida por f (x, y) = (x2 + 3y 2 ) e1 (x +y ) .


2 2

¿Cuáles son los puntos crı́ticos de f ?


Solución.
De la condición de punto crı́tico tenemos:
= 2xe1 (x +y ) 2x (x2 + 3y 2 ) e1 (x +y ) = 0 =)
@f 2 2 2 2

@x

231
= e1 (x +y ) (2x
@f 2 2

@x
2x3 6xy 2 ) = 0
= 6ye1 (x +y ) 2y (x2 + 3y 2 ) e1 (x +y ) = 0 =)
@f 2 2 2 2

@y

= e1 (x +y ) (6y
@f 2 2

@y
6y 3 2x2 y) = 0

De donde obtenemos, el sistema de ecuaciones


x x3 3xy 2 = 0 x (1 x2 3y 2 ) = 0
=) =)
3y 3y 3 x2 y = 0 y (3 3y 2 x2 ) = 0
Puntos Crı́ticos: (0, 0), (0, 1), (0, 1), (1, 0), (1, 0), ( 1, 0)

Determine en qué puntos hay máximos, mı́nimos o puntos sillas.

y
Ejemplo 3: Sea f : R4 {0, 0, 0, 0} ! R y f (x, y, z, u) = x + +
x
z u 1
+ +
y z u
Determine los valores extremos de f.
Solución
Determinemos los puntos crı́ticos de la función f

@f y
= 1 =0
@x x2
@f 1 z
= =0
@y x y2
@f 1 u
= =0
@z y z2
@f 1 1
= =0
@y z u2

=) (1, 1, 1, 1) Punto Crı́tico.


La matriz Hessiana está definida como
0 1
@2f @2f @2f @2f
@x2 @y@x @z@x @u@x
B @2f @2f @2f @2f C
B C
H(x, y, z, u) = B
B
@x@y
@2f
@y 2
@2f
@z@y
@2f
@u@y
@2f
C
C
@ @x@z @y@z @z 2 @u@z A
@2f @2f @2f @2f
@x@u @y@u @z@u @u2

232
0 2y 1 1
x3 x2
0 0
B 1 2z 1
0 C
H(x, y, z, u) = B
@
x2 y3
1
y2
2u 1
C
A
0 y2 z3 z2
1 2
0 0 z2 u3

0 1
2 1 0 0
B 1 2 1 0 C
H(1, 1, 1, 1) = B
@ 0
C
1 2 1 A
0 0 1 2

41 = 2, 42 = 3, 43 = 2, 44 = 5

)Hay un mı́nimo local en (1, 1, 1, 1).

Teorema 3.3.10. (Condiciones suficientes para la existencia de ex-


tremos locales en dos variables.)
Sea f : U ✓ R2 ! R una función de dos variables que tiene derivadas
parciales de segundo orden continuas en el conjunto abierto U. Sea
fxx fxy
det(H (x, y)) = (x, y) 8 (x, y) 2 U. Sea (x0 , y0 ) 2 U tal
fyx fyy
que rf (x0 , y0 ) = 0. Entonces
i) Si det(H (x, y)) > 0 y fxx (x0 , y0 ) < 0, f tiene una máximo local en
(x0 , y0 ) .
ii) Si det(H (x, y)) > 0 y fxx (x0 , y0 ) > 0, f tiene un punto silla en
(x0 , y0 ) .
iii) Si det(H (x, y)) = 0 no podemos concluir nada.

3.3.7. Extremos Restringidos


Pensemos, que se tiene una función f (x, y) sujeta a cierta condición
g(x, y) = 0. Se quiere maximizar o minimizar f (x, y) con la condición
que (x, y) satisfaga la ecuación g(x, y) = 0.
En este caso podemos elaborar una respuesta aplicando la condición
necesaria de valor extremo para f (x, y)

df @f @f dy
= + · =0
dx @x @y dx

233
dy
Aplicando el teorema de la función implı́cita podemos calcular a
dx
partir de la restricción:
@g @g dy
g(x, y(x)) = 0 =) + · =0
@x @y dx

@g
dy @x
=) = @g
dx @y

Sustituyendo en la primera ecuación tenemos:


!
@g
df @f @f @x @g
= + · @g
=0 , 6= 0
dx @x @y @y
@y

@f @g @f @g
=) + · =0
@x @y @y @x

@(f, g)
=) =0
@(x, y)
Lo que implica que los puntos crı́ticos deben cumplir dos condiciones:

g(x, y) = 0.
@(f, g)
= 0
@(x, y)

d2 f
Además, la decisión de valor extremo resulta del signo de en cada
dx2
punto crı́tico, ya que f (x, y) = f (x, y(x)) = f (x).

Ejemplo 1. Obtenga las dimensiones de un rectángulo de perı́metro


dado que determinan la mayor y la menor área de éste.
Solución.
Si x e y denotan las longitudes de los lados del rectángulo, el problema
consistirá en buscar los extremos de la función

f (x, y) = xy, x, y > 0

234
L
sujeta a la restricción g(x, y) = x+y 2
= 0 , donde L es el perı́metro
dado.
Entonces aplicando las condiciones necesaria para obtener los puntos
crı́ticos tenemos:
L
g(x, y) = x + y = 0.
 2
@(f, g) y x
= =) y = x
@(x, y) 1 1

Sustituyendo el resultado de la segunda ecuación en la primera, se tiene:

L
x=y=
4
Luego hemos encontrado que la función f tiene un único punto crı́tico
en
✓ ◆
L L
P = , .
4 4
Además, determinemos la naturaleza del punto usando el criterio de la
segunda derivada

df dy
f (x, y (x)) = xy =) =y+x
dx dx
L dy
g(x, y) = x + y = 0 =) = 1
2 dx
df
) =y x
dx
d2 f dy
= 1= 2<0
dx2 dx
2
Por tanto, en el punto ( L4 , L4 , L16 ) hay un máximo relativo sobre la curva
de
L
intersección de las dos superficies z = xy y x + y = 2

La situación se puede generalizar para funciones de más variables o más


condiciones.

235
Por otra parte, para resolver este mismo tipo de problemas de máximo
y
mı́nimo podemos utilizar otro método que se fundamenta en el siguiente
teorema.
Teorema 3.3.11. (Multiplicadores de Lagrange)
Sean f : U ✓ Rn ! R, g : U ✓ Rn ! R funciones de C 1 . Sean x0 2 U
tal que g(x0 ) = 0 y S = {x 2 U : g(x) = 0} supongamos además que
rg(x0 ) 6= 0. Si f |S (f restringida a S) tiene un máximo o mı́nimo
local en S, en x0 ,entonces existe 2 R tal que:
rf (x0 ) + rg(x0 ) = 0

Nota: Lo anterior significa que x0 es un punto crı́tico de f |S .


Método: Construir F (x, y, ) = f + g y determinar puntos crı́ticos de
F

Ejemplo 1. Utilice el procedimiento que se origina a partir de este


teorema para obtener el máximo local del ejemplo anterior.
Solución.
En este caso
f (x, y) = xy, x, y > 0
sujeta a la condición:
L
g(x, y) = x + y =0
2
.
L
Sea F (x, y, ) = xy + (x + y 2
), entonces
Fx = y + =0
Fy = x + =0
L
F = x+y =0
2
de aquı́ x = , y= y sustituyendo en la tercera ecuación:
L L
2
= 0 =) = 4
.
L L
El punto crı́tico es (x0 , y0 ) = ( , ).
4 4
2
El punto ( L4 , L4 , L16 ) es el máximo relativo sobre la curva de intersección
L
de las dos superficies z = xy y x + y = 2

236
x2 y 2
Ejemplo 2. De todos los rectángulos inscritos en la elipse 2 + 2 =
a b
1,
a > 0 y b > 0 con lados paralelos al los ejes, determine el de mayor
área.
Solución.
En este caso
f (x, y) = 4xy, x, y > 0

x2 y 2
y g(x, y) = + 2 1=0
a2 b
✓ 2 ◆
x y2
Sea F (x, y, ) = 4xy + + 1 = 0 , entonces
a2 b2

x 2a2 y
Fx = 4y + 2 = 0 =) =
a2 x
y 2b2 x
Fy = 4x + 2 2 = 0 =) =
b y
2
x y2
) =
a2 b2

Reemplazando esta última expresión en la ecuación

x2 y 2
F = 2+ 2 1 = 0 =)
a b
✓ ◆
a b
obtenemos a un único punto crı́tico: p , p
2 2
Además, determinemos la naturaleza del punto usando el criterio de la
segunda derivada

df dy
f (x, y (x)) = 4xy =) = 4y + 4x
dx dx
d2 f dy d2 y
=) = 4 + 4x 2
dx2 dx dx
2 2
x y dy b2 x
g(x, y) = 2
+ 2 1 = 0 =) =
a b dx a2 y
✓ ◆
d2 y b2 1 b2 x 2
=) = +
dx2 a2 y a2 y 3

237
◆ ✓
a b
Evaluando segunda derivada en el punto P p , p , pro-
2 2
duce
✓ ◆
d2 f b2 x b4 x 3 d2 f b
= 8 2 4 4 =) = 12 < 0
dx2 ay a y 3 dx 2 a
✓ ◆
a b
Por tanto, el punto p ,p hay máximo relativo cuyo valor
2 2
✓ ◆
a b
es f p , p = 2ab
2 2

Ejemplo 3: Obtenga los extremos posibles de f (x, y) = x2 + 24xy +


8y 2 con la restricción x2 + y 2 25 = 0.
Solución:
Sea F (x, y, ) = x2 + 24xy + 8y 2 + (x2 + y 2 25) .

Fx = 2x + 24y + 2 x = 0 =) (1 + )x + 12y = 0 (1)


Fy = 24x + 16y + 2 y = 0 =) 12x + (8 + )y = 0 (2)
F = x2 + y 2 25 = 0 (3)

Multiplicando la ecuación (1) por 12 y (3) por(1 + ) produce.

12(1 + )x + 144y = 0
12(1 + )x + (8 + )(1 + )y = 0

Restando ambas ecuaciones


2
=) +9 136 = 0 =) 1 = 8; 2 = 17

18x + 24y = 0 3
Si = 8 =) =) 3x + 4y = 0 =) y = 4
x
24x + 32y = 0
3 2
x2 + 4
x = 25 =) x = ±4
Lo que da los puntos crı́ticos (±4, ⌥3)

238
Si = 17 en forma similar se obtiene que (±3, ±4) son puntos
crı́ticos.
Localización del máximo y el mı́nimo absoluto de f sobre un
dominio compacto.
Para determinar máximos y mı́nimos absolutos e una región cerrada
y acotada encerrada por una curva suave, se debe:
• Determinar todos los puntos crı́ticos en el interior de la región.
• Usar Lagrange para determinar puntos crı́ticos en la frontera.
• Evaluar f en los puntos, crı́ticos.
• Comparar todos estos valores y seleccionar el máximo y el mı́ni-
mo.

Criterio de la Segunda Derivada para Extremos Restringidos.

Sea f (x, y) función a maximizar o minimizar y g(x, y) = 0 la condición:


S = {(x, y) 2 R2 | g(x, y) = 0} .Sea F (x, y, ) = f (x, y) + g(x, y)
Sea (x0 , y0 , ) punto crı́tico de F

@f @g
|
@x (x0 ,y0 )
+ @x |(x0 ,y0 ) = 0
@f @g
|
@y (x0 ,y0 )
+ @y |(x0 ,y0 ) = 0
g (x0 , y0 ) = 0

Se desea utilizar el criterio de la segunda derivada para identificar ex-


tremos locales de la función f (x, y) bajo la condición g (x, y) = 0 lo
que conduce encontrar los máximos y mı́nimos de funciones de una
variable f (x, y(x)) .
De la condición, calculemos y = y(x) y reemplacemos en f |S .
Aquı́ podemos considerar f como función de una variable, es decir
f (x, y) = f (x, y(x)) en S.
@2f
Nos proponemos calcular @x2

df @f @f dy
= + ·
dx @x @y dx

239
✓ ◆2
d2 f @ 2f @ 2 f dy @ 2 f dy @ 2 f dy @f d2 y
= + · + · + · + ·
dx2 @x2 @y@x dx @x@y dx @y 2 dx @y dx2
✓ ◆2
@ 2f @ 2 f dy @ 2 f dy @f d2 y
= + 2 · + · + ·
@x2 @y@x dx @y 2 dx @y dx2

@g @g dy
g(x, y(x)) = 0 =) + · =0
@x @y dx

@g
dy @x
=) = @g
dx @y

Derivando nuevamente g respecto de x

✓ ◆2
@ 2g @ 2 g dy @ 2 g dy @ 2 g dy @g d2 y
+ · + · + · + · =0
@x2 @y@x dx @x@y dx @y 2 dx @y dx2

dy
Sustituyendo se tiene:
dx

! !2 ✓ ◆
2 2 @g 2 @g
@ g @ g @x @ g @x @g d2 y @g
2
+2 · @g
+ 2· @g
+ · =0 /·
@x @y@x @y
@y @y
@y dx2 @y

✓ ◆2 ✓ ◆2
@ 2 g @g @ 2 g @g @ 2 g @g 1 @g d2 y
· 2 · + · · ⇣ ⌘2 + · =0
@x2 @y @y@x @x @y 2 @x @g @y dx2
@y

" ✓ ◆2 #
d2 y 1 @ 2 g @g @ 2 g @g @ 2g @g 1
=) 2 = ⇣ ⌘2 · + 2 · · · @g
dx @g @x2 @y @y@x @x @y 2 @x @y
@y

d2 f
Reemplazando en dx2

240
! !2
@g @g
d2 y @ 2f @ 2f @ 2
f
= 2 · @x
@g
+ 2 · @x @g
dx2 @x2 @y@x @y
@y @y
" ✓ ◆2 #
@f 1 @ 2 g @g @ 2 g @g @ 2g @g 1
+ · ⇣ ⌘2 · + 2 · · · @g
@y @g @x2 @y @y@x @x @y 2 @x @y
@y

@f @g @f @g
y0 = y(x0 ), |x = | x0 ; |x = |x
@x 0 @x @y 0 @y 0

Luego

@f @f
|
@x x0 @y
| x0
(x0 , y0 ) , @g
= , @g
=
|
@x x0 @y
| x0

" ✓ ◆2 ✓ ◆2
d2 f 1 @ 2f @g @ 2 f @g @g @ 2 f @g
2
= ⇣ ⌘2 2
· 2 · · + ·
dx @g @x @y @y@x @x @y @y 2 @x
@y
✓ ◆2 ✓ ◆2 #
@ 2g @g @ 2 g @g @g @ 2g @g
+ 2
· 2 · · + 2
·
@x @y @y@x @x @y @y @x
"✓ ◆ ✓ ◆ ✓ ◆
2
1 @ 2f @ 2g @g @ 2f @ 2g
= ⇣ ⌘2 + · 2 + ·
@g @x2 @x2 @y @y@x @y@x
@y
✓ ◆ ✓ ◆2 #
@g @g @ 2f @ 2g @g
· · + + ·
@x @y @y 2 @y 2 @x

Podemos escribir entonces:

" ✓ ◆2 ✓ ◆2 #
d2 f 1 @ 2F @g @ 2 F @g @g @ 2 F @g
2
= ⇣ ⌘2 2
· 2 · · + ·
dx @g @x @y @y@x @x @y @y 2 @x
@y

En el paréntesis, cuadrado se tiene

241
@g @g @g @g
@g @x @y @g @y @x
⇤ = @2F @2F + @2F @2F
@x @y@x @y 2
@y @y@x @x2
@g @g @g @g
0 @x @y
0 @x @y
@g @2F @2F @g @2F @2F
= @x @x2 @y@x = @x @x2 @y@x
@g @2F @2F @g @2F @2F
@y @y@x @y 2 @y @y@x @y 2

De lo anterior se plantea la siguiente definición


Si f y g son funciones como las definidas antes y F = f + g. Llamamos
hessiano limitado de la función F a
@g @g
0 @x @y
@g @2F @2F
HF = @x @x2 @y@x en (x0 , y0 )
@g @2F @2F
@y @y@x @y 2

De tal modo que

d2 f HF (x0 , y0 )
= ⇣ ⌘2
dx2 @g
@y
(x ,
0 0y )

Teorema en extremos restringidos

Consideremos nuevamente

d2 f HF (x0 , y0 )
= ⇣ ⌘2
dx2 @g
@y
(x 0 , y 0 )

2
De esta expresión, dado que el signo de la segunda derivada ddxf2 depende
⇣ ⌘2
@g
sólo del determinante Hessiano limitado, ya que @y (x0 , y0 ) > 0,
inferimos el siguiente criterio de la segunda derivada para extremos
restringidos.
d2 f
a) Si HF (x0 , y0 ) > 0, entonces dx2
< 0 y la función tiene un máximo
local condicionado en (x0 , y0 ).

242
d2 f
b) Si HF (x0 , y0 ) < 0, entonces dx2
> 0 y la función tiene un mı́nimo
local condicionado en (x0 , y0 ) .
c) Si HF (x0 , y0 ) = 0 no hay información del punto(x0 , y0 ).

Ejemplo: Hallar los extremos de f (x, y) = (x y)2 sujeta a la


restricción x + y 2 1 = 0
2

Solución:
Sea F (x, y, ) = (x y)2 + (x2 + y 2 1)
Fx = 2 (x y) + 2 x = 0
=) 2 (x + y) = 0
Fy = 2 (x y) + 2 y = 0
F = x2 + y 2 = 1

a) Si escogemos = 0 , y (x + y) 6= 0 obtenemos los puntos crı́ticos


de f libre de restricción.

b) Si 6= 0 =) y = x. Reemplazando en la tercera ecuación , se


obtienen dos puntos crı́ticos de la función f sujetos a la restricción.
⇣ ⌘ ⇣ ⌘
1 1 1 p1
P1 2 ,
p p
2
y P2 p
2
, 2
Determinemos el Hessiano lı́mitado para decidir la naturaleza de estos
puntos crı́ticos

0 2x 2y
HF (x, y) = 2x 2 + 2 2
2y 2 2+2
= 8 x2 + 2xy + y 2 8 x2 + y 2

Sustituyendo y = x en la primera ecuación se tiene = 2 6=


0,entonces el Hessiano se reduce a

HF (x, y) = 8 x2 + 2xy + y 2 + 16 x2 + y 2
= 8 x2 2xy + y 2

Evaluando en el primer punto crı́ticos produce

243
✓ ◆
1 1
HF p , p = 16 > 0
2 2

d2 f ⇣ 1 1
⌘ ⇣ ⌘
entonces p , p < 0, luego en el punto p12 , p1 la función
dx2 2 2 2
f tiene un máximo local condicionado.
Por simetrı́a, el segundo punto también corresponde a un máximo local
condicionado de f.
El criterio de la segunda derivada para extremos restringidos se puede
generalizar para funciones de más de dos variables.
En el caso de una función de tres variables x, y, z y sujeta a la sola
restricción g(x, y, z) = 0, formamos el Hessiano correspondiente a

F (x, y, z, ) = f (x, y, z) + g (x, y, z)

de la siguiente forma

@g @g @g
0 @x @y @z
@g @2F @2F @2F
@x @x2 @y@x @x@z
HF = @g @2F @2F @2F
@y @y@x @y 2 @y@z
@g @2F @2F @2F
@z @x@z @y@z @z 2

y sea
@g @g
0 @x @y
@g @2F @2F
A3 = @x @x2 @y@x
@g @2F @2F
@y @y@x @y 2

El criterio de la segunda derivada en este caso expresa:


Sea (x0 , y0 , z0 ) punto crı́tico.
a) Si HF < 0 y A3 > 0,entonces la función f tendrá un máximo
condicionado en (x0 , y0 , z0 ).
b) Si HF < 0 y A3 < 0,entonces la función f tendrá un mı́nimo
condicionado en (x0 , y0 , z0 ).
c) Si HF > 0,entonces la función f no tiene extremos condicionado
en (x0 , y0 , z0 ).
d.) Si HF = 0, no hay información acerca del punto (x0 , y0, z0 ).

244
Ejemplo Hallar los extremos de la función f (x, y, z) = x2 + y 2 + z 2
sujeta a la restrición g (x, y, z) = z 2 + 2x y 2 1 = 0.
Solución:
Sea la función F = x2 + y 2 + z 2 + (z 2 + 2x y2 1)

Fx = 2x + 2 = 0 =) 2 (x + ) = 0 =) x =
Fy = 2y 2 y = 0 =) 2y(1 ) = 0 =) y = 0
Fz = 2z + 2 z = 0 =) 2z(1 + ) = 0 =) z = 0
F = z 2 + 2x y 2 1 = 0

1
Reemplazando en F : 02 +2( ) 02 1 = 0 =) = 2
, y obtenemos
el punto crı́tico 12 , 0, 0

0 2 2y 2z
2 2 0 0
HF =
2y 0 2 2 0
2z 0 0 2+2
= 4 4 4x2 2y · 2 · 2y (2 2 ) + 2z · 2 (2 2 ) ( 2z)
✓ ◆
1 1
HF , 0, 0, = 16 < 0
2 2

A3 = 8y 2 4 (2 2 )=0 12 < 0

)Hay un mı́nimo condicionado de f en el punto 1


2
, 0, 0 .
Obsérvese que las otras opciones no producen puntos crı́ticos en IR3

3.4. Problemas Resueltos

3.4.1. Continuidad y diferenciabilidad


Problema

Dada la función f : R2 ! R definida como


( xy
arctg 2 , si (x, y) 6= (0, 0)
f (x, y) = x + y2 .
0 , si (x, y) = (0, 0)

245
a) Verificar si f es continua en IR2
@f @f
b) Calcular si existen las derivadas parciales , en IR2
@x @y
Solución.
a) Tenemos que f (x, y) es continua 8 (x, y) 6= (0, 0) puesto que es
xy
composición de dos funciones continuas, como son arctg y 2 .
x + y2
Para estudiar la continuidad en el punto (0, 0) tenemos que calcular
lı́m f (x, y) lo que haremos a través de la trayectoria y = mx,
(x,y)!(0,0)

mx2 m
entonces lı́m f (x, mx) = lı́marctg 2 2
= lı́marctg
(x,y)!(0,0) x!0 x +y x!0 2 + m2
que depende de la pendiente m, por lo que este lı́mite no existe.
Por lo tanto f no es continua en el punto(0, 0)

b) Para (x, y) 6= (0, 0) la función admite derivadas parciales, que son:

@f y 2 x2
(x, y) = y 2 ,
@x (x + y 2 )2 + x2 y 2
@f x2 y 2
(x, y) = x 2
@y (x + y 2 )2 + x2 y 2

Para (x, y) = (0, 0) ,se tiene

@f f (h, 0) f (0, 0)
(0, 0) = lı́m
@x h!0 h
arctg0 0
= lı́m = lı́m0 = 0
h!0 h h!0

Analogamente, derivando con respecto a y

@f f (0, h) f (0, 0)
(0, 0) = lı́m
@y h!0 h
arctg0 0
= lı́m = lı́m0 = 0
h!0 h h!0

Por lo tanto, existen las derivadas parciales en (x, y) = (0, 0) .

246
Problema

Dada la función f : IR2 ! IR definida como


8 2
< x seny 2
, si (x, y) 6= (0, 0)
f (x, y) = x2 + y 2 , probar que es
:
0 , si (x, y) = (0, 0)

diferenciable en el punto P0 = (0, 0) .¿Es continua la función


en ese punto?
Solución.
Tenemos que utilizar la definición y ver si el siguiente lı́mite es cero:
| f df |
L= lı́m p
(h,k)!(0,0) h2 + k 2

h2 senk 2
donde f = f (h, k) f (0, 0) = ,y
h2 + k 2
@f @f
df = (0, 0) h + (0, 0) k
@x @y
En este caso
@f f (h, 0) f (0, 0)
(0, 0) = lı́m
@x h!0 h
2
h ·0
2
0
= lı́m h =0
h!0 h
Asimismo
@f f (0, k) f (0, 0)
(0, 0) = lı́m
@y h!0 k
0 · senk 2
0
= lı́m k2 =0
h!0 k
h2 senk 2
Luego, df = 0, entonces L = lı́m p
(h,k)!(0,,0) (h2 + k 2 ) h2 + k 2

h2 senk 2 h2 k 2
g (h, k) = 
(h2 + k 2 )3/2 (h2 + k 2 )3/2
(h2 + k 2 )(h2 + k 2 ) p 2
 3/2
= (h + k 2 ) < "
2
(h + k ) 2

247
Si = " . Ası́ L = 0 y f es diferenciable en P0 = (0, 0) .
De lo anterior se deduce que f es es continua en (0, 0) ya que es
diferenciable en dicho punto.

3.4.2. Regla de la cadena


Problema 1

Sea la ecuación zxx + 2zxy + zy = 0 , donde

u+v u v u2 v 2
x(u, v) = , y(u, v) = , z(u, v) = w (u, v)
2 2 4

Muestre que al cambiar las variables independientes (x, y) por (u, v)


y la
función z por w la ecuación se reduce a 2 4wuu = 0.
Solución.
En primer lugar, calculamos la aplicación inversa

u(x, y) = x + y, v (x, y) = x y.

Derivando parcialmente estas últimas expresiones se tiene:


ux = 1, uy = 1; vx = 1, vy = 1
Usando estos resultados y la regla de la cadena, obtenemos

z x = z u ux + z v vx = z u + z v
z y = z u uy + z v vy = z u zv
Reiterando la derivación parcial y usando la regla de la cadena por
segunda
vez
zxx = (zx )u ux + (zx )v vx = zuu + zvu + zuv + zvv
zxy = (zx )u uy + (zx )v vy = zuu + zvu (zuv + zvv )
zyy = (zx )u uy + (zv )v vy = zuu zvu (zuv zvv )
Suponiendo que z es una función continua con primeras derivadas par-
ciales

248
continuas, entonces
zxx + 2zxy + zy = 4zuu = 0
2u 1
Finalmente, zu = wu =) zuu = wuu
4 2
Por tanto, se tiene
1
wuu = 0
2

Problema 2

Una función z = z (x, y) se dice que es armónica si tiene derivadas


parciales de segundo orden continuas y además zxx + zyy = 0.
x y
Sean u = 2 2
, v= 2 . Pruebe que:
x +y x + y2
i) u y v son armónicas
ii) (ux )2 = (vy )2
iii) (uy )2 = (vx )2
iv) ux vx = uy v y
b)✓Si f (x, y) es una◆función armónica, entonces la función w (x, y) =
x y
f , 2
x + y x + y2
2 2

es también armónica
Solución.
x y 2 x2 2xy
a) i) u = =) u x = , uy =
x2 + y 2 (x2 + y 2 )2 (x2 + y 2 )2
y 2xy x2 y 2
v= =) vx = , v y =
x + y2
2 (x2 + y 2 )2 (x2 + y 2 )2

Derivando parcialmente por segunda vez se tiene

2x(x2 + y 2 )2 (y 2 x2 )2(x2 + y 2 )2x


uxx =
(x2 + y 2 )4
2x3 6xy 2
=
(x2 + y 2 )3

249
2x(x2 + y 2 )2 (2xy)2(x2 + y 2 )2y
uyy =
(x2 + y 2 )4
2x3 + 6xy 2
=
(x2 + y 2 )3

Lo anterior implica que


uxx + uyy = 0

Analogamente para ,
vxx + vyy = 0

Además
✓ ◆2 ✓ ◆2
2 y 2 x2 x2 y 2
ii) (ux ) = = = (vy )2
(x2 + y 2 )2 (x2 + y 2 )2
✓ ◆2 ✓ ◆2
2 2xy 2xy
iii)(uy ) = 2 2 2
= 2 2 2
= (vx )2
(x + y ) (x + y )
✓ 2 2
◆✓ ◆ ✓ ◆
y x 2xy 2xy(y 2 x2 )
iv) ux vx = = = uy v y
(x2 + y 2 )2 (x2 + y 2 )2 (x2 + y 2 )2
b) Aplicando derivación compuesta tenemos:

@f @f
wx = ux + vx ,
@u @v
@f @f
wy = uy + vy
@u @v

Derivando parcialmente por segunda vez, queda


@ 2f @ 2f @f
wxx = 2
u x + v x ux + uxx +
@u @v@u @u
✓ 2 ◆
@ f @ 2f @f
+ ux + 2 v x v x + vxx
@u@v @v @v
 2
@ f @ 2f @f
wyy = 2
u y + v y uy + uyy +
@u @v@u @u
✓ 2 ◆
@ f @ 2f @f
+ uy + 2 v y v y + vyy
@u@v @v @v

Finalmente

250
@ 2f 2 @ 2f @f
wxx = 2
(u x ) + vx ux + uxx +
@u @v@u @u
@ 2f @ 2f @f
+ ux vx + 2 (vx )2 + vxx
@u@v @v @v
@ 2f 2 @ 2f @f
wyy = 2
(u y ) + uy vy + uyy +
@u @v@u @u
@ 2f @ 2f @f
+ uy vy + 2 (vy )2 + vyy
@u@v @v @v

Sumando términos y utilizando las igualdades establecidas en a) se


tiene
✓ ◆
@ 2f @ 2f @f
wxx + wyy = 2
+ 2 (ux )2 + (uxx + uyy )+
@u @v @u
✓ 2 ◆
@ f @ 2f @f
+ 2
+ 2 (vx )2 + (vxx + vyy ) = 0
@u @v @v

3.4.3. Derivación Implı́cita


Problema 1

a) Sea f : R ! R2 una función tal que grad f (1, 1) = (2, 4) y g :


R3 ! R2 una función tal que sus funciones coordenadas gi : R3 !
R, i = 1, 2 tienen los siguientes gradientes grad g1 (1, 1, 1) = (2, 3, 1),
@(f g)
grad g2 (1, 1, 1) = ( 5, 4, 2). Si g(1, 1, 1) = (1, 1). Obtener (1, 1, 1).
@x

b) Utilizando el teorema de la función implı́cita determine si es posible


3
escribir y en términos de x para la función F (x, y) = x4 exy 1 = 0
en una vecindad del punto (1, 1), y además encuentre su derivada.

Solución
Sean las coordenadas cartesianas designadas en R3 por (x, y, z) y en R2
por (u, v), y tomando u = u(x, y, z), v = v(x, y, z) tenemos que:

@f g @f @u @f @v
(1, 1, 1) = (g(1, 1, 1)) · (1, 1, 1) + (g(1, 1, 1)) · (1, 1, 1)
@x @u @x @v @x
@f @u @f @v
= (1, 1) · (1, 1, 1) + (1, 1) · (1, 1, 1)
@u @x @v @x
251
Notemos que
✓ ◆
@f @f
grad f (1, 1) = (1, 1), (1, 1) = (2, 4),
@u @v
✓ ◆ ✓ ◆
@g1 @g1 @g1 @u @u @u
grad g1 = , , = , , = (2, 3, 1),
@x @y @z @x @y @z
✓ ◆ ✓ ◆
@g2 @g2 @21 @v @v @v
grad g2 = , , = , , = ( 5, 4, 2),
@x @y @z @x @y @z

Ası́
@f g
(1, 1, 1) = 2 · 2 + 4 · 5= 16.
@x
1
Primeramente note que (1, 1) 2 F (0, 0) y además

@F 3 @F 3
= 4x3 y 3 exy 1
, = 3xy 2 exy 1
@x @y

las cuales son continuas en R2 en particular para alguna vecindad


V ((1, 1), ) de (1, 1) donde @F
@y
(1, 1) = 3 6= 0 por lo tanto podemos
ocupar el teorema de la función implı́cita, y definir f : V ((1, 1), ) ! R
con y = f (x) y 1 = f (1) cuya derivada es
3
0 4x3 y 3 exy 1
y = .
3xy 2 exy3 1

Problema 2

a) Determine las derivadas parciales @u/@x, @u/@y, @v/@x, @v/@y, donde


u, v son funciones definidas implicitamente por el sistema

F (x, y, u, v) = xeu+v + uv 1 = 0,
G(x, y, u, v) = yeu v 2uv 1 = 0.

alrededor del punto p = (1, 1, 0, 0).

2 2 @ 2z
b) Sea la función z = f (u + v , u/v) obtener .
@u2

252
Solución.
a) Verificando las hipótesis del Teorema de la función implı́cita podemos
concluir que:

@u (yeu v + 2u)eu+v
=
@x 2xu+v yeu v + yeu v u 2vxeu+v + 2xeu+v u + vyeu v

@v eu+v (yeu v 2v)


=
@x 2xu+v yeu v + yeu v u 2vxeu+v + 2xeu+v u + vyeu v

@u e( u v)(xeu+v + u)
=
@y 2xu+v yeu v + yeu v u 2vxeu+v + 2xeu+v u + vyeu v

@v (xeu+v + v)eu v
= u+v u v + yeu v u
@y 2x ye 2vxeu+v + 2xeu+v u + vyeu v

b) Definiendo x = x(u, v) = u2 + v 2 , y = y(u, v) = u/v tenemos que


z = f (x, y), entonces
@z @z @x @z @y @z @z 1
= + = 2u + ,
@u @x @u @y @u @x @y v
luego
✓ 2 ◆ ✓ ◆
@ 2z @z @ z @x @ 2 z @y 1 @ 2 z @y @ 2 z @x
= 2 + 2u + + +
@u2 @x @x2 @u @y@x @u v @y 2 @u @x@y @u
✓ 2 ◆ ✓ ◆
@z @ x @ 2z 1 1 @ 2z @ 2z 1
= 2 + 2u 2u + + 2u +
@x @x2 @y@x v v @y 2 @x@y v
Utilizando finalmente el teorema de Schwarz tenemos que
@ 2z @z 2
2@ z 1 @ 2 z 4u @ 2 z
= 2 + 4u + + .
@u2 @x @x2 v @y 2 v @x@y

Problema 3

a) Si u = f (x, y, z) define una función diferenciable, y z se define im-


plicitamente como una función de x e y por la ecuación g(x, y, z) =
0 con los atributos pedido en el teorema de la función implı́cita.
Pruebe que u tiene primeras derivadas parciales de x e y dadas
por:
@(f,g) @(f,g)
@u @(x,z) @u @(y,z)
= @y ; = @y
@x @z
@y @z

253
b) Si u = x2 y + z 2 , y z = g(x, y) se define implı́citamente por la
ecuación

x2 y 3z + 8yz 3 = 0

Calcule:
@u @u
(1, 0, 0) y (1, 0, 0)
@x @y
Solución.

a) Utilizando la regla de la cadena tenemos

@u @f @f @z
= +
@x @x @z @x

por otra parte si g(x, y, z) = 0 define implı́citamente a z = z(x, y)


entonces
@g
@z
= @x @g
@x @z

reemplazando en la ecuación anterior


@g
@u @f @f @x
= + ( @g
)
@x @x @z @z
@f @g @f @g @(f,g)
@x @z @z @x @(x,z)
= @g
= @g
@z @z

Similarmente
@g
@u @f @f @z @z @y
= + y = @g
@y @y @z @y @y @z

De lo anterior se deduce
@g
@u @f @f @y
= + ( @g
)
@y @y @z @z
@f @g @f @g @(f,g)
@y @z @z @y @(y,z)
= @g
= @g
@z @z

254
b) En este caso u = f (x, y, z) = x2 y + z 2 y z = z(x, y) se define
implı́citamente

por g(x, y, z) = x2 y 3z + 8yz 3 = 0 ,luego tenemos

@g @g @g
= 2xy , = x2 + 8z 3 , = 3 + 24yz 2
@x @y @z

derivadas que son todas continuas por lo que se afirma que g es de C1

Además g(1, 0, 0) = 0 y @g
@z
(1, 0, 0) = 3 6= 0
Entonces por el teorema de la función implı́cita se tiene que existe
V = V (1, 0) y una vecindad ( a, a) de z = 0 y una función z(x, y)
de
C1 sobre V tal que

z(1, 0) = 0 y z(1, 0)✏( a, a)

Calculemos el Jacobiano
@(f, g) 2xy 2z
=
@(x, z) 2xy 3 + 24yz 2
= 2xy( 3 + 24yz 2 ) 2xy2z
= 2xy( 3 + 24yz 2 2z)

Por otra parte


@g
= 3 + 24yz 2
@z
Entonces

@u 2xy( 3 + 24yz 2 2z)


= )
@x 3 + 24yz 2
@u 0
(1, 0, 0) = =0
@x 3
También:

@(f, g) x2 2z
=
@(y, z) x2 + 8z 3 3 + 24yz 2
= 3x2 + 24x2 yz 2 2x2 z 16z 3
= x2 (24yz 2 2z 3) 16z 3

255
Finalmente

@u x2 (24yz 2 2z 3) 16z 3
) =
@y 3 + 24yz 2
@u 3
) (1, 0, 0) = =1
@y 3

3.4.4. Plano Tangente a una Superficie

Problema 1

Probar que los planos tangentes a la superficie S: xyz = a, a > 0


constante, en cualquier punto de S forma con los planos coordenados
un tetraedro de volumen constante.
Solución.
Sea la superficie S descrita por la función implı́cita F (x, y, z) = xyz
a = 0.
los vectores normales a la superficie S satisfacen rF (x, y, z) = (yz, xz, xy)
! !
Si P 0 = (x0 , y0 , z0 ) 2 S,entonces N (P0 ) = rF (x0 , y0 , z0 ) = (y0 z0 , x0 z0 , x0 y0 )
!
La ecuación del plano tangente al punto P 0 2 S, esta definida por
! !
(!r P )· N (P ) = 0,luego y z (x x )+x z (y y )+x y (z z ) =
0 0 0 0 0 0 0 0 0 0 0
0.
Entonces, las trazas de este plano con los ejes coordenados son
3x0 y0 z0
i) Si x = ↵, y = 0, z = 0 =) ↵ = = 3x0
y0 z 0
3x0 y0 z0
ii) Si x = 0, y = , z = 0 =) = = 3y0
x0 z 0
3x0 y0 z0
iii) Si x = 0, y = 0, z = =) ↵ = = 3z0
x0 y0
El volumen del tetraedro es:
↵ (3x0 ) (3y0 ) (3z0 ) 9
V = = = a, constante.
6 6 2

256
Problema 2

a) Probar que S1 dada por F (x, y, z) = 0 y S2 dada por G(x, y, z) = 0


son ortogonales en sus puntos de intersección sı́ y solo si
Fx Gx + Fy Gy + Fz Gz = 0.

b) Probar que las superficies S1 : x2 + y 2 + z 2 2x + 4y 8z = 0 y


S2 : 3x2 y 2 + 2z 2 6x 4y 16z + 31 = 0 son ortogonales.
Solución.
!
a) La normal a S1 dada por F (x, y, z) = 0 es N 1 = rF (x, y, z) =
(Fx , Fy , Fz ).
Del mismo modo, la normal a S2 dada por G(x, y, z) = 0 es

!
N 2 = rG(x, y, z) = (Gx , Gy , Gz )

Por definición, ambas superficies son ortogonales si rF · rG = 0 ()


(Fx , Fy , Fz ) · (Gx , Gy , Gz ) = 0 =)
F x Gx + F y Gy + F z Gz = 0

b) Sea S1 : x2 + y 2 + z 2 2x + 4y 8z = 0 =)

(x 1)2 + (y + 2)2 + (z 4)2 21 = 0

y S2 : 3x2 y 2 + 2z 2 6x 4y 16z + 31 = 0 =)

3(x 1)2 (y + 2)2 + 2(z 4)2 = 0

Entonces S1 tiene normal


!
N 1 = rF (x, y, z) = (2x 2, 2y + 4, 2z 8)

Asimismo , S2 tiene normal


!
N 2 = rG(x, y, z) = (6x 6, 2y 4, 4z 16)

=) rF · rG = (2x 2, 2y + 4, 2z 8) · (6x 6, 2y 4, 4z 16)


= (2x 2) (6x 6) + (2y + 4) ( 2y 4) + (2z 8) (4z 16)
2 2 2
= 4 [3(x 1) (y + 2) + 2(z 4) ] = 4 · 0 = 0
Por lo tanto, la superficie S1 es ortogonal a S2 .

257
3.4.5. Derivadas Direccionales

Problema 1
8
< 2xy 2
, si (x, y) 6= (0, 0)
Sea f (x, y) : x2 + y 4 . Determine, si existe, la
:
0 , si (x, y) = (0, 0)
derivada direccional de f en P0 = (0, 0).
Solución.
Sea eb = (e1 , e2 ) un versor en una dirección cualquiera de IR2 tal que
f ((0, 0) + (e1 , e2 )) f (0, 0) f ( e1 , e2 ) f (0, 0)
Dbe f (0, 0) = lı́m = lı́m
!0 !0
2
2 ( e1 ) ( e2 ) 2 3 e1 e22
( e1 )2 + ( e2 )4 2 (e2 + 2 e4 )
1 2
Dbe f (0, 0) = lı́m = lı́m
!0 !0

Por lo tanto
2e22
Dbe f (0, 0) = lı́m ,este lı́mite existe si y solo si e1 6= 0
!0 e1

Problema 2
8
< x (x + y)
, si (x, y) 6= (0, 0)
Sea f (x, y) = x2 + y 2 . Determine, si existe,
: 0 , si (x, y) = (0, 0)
la
derivada direccional de f en P0 = (0, 0).
Solución.
Sea eb = (e1 , e2 ) un versor en una dirección cualquiera de IR2 tal que
f ((0, 0) + (e1 , e2 )) f (0, 0) f ( e1 , e2 ) f (0, 0)
Dbe f (0, 0) = lı́m = lı́m
!0 !0

e1 (e1 + e2 )
Dbe f (0, 0) = lı́m
!0 (e2 2
1 + e2 )

Este lı́mite existe si y solo si e1 (e1 + e2 ) = 0 =) e1 = 0 ó (e1 + e2 ) = 0


i) Si eb = (0, e2 ) = 0, Dbe f (0, 0) = 0
ii) Si eb = (e1 , e1 ) = 0, Dbe f (0, 0) = 0.

258
Problema 3

Hallar la derivada direccional de f (x, y, z) = x2 yz 3 en el punto P0 =


(1, 1, 1) en la dirección de la tangente a la trayectoria : ! r (t) =
t
(e , 1 + 2sen (t) , t cos (t)) .
Solución.
Como f (x, y, z) = x2 yz 3 es una función diferenciable en R3 ,entonces
Dbt f (P0 ) = rf (P0 ) · b
t, donde rf (P ) = (2xyz 3 , x2 z 3 , 3x2 yz 2 )
El punto P0 que corresponde a t = t0 es:
!
r (t0 ) = (e t0 , 1 + 2sen (t0 ) , t0 cos (t0 )) = (1, 1, 1) =) e t0
=1
Ası́, t0 = ln (1) = 0
El vector tangente a la curva es:
!
r (t) = ( e t , 2 cos (t) , 1 + sen (t)) , entonces !
0 0
r (0) = ( 1, 2, 1) y
el vector tangente unitario en ésta dirección queda
! 0
b r (0) ( 1,2,1)
t= !
r 0 (0)
= p
k k 6

Por tanto, la derivada direccional es


( 1,2,1)
Dbt f (P0 ) = ( 2, 1, 3) · p = p3 >0
6 6
El valor positivo, significa que f aumenta en ésta dirección.

Problema 4

Calcular
p la pderivada direccional de f (x, y, z) = xy +xz yz en el punto
P0 = 2, 2, 0 en dirección de la tangente a la curva determinada
por las superficies

x2 + y 2 + z 2 = 4, x + y + z = 0

Solución.
Como f (x, y, z) = xy + xz yz es una función diferenciable en
R3 ,entonces
Dbt f (P0 ) = rf (P0 ) · b
t, donde rf (P ) = (y + z, x z, x y) =)
⇣ p p p ⌘
rf (P0 ) = 2, 2, 2 2

259
! 0
r (P0 )
Aún falta calcular el vector b
t= ! , que es tangente a la curva
r 0 (P )
k 0 k

determinada por las superficies dadas.


Sea C dada por !
r (x) = (x, y (x) , z (x)) =) !
0
r 0 (x) = 1, y (x) , z 0 (x) ,
0
donde y (x) , z 0 (x) se calculan implı́citamente a partir del sistema
de ecuaciones por derivación con respecto a x, en el entendido
que y = y (x) , z = z (x) . En efecto:
2x + 2yy 0 + 2zz 0 = 0 · 12 yy 0 + zz 0 = x
0 0 () 0 0
1+y +z =0 y +z = 1
Resolviendo el último sistema obtenemos
x z y x
0 1 1 x+z 0 1 1 x y
y (x) = = , z (x) = =
y z y z y z y z
1 1 1 1
Evaluando
p p
0 p 2 0 p 2 2
y 2 = p =1yy 2 = p = 2
2 2
Luego
p ! 0
r (P0 ) (1, 1, 2)
=) r ! 0 b
2 = (1, 1, 2) =) t = !0 = p
k r (P0 )k 6
Por lo tanto, se tiene que

p p p (1, 1, 2) 4
Dbt f (P0 ) = ( 2, 2, 2 2) · p =p
6 6

3.4.6. Valores extremos


Problema 1

Sea f : U ✓ R2 ! R definida en el abierto U , dada por f (x, y) =


x4 2px2 y 2 + 3 donde p es una constante entera. Calcular los valores
extremos de la función
Solución
Apliquemos la condición necesaria de punto crı́tico, calculando las derivadas
parciales e igualándolas a cero, luego
fx = 4x3 4px = 4x(x2 p); fy = 2y.

260
Tenemos que fy = 2y = 0 =) y = 0, pero en la derivada parcial
respecto de x hay que analizar el signo de p.
p
i) Si p > 0, f x = 4x(x2 p) = 0 =) x = 0, ± p y los puntos
crı́ticos son
p p
P1 = (0, 0), P2 = ( p, 0) y P3 = ( p, 0).
ii) Si p = 0, f x = 4x3 = 0 =) x = 0, y el único punto crı́tico es
P1 = (0, 0).
iii) Si p < 0, f x = 4x(x2 p) = 0 =) x = 0, y el único punto
crı́tico es
P1 = (0, 0)
Estudiemos el Hessiano para ver si los puntos obtenidos son máximos,
mı́nimos o puntos silla.
fxx = 12x2 4p, fxy = 0, fyx = 0, fyy = 2
Por tanto el determinante de la matriz Hessiana es

12x2 4p 0
H (x, y) =
0 2

Estudiemos los puntos crı́ticos según los valores del parámetro p


1) Si p > 0: En este caso hay 3 puntos crı́ticos.

4p 0
H (0, 0) = = 8p > 0 y fxx = 4p < 0
0 2

Entonces, el punto P1 = (0, 0) es un Máximo Relativo.

p 8p 0
H (± p, 0) = = 16p < 0
0 2

p p
Por lo tanto, los puntos P2 = ( p, 0) y P3 = ( p, 0) son Puntos Silla.
2) Si p < 0: En este caso en P1 = (0, 0) hay un punto crı́tico. H (0, 0) =
4p 0
= 8p < 0, luego en P1 = (0, 0) hay un Punto Silla.
0 2
3) Si p = 0: En este caso hay un único punto crı́tico en el origen, pero
al calcular el determinante del Hessiano, obtenemos que H(0, 0) = 0,
luego no se tiene información.

261
Sin embargo, podemos comprobar la naturaleza del punto aplicando la
definición de extremo local en las cercanı́as del origen, siguiendo dos
trayectorias distintas, para la función: f (x, y) = x4 y 2 + 3. Entonces
f (x, 0) f (0, 0) = x4 > 0 para todo (x, 0) 2 V (0, 0)
f (0, y) f (0, 0) = y 2 < 0 para todo (0, y) 2 V (0, 0).
Por lo tanto, P1 = (0, 0) es un Punto de Silla

Problema 2

1 4
Encuentre los valores extremos de la f (x, y) = x2 + y 2 2
x.
Solución.
Derivando parcialmente con respecto a x e y tenemos:
fx (x, y) = 2x 2x3
fy (x, y) = 2y
Esta claro que fx y fy son funciones continuas en R2
Aplicando la condición necesaria para los puntos crı́ticos de f
tenemos: 2x 2x3 = 0; 2y = 0.
Al resolver el sistema obtenemos tres puntos crı́ticos.
P0 = (0, 0) , P1 = (1, 0) , P2 = ( 1, 0)
Determinemos el Hessiano H (x, y) :
fxx (x, y) fxy (x, y) 2 6x2 0
H (x, y) = = =4 12x2
fyx (x, y) fyy (x, y) 0 2
Evaluemos el Hessiano H (x, y) en cada uno de los puntos:
i) Para P0 = (0, 0) =) H (0, 0) = 4 > 0 y fxx (0, 0) = 2 > 0
Entonces en P0 hay un mı́nimo relativo f (0, 0) = 0.
ii) Para P1 = (1, 0) =) H (1, 0) = 8 <0.
Entonces, en P1 hay punto silla de f.
iii) Para P2 = ( 1, 0) =) H ( 1, 0) = 8 <0.
Ası́, en P2 tambien hay un punto silla de f.

262
Problema 3

Encuentre los valores extremos de la f (x, y) = x2 + y 2 xy + x + y


en el dominio D = {(x, y) 2 R / x  0, y  0, x + y
2
3} .
Solución.
En primer lugar, determinemos los valores extremos en el conjunto
abierto: D⇤ = {(x, y) 2 R2 / x < 0, y < 0, x + y > 3} .
Derivando parcialmente con respecto a x e y tenemos:
fx (x, y) = 2x y+1
fy (x, y) = 2y x+1
Observe que fx y fy son continuas en R2
Aplicando la condición necesaria para los puntos crı́ticos de f
tenemos el sistema:
2x y= 1; x + 2y = 1.
Al resolver este sistema obtenemos un único punto crı́tico
P0 = ( 1, 1) 2 D⇤
Determinemos el Hessiano H (x, y) :
fxx (x, y) fxy (x, y) 2 1
H (x, y) = = = 3 8 (x, y) 2 D⇤ :
fyx (x, y) fyy (x, y) 1 2
Ası́, P0 = ( 1, 1) =) H ( 1, 1) = 3 > 0 y fxx ( 1, 1) = 3 > 0
Se concluye, que en P0 hay un mı́nimo relativo f ( 1, 1) = 1.
En segundo lugar, estudiemos la condición que se presenta en la
frontera de D.
a) Si y = 0, f (x, 0) = x2 + x con x 2 [ 3, 0]
Determinemos los puntos crı́ticos en este borde
0 1
f (x) = 2x + 1 = 0 =) x = 2
1
Luego, hay un punto critı́co en P1 = 2
,0 2 D.
00
1
Como f (x) = 2 > 0, 8x 2 [ 3, 0] , entonces en P1 = 2
,0 hay
1 1
un mı́nimo f 2
,0 = 4
.
2
b) Si x = 0, f (0, y) = y + y con y 2 [ 3, 0]
Determinemos los puntos crı́ticos en este borde

263
0 1
f (y) = 2y + 1 = 0 =) y = 2
1
Luego, se tiene un punto critı́co en P2 = 0, 2
2 D.
00
1
Como f (y) = 2 > 0, 8y 2 [ 3, 0] ,entonces en P2 = 0, 2
hay
1 1
un mı́nimo f 0, 2
= 4
.
c) Si x+ y = 3, f (x, x 3) = 3x2 + 9x + 6 con x 2 [ 3, 0]
Determinemos los puntos crı́ticos en este borde
0 3 3
f (x) = 6x + 9 = 0 =) x = 2
=) y = 2
3 3
Luego, hay un punto critı́co en P2 = 2
, 2
2 D.
00
3 3
Como f (x, x 3) = 6 > 0, 8x 2 [ 3, 0] ,entonces P2 = 2
, 2
3 3 3
en hay un mı́nimo f 2
, 2
= 4
.

3.4.7. Multimplicadores de Lagrange para extremos


restringidos
Problema 1

Obtener los extremos de la función f (x, y) = x3 + 2xy + y 2 que se


encuentren en la recta x + y = 0.
Solución
En primer lugar consideremos la función de Lagrange y apliquemos la
condición necesaria de punto crı́tico:
F (x, y, ) = x3 + 2xy + y 2 + (x + y)
Fx = 3x2 + 2y + =0 (1)
F y = 2x + 2y + =0 (2)
Fz = x + y = 0 (3)
De la (3) ecuación obtenemos: y = x que sustituida en la segunda:
2x 2x + = 0 =) =0
Sustituido este valor en la ecuación (1), queda: 3x2 2x = 0, x = 0,
x = 2/3
✓ ◆
2 2
Entonces los puntos crı́ticos del lagrangiano son. P1 (0, 0), P2 ,
3 3
.
Clasificaremos los puntos con el determinante del Hessiano Limitado:

264
0 1 1
H (x, y) = 1 6x 2
1 2 2

evaluándo en los puntos crı́ticos obtenidos anteriormente.


0 1 1
d2 f
Consideremos H (0, 0) = 1 0 2 = 2 > 0, entonces < 0 y en
dx2
1 2 2
P1 (0, 0) hay un máximo local condicionado de f, cuyo valor es f (0, 0) =
0.
✓ ◆ 0 1 1
2 2 d2 f
Por otra parte H , = 1 4 2 = 2 > 0, entonces <0
3 3 dx2
✓ ◆ 1 2 2
2 2
y en P2 , hay un mı́nimo local condicionado de f, cuyo valor
✓ 3 ◆3
2 2 4
es f , = .
3 3 27

Problema 2
x2 y2
En que puntos de la elipse 2 + 2 = 1, la tangente a este lugar
a b
geométrico forma con los ejes coordenados un triángulo de área mı́nima.
Solución.
Sea la ecuación de la tangente a la elipse en el punto (x0 , y0 ) .
x0 x y0 y
+ 2 =1
a2 b
1
Sea f (x, y) = xT yT , el área que forma la recta tangente con los ejes
2
coordenados, donde xT yT se determinan a partir de la ecuación de
la tangente.
a2 b2
Si yT = 0 =) xT = xT = 0 =) yT =
x0 y0
1 a2 b 2
Ası́ f (x, y) = es la función a estudiar, que verifica la condición:
2 x0 y0
x20 y02
+ 2 1 = 0.
a2 b
Consideremos la función :

265
✓ ◆
1 a2 b 2 x20 y02
L (x, y, ) = + + 2 1
2 x0 y0 a2 b

1 a2 b 2 x0
Lx (x, y, ) = 2
+ 2 2 = 0 ·1/y0
2 x0 y0 a
1 a2 b 2 y0
Ly (x, y, ) = 2
+ 2 2 = 0 ·/x0
2 x0 y0 b
x20 y02
L (x, y, ) = 2 + 2 1 =0
a b
Multiplicando las ecuaciones anteriores por los coeficientes que se
indican, tenemos
1 a2 b 2 2 x0
1,0) 2 2
+ =0 ·1/y0
2 x0 y0 a2 y 0
1 a2 b 2 2 y0
2,0) 2 2
+ =0 ·1/x0
2 x0 y0 b2 x 0
x2 y 2
3,0) 20 + 20 1 = 0
a b

Restando 2,0 1,0 se tiene


2 x0 2 y0 a2 2
= 2 =) x20 = y
2
a y0 b x0 b2 0
Sustituyendo este resultado en 3.0) se tiene un único punto crı́tico de
f en
✓ ◆
a b
P0 = p , p
2 2
Mediante el criterio de la segunda derivada se puede determinar la
naturaleza del punto crı́tico
✓ 0 ◆
1 a2 b 2 0 a2 b 2 y0 x0 y0 (x)
f (x, y(x)) = =) f (x) =
2 x0 y0 2 (x0 y0 )2
donde a partir de la condición obtenemos
 00
2x0 2y0 0 0 b2 x 0 0 b2 y0 x0 y0
+ 2 y0 (x) = 0 =) y0 (x) = =) y0 (x) =
a 2 b 2
a y0 a2 y02
Ası́
✓ 0 00 0 ◆
00 a2 b 2 (x0 y0 )2 (2y0 + x0 y0 (x)) 2(x0 y0 )(y0 + x0 y0 (x))2
f (x) =
2 (x0 y0 )4
Produce

266
✓ ◆
00 a b
f p ,p >0
2 2
✓ ◆
a b
Por lo tanto, en el punto P0 = p ,p un mı́nimo de f
2 2

Problema 3

Se desea construir una tolva para un silo, que tenga una capacidad
de 100 m3 y forma de cono circular recto de 2m de radio, coronado
por un cilindro circular recto, empleando un mı́nimo de material para
la superficie. Calcular las alturas x del cilindro e y del cono para tal
objeto.
Solución:
p
Sea la función superficie definida por f (x, y) = 2⇡ 4 + y 2 +4⇡x
Con la condición que el volumen sea g (x, y) = 43 ⇡y + 4⇡x 100 = 0
Entonces formemos la función:
p 4
L (x, y, ) = f (x, y) = 2⇡ 4 + y 2 +4⇡x + 3
⇡y + 4⇡x 100
Lx (x, y, ) = 4⇡ + 4⇡ =0 =) = 1
4⇡y 4 y 1
Ly (x, y, ) = p + ⇡ =0 =) p =
2 4 + y2 3 2 4+y 2 3
L (x, y, ) = 43 ⇡y + 4⇡x 100 = 0

4
9y 2 = 4 (4 + y 2 ) =) 5y 2 = 16
=) y = p
5
Sustituyendo en la restricción se tiene

16 100 4
4⇡x = 100 p ⇡
3 5
=) x = p
3 5
4⇡
En consecuencia, se tiene un único punto crı́tico en
✓ ◆
100 4 4
P0 = p ,p
4⇡ 5 5
La condición de mı́nimo de f se estable mediante la segunda derivada
p 0 4⇡yy 0
f (x, y(x)) = 2⇡ 4 + y + 4⇡x =) f (x) = p
2 + 4⇡
2 4 + y2
4
3
⇡y + 4⇡x 100 = 0 =) y 0 (x) = 3

Por lo tanto, sustituyendo y 0 (x) , y derivando por segunda vez

267
!
0 6⇡y 00 ( 3(4 + y 2 ) y2
f (x) = p + 4⇡ =) f (x) = 6⇡
4 + y2 (4 + y 2 )3/2
00
f (P0 ) > 0 =) Valor mı́nimo
Ası́ el valor mı́nimo de la función es:
✓ ◆ q ✓ ◆
100 4 4 16 100 4 20⇡
f p ,p = 2⇡ 4 + 5 +4⇡ p = 100 p
4⇡ 5 5 4⇡ 5 3 5

Problema 4

Determine la distancia mı́nima y máxima del origen a la curva de in-


7
tersección del paraboloide z = x2 y 2 y el plano x + y + z = 2.
4
Solución:
En este caso es conveniente los valores extremos del cuadrado de la
distancia con respecto al origen en vez de la distancia misma. Por lo
tanto, se deben hallar los valores extremos de la función:

f (x, y, z) = x2 + y 2 + z 2

sujeta a las restriciones

7
g (x, y, z) = z + x2 + y 2 = 0
4
h (x, y, z) = x + y + z 2 = 0

Para aplicar el método de los multiplicadores de Lagrange se define


✓ ◆
2 2 2 7 2 2
F (x, y, z, 1 , 2 ) = x +y +z + 1 z + x + y + 2 (x + y + z 2)
4
Fx = 2 (1 + 1 ) x + 2 = 0 (1,0)
Fy = 2 (1 + 1 ) y + 2 2 = 0 (2,0)
Fz = 2z + 1 + 2 = 0 (3,0)
7
F 1=z + x2 + y 2 = 0 (4,0)
4
F 2 =x+y+z 2=0 (5,0)
1,0) 2,0) : 2 (1 + 1 ) (x y) = 0 =) 1 =
1 o y=x
1
Si 1 = 1 ,entonces de 1) 2 = 0 y de 3) z =
2
268
1
Si z = , entonces de 4) y 5) se obtiene: 2x2 3x + 1 = 0
2
1
Resolviendo la ecuación anterior, sus soluciones son: x1 = 1; x2 =
2
✓ ◆
1 1 1
x1 = 1 =) y1 = =) 1, , es punto crı́tico.
2 2 2
✓ ◆
1 1 1
x2 = =) y2 = 1 =) , 1, es punto crı́tico.
2 2 2
1
Por otra parte:4) 5) =) x2 + y 2 x y+ =0
4
p
1 2 ± 2
Si y = x =) 2x2 2x + = 0, resolviendo la ecuación x =
4 4
p p p p p !
2+ 2 4 2 2 2+ 2 2+ 2 4 2 2
y=x= =) z = =) , ,
4 4 4 4 4
es punto crı́tico de f.
p p p p p !
2 2 4+2 2 2 2 2 2 4+2 2
y=x= =) z = =) , ,
4 4 4 4 4
es punto crı́tico de f.
Ası́
p p p !
2± 2 2± 2 4⌥2 2 1 p
fmáx , , = 9+2 2
4 4 4 4
✓ ◆
1 1 3
fmı́n , 1, =
2 2 2

Como la curva intersección del paraboloide y el plano es una curva


cerrada, la distancia
r mı́nima y la distancia máxima al origen son re-
q p
3 1
spectivamente y 9 + 2 2 .No necesitamos más pruebas por
2 2
las caracteristicas geométricas del problema.

Problema 5

Demuestre que las distancias máxima y mı́nima desde el origen a la


x2 y 2 z 2
curva de intersección definida por + + = 1, z = x + y.
4 5 25
Solución:

269
Debenos encontrar los valores extremos de la función

f (x, y, z) = x2 + y 2 + z 2

sujeta a las restriciones

x2 y 2 z 2
g (x, y, z) = + + 1=0
4 5 25
h (x, y, z) = x + y z = 0

Para aplicar el método de los multiplicadores de Lagrange se define


✓ 2 ◆
2 2 2 x y2 z2
F (x, y, z, 1 , 2 ) = x +y +z + 1 + + 1 + 2 (x + y z)
4 5 25
Aplicando la condición necesaria de punto crı́tico
✓ ◆
1
Fx = 2 1 + x+ 2 =0 (1,0)
✓ 4◆
1
Fy = 2 1 + y+ 2=0 (2,0)
5
1
Fz = 2(1 + + 2=0 (3,0)
5
x2 y 2 z 2
F 1= + + 1=0 (4,0)
4 5 25
F 2 =x+y z =0 (5,0)
Despejando de estas ecuaciones x, y, z se tiene
2 2 5 2 25 2
x= ; y= ; z= ; 6,0)
1+4 2 1 + 10 2 1 + 50
Al dividir 5,0 por 2 6= 0 (lo cual está justificado porque de otro
modo de 1,0, 2,0 y 3,0, se tendrı́a x = y = z = 0).
2 5 25
+ + = 0. Multiplicando por 2 ( 1 + 4) (2 1 + 10) (2 1 + 50)
1 + 4 2 1 + 10 2 1 + 50
y simplificando da
2
17 1 + 245 1+ 750 = 0 =) ( 1 + 10)(17 1 + 75) = 0
75
de donde: 1 = 10, 1 =
17
2 25 2
Caso i) Si 1 = 10, entonces de 6,0 : x = ; y= ;z =
.
3 2 6
2 2
5 22
2 2 180
Sutituyendo en 4.0 da: + + 1 = 0 =) 22 = =)
36 20 66 19
270
r
5
2 = ±6
19
Por lo tanto, se tienen dos puntos crı́ticos.
r r r ! r r r !
5 5 5 5 5 5
P1 = 2 ,3 ,5 y P2 = 2 . 3 , 5
19 19 19 19 19 19
r r r !
5 5 5
Evaluando en la función se tiene f ±2 , ±3 , ±5 = 10
19 19 19
75 34 2 17 2
Caso ii) Si 1 = , entonces de 6,0 : x = ; y = ;z =
17 7 4
17 2
.
28
2 2
2 5 222 2 (140)2
Sutituyendo en 4.0 da: + +
1 = 0 =) 2 = =)
36 20 66 (17)2 (646)
p
2 = ±14017 646
Por lo tanto, se tienen otros dos puntos crı́ticos más.
✓ ◆ ✓ ◆
40 35 5 40 35 5
P1 = p , p p y P2 = p ,p , p
646 646 646 646 646 646
✓ ◆
40 35 5 75
Evaluando en la función se tiene f ± p , ⌥p , ±p =
646 646 646 17
75
Asi el valor máximo buscado es 10 y el valor mı́nimo es
17

Problema 6

Se desea construir un silo, que tenga una capacidad de V0 con forma de


cilindro circular recto de altura h y radio basal r . Calcular la altura
h del cilindro y radio basal r de manera que la superficie total sea
mı́nima.
Solución:
Sea la función superficie definida por f (r, h) = 2⇡r2 +2⇡rh
Con la condición que el volumen sea g (x, y) = ⇡r2 h V0 = 0
Entonces formemos la función:
L (r, h, ) = 2⇡r2 + 2⇡rh + (⇡r2 h V0 )

271
Lr (r, h, ) = 4⇡r + 2⇡h + 2 ⇡rh = 0 1,0)
Lh (r, h, ) = 2⇡h + ⇡r2 = 0 2,0)
L (r, h, ) = ⇡r2 h V0 = 0 3,0)

2
De 2,0) se tiene: = y sustituyendo este valor en 1,0) obtenemos
r
h = 2r
r r
3 V0 V0
Si h = 2r, entonces de 3.0) r = ;h = 2 3
2⇡ 2⇡
En consecuencia, se tiene un único punto crı́tico en
r r !
3 V0 V0
P0 = ,2 3
2⇡ 2⇡
La condición de mı́nimo de f se establece mediante la segunda derivada
0
f (r, h(r)) = 6⇡r2 =) f (r) = 12⇡r
00
=) f (r) = 12⇡ > 0

Por lo tanto, se tiene un valor mı́nimo de f si h = 2r


Ası́ el valor mı́nimo de la superficie es:
r r ! ✓ ◆2/3
3 V 0 3 V0 V0
f ,2 = 6⇡
2⇡ 2⇡ 2⇡

Problema 7:

Determinar los extremos absolutos de la función f (x, y) = y 3 + x2 y +


2x2 + 2y 2 4y 8 en el conjunto D = {(x, y) 2 IR2 /x2 + y 2  1} .
Solución:
En primer lugar estudiemos los puntos del interior de D, para ver si
existen máximos o mı́nimos locales.
La condicion necesaria, de los puntos interiores candidatos a extremos,
es
@f
= 2x(y + 2) = 0
rf (x, y) = (0, 0) =) @f @x
= 3y 2 + x2 + 4y 4 = 0
@y
i) La primera ecuación implica que x = 0 ó y = 2. Si y = 2, la
segunda ecuación implica que x = 0, luego se tiene un punto crı́tico en
P0 = (0, 2) , sin embargo , P0 2/ D.

272
ii) Si x = 0, la segunda ecuación es 3y 2 + 4y 4 = 0 =) y = 2,
2
y= .
3
✓ ◆
2 4
Las coordenadas del punto P1 = 0, verifican < 1,entonces P1 2
3 9
8 8 8 256
D. Además f (P1 ) = + 8= = 9, 48.
27 9 3 27
En segundo lugar, estudiemos los puntos de la frontera de D usando
la función f (x, y) = y 3 + x2 y + 2x2 + 2y 2 4y 8 bajo la restrición
g (x, y) = x2 + y 2 1 = 0.
Usemos el método de los multiplicadores de Lagrange.
Sea L (x, y, ) = y 3 + x2 y + 2x2 + 2y 2 4y 8 + (x2 + y 2 1),
y obtenemos:
@L
= 2x(y + 2) + 2x = 0 (1,0)
@x
@L
= 3y 2 + x2 + 4y 4 + 2y = 0 (2,0)
@y
@L
= x2 + y 2 1 = 0 (3,0)
@
De la ecuación 1,0 se tiene que x = 0 ó (y + 2) + =0
a) Si x = 0,en 3.0 se tiene y 2 1 = 0 =) y = ±1.Luego se tienen
otros dos puntos crı́ticos
P2 = (0, 1) y P2 = (0, 1) que satisfacen las ecuaciones 1,0 y 3,0.
Para comprobar que también satisfacen la ecuación 2,0 , sustituyamos
el ella
3
P1 = (0, 1) =) = 2 IR
2
5
P2 = (0, 1) =) = 2 IR.
2
Si evaluamos las función en los puntos encontrados obtenemos:
f (P2 ) = 1 + 2 4 8= 9
f (P3 ) = 1+2+4 8= 3
b) Si (y + 2) + = 0 () = (y + 2), en 2,0 se tiene 3y 2 + x2 +
4y 4 + 2y(y + 2) = 0
=) x2 + y 2 = 4, resultado que contradice la ecuación 3,0, x2 + y 2 = 1.
Luego, esta condición no produce un punto crı́tico.

273
Por lo tanto, comparando los valores de la función en los tres puntos
encontrados, podemos inferir que el máximo absoluto se alcanza en P3
y que el mı́nimo absoluto se alcanza en P1

Problema 8:

Determine las dimensiones de una caja rectangular, sin tapa


superior,que ha de tener un volumen dado V0 ,de manera que
su superficie sea mı́nima.
Solución:

Sea la función superficie definida por f (x, y, z) = xy + 2xz + 2yz


Con la condicion que el volumen sea g (x, y, z) = xyz V0 = 0
Entonces formemos la función:
L (x, y, z, ) = f (x, y) = xy + 2xz + 2yz + (xyz V0 )

Lx (x, y, z, ) = y + 2z + yz = 0 (1,0)
Ly (x, y, z, ) = x + 2z + xz = 0 (2,0)
Lz (x, y, z, ) = 2x + 2y + xy = 0 (3,0)
L (x, y, z, )= xyz V0 = 0 (4,0)

2z
Despejando y y x de 1,0 y 2,0 se tiene 5.0) y = x = 1+ z
, sutituyendo
en 3,0) produce:
8z 4 z2 1
1+ z
+ (1+ z)2
= 0 =) z = 6,0)
Reemplazando en 6,0) en 5,0) se tiene
4
y=x= . Sustituyendo en 4.0
⇣ ⌘1/3
4 4 1
= V0 =) = V160
Por lo tanto, se tiene un único punto crı́tico de f en
⇣ ⌘
1/3 1/3 1/3
P0 = 4 V160 , 4 V160 , V160 .
Examinemos la naturaleza del punto crı́tico usando el Hessiano limita-
do:
@g @g @g
0 @x @y @z
@g @2L @2L @2L
@x @x2 @x@y @x@z
H (x, y, z) = @g @2L @2L @2L =)
@y @x@y @y 2 @y@z
@g @2L @2L @2L
@z @x@z @y@z @z 2

274
0 yz xz xy
yz 0 1+ z 2+ y
H (x, y, z) =
xz 1 + z 0 2+ x
xy 2 + y 2 + x 0
⇣ ⌘
1/3 1/3 V0 1/3
Evaluemos el Hessiano H 4 V160 , 4 V160 , 16

V0 2/3 V0 2/3 V0 2/3


0 4 16
4 16
16 16
V0 2/3
4 16
0 2 6
H= V0 2/3
4 16
2 0 6
V0 2/3
16 16
6 6 0
Usando propiedades de determinantes, obtenemos que el valor de Hes-
siano es:

0 1 1 4
V0 4/3 1 0 2 6
H = 16 16 1 2 0 6
4 6 6 0
4
H= 2048,0 ⇥ (0,062 5V0 ) 3 < 0
0 1 1
Además: 3 = 1 0 2 =4>0
1 2 0
Entonces
⇣ la función f tendrá un⌘ máximo condicionado en el punto
1/3 1/3 1/3
P0 = 4 V160 , 4 V160 , V160

3.4.8. Aplicación al cálculo de errores


Problema 1:

El periodo T de un péndulo simple depende de la longitud l y de la


aceleración de gravedad g del lugar y está dado por:
r
l
T = 2⇡ . Hallar a a) el error absoluto y b) el error relativo , al
g
calcular T con l = 0, 6 m y g = 10m/s2 si los valores verdaderos eran
l = 58, 5cm y g = 9, 8m/s2 .
Solución:
r
l
a) Sea T = 2⇡ . el perı́odo de un péndulo simple.
g

275
El error absoluto de T es T, que en este caso es aproximadamente
dT. ası́ se tiene:
r
@T @T ⇡ l
El error absoluto de T = dT = dl + dg = p dl ⇡ dg
@l @g lg g3
Error de l = l = dl = (0, 6 0, 585) m = 0, 015m

Error de g = 9, 8) m/s2 = 0, 2m/s2


g = dg = (10
r
⇡ 0, 6
El error absoluto de T = dT = p (0, 015) ⇡ (0, 2)
0, 6x10 1000
✓ r ◆
dT 1 ⇡ l
b) El error relativo de T = = r p dl ⇡ dg
T l lg g3
2⇡
g

✓ ◆
1 1
El error relativo de T = dg .dl
2g 2l
✓ r ◆
dT 1 ⇡ l
El error relativo de T = = r p dl ⇡ dg
T l lg g3
2⇡
g

3.5. Ejercicios Propuestos

3.5.1. Lı́mites
Problema 1

Determinar si existen los siguientes limites, y si existen, calcular su


valor

a) lı́m ln xseny
ysenx
(x,y)!(0,0)

x2 y 2
b) lı́m
(x,y)!(0,0) x2 + y 2

Solución:
a) lı́m ln xseny
ysenx
= 0, b) El lı́mite de la función f no existe
(x,y)!0

276
Problema 2

En los siguientes casos, usar coordenadas polares para concluir que el


lı́mite de f (x, y) cuando (x, y) tiende a (0, 0) existe y vale cero.
y3
a) lı́m
(x,y)!(0,0) x2 + y 2

x3 y 4
b) lı́m
(x,y)!(0,0) x4 + y 4

3.5.2. Diferenciabilidad, continuidad


Problema 1
8
< 1
xysen si y 6= 0
Dada la función f (x, y) = y , probar que es
: 0 8 (x, 0)
diferenciable en el punto P0 = (0, 0) ,¿es continua en este punto?
Solución:
La función f es diferenciable en P0 = (0, 0) , implica que es continua
en (0, 0)

Problema 2

8
< p |x| y si (x, y) 6= (0, 0)
Dada la función f (x, y) = x2 + y 2 .
:
0 si (x, y) = (0, 0)
a) Estudiar su continuidad en IR2
b) Estudiar la existencia de derivadas parciales en IR2 y determinarlas,
si es que existen.
c) Estudiar su diferenciabilidad en IR2

Solución:
a) La función f es en continua en IR2
b) Existen derivadas parciales en
@f @f
IR2 {(o, a) /a 6= 0} ,No existe (0, a) , (0, a) = 0,
@x @y

277
@f @f
(0, 0) = 0, (0, 0) = 0
@x @y
c) La función f no es diferenciable en P0 = (0, 0) ,puesto que
| f df |
lı́m p 6= 0
(h,k)!(0,0) h2 + k 2

Problema 3
p
Verificar que f (x, y) = |xy| es continua y no diferenciable en (0, 0) .

Problema 4
( 4 4
arctg xx2 +y
+y 2
si (x, y) 6= (0, 0)
Dada la función f (x, y) =
a si (x, y) = (0, 0)
a) Determinar el valor de a para que la función sea continua en el
origen.
b) Para este valor de a calcular fx (0, 0), fy (0, 0)
b un vector unitario
c) Hallar la derivada direccional Dub f (1, 0), siendo u
o
que la forma un ángulo de 60 con la parte positiva del eje OX.
Solución:
a) f(x,y) es continua en (0,0) si a= f(0,0)
b) fx (0,0)=0 , fy (0,0)=0
1
c) Dub f (1, 0) = 2

3.5.3. Derivadas parciales


Problema 1
8
< x2 y 2
xy si (x, y) 6= (0, 0)
Sea f (x, y) = x2 + y 2
:
0 si (x, y) = (0, 0)
@f @f
a) Deducir que (0, y) = y 8y; (x, 0) = x 8x
@x @y
@ 2f @ 2f
b) Verificar que (0, 0) 6= (0, 0)
@y@x @x@y

278
Problema 2

x + y si x = 0 ó y = 0
Dada la función f (x, y) =
1 si x =6 0, y 6= 0
@f @f
a) Verifique que (0, 0) = 1 ; (0, 0) = 1
@x @y
b) Muestre que la función f no es continua en (0, 0)

Problema 3
xy
Sea u = , muestre que u satisface la ecuación:
x+y
@ 2u @ 2u 2
2@ u
x2 + 2xy + y =0
@x2 @y@x @y 2

Problema 4
1
Sea u = p , probar que u satisface la ecuación:
x2 + y2 + z2
@ 2u @ 2u @ 2u
+ + =0
@x2 @y 2 @z 2

Problema 5

Enuncie las condiciones del teorema de Schwarz de igualdad de las


derivadas cruzadas.

Problema 6

Justifique si es cierto que una función f : IR3 ! IR diferenciable en


IR3 tiene derivadas parciales.
Solución
Si una función f es diferenciable en !
x o y la diferencial es Df (!
x o) ,
!
entonces para todo vector unitario vb 6= 0 existe D f ( x ) que verifica
b
v o

Dvbf (!
x o ) = Df (!
x o ) · vb. En particular, esto sucede para los vectores
(1, 0, 0) , (0, 1, 0) , (0, 0, 1) , por lo que existen las derivadas parciales

279
Problema 7

Suponga que z es función de las variables x e y , que satisface la ecuación


que se da en cada caso . Encontrar las derivadas parciales de primer
@z @z
orden y
@x @y
a) x3 + y 3 + z 3 + senxz + cos yz = 15
b) ez + x2 ln z + y = 0.
Solución:
@z (3x2 + z cos xz) @z (3y 2 zsenxz)
a) = 2
, =
@x 3z + x cos xz ysenyz @y 3z 2 + x cos xz ysenyz
@z 2xz ln z @z z
b) = z 2
, =
@x ze + x @y ze + x2
z

Problema 8

Suponga que existen funciones u y v , que satisfacen las siguientes


ecuaciones.

u cos v = x + 1
u sin y = x + y

@u @u @v @v
Calcular , , ,
@x @y @x @y
Solución:
@u @u @v cos v senv @v cos v
= cos v + senv , = senv, = , =
@x @y @x u @y u

3.5.4. Derivadas Direccionales


Problema 1

Calcule la derivada direccional de la función dada en la dirección del


vector indicado.
a) f (x, y, z) = 2x2 y2 z 2 , en (1, 2, 2) hacia (4, 5, 0) .
b) f (x, y, z) = 2x2 8xy + z 2 , en el punto (4, 4, 1) en la dirección de
la

280
normal exterior a la superficie x2 + y 2 + z = 17
Solución:
8 446
a) Dvbf (1, 2, 2) = p , b) Dvbf (4, 4, 1) = p
22 129

Problema 2

Determine la ecuación del plano tangente al paraboloide x2 +y 2 +z 1 =


0 y que pasa por los puntos 1, 0, 12 y 0, 1, 12 .
Solución:
3
La ecuación de plano tangente es: x + y + z =
2

Problema 3

Sea S una superficie dada por la ecuación x3 + y 3 + z 3 a3 = 0, con a


constante, y ⇧ un plano tangente en P0 = (xo , yo, zo ) 2 S que intersec-
ta los ejes coordenados X,Y,Z en ↵, , respectivamente. Probar que
↵ 3/2 + 3/2 + 3/2 = a 3/2 es constante.

Problema 4

Determinar el plano tangente a la superficie S

p p p p
x+ y+ z= a, (a > 0)

en el punto P0 = (xo , yo, zo ) 2 S y demostrar que este plano corta los


ejes
coordenados en segmentos cuya suma de longitudes es constante.
Solución:
x xo y yo z z o
p + p + p =0
xo yo zo
p p p p
l = a( xo + yo + zo ) = a, (a > 0)

281
Problema 5

Muestre que si el punto P0 = (xo , yo, zo ) pertenece al elipsoide de


ecuación:
x2 y2 z2
⇡: 2 + 2 + 2 = 1 con (↵ < c2 < b2 < a2 )
a ↵ b ↵ c ↵
y al manto del hiperboloide de ecuación:
x2 y2 z2
✓: 2 + 2 + 2 = 1 con (c2 < < b2 < a2 )
a b c
Entonces las superficies ⇡ y ✓ se cortan ortogonalmente en P0 = (xo , yo, zo ) .

Regla de la cadena

Problema 1

Sean f : R2 ! R y g : R ! R, con f (x, g (x)) = x sin (g (x)) .


0
Sabiendo que g (1) = 0 y que f (1, 0) = 1, calcular g 0 (1) .
Solución:
El valor de la derivada es g 0 (1) = 1

Problema 2

Sea f : D ⇢ IR2 ! IR una función diferenciable homogénea de


grado p, es
decir que verifica la condición f (tx, ty) = tp f (x, y) 8t > 0, 8 (x, y) 2
IR2 .
@f @f
a) Demuestre la ecuación x +y = pf.
@x @y
b) Pruebe que la siguientes funciones son homogéneas f (x, y) = xy,
f (x, y) = x2 + 3xy + y 2 y luego verifique la ecuación anterior.

Problema 3

Sea g(t) = f (x (t) , y (t)) de clase C2 , deduzca que:

2 2
g 00 (t) = fxx (x, y) (x0 ) +2fxy (x, y) x0 y 0 +fyy (x, y) (y 0 ) +fx (x, y) x00 +fy (x, y) y 00

282
Problema 4

Sea f = f (x, y) de clase C2 con fx (0, 1) = 2, fy (0, 1) = 1; fxx (0, 1) = 0,


fxy (0, 1) = 1, fyy (0, 1) = 1.Si h(t) = f (t2 , 1 + t3 ) .
Calcule la derivada compuesta (f !
0
r ) (t) y evalue h´(0) = 4
Solución:
00
h (t) = fxx (x, y) (x0 )2 + 2fxy (x, y) x0 y 0 + fyy (x, y) (y 0 )2 + fx (x, y) x00 +
fy (x, y) y 00
h00 (0) = 4

Derivación implı́cita
Sea z = f (x, y) definida por z = u + vdonde u = u(x, y) y v = v(x, y)
son funciones
definidas de manera implı́cita por las ecuaciones
F = u + eu+v x=0
G = v + eu v
y=0
Si u = v = 0 entonces x = y = 1: Calcular zx (1, 1).
Solución:
zx (1, 1) = 1

Problema 5
✓ ◆
x
Verifique que la función f (x, y) = g satisface la ecuación:
y2
@f @f
2x +y =0
@x @y

Problema 6
⇣ xy ⌘
Sea g , x2 + y 2 = 0 una ecuación que define a z como una función
z
de x e
y. Verifique que si gx ; gy y gz existen y son continuas en toda la región
en la que
gz 6= 0; entonces

283
z(x2 y 2 )
yzx xzy =
xy

Problema 7

Si F (xz, yz) = 0 define a z como función implı́cita de x e y y además


cumple con las
condiciones del teorema de la función implı́cita en cada punto de una
región R; entoncesverifique que, en R; se satisface la ecuación

xzx + yzy = z

3.5.5. Puntos crı́ticos máximos y mı́nimos


Problema 1

Sea f : U ✓ R2 ! R definida en el abierto U , dada por:


f (x, y) = x4 + y 4 + 4axy + 8a4 , (a 2 R). Calcular los valores extremos
de la función.
Solución:
Si a = 0, entonces, el único punto crı́tico es P0 = (0, 0) (mı́nimo) .
Si a >p 0, tenemos
p tres puntos
p p crı́tico es P0 = (0, 0) (punto silla),
P1 = ( a, a), P2 = ( a, a) (mı́nimos).
Si a <p 0, tenemos
p tres puntos
p crı́tico
p es P0 = (0, 0) (punto silla),
P1 = ( a, a), P2 = ( a, a) (mı́nimos).

Problema 2

Hallar los extremos absolutos de la función f (x, y, z) = x + y + z en el


conjunto D = ((x, y, z) 2 IR3 /x2 + y 2  z  1) .
Solución:
✓ ◆
1
1 1 1
El mı́nimo de f se alcanza en ,, y vale , y el máximo se
✓ ◆ 2
2 2 2
1 1 1 p
alcanza en p , p , y vale 1 + 2
2 2 2

284
Problema 3

Sea la función f (x, y) = xy (y 2 x2 )


a) Determine y clasifique los puntos crı́ticos de f
b) Detemine los máximos y los mı́nimos de f en la región
0  x  1; 0  y  1.
Solución:
a) En P0 = (0, 0) hay un punto silla. En P1 = ( p13 , 1) hay máximo de
f ,
y en P2 = (1, p13 , ) hay un mı́nimo de f
2 2
b) El máximo es f ( p13 , 1) = p y el mı́nimo f (1, p13 ) = p
3 3 3 3

Problema 4

Calcular la distancia mı́nima del punto (0, b) a la parábola x2 4y = 0.


Solución:
d=2

Problema 5

Calcular el paralelepı́pedo de volumen máximo que tiene una diagonal


de longitud 1.
Solución:
✓ ◆
1 1 1 1
V p ,p ,p = p
3 3 3 3 3

Problema 6

Hallar el punto de la elipse 3x2 2xy+3y 2 = 8 para la cual la respectiva


recta tangente está a la menor distancia del origen.
Solución:
Se tienen cuatro puntos crı́ticos P0 = (1, 1) , P1 = ( 1, 1) ,
p p p p
P2 = 2, 2 , P3 = 2, 2 .

285
Los puntos donde esta la menor distancia son:
p p p p
d 2, 2 = d 2, 2 =2
Los puntos donde está la mayor distancia :
p
d (1, 1) = d ( 1, 1) = 2

Problema 7

Un canaleta cuya sección transversal tiene forma de trapecio, con ángu-


los iguales en la base, se fabrica doblando bandas iguales a lo largo de
ambos lados de una larga plancha de metal,de 12 pulgadas de ancho.
Encuentre los ángulos de la base y el ancho de los lados que producen
la máxima capacidad de la canaleta.
2⇡
Solución: ángulos , ancho de lados 4 pulgadas
3

Problema 8

Pruebe que para ángulos x, y, z cuya suma es x + y + z = ⇡2 ,


se cumple la desigualdad
1
sin x sin y sin z 
8

3.6. Aplicaciones Derivada Direccional


Ejemplo 1

Suponga que la temperatura en grados Celsuis en el punto (x, y) cerca


de un aeropuerto está dada por
1
f (x, y) = [7400 4x 9y 0, 03xy] ( con las distancias x e y
180
medidas en kilometros).
a) Suponga que un avión despega del aeropuerto en la ubicación P (200, 200)
y se dirige al noreste en la dirección especificada por el vector ! v =
(3, 4) .¿Cuál es la tasa de cambio inicial de la temperatura en la direc-
ción dada?.

286
b) Si el avión sale del aeropuerto en P y vuela en la dirección !
v con
ds
una rapidez v = = 5 km/min. ¿Cuál es la tasa de cambio inicial de
dt
la temperatura en la dirección dada por unidad de tiempo?
Solución:
a) Como f es una función diferenciable en IR2 , la derivada direccional
de la función f en el punto P en la dirección !
v está dada por:

Dub f (P ) = rf (P ) · u
b

! ✓ ◆
u (3, 4) 3 4
Determinemos el versor ub= ! = p = ,
kuk 32 + 42 5 5
✓ ◆
1 1
Ahora calculemos el gradiente rf (P ) = ( 4 0, 03y) , ( 9 0, 03x)
180 180
La derivada direccional en cualquier punto es
✓ ◆ ✓ ◆
1 1 3 4
Dvbf (P ) = ( 4 0, 03y) , ( 9 0, 03x) · ,
180 180 5 5

Evaluemos la derivada direccional el punto P (200, 200), se encuentra


que:
✓ ✓ ◆ ✓ ◆◆
3 10 4 15 18
Dvbf (200, 200) = · + =
5 180 5 180 180
o
C
Dvbf (200, 200) = 0, 1
km

Lo que significa que se observará una disminución de 0, 1o C de temper-


atura en esa dirección por cada km de viaje.
b) La tasa de variación de la temperatura con respecto al tiempo
está dada por
✓ o

df df ds ! C
= = Dvbf (P ) k v k = 0, 1 (5km/min)
dt ds dt km
df km
= 0, 5
dt mı́n

287
Ejemplo 2

Suponga ahora que consideramos que la temperatura en grados Celsuis


en el punto (x, y, z) cerca de un aeropuerto está dada por
1
f (x, y, z) = [7400 4x 9y 0, 03xy] 2z , donde f es la tem-
180
peratura por kilometro de altitud.
a) Si un halcón esta inmóvil en el aire, en el punto P (200, 200, 5) y
sobre el aeropuerto desciende en forma súbita con una rapidez de 3
km/min en la dirección dada por el vector (3,4,-12) . ¿Cuál es la tasa
de cambio instantánea que experimenta el ave?
b) ¿En qué dirección debe descender el halcón que está en el punto
P(200,200) a una altitud de 5 km, a fin de calentarse lo más rápido?
c) ¿Qué tan rápido súbira su temperatura conforme el ave baje a una
rapidez de 3 km/min?
d) ¿Cuál será la dirección de la brújula y el ángulo de descenso conforme
vuele en esa dirección particular?
Solución:
a) La derivada direccional de la función f en el punto P en la dirección
!v está dada por:
Dub f (P ) = rf (P ) · u b
! ✓ ◆
u (3, 4, 12) 3 4 12
Determinemos el versor ub= ! = q = , ,
kuk 2 2 2 5 5 13
3 + 4 + ( 12)
El vector gradiente de temperatura es
✓ ◆
1 1
rf (P ) = ( 4 0, 03y) , ( 9 0, 03x) , 2
180 180

El gradiente en la posición inicial es


✓ ◆
10 15
rf (200, 200, 5) = , , 2
180 180

Por tanto , la tasa de cambio inicial de la temperatura del ave respecto


de la distancia en la dirección dada es

df
= Dub f (P ) = rf (P ) · u
b
ds
288
Evaluemos la derivada direccional la posición inicial P (200, 200, 5),

✓ ✓ ◆ ✓ ◆ ✓ ◆◆
3 10 4 15 12 47
Dvbf (200, 200, 5) = · + 2 =
13 180 13 180 13 26

ds
Como la rapidez del ave es =3 km/min, entonces la tasa de cambio
dt
por unidad de tiempo que experimenta el halcón es
✓ o ◆ o
df df ds ! 47 C C
= = Dvbf (P ) k v k = (3km/min) ⇡ 5, 4
dt ds dt 26 km mı́n

c) El valor máximo está dado por

df
= [Dvbf (P )]máx = krf (P )k
ds s✓ ◆2 ✓ ◆2
10 15
= + + ( 2)2
180 180
= 2,0025

Y se alcanza en en la dirección dada por el gradiente

rf (P ) ( 10, 15, 360)


b=
n =
krf (P )k 360, 45

La rapidez del halcón es 3 , por lo que la tasa temporal de cambio de


la temperatura en esa dirección experimentada por el halcon es
✓ o
◆ o
df df ds ! C C
= = Dvbf (P ) k v k = 2, 0025 (3km/min) ⇡ 6, 0075
dt ds dt km mı́n

3.7. Aplicaciones de Máximos y Mı́nimos

3.7.1. Aplicación al campo de la mecánica


!
Sea F : D ! R3 un campo de Fuerza definido en cierto dominio D de
R3 . Consideremos ahora una partı́cula de masa m que se mueve a lo
largo de una trayectoria !
r (t) bajo la acción de este campo de fuerza.

289
La ecuación de movimiento de la partı́cula esta dada por la segunda
ley de Newton:
! !
F ( r (t)) = m!r ” (t) (⇤)
! !
Si el campo vectorial F es conservativo, esto es, F = rV , entonces:

1
m k!
r 0 (t)k + V (!
r (t)) = c
2
donde c es una constante. El primer término se llama energı́a cinética
y el segundo corresponde a la función potencial V.
Si diferenciamos la expresión anterior usando la regla de la cadena:

d 1
m k!r 0 (t)k + V (!r (t)) = m! r 0 (t)· !
r (t)+rV (! r (t))· !
00 0
r (t) =
dt 2
0
⇥ !00 ⇤
m r (t) + rV (! r (t)) · !
r (t) = 0 () m! r (t) + rV (!
0 00
r (t)) = 0
!
Por tanto, m! r (t) = rV (! r (t)) = F (!
00
r (t)) lo que demuestra la
ecuación de Newton (⇤) .
Un punto ! r o 2 D se llama posición de equilibrio si la fuerza en ese
!
punto es cero: F (!r ) = 0.Un punto !
o r que sea de equilibrio se llama
o
estable si para todo ⇢ > 0 y " > 0, podemos escoger números ⇢o > 0
y "o > 0 tales que un punto situado en cualquier lugar a una distancia
menor que ⇢o de ! r o ,después de recibir inicialmente energı́a cinética en
una cantidad menor que "o , permanecerá para siempre a una distancia
de !r o menor que ⇢ y poseera energı́a cinética menor que ".

Ası́, si tenemos una posición de equilibrio, la estabilidad en ! r o sig-


!
nifica que una partı́cula que se mueve lentamente cerca de r o siempre
permanecerá cerca de ! r o y se mantendrá moviendose lentamente.
Ahora, si tenemos un punto de equilibrio inestable ! r o , entonces
! ! !
r (t) = r o resuelve la ecuación de Newton F ( r (t)) = m!
! r ” (t) ,
!
pero las soluciones cercanas pueden alejarse de r conforme trascurra
o
el tiempo.

Proposición 3.7.1. i) Los puntos crı́ticos de un potencial son posi-


ciones de equilibrio.
ii) En un campo conservativo, un punto ! r o en el cual el potencial
alcance un mı́nimo local estricto, es una posición de equilibrio estable.

290
Demostración

1) La primera afirmación es bastante obvia debido a la definición de


!
campo conservativo: F = rV, los puntos de equilibrios ! r o son
exactamente los puntos crı́ticos de V, en los cuales rV (!
r o) = 0
2) Para probar la afirmación ii), haremos uso de la ley de conservación
de energı́a . Tenemos

1 1
m k!
r 0 (t)k + V (!
r (t)) = m k! r 0 (0)k + V (!
r (0))
2 2
Escojamos un pequeña vecindad de ! r , y asumamos que la partı́cula
o
tienen poca energı́a cinética. Conforme t crece, la partı́cula se aleja
de !r o sobre una trayectoria ! r (t) y V (!r (t)) crece pues V (! r (0)) es
un mı́nimo estricto , de modo que la energı́a cinética debe decrecer .
Si la energı́a cinetica inicial es suficientemente pequeña, entonces, para
que la partı́cula escape de la vecindad de ! r o , fuera de la cual V ha
crecido en una cantidad definida, la energia cinetica tendria que volverse
negativa, lo cual es imposible. Asi, la particula no puede escapar de la
vecindad.
Sea una partı́cula en un campo de potencial V restringido a manten-
erse sobre la superficie de nivel S dada por la ecuación (x, y, z) = 0
!
con r 6= 0. Si en la ecuación de Newton F (! r (t)) = m!r ” (t) (⇤) ,
! !
reemplazamos F con la componente de F paralela a S, aseguramos
que la partı́cula permanecerá en S.

Proposición 3.7.2. i) Si en un punto P sobre la superficie S el poten-


cial V |S tiene un valor extremo, entonces el punto P es una posición
de equilibrio sobre la superficie.
ii) Si un punto P 2 S es un mı́nimo local estricto del potencial V|S ,
entonces el punto P es una posicion de equilibrio estable.

Ejemplo Sea el campo gravitacional cerca de la superficie de la tier-


!
ra; esto es, sea F = (0, 0, mg) donde g es la aceleración de gravedad
. Determine la función potencial gravitacional y ¿cuáles son las posi-
ciones de equilibrio, si una partı́cula con masa m esta restrigida a la
esfera g (x, y, z) = x2 + y 2 + z 2 r2 = 0, (r > 0)?¿Cuáles son estables?.
Solución:

291
@V
Tenemos que Fz = = mg =) V (x, y, z) = mgz
@z
Usando el método de los multiplicadores de Lagrange podemos localizar
los extremos posibles, tenemos que:
L (x, y, z, ) =
V (x, y, z) + g (x, y, z)
L (x, y, z, ) mgz + x2 + y 2 + z 2 r2
=
Lx (x, y, z, ) 2 x = 0 =) 6= 0 y x = 0
= (1,0)
Ly (x, y, z, ) 2 y = 0 =) 6= 0 y y = 0
= (2,0)
mg
Lz (x, y, z, ) = mg + 2 z = 0 =) = y z 6= 0 (3,0)
2z
L (x, y, z, ) = x2 + y 2 + z 2 r2 = 0 ( 4,0)

Reemplazando (1,0), (2,0), (3,0) en ( 4,0)

mg
z2 r2 = 0 =) z = ±r y =
2r

Luego, se deduce que los puntos P1 = (0, 0, r) y P2 = (0, 0, r) son


posiciones de equilibrio.

3.7.2. Aplicaciones a la geometrı́a


Ejemplo 1 Determine la distancia mı́nima desde el origen (0,0,0) a
y2 z2
la superficie S del elipsoide x2 + + = 1.
4 9
Solución:
Sabemos que la distancia pentre un punto P y el origen está dada por
la función d(x, y, z) = x2 + y 2 + z 2 . Sin embargo, por razones de
simplicidad en los cálculos , en lugar de la función anterior vamos a
considerar la función f (x, y, z) = x2 + y 2 + z 2 en atención a que f
tendra un mı́nimo en un punto si y solo si d lo tiene.
Se trata de obtener los extremos condicionados de la función distancia
y2 z2
f (x, y, z) = x2 + y 2 + z 2 sujeta a la condición x2 + + = 1.
4 9
Formemos la función auxiliar de Lagrange

2 2 2 y2 z2
2
F (x, y, z, ) = (x + y + z ) + (x + + 1)
4 9
292
y consideremos entonces el sistema.
Fx (x, y, z, ) = 2x + 2 x = 0
1
Fy (x, y, z, ) = 2y + y = 0
2
2
Fz (x, y, z, ) = 2z + z = 0
9
2
y z2
F (x, y, z, ) = x2 + + 1=0
4 9
De las tres primeras ecuaciones obtenemos
2x(1 + ) = 0 =) x = 0 _ = 1
1
y(2 + ) = 0 =) y = 0 _ = 4
2
1
2z(1 + ) = 0 =) z = 0 _ = 9
9

Sustituyendo y = z = 0 en la cuarta ecuación produce


x2 1 = 0 =) x = ±1
.
Reemplazando x = z = 0 en la cuarta ecuación produce
y2
1 = 0 =) y = ±2
4
Sustituyendo x = y = 0 en la cuarta ecuación produce
z2
1 = 0 =) z = ±3
9
Luego , se obtienen seis puntos crı́ticos
P0 = (1, 0, 0) , P1 = ( 1, 0, 0) , P2 = (0, 2, 0) , P3 = (0, 2, 0)
P4 = (0, 0, 3) , P5 = (0, 0, 3) .
Evaluando la función en los puntos encontrados deberá haber un máxi-
mo y
un mı́nimo
f (±1, 0, 0) = 1, f (0, ±2, 0) = 4, f (0, 0, ±3) = 9
Se tiene que el mı́nimo de f se encuentra en los puntos (±1, 0, 0) y es
igual 1. y el máximo está localizado en los puntos (0, 0, ±3) y vale 9.

293
Ejemplo 2 Determine la distancia mı́nima y máxima del origen a la
7
curva de interseccion del paraboloide x2 + y 2 + z = 0 y el plano
4
x + y + z 2 = 0.
Solución:
Igual que el ejemplo anterior resulta mas conveniente hallar los ex-
tremos del cuadrado de la distancia
p respecto del origen en vez de la
función distancia d (x, y, z) = x + y 2 + z 2
2

Por consiguiente, se trata de obtener los extremos condicionados de


la función distancia f (x, y, z) = x2 + y 2 + z 2 sujeta a las condiciones
7
g(x, y, z) = x2 + y 2 + z = 0 y h(x, y, z) = x + y + z 2 = 0
4

Formemos la función auxiliar de Lagrange


✓ ◆
7
F (x, y, z, 1 , 2 ) = (x +y +z )+ 1 x2 + y 2 + z
2 2 2
+ 2 (x + y + z 2)
4

y consideremos entonces el sistema.

Fx (x, y, z, 1, 2) = 2x(1 + 1 ) + 2 2 = 0 (1)


Fy (x, y, z, 1, 2 ) = 2y(1 + 1 ) + 2 2 = 0 (2)
Fz (x, y, z, 1, 2 ) = 2z + 1 + 2 = 0 (3)

2 2 7
F 1 (x, y, z, 1, 2) = x + y + z = 0 (4)
4
F 2 (x, y, z, 1, 2) = x + y + z 2 = 0 (5)

De las dos primeras ecuaciones se obtiene


1 = 1 ó y = x
Consideremos primero el caso 1 =
1.A partir de (1) se obtiene 2 =
1
0 . Sustituyendo estos valores en (3) z = .
2
Reemplazando z en (4) y (5), produce
5
x2 + y 2 = 0
4
3
x+y = 0
2

Resolviendo el sistema se obtienen los puntos crı́ticos

294
✓ ◆ ✓ ◆
1 1 1 1
P0 = 1, , , P1 = , 1,
2 2 2 2
Al evaluar ambos puntos en la función distancia, obtenemos
r r
1 1 3
d (x, y, z) = 1 + + =
4 4 2
Consideremos ahora y = x , a partir de las ecuaciones (4) y (5) se
obtiene
7
2x2 + z = 0
4
2x + z 2 = 0

Resolviendo el sistema se obtienen los puntos crı́ticos


p p p !
1 2 1 2 2
P3 = ± , ± ,1 ⌥ ,
2 4 2 4 2
Al evaluar todos estos puntos en la función distancia, obtenemos
p p p !
1 2 1 2 2 1p p
d ± , ± ,1 ⌥ = 9⌥2 2
2 4 2 4 2 2
Como la curva de intersección del paraboloide y el plano es cerrada, las
distancias máxima y minima absoluta del la curva al origen son:
r
1p p 3
dmáx = 9 + 2 2, dmı́n =
2 2

Ejemplo 3 Encuentre el volumen máximo de una caja rectangular


x2 y 2 z 2
inscrita en el elipsoide 2 + 2 + 2 = 1 con sus caras paralelas a los
a b c
planos coordenados.
Solución:
Sea P(x,x,z) el vértice de la caja que esta en el primer octante donde
x > 0, y > 0, z > 0. Por la simetria del problema se desea maximizar
la función volumen f (x, y, z) = 8xyz sujeta a la condición g(x, y, z) =
x2 y 2 z 2
+ 2 + 2 1=0
a2 b c
Formemos la función auxiliar de Lagrange
✓ 2 ◆
x y2 z2
F (x, y, z, ) = 8xyz + + 2 + 2 1
a2 b c

295
y consideremos entonces el sistema.
2
Fx (x, y, z, ) = 8yz + x=0 (1)
a2
2
Fy (x, y, z, ) = 8xz + 2 y = 0 (2)
b
2
Fz (x, y, z, ) = 8xy + 2 z = 0 (3)
c
2 2
x y z2
F (x, y, z, ) = 2 + 2 + 2 1 = 0 (4)
a b c
Multiplicando las ecuaciones (1), (2)y (3) por x, y, x respectivamente,
produce

2 2
8xyz + x = 0 (1,1)
a2
2
8xyz + 2 y 2 = 0 (2,2)
b
2
8xyz + 2 z 2 = 0 (3,3)
c
Entonces , obtenemos
2 2 2 2 2 2
x = 2y = 2z = 8xyz
a2 b c
Para obtener el volumen máximo se requiere que x, y, x 6= 0 y 6= 0
Concluimos entonces que
x2 y2 z2
= =
a2 b2 c2
Sustituyendo esta expresión en la ecuación (4), obtenemos

x2 a
3 1 = 0 =) x = p
a2 3
Ası́, sucesivamente se tiene un único punto crı́tico
✓ ◆
a b c
P0 = p , p , p
3 3 3
Por lo tanto , la caja tiene un volumen máximo dado por
✓ ◆
a b c 8
fmáx p , p , p = p abc
3 3 3 3 3

296
3.7.3. Aplicación al campo de la economı́a

Supongase que la producción de cierto producto de una compañı́a man-


ufacturera es una cantidad Q , donde Q es una función de f (K, L) donde
K es la cantidad de capital (o inversión) y L es la cantidad de trabajo
realizado. Si el precio del trabajo es p, el precio del capital es q y la
compañia no puede gastar más de B dólares, ¿cómo podemos hallar la
cantidad de capital y de trabajo que maximice la producción Q?
Solución:
Se esperarı́a que si se incrementa la cantidad de capital o de trabajo,
entonces la producción deberá incrementarse; esto es:

@Q @Q
0 y 0
@K @L

También se esperaria que conforme se añada trabajo a una cantidad


dada de capital, obtendremos menos productos adicionales por nuestro
esfuerzo, esto es:
@ 2Q
<0
@K 2

De manera análoga,
@ 2Q
<0
@L2

Con estas hipótesis sobre Q , es razonable esperar que las curvas de


nivel de la producción- llamadas isocuantas- Q(K, L) = c, se vean
como las esbozadas en la figura, con c1 < c2 < c3.

pendiente imagen

Podemos interpretar la convexidad de las isocuantas como sigue: si nos


movemos hacia la derecha a lo largo de una isocuanta dada , se em-
plea más capital para reemplazar una unidad de trabajo y producir
la misma cantidad. La restricción de presupuesto significa que debe-
mos mantenernos dentro del triángulo acotado por los ejes y la recta
pL + qK = B. Geometricamente, es claro que producimos más al gas-
tar nuestro dinero de tal manera que seleccionemos la isocuanta que
solamente toca, pero no cruza, la recta presupuesto.

297
Como el punto máximo está en la frontera de nuestro dominio, aplicare-
mos el método de los multiplicadores de Lagrange para hallar el máxi-
mo. Para maximizar Q = f (K, L) sujeto a la restricción pL + qK = B,
buscamos los puntos crı́ticos de la función auxiliar

H (K, L, ) = f (K, L) + (pL + qK B)

Ası́ queremos:

HK (K, L, ) = fK (K, L) + q = 0 =) fK (K, L) = QK (K, L) = q


HL (K, L, ) = fL (K, L) + p = 0 =) fL (K, L) = QL (K, L) = p
H (K, L, ) = pL + qK B = 0 =) pL + qK B = 0

Con estas ecuaciones podemos encontrar los puntos crı́ticos de la fun-


ción Q. Luego, usando derivación implicita podemos determinar el pun-
to donde se maximiza la producción.
En el ejemplo anterior , representa algo interesante. Más adelante
vamos a interpretar .
Sean k = qK y l = pL, de modo que k es el valor en dólares del capital
empleado, y l es el valor en dolares del trabajo empleado, entonces las
ecuaciones se convierten en:

H (k, l, ) = f (k, l) + (l + k B)

Hk (k, l, ) = fk (k, l) + = 0 =) fk (k, l) = Qk (k, l) =


Hl (k, l, ) = fl (k, l) + = 0 =) fl (k, l) = Ql (k, l) =
H (k, l, ) = l + k B = 0 =) l + k B = 0

Comparando con las primeras dos ecuaciones del caso anterior se tiene
que:

@Q 1 @ @Q 1 @
= = = =
@k q @K @l p @K
Ası́ , en el punto óptimo de producción , el cambio marginal en la pro-
ducción por dólar de inversión de capital adicional, es igual al cambio
marginal de la producción por dólar de trabajo adicional, y es este

298
valor común. En el punto óptimo, el intercambio de un dólar de capi-
tal por un dólar de trabajo no cambia la producción. Fuera del punto
óptimo, la producción marginal es distinta, y un intercambio, o el otro,
incrementan la producción.

Ejemplo Suponga que la producción total de una compañia está da-


da por la función P = P (K, L) = K 1/4 L1/2 donde K denota el número
de unidades de capital usado y L es el número de unidades de trabajo
usado.
a) Suponga que cada unidad de capital K cuesta 1 millón de dólares
y cada unidad de trabajo L cuesta 7 millones de dólares.Considere el
problema de maximizar la producción total P si el presupuesto total
es de 10 millones de dólares.
b) Suponga que el capital K aumenta en una razón de 4 % por año, y
el trabajo L aumenta en una razón de 5 % por año. Determine la razón
de crecimiento de la producción P por año.
Solución:
Tenemos que extremar P = P (K, L) = K 1/4 L1/2 bajo la condición
K + 7L 10 = 0
Sea la función auxiliar de Lagrange

F (K, L. ) = K 1/4 L1/2 + (K + 7L 10)

y consideremos entonces el sistema.


1
FK (K, L. ) = K 3/4 L1/2 + = 0 (1)
4
1 1/4 1/2
FL (K, L. ) = K L +7 =0 (2)
2
F (K, L. ) = K + 7L 10 = 0 (3)

Multiplicando (1,0) por 4K y (2,0) por 2L, produce:

K 1/4 L1/2 + 4K = 0 (1,1)


K 1/4 L 1/2 + 14L = 0 (2,1)

Restando las últimas ecuaciones tenemos:

299
4K 14L = 0

Entonces

2 (2K 7L) = 0 =) = 0ó (2K 7L) = 0

Si = 0 nos quedamos sin restricion, luego

7
6= 0y(2K 7L) = 0 =) K = L
2

Reemplazando el resultado anterior en (3,0) ,queda


7 20 10
L + 7L = 10 =) L = entonces K =
2 21 3
✓ ◆
10 20
Asi, se tienen un único punto crı́tico en P0 = , .
3 21
✓ ◆ ✓ ◆1/4 ✓ ◆1/2
10 20 10 20
Por tanto, la produccion maximizada será P , =
3 21 3 21
ii) Supongamos que K y L aumentan en una razón de 4 % y 5 %,
respectivamente, por año
dK
= 0, 04K (1,0)
dt
dL
= 0, 05L (2,0)
dt

Usando la regla de la cadena sobre la función P (K, L) tenemos:

dP @P @K @P @L
= +
dt @K @t @L @t
dP 1 @K 1 1/4 1/2 @L
= K 3/4 L1/2 + K L (3,0)
dt 4 @t 2 @t

Sustituyendo (1,0) y (2,0) en (3,0)


dP 1 3/4 1
= K L1/2 (0, 04K) + K 1/4 L 1/2
(0, 05L)
dt 4 ✓ 2 ◆
dP 1 1
= K 1/4 L1/2 0, 04 + 0, 05
dt 4 2

300
Reemplazado en la expresion anterior P = K 1/4 L1/2
dP
Finamente, se tiene: = 0, 035P,lo que significa que la producción
dt
aumenta 3,5 % por año.

Ejemplo Una compañia planea gastar 10.000 dólares en publicidad.


Cuesta 3.000 dólares un minuto de publicidad en la televisión y 1.000
dólares un minuto de publicidad en la radio. Si la empresa compra
x minutos de comerciales en televisión e y minutos de comerciales en
la radio, su ingreso, en miles de dólares, esta dado por f (x, y) =
2x2 y 2 + xy + 8x + 3y. ¿Como puede la empresa maximizar su
ingreso invirtiendo en la publicidad?
Solución:
Se desea maximizar la función f (x, y) = 2x2 y 2 + xy + 8x + 3y
bajo la restricción g(x, y) = 3x + y 10 = 0.
Consideremos la función auxiliar

L (x, y, ) = 2x2 y 2 + xy + 8x + 3y + (3x + y 10)

Primero,apliquemos la condición necesaria de punto crı́tico rL (x, y, ) =


0.

Lx (x, y, ) = 4x + y + 8 3 = 0 (1)
Ly (x, y, ) = 2y + x + 3 =0 (2)
L (x, y, ) = 3x + y 10 = 0 (3)

De la ecuaciones (1) y (2)se obtiene

y = 3 + 4x 8 (1,1) , x = + 2y 3 (2,1)

Reemplazando (2,1) en (1,1) queda:

y = 3 + 4 ( + 2y 3) 8 =) y = 7 + 8y 80

Luego, se tiene
20
y= (1,2)
7

Asi, (1,2) en (2,1) produce:

301
19
x= (2,2)
7

Sustituyendo (1,2) y (2,2) en (3,0) , obtenemos:


1
4 1 = 0 =) = .
4
✓ ◆
73 69
Entonces (1,2) y (2,2) nos da un único punto crı́tico P0 = ,
28 28
.
Determinemos la naturaleza de este punto critico usando el criterios de
la segunda derivada.
El hessiano para f (x, y) es
✓ ◆ ✓ ◆
73 69 fxx fxy 4 1 73 69
H , = = =7>0 y fxx ,
28 29 fxy fyy 1 2 28 28
= 4<0
✓ ◆
73 69
Por lo tanto, la función es máxima en el punto P0 = , .
28 28
73
Ası́ , la empresa tendria que comprar minutos de comerciales en
28
69
televisión y minutos de comerciales en radio.
28

3.7.4. Problemas Propuestos de Aplicaciones


1.- Sea una partı́cula que se mueve en un campo de potencial en R2
dado por V (x, y) = 3x2 + 2xy + 2x + y 2 + y + 4. Hallar los puntos de
equilibrio estable si los hay.
✓ ◆
1 1
Solución: Hay un único punto de equilibrio estable en , .
4 4
2.- Sea una partı́cula moviéndose en un campo de potencial en R2 dado
por V (x, y) = x2 + 4xy y 2 8x 6y. Hallar todos los puntos de
equilibrio.¿Cuáles, si los hay, son estable?
Solución : Hay un único punto de equilibrio inestable en (2, 1) .
3.- Sea una partı́cula restringida a moverse sobre la esfera x2 +y 2 +z 2 =
1, sujeta a fuerzas gravitacionales, asim como al potencial adicional
V (x, y, z) = x + y.Hallar los puntos de equilibrio estable, si los hay.

302
1/2
Solución : Hay un único punto de equilibrio estable en (2 + m2 g 2 ) ( 1, 1, mg) .

4.- Usando la información anterior , encuentre el punto óptimo para la


función de producción Q (K, L) = AK ↵ L1 ↵ , donde A y ↵ son con-
stantes positivas y 0 < ↵ < 1, que se usa para modelar la econonomı́a
nacional. Q es, entonces, la producción agregada de la economı́a para
una entrada de capital y trabajo dada .
KQ pL
Solución: En el óptimo: =
↵ 1 ↵

5.- Una compañia usa aluminio, hierro y magnesio para producir acc-
cesorios de automóviles. La cantidad de accesorios que puede producir
usando x toneladas de aluminio, y toneladas de hierro y z toneladas de
magnesio es Q (x, y, z) = xyz. El costo de la materia prima es: aluminio
6 dólares por tonelada; hierro 4 dólares por tonelada ; y magnesio 8
dolares por tonelada. ¿Cuántas toneladas de aluminio, hierro y magne-
sio deberán usarse para manufacturar 1000 accesorios al menor costo
posible?
20 p
3
p p
Solución: x = 3 , y = 10 3 3, z = 5 3 3
3

6.- Una Pyme cuenta con 8.000 dólares para importar dos tipos de
bebidas energéticas . Si x son la unidades de bebidas energéticas que se
12x
importarán desde Holanda, y se estima que venderán unidades de
x+6
esta bebidas a un precio de 200 dólares cada una. Si y son la unidades
de bebidas energéticas que se importarán desde Alemania, estimandose
24y
que venderán unidades, a un precio de 200 dólares cada una.
y+3
Si el costo por unidad vendida de cada bebida es de 50 dólares.
a) Determine cuantas unidades de cada bebida energética deben im-
portar para maximizar su utilidad.
b) Determine la utilidad máxima.

Solución:La función utilidad esta dada por la diferencia entre el precio


de venta de las bebidas y el costo de importación
a) 78, 5 bebidas Holandesas y 81, 5 bebidas Alemanas.
b) Utilidad U (78, 5; 81, 5) = 5144, 7 dólares.

303
7.- La función de producción de una compañı́a es Q (x, y) = xy. El
costo de producción es C (x, y) = 2x+3y.Si esta compañı́a puede gastar
C (x, y) = 10, ¿cuál es la máxima cantidad que puede producir?

✓ ◆
5 5 25
Solución: P , =
2 3 6

3.8. Auto evaluaciones


Autoevaluación No 1
El estudiante:
1) Determinará si una función f : D ✓ R2 ! R definida en el conjun-
to abierto D, es o no diferenciable en un punto del dominio, empleando la
definición de diferenciabilidad.
2) Calculará la derivada direccional máxima de una función f diferenciable
en un punto evaluando el gradiente y el vector unitario necesario.
3) Utilizará el método de los multiplicadores de Lagrange para localizar
los posibles valores extremos de una función f : ⌦ ✓ RN ! R, definida en
un conjunto abierto ⌦, sujeta a una o dos condiciones gm (x) = 0 , m = 1,2 y
m<N

Tiempo: 2 horas
Pregunta 1
Estudie la diferenciabilidad de la función f (x; y) = x2 + 2y 2 , en el punto
(x0 ; y0 ) = (1; 2).

Pregunta 2
Hallar los coeficientes a y b para que la derivada direccional máxima
p
de la función eax+by cos(x + y) z = 0 en el punto (0, 0) sea 3 2 en la
dirección de la bisectriz del primer cuadrante.
Pregunta 3

Calcular la mı́nima distancia desde el origen (0, 0) hasta la recta


2x + y = 1,usando el teorema de los Multiplicadores de Lagrange.

Pauta Autocorrección

304
Pregunta 1
En primer lugar, sus derivadas parciales en el punto (x0 ; y0 ) = (1; 2)
existen y valen
@f @f
(x, y) = 2x =) (1, 2) = 2
@x @x
@f @f
(x, y) = 4y =) (1, 2) = 8
@y @y
En segundo lugar, para mostrar que f es diferenciable tenemos que
demostrar
que se cumple

@f @f
f (x, y) f (x0 , y0 ) + (x0 , y0 ) (x x0 ) + (x0 , y0 ) (y y0 )
@x @y
lı́m q =0
(x,y)!(x0 ,y0 )
(x x0 )2 + (y y0 )2

En efecto, tenemos que mostrar que

(x2 + 2y 2 ) (9 + 2(x 1) + 8(y 2))


lı́m q =0
(x,y)!(1,2)
(x 1)2 + (y 2)2
Para demostrar esto , traslademos el origen usando el cambio de variables
u = (x 1) y v = (y 2) .De este modo, tenemos que
((u + 1)2 + 2(v + 2)2) (9 + 2u + 8v)
lı́m p
(u,v)!(0,0) u2 + v 2
si se desarrollan los paréntesis, se obtiene

u2 + 2v 2
lı́m p
(u,v)!(0,0) u2 + v 2

Si nos acercamos al origen por una recta v = ku, obtenemos


u2 + 2v 2 u2 (1 + 2k 2 )
lı́m p = lı́m p =0
(u,v)!(0,0) u2 + v 2 u!0 u 1 + k 2

Esto no prueba que el valor del lı́mite sea cero. Lo que nos indica es que
si
tal lı́mite existe, debe ser cero.
Ası́, se requiere la aplicación directa de la definición de lı́mite para concluir
que éste existe y vale cero
Para aplicar la definición se tiene que

u2 + 2v 2  2u2 + 2v 2  2 u2 + v 2

305
Luego

u2 + 2v 2 2(u2 + v 2 ) p
p 0  p = 2 u2 + v 2
u2 + v 2 u2 + v 2
vemos que
p 2(u2 + v 2 ) p
u2 + v 2 < =) p = u2 + v 2 <
u2 + v 2 2
Es decir, con = 2✏ nos queda
u2 + 2v 2
k(u, v) (0, 0)k < =) p 0 <✏
u2 + v 2
lo que nos dice que efectivamente

u2 + 2v 2
lı́m p =0
(u,v)!(0,0) u2 + v 2

Por lo tanto la función f es diferenciable en el punto (1, 2) .

Pregunta 2
La función z = eax+by cos(x + y) es continua en todo IR2 por ser com-
posición
de funciones continuas.

@f
z0x = = aeax+by cos(x + y) eax+by sen(x + y)
@x
@f
z0y = = beax+by cos(x + y) eax+by sen(x + y)
@y

Además, las derivadas parciales son continuas en todo IR2 .Por tanto la
función es diferenciable en todo en todo IR2 .
Esto significa que la derivada direccional en un punto P (x, y) en una
b se puede obtener como el producto escalar del gradiente de la
dirección u
función en el punto y el versor que señala la dirección en el punto considerado
p
Dub f (x, y) = rf (x, y) · u
b=3 2
Por otro lado, el gradiente de f nos da la dirección hacia donde la derivada
direccional es máxima, que en este caso corresponde a la bisectriz del primer
cuadrante, luego tenemos:
p p !
p rf (0, 0) 2 2
krf (0, 0)k = 3 2 y u b= = ,
krf (0, 0)k 2 2

306
Calculando el gradiente en el punto P(x, y) se tiene:

rf (x, y) = (aeax+by cos(x + y) eax+by sen(x + y), beax+by cos(x + y)


eax+by sen(x + y))

Evaluando en el origen, obtenemos:

rf (0, 0) = abi + bb
j
se tiene que cumplir que:

✓ ◆ p p !
p p a b 2 2
a2 + b 2 = 3 2 y u b= p , p = , =) a = b
3 2 3 2 2 2
Por lo tanto, al sustituir el resultado en la primera ecuación, queda:
a=b=3

Pregunta 3 p
Hay que calcular los mı́nimos de la función d (x, y) = x2 + y 2 bajo la
condición 2x + y 1 = 0. Sin embargo, podemos simplificar
cálculos si se trabaja con la función f (x, y) = x2 + y 2 , puesto que
los máximos y minı́mos de ambas funciones coinciden.
Determinemos los puntos crı́ticos usando la función de Lagrange
Sea la funcion de Lagrange F (x, y, ) = x2 + y 2 + (2x + y 1) ,
aplicando la condicion necesaria de punto critico , se tiene
Fx (x, y, ) = 2x + 2 = 0 =) x =
Fy (x, y, ) = 2y + = 0 =) y =
2
F (x, y, ) = 2x + y 1=0

Sustituyendo los dos primeros resultados en la tercera ecuación, produce


2 1 2
x = ,y = , = .
5 5 5 ✓ ◆
2 1
Entonces, existe un único punto crı́tico, que es P = , .
5 5
Determinemos si es máximo o mı́nimo, usando la función de Lagrange
2
para el valor obtenido F (x, y) = x2 + y 2 + (2x + y 1)
3
Derivando parcialmente, queda
4
Fx (x, y) = 2x + =) Fxx (x, y) = 2, Fxy (x, y) = 0
3
2
Fy (x, y) = 2y + =) Fyy (x, y) = 2
3
307
Luego, el determinante de la matriz Hessiana es:
2 0
H (x, y) = = 4 6= 0. Como Fxx = 2 > 0 , se trata de un mı́nimo
0 2
de F .

Autoevaluación No 2

El estudiante:
1) Determinará la continuidad de una función f : R2 ! R, dada f (x, y)
y un punto (x0 , y0 ) del dominio.
2) Evaluará las derivadas parciales, de primer y segundo orden, de una
función f (x, y) en un punto dado del dominio , por definición.
3) Calculará las derivadas parciales de primer y segundo orden de una
función f (x, y) usando el álgebra de derivadas.
4) Empleará la definición de derivada direccional para determinar la ex-
istencia y variación de una función f en un punto (x, y) de su dominio en
una dirección vb,
5) Empleará el teorema de la función implicita, en la vecindad de un
punto (x0 , y0 ) del dominio y calculará las derivadas de una función implı́cita
de primer y segundo orden.
6) Usará la regla de la cadena para probar que una función compuesta es
solución de una ecuación diferencial parcial.

Tiempo: 2 horas
Pregunta 1 ⇢ x sin(xy)
si (x, y) 6= (0, 0)
x2 +y 2
Dada la función f (x, y) =
0 si (x, y) = (0, 0)
a) estudie la continuidad de f en todo IR2
@f @f
b) calcular y en todo IR2
@x @y
c) estudie la derivada direccional de f en (0, 0)
d) determine si f es diferenciable en (0, 0).

Pregunta 2
Dada F (x, y, z) = x2 + y 2 + z 2 + xy + 2z 1 = 0
a) Verificar si F (x, y, z) = 0 define en el punto P (0, 1, 0) a z como
función
implı́cita de x e y, es decir, z = f (x, y)
b) Calcular zx (0, 1) , zy (0, 1) , zxx (0, 1) y zyy (0, 1)

Pregunta 3

308
La ecuación de onda
@ 2u @ 2u
= a
@t2 @x2
donde a es una constante, describe el movimiento de una onda, que puede
ser una onda de sonido, una onda de luz o una onda que viaja a lo largo
de una cuerda vibrante.Si f y g son funciones de una sola variable dos
veces derivables, compruebe que la función u(x, t) = f (x + at) + g(x at)
satisface la ecuación de onda.
Pauta Autocorrección
Pregunta 1
(a) En los puntos de IR2 distintos de (0, 0) la función es continua por ser
cuociente de funciones continuas con denominador no nulo.
En (0, 0) estudiemos la continuidad de f :
i) Existe f (0, 0) = 0
ii)
x sin (xy)
lim f (x, y) = lim
(x,y)!(0,0) (x,y)!(0,0) x2 + y 2

Al usar coordenadas polares se tiene


x sin (xy) r cos ✓ sin (r2 cos ✓ sin ✓)
lim = lı́m
(x,y)!(0,0) x2 + y 2 r!0 r2
2
= lı́mr cos ✓ sin ✓ = 0 = f (0, 0)
r!0

Puesto que sin ↵ ' ↵ si ↵ ! 0,y ✓ 2 [0, 2⇡] , 1 < cos2 ✓ sin ✓ < 1,en el
último lı́mite se tiene que el producto de una cantidad infinitésimal
por una función acotada es cero.
Por lo tanto la función es continua en todo IR2
b) Las derivadas parciales 8 (x, y) 6= (0, 0) son:
@f [sin (xy) + xy cos (xy)] (x2 + y 2 ) 2x [x sin (xy)]
=
@x (x2 + y 2 )2
@f [x cos (xy)] (x + y 2 ) 2y [x sin (xy)]
2 2
=
@y (x2 + y 2 )2
Además, las derivadas parciales para (x, y) = (0, 0) son
h sin 0
@f f (h, 0) f (0, 0) 2 0 0
(0, 0) = lı́m = lı́m h = lı́m=0
@x h!0 h h!0 h h!0 h
0 sin 0
@f f (0, k) f (0, 0) 2 0 0
(0, 0) = lı́m = lı́m k = lı́m = 0
@y k!0 k k!0 k k!0 k

309
Por lo tanto las derivadas parciales de primer orden existen en todo IR2
c) Calculamos b = (a, b) tal
p la derivada direccional en cualquier dirección u
que kb
uk = a + b = 1utilizando, la definición
f (ta, tb) f (0, 0)
Dub f (0, 0) = lı́m
t!0 t
ta sin(t2 ab) ta(t2 ab)
t2
0 t2
0
= lı́m = lı́m
t!0 t h!0 t
2
Dub f (0, 0) = a b

Por tanto, existe la derivada direccional de la función f en el origen


en cualquier dirección ub = (a, b).
d) Examinemos si función es diferenciable en el origen.
Utilizando la definición de diferenciabilidad, se tiene:

f (h, k) f (0, 0) fx (0, 0) h fy (0, 0) k


lı́m =
(h,k)!(0,0) k(h, k)k
h sin (hk)
0 0·h 0·k h sin (hk)
lı́m h2 + k 2 p = lı́m
(h,k)!(0,0) h2 + k 2 (h,k)!(0,0) (h2 + k 2 )3/2

Calculemos este lı́mite utilizando coordenadas polares


h sin (hk) r cos ✓ sin (r2 cos ✓ sin ✓)
lı́m = lı́m
(h,k)!(0,0) (h2 + k 2 )3/2 r!0 r3
r cos ✓ (r2 cos ✓ sin ✓)
= lı́m
r!0 r3
= cos2 ✓ sin ✓ 6= 0

El lı́mite no es cero y por lo tanto la función no es diferenciable en (0, 0).

Pregunta 2
a) F (x, y, z) = 0 define a z = f (x, y) en una vecindad de P (0, 1, 0) si:
i) El punto P es un punto de la superficie, es decir, F (0, 1, 0) = 0.
En efecto, al evaluar F (0, 1, 0) = 0 + 1 + 0 + 0 + 0 1 = 0.
ii) Fx , Fy , Fz son continuas en una vecindad de P.
En efecto:
Fx (x, y, z) = 2x + y , Fy (x, y, z) = 2y + x, Fz (x, y, z) = 2z + 2
son funciones polinómicas y estas son continuas en IR2 .
iii) Además, se debe cumplir Fz (0, 1, 0) 6= 0
Como Fz (x, y, z) = 2z + 2 entoncesFz (0, 1, 0) = 0 + 2 = 2 6= 0.

310
El teorema de la función implı́cita garantiza una vecindad V (0, 1) en la
cual podemos definir una función z = f (x, y) tal que F (x, y, f (x, y)) = 0.

b) La función tiene derivadas continuas en V (0, 1) que pueden calcularse


por:
Fx (x, y, z) 2x + y Fx (0, 1, 0) 1
zx (x, y) = = =) zx (0, 1) = =
Fz (x, y, z) 2z + 2 Fz (0, 1, 0) 2
Fy (x, y, z) 2y + x Fy (0, 1, 0)
zy (x, y) = = =) zy (0, 1) = =1
Fz (x, y, z) 2z + 2 Fz (0, 1, 0)
Para calcular las derivadas de segundo orden basta derivar (1) y (2) re-
specto
a x e y respectivamente:
[(2z + 2)2 (2x + y) (2zx )] 5
zxx (x, y) = 2
=) zxx (0, 1) =
(2z + 2) 4
[(2z + 2) 2 (2y + x) (2zy )]
zyy (x, y) = =) zyy (0, 1) = 2
(2z + 2)2
Pregunta 3
Derivando u(x, t) con respecto a x se tiene que :
@u
= f 0 (x + at) + g 0 (x at)
@x
@ 2u
= f 00 (x + at) + g 00 (x + at)
@x2
Asimismo las derivadas u(x, t) con respecto a t están dadas por :
@u
= af 0 (x + at) ag 0 (x at)
@t
@ 2u
= a2 f 00 (x + at) + a2 g 0 (x at)
@t2
Sustituyendo obtenemos que
@ 2u 2 00 2 0 2 00 0
2
2@ u
= a f (x + at) + a g (x at) = a (f (x + at) + g (x at)) = a
@t2 @x2

Autoevaluación No 3
El estudiante:
1) Resolverá problemas de aplicación empleando la derivada direccional
de una función f diferenciable en un punto (x, y) en una dirección dada por
vector unitario ub,
2) Empleará el teorema de la derivada implicita para obtener las derivadas
parciales de primer y segundo orden de una función implicita.
3) Hallará los puntos crı́ticos de una función f : ⌦ ✓ R2 ! R aplicando
la condición necesaria de punto crı́tico.

311
4) Utilizará el criterio de la segunda derivada para determinar los ex-
tremos locales de funciones de dos variables.

Tiempo: 2 horas
Pregunta 1
El conjunto de los puntos (x, y) tal que 0  x  5 , 0  y  5 es un
cuadrado
colocado en el primer cuadrante del plano XY . Supongamos que se calien-
ta
ese cuadrado de tal manera que T (x, y) = x2 + y 2 es la temperatura en el

puntoP (x, y). ¿En qué direccion se establecerá el flujo de calor en el punto
P0 (3, 4) ?.
Pregunta
⇣ xy 2 ⌘
Sea g , x2 + y 2 = 0 una ecuación que define a z como una función
z
de
x e y. Verifique que si gx ; gy y gz existen y son continuas en toda la región
en la que gz 6= 0; entonces

z (x2 y 2 )
yzx xzy =
xy

Pregunta 3
Hallar los valores extremos locales, absolutos y puntos sillas de
f (x, y) = xy(1 x2 y 2 ) en [0, 1]⇥ [0, 1].

Pauta Autocorrección
Pregunta 1
!
El flujo de calor en la región está dado por una función vectorial f (x, y)
y
su valor en cada punto depende de las coordenadas de éste. Sabemos
!
que f (x, y) es perpendicular a las curvas de isonivel T (x, y) = c donde
c es constante. Por consiguiente, el gradiente de la función T (x, y)
!
verifica esta condición. Entonces f (x, y) = rT (x, y) donde  es una
constante positiva ,llamada conductividad térmica. Nótese que el signo
negativo indica que el calor fluye desde puntos de mayor temperatura
a puntos de menor temperatura.

Como T (3, 4) = 25 el punto P está en la isoterma T (x, y) = 25 , que es


un cuadrante de la circunferencia x2 + y 2 = 25. Sabemos que el flujo

312
!
de calor en P0 (3, 4) es f (3, 4) = KrT (3, 4) .
Apartir de rT (x, y) = 2xbi + 2yb jse tiene que rT (3, 4) = 6bi + 8b
j.Ası́, el
! b b
flujo de calor en P0 es: f (3, 4) = (6i + 8j).
Como la conductividad térmica es positiva se puede afirmar que el
calor fluye en P0 en la dirección del vector unitario.
✓ ◆
(6bi + 8b
j) 3b 4 b
b= p
u = i+ j
36 + 64 5 5

Pregunta
⇣ xy 2 ⌘ xy
Sea g , x2 + y 2 = 0 y u = , v = x2 + y 2 .
z z
Entonces h i
y
gx g u · + g v · 2x
zx = = ⇣ z xy ⌘
gz gu · 2
h z i
x
gy g u · + g v · 2y
zy = = ⇣ z xy ⌘
gz gu · 2
z
Sumando términosh i h i
y x
gu · + gv · 2x gu · + gv · 2y
yzx xzy = y z x z
xy xy
gu · 2 gu · 2
z z
(x2 y 2 )
gu · z (x2 y 2 )
= z =
xy xy
gu · 2
z
Pregunta 3
Sea f (x, y) = xy(1 x2 y 2 ) en el interior del cuadrado (0, 1)⇥ (0, 1).
Obtengamos sus puntos crı́ticos
rf (x, y) = 0 ()
fx (x, y) = y(1 x2 y 2 ) + xy( 2x) = y 3x2 y y 3 = 0
fy (x, y) = x(1 x2 y 2 ) + xy( 2y) = x x3 3xy 2 = 0
Factorizando las ecuaciones anteriores, se tiene
y(1 3x2 y 2 ) = 0 =) y = 0 ó 1 3x2 y 2 = 0
x(1 x2 3y 2 ) = 0 =) x = 0 ó 1 x2 3y 2 = 0
Consideremos y = 0, x = 0 =) P0 (0, 0) no pertenece al interior del
cuadrado.
Tomemos ahora las expresiones
y = 0; 1 x2 3y 2 = 0 =) dos puntos criticos P1 (1, 0); P2 ( 1, 0)
estos puntos no pertenecen al interior del cuadrado.
Resolvamos las ecuaciones

313
x = 0; 1 3x2 y 2 = 0 =) P3 (0; 1); P4 (0; 1) estos puntos
tampoco pertenecen al interior del cuadrado.
Examinemos, las ecuaciones
1 x2 3y 2 = 0
=) y 2 = 1 3x2 =) 1 x2 3(1 3x2 ) = 0
1 3x2 y 2 = 0
1 1
4x2 = 1 =) x = ; x = esta última coordenada no pertenece al
2 2
interior del cuadrado, luego , al sustituir en la segunda ecuación
anterior produce: ✓ ◆
1 2 2 1 1 1 1
x = ;1 x 3y = 0 =) y = ; y = =) dos punto P5 ;
2 2 2 2 2
y ✓ ◆
1 1
P6 ; 2/ al interior del cuadrado.
2 2 ✓ ◆
1 1
Entonces estudiemos el Hessiano en el punto P5 ;
2 2
fxx✓(x, y)◆= 6xy, fxy (x, y) = 1 3x2 ✓ 3y 2 , f◆yy (x, y) = 6xy
1 1 3 1 1 1 3
H , = 2 2 = 2 > 0 y fxx , = < 0 =)
1 3
2 2 2 2 ✓ ◆ 2 2 2 ✓ ◆
1 1 1 1 1
hay un mı́nimo local de f en P5 , cuyo valor es f , = .
2 2 2 2 8
Finalmente, estudiemos la función en la frontera del cuadrado:
Para x 2 [0, 1]; y = 0 ; f (x, 0) = 0 =) mı́nimo en este segmento
abierto .
Ahora, en los vértices, f (0; 0) = 0 =) mı́nimo local y f (1, 0) = 0.
0
Para y 2 [0, 1]; x = 1; f (1; y) = y 3 =) f (y) = 3y 2 = 0 ()
00
y = 0; f (y) = 6y; f 00 (0) = 0
no existen valores extremos en el segmento abierto .
Ahora, en los vértices f (1, 0) = 0; f (1, 1) = 1. Por lo tanto hay mı́nimos
locales en (1, 0),(1, 1).
Para x 2 [0, 1]; y = 1; f (x; 1) = x3 =) f 0 (x) = 3x2 = 0 ()
x = 0; f 00 (x) = 6x; f 00 (0) = 0 entonces no existe ni máximo ni mı́nimo
en el segmento abierto.
En los vértices x = 0; x = 1 tenemos f (0, 1) = 0; f (1, 1) = 1 =)

mı́nimos locales.

Para x = 0; y 2 [0, 1]; f (0; y) = 0 =) mı́nimo local en el segmento


abierto.

314
Capı́tulo 4

Integración Multiple

Falta una introducción

4.1. Integrales dobles y triples


4.1.1. Integrales Dobles
Aspectos geométricos
Sea R un rectángulo representado por R = [a, b] ⇥ [c, d] y f una función
continua definida sobre R, es decir

f : R ✓ R2 ! R

Caso de funciones no negativas:


Supongamos que f (x, y) 0 y 8(x, y) 2 R tal que la gráfica de z = f (x, y)
está arriba del plano xy ,determinando una región V del espacio R3 , bajo la
superficie z = f (x, y) y sobre la región R.
Antes de dar una definición en el lenguaje de las Sumas de Riemann,
podemos decir que; bajo las condiciones anteriores el volumen de la región
V corresponde en este caso a lo que llamaremos integral doble de f sobre
R y que denotaremos:
Z Z Z Z
f (x, y)dA o f (x, y)dxdy
R R

315
Ejemplo:

Sea f (x, y) = x2 + y 2 y R = [0, 1] ⇥ [0, 2].

Figura 4.1: paraboloide

El volumen bajo el paraboloide z = x2 + y 2 sobre el rectángulo R corre-


sponde a la integral doble de f sobre R en este caso.
Z Z
(x2 + y 2 )dA
R

Debemos estar claros eso si, que el concepto de integral doble es mucho
más que esta interpretación geométrica

Integral doble sobre un rectángulo


Sea R = [a, b] ⇥ [c, d] un rectángulo y f una función acotada definida
sobre R es decir existe M > 0 tal que
M  f (x, y)  M para (x, y) 2 R
Observación: Una función continua sobre un rectángulo cerrado siempre es
acotada.

Partición
Sean P1 = {x0 , x1 , ..., xn } partición de [a, b]
P2 = {y0 , y1 , ..., yn } partición de [c, d]
Al conjunto P = P1 ⇥ P2 = {(xi , yj ) / 0  i  n, 0  j  n} lo llamare-
mos partición de R de valor n ⇥ n.
Sea kP1 k = máx {4xi = xi xi 1 / i = 1, 2, ..., n}
kP2 k = máx {4yi = yi yi 1 / j = 1, 2, ..., n}

316
Norma de la partición.
La norma de P denotada kP k se define por

kP k = máx {kP1 k , kP2 k}


kP k = máx {lij : diagonal de Rij }
(no es única forma de definir norma de P , pero ésta es la que usaremos)

Sumas superiores y sumas inferiores


Sean Rij rectángulo [xi 1 , xi ] ⇥ [yj 1 , yj ] 1  i, j  n y 4ij = área del
rectángulo Rij = (xi xi 1 ) · (yj yj 1 )
Definimos ahora, sumas inferiores y sumas superiores de Riemann de f
respecto de la partición dada, por
n
X
sP (f ) = mij (f )4ij ,
i,j=1
X n
SP (f ) = Mij (f )4ij
i,j=1

Como consecuencia de estas definiciones podemos decir de estas sumas:


i) Si P es una partición cualquiera de R

sP (f )  SP (f )

ii) Si P 0 es partición mas fina que P ( P ✓ P 0 ) entonces

sP (f )  sP 0 (f ) y SP 0 (f )  SP (f )

iii) Si P1 y P2 son dos particiones cualquiera de R

sP1 (f )  SP2 (f )

Con estas sumas formamos los respectivos conjuntos:


Conjunto de sumas inferiores.

{sP (f )/P es partición de R}


Conjunto de sumas superiores.

{SP (f )/P es partición de R}

317
Si m y M son cortas inferior y superior respectivamente de f en R en-
tonces
si A = (b a) · (d c).

i) sP (f ) m · A para todo P partición de R


es decir el conjunto de sumas inferiores es acotado inferiormente
ii) SP (f )  M · A 8 P partición de R, es decir, el conjunto de sumas
superiores es acotado superiormente.
Por lo tanto, haciendo uso del axioma del supremo (o del ı́nfimo) de la
axiomática de los números reales podemos definir.
Si R es un rectángulo de R2 y f una función acotada sobre R definimos:
a) Integral Inf erior de f sobre R por
Z Z
f dA = sup {sP (f ) : P es partición de R}
R

b) Integral Superior de f sobre R por


Z Z
f dA = ı́nf {SP (f ) : P es partición de R}
R

Sumas e Integrales
Las definiciones de estas respectivas integrales permiten afirmar que para
toda partición P de R y toda función acotada definida sobre R
Z Z Z Z
sP (f )  f dA  f dA  SP (f )
R R

Estamos ahora en condiciones de formular la definición de integral doble


sobre un rectángulo en base a sumas superiores e inferiores.
Una función f (x, y) definida y acotada sobre un rectángulo R se dice que
es Riemann integrable sobre R si
Z Z Z Z
f dA = f dA
R R

Si f es integrable
R R sobre R,R Rentonces la integral doble definida de f sobre R
se denota por R
f dA o R
f dxdy y en tal caso
Z Z Z Z Z Z
f dA = f dA = f dA
R R R

318
Nota: Alternativamente en cursos de cálculo se define:
Z Z n
X
f dA = lı́m f (xi , yj )Aij, (xi , yj ) 2 Rij
R kpk!0
i,j=1

lo que no es contradictorio sino que complementario y resultan planteamien-


tos equivalentes. Esto es la definición utilizando el concepto de sumas inter-
medias de Riemann.

Teorema 4.1.1. Cualquier función continua definida en un rectángulo cer-


rado R es integrable

Demostración:
La demostración de este hecho no resulta de interés en este curso a pesar
de su enorme importancia, dejemos las cosas aquı́ a la imaginación del
estudiante.

Proposición 4.1.1. (Propiedades básicas de la Integral Doble)


De la definición se desprende que:
1) Si f (x, y) = 1 todo (x, y) 2 R,la integral resulta el área de la región
Z Z
A = Área de R = dA
R

2) Si f es integrable en R
Z Z Z Z
cf dA = c f dA
R R

3) Si f y g son funciones integrables en R


Z Z Z Z Z Z
(f + g)dA = f dA + gdA
R R R

4) Si f y g son funciones integrables en R y f (x, y)  g(x, y) para todo


(x, y) 2 R entonces Z Z Z Z
f dA  gdA
R R

5) Si f es integrable sobre R, entonces |f | es integrable sobre R y


Z Z Z Z
f dA  |f | dA
R R

319
Teorema 4.1.2. (Teorema del Valor Medio para Integrales Dobles)
Si f (x, y) es continua sobre rectángulo R con área A(R), entonces existe
un punto (", ⌘) en el interior de R tal que
Z Z
f (x, y)dA = f (", ⌘) · A(R)
R

Demostración:
Sea

m = mı́n {f (x, y) : (x, y) 2 R}


M = máx {f (x, y) : (x, y) 2 R}

y supongamos que m < M. Entonces m  f (x, y)  M, y si f no


es identicamente igual a m o M , entonces
Z Z
m · A(R) < f (x, y)dA < M · A(R)
R

El teorema del valor medio de las funciones continuas asegura que existe
un
punto (", ⌘) en el interior R tal que
Z Z
f (", ⌘) = f (x, y)dA A(R)
R

Z Z
=)) f (", ⌘) · A(R) = f (x, y)dA
R

4.1.2. Integrales sobre conjuntos acotados de R2


En este caso extenderemos la definición de integral doble a regiones que
no son necesariamente rectángulos, sino que regiones acotadas en general.
Supongamos que S es una región cerrada y acotada de R2 , por ejemplo un
circulo, un triángulo, un rombo etc. , cualquier región con estas caracterı́sticas
se puede poner dentro de un rectángulo R
Sea R rectángulo que contiene a región cerrada y acotada S y f una
función definida y acotada en S, extendemos f a R de la siguiente forma

f (x, y), (x, y) 2 S
fR (x, y) =
0, (x, y) 2 R S
fR la consideraremos como la extensión de f a todo R.

320
Sea S una región acotada de R2 y f una función definida y acotada sobre
S, si R es en rectángulo tal que R R R◆ S y fR la extensión de f a R del tipo
definido aquı́, entonces si existe f dA , definimos
R R
Z Z Z Z
f dA = fR dA
S R

Importante.

Las propiedades enunciadas, de la integral doble en rectángulos siguen


siendo válidas en conjuntos más generales lo que se puede justificar por la
definición anterior

Integrales Iteradas
Una integral de la forma
Z bZ h(x)
f (x, y)dydx
a g(x)
Rb
se llama integral iterada y se interpreta como a F (x)dx donde para cada
R h(x)
x 2 [a, b],con F (x) = g(x) f (x, y)dy.
Si f (x, y) es función continua sobre {(x, y) : a  x  b, g(x)  y  h(x)}
y
G(x, y) una primitiva en la segunda variable, de f (x, y), es decir ,
@G(x, y)
= f (x, y) para cada x 2 [a, b] y todo g(x)  y  h(x), el
@y
teorema
Fundamental del cálculo permite que
Z h(x)
y=h(x)
F (x) = f (x, y)dy = G(x, y) |y=g(x) = G(x, h(x)) G(x, g(x))
g(x)

Se puede interpretar entonces la integral iterada como un proceso sucesivo


de integración ası́
Z b Z h(x) Z b Z h(x) !
f (x, y)dydx = f (x, y)dy dx
a g(x) a g(x)

De manera similar se tiene


Z d Z h(y) Z Z !
d h(y)
f (x, y)dxdy = f (x, y)dx dy
c g(y) c g(y)

321
Ejemplo.
Z 1 Z x Z 1 Z x
2
(x + 4xy)dydx = (x2 + 4xy)dy dx
0 0 0 0
Z 1 ⇥ ⇤x
= x2 y + 2xy 2 0
dx
0

Z 1 Z 1 1
3 3 3 4 3
= (x + 2x )dx = 3x3 dx = x =
0 0 4 0 4

Otros
R 4ejemplos:
R 2 p Interprete y evalúe:
i) 0 0 x ydxdy
R2R4 p
ii) 0 0 x ydydx
R1Rx
iii) 0 0 sin(x2 )dydx
R2R3
iv) 0 px (x2 + y) dydx
R 5 R x2
v) 0 2x (x + y)dydx

Algunas Respuestas
R4R2 p R 4 hR 2 p i R 4 h 2 p i2
i) 0 0 x ydxdy = 0 0 x ydx dy = 0 x2 y dy =
h i4 0
R4 p 2 32 32
= 0 2 ydy = 2 · 3 y = 3
R1Rx 0
iii) 0 0 sin(x2 )dydx = 1 cos
2h
1
R2R3 p i
2 2
iv) 0 x (x + y) dydx 16 1
p
7

A continuación examinaremos la evaluación de la integral doble por medio


de integrales iteradas.

4.1.3. Teorema de Fubini


Sea f una función continua en una región R cerrada y acotada, entonces
a) Si R = {(x, y) a  x  b, g1 (x)  y  g2 (x)} y g y son funciones
continuas en [a, b] se tiene:
Z Z Z bZ g2 (x)
f dA = f (x, y)dydx
R a g1 (x)

b) Si R = {(x, y) c  y  d, h1 (y)  x  h2 (y)} y h1 y h2 son

322
Figura 4.2: Región tipo a

Figura 4.3: Región tipo b

funciones continuas en [c, d] se tiene


Z Z Z d Z h2 (y)
f dA = f (x, y)dxdy
R c h1 (y)

Ejemplo

Este ejemplo ilustra como este teorema se adapta a la situación del prob-
lema, en este caso se pide calcular
Z Z
xydA
R

y R es la región triángular del plano con vértices en los puntos A(-6,-2),


B(-1,3) y C(9,-7).
Solución:

323
Figura 4.4: Región triángular del plano

La región se debe subdividir en dos subregiones del tipo (a)., tal como
lo muestra la figura siguiente
Los segmentos de recta AB, BC Y AC tienen ecuaciones y = x + 4,
y = x + 2 e y = 13 x 4 respectivemente. Las regiones pueden
escribirse.

I : 6  x  1, 13 x 4  y  x + 4
II : 1  x  9, 13 x 4  y  x + 2
Entonces aplicando T. de Fubini a ambas regiones se tiene

Z Z Z 1 Z x+4 Z 9 Z x+2
xydA = xydydx + xydydx
1 1
R 6 3
x 4 1 3
x 4
Z 1 Z
1 ⇥ 2 ⇤x+4 1 9 ⇥ 2 ⇤ x+2
= xy 1 x 4 dx + xy 1 x 4 dx
2 6 3 2 1 3
Z 1 Z 9
1 1
= (4x3 + 24x2 )dx + (4x3 30x2 54x)dx
9 6 9 1
1025
=
27
Se observa que R también puede subdividirse en dos regiones del tipo
(b) mediante una recta paralela al eje horizontal que pase por A.
Proposición 4.1.2. Suponga que S es una región acotada y sea C una
curva la cual divide a S en dos subregiones S1 y S2 .
Si f es continua en S , lo es tambien en S1 y S2 , y
Z Z Z Z Z Z
f (x, y)dA = f (x, y)dA + f (x, y)dA
S S1 S2

324
Figura 4.5: Región triángular del plano, tipo b

Demostración.- Directamente de la definición eligiendo un rectangulo sufi-


cientemente grande que contenga a S y extendiendo f de S a R,
de S1 a R, S2 a R.

RR 2
Ejemplo. Calcular S
(x + y)dA, donde S es la región limitada por
la recta y = x y la curva y = x3

Figura 4.6: Región comprendida entre las curvas y = x y y = x3

325
Solución.- En este caso

S1 = (x, y) 2 R2 : 1  x  0, x  y  x3

S2 = (x, y) 2 R2 : 0  x  1, x3  y  x

Z Z Z Z Z Z
2 2
(x + y)dA = (x + y)dA + (x2 + y)dA
S S1 S2

Z Z Z 0 Z x3 Z 0  x3
y2
(x2 + y)dA = (x2 + y)dydx = x2 y + dx
S1 1 x 1 2 x
Z 0
x6 x2
= [(x5 + ) (x3 + )]dx
1 2 2
 6 0
x x7 x4 x3
= +
6 14 4 6 1
6 7
( 1) ( 1) ( 1)4 ( 1)3
= + +
6 14 4 6
1
=
84

Z Z Z 1 Z x Z 1  x
2 2 2 y2
(x + y)dA = (x + y)dydx = x y+ dx
S2 0 x3 0 2 x3
Z 1
x2 x 6
= [(x3 + ) (x5 + )]dx
0 2 2
 1
x4 x3 x6 x7
= ( + ) ( + )
4 6 6 14 0
4 3 6 7
1 1 1 1
= ( + ) ( + )
4 6 6 14
5
=
28

Por lo tanto Z Z
1 5 1
(x2 + y)dA = + =
S 84 28 6

326
4.1.4. Áreas y Volumenes
Área
Como se dijo en la introducción y de acuerdo a la idea geométrica si R
es una región plana entonces el área de R se calcula con la integral doble.
Z Z
A(R) = dA
R

Ejemplo. Calcule el área de la región interior a la circunferencia x2 + y 2 =


2ax arriba de la parábola ay = x2 , a > 0.

Solución:

Figura 4.7: Región en el plano xy, acotada por la circunferencia (x a)2 + y 2 = a2


x2
y la parábola y = a2
n 2 p o
Sea R = (x, y) 2 IR2 / 0  x  a, xa  y  2ax x2

Z Z Z Z p
a 2ax x2
A = dA = dydx
x2
R 0 a
Z a p x2
= ( 2ax x2 )dx
0 a
a2
= (3⇡ 4)
12

Volumen
Si R es una región plana, z = f (x, y), z = g(x, y) son dos superficies
tal que f (x, y) g(x, y) 8(x, y) 2 R , el volumen entre ambas superficies

327
al interior de la región se puede calcular usando la siguiente integral doble

Z Z
V = [f (x, y) g(x, y)]dA
R

Ejemplo
1) Use la integral doble para determinar el volumen del tetraedro acotado
por los planos coordenados y el plano 3x + 6y + 4z 12 = 0.
Solución: Para determinar la región R hacemos z = 0 y encontramos
su intersección con el plano dado xy . Ası́ z = 0 =) 3x + 6y = 12 =) y =
1
2
x+2
La región en el plano xy está acotada por el eje x, el eje y y la recta
y = 12 x + 2, por lo tanto

x
Figura 4.8: Región en el plano xy, acotada por el x = 0, y = 0 y la recta y = 2 +2

Figura 4.9: Tetraedro

1
R = (x, y) 2 IR2 / 0  x  4, 0  y  2
x +2

328
12 3x 6y 3 3
z = f (x, y) = =3 4
x 2
y y z = g(x, y) = 0
4
Z Z
V = f (x, y)dA
R
Z Z ✓ ◆ Z 4 Z 1 x+2 ✓ ◆
3 3 2 3 3
= 3 x y dA = 3 x y dydx =
R 4 2 0 0 4 2
Z 4 ✓ ◆ 1
x+2 Z 4✓ ◆
3 3y 2 2 3 2 3
= 3 x y dx = x x + 3 dx
0 4 4 0 0 16 2
 3 4
x 3x2
= + 3x = 4
16 4 0

Otros ejemplos: RR
1) Usando integral iterada calculep s (x2 + 2y) dA, donde S: región
comprendida entre y = x2 e y = x
2) Calcule el volumen del sólido limitado por los cilindros x2 + z 2 = 16 y
y 2 + z 2 = 16 .
Este es un interesante ejercicio, puede empezar por bosquejar el
sólido o una parte de él, pues se puede aprovechar su simetrı́a.

4.1.5. Cambio de variable


Un cambio de variables adecuado puede no sólo simplificar el integrando
sino también la región donde se evalúa la integral.
Sea f una función continua definida sobre la region R cerrada y acotada.
Considerese la integral doble
Z Z
f (x, y)dxdy
R
Definimos T , transformación invertible

x = x(u, v), (u, v) 2 S


y = y(u, v), (u, v) 2 S
@(x, y)
tal que 6= 0, la que produce una correspondencia biunı́voca
@(u, v)
entre R y S donde R es una región en xy y S es la nueva región en plano
uv. resultado de la transformación T .
Si P es una partición definida en R la transformación induce a su vez una
correspondiente partición en S de tal modo que si Rij es un subrectángulo

329
generado por la partición P en R, denotaremos por Sij el correspondiente
subrectángulo en S

Si 4 Aij = Área de Rij y 4 A0ij = Área de Sij


Se tiene la siguiente razón entre las áreas.

4Aij @(x, y) @(x, y)


0
t =) 4Aij t 4 A0ij
4Aij @(u, v) @(u, v)
Entonces

@(x, y)
f (x, y) 4 Axy t f (x, y) 4 Auv
@(u, v)
De esta relación y la definición de integral doble se tiene el siguiente
teorema de cambio de variable.

Teorema 4.1.3. Sea R una región en el plano xy acotado por una curva
simple cerrada y suave y que S es la imagen de R bajo la transformación T
invertible, definida:

x = x(u, v), (u, v) 2 S


y = y(u, v), (u, v) 2 S

donde x(u, v), y(u, v) son continuamente diferenciables en un dominio que


contiene a S en cual
@(x, y)
J⌘ 6= 0
@(u, v)
Si f (x, y) define una función continua sobre R se tiene:
Z Z Z Z
@(x, y)
f (x, y)dxdy = f (x (u, v) , y (u, v)) dudv
R S @(u, v)
Ejemplo

Calcular Z Z
3xydA
R
Sea R la región limitada por las rectas x 2y = 0, x 2y = 4, x + y = 4,
x + y = 1.
Solución:
Sea u = x + y, v = x 2y
Resolviendo el sistema lineal obtenemos

330
Figura 4.10: Aplicación definida por una ecuación vetorial

Figura 4.11: Aplicación definida por una transformación lineal

%
u=x+y x = 13 (2u + v)
=)
v = x 2y y = 13 (u v)

Además el jacobiano de la transformación es

@x @x 2 1
@(x, y) @u @v 3 3
2 1 1
= @y @y = 1 1 = =
@(u, v) @u @v 3 3 9 9 3

Aplicando el teorema del cambio de variable , obtenemos

331
Z Z Z Z
1 1 1
3xydA = 3( (2u + v) · (u v)) dA
R S 3 3 3
Z Z
1
= (2u + v) (u v) dA
9 S
Z Z
1 4 0
= (2u + v) (u v) dvdu
9 1 4
104
=
9

Ejemplo
Calcular
Z Z p
x2 + y 2 dA
R

R región del plano xy limitada por x2 + y 2 = 4, x2 + y 2 = 9

Figura 4.12: Región del plano xy limitada por x2 + y 2 = 4, x2 + y 2 = 9

Solución:
Sea x = r cos ✓, y = r sin ✓ el cambio de variable a polares
entonces
@x @x
@(x, y) @r @✓ cos ✓ r sin ✓
= @y @y = =r
@(r, ✓) @r @✓
sin ✓ r cos ✓

332
Z Z p Z Z
x2 + y 2 dA = r · |r| drd✓
R S
Z 2⇡ Z 3
= r2 drd✓
Z0 2⇡ 2
19 38
= d✓ = ⇡
0 3 3

4.2. Aplicaciones de la integral doble

4.2.1. Masa de una región plana de densidad variable.

Sea (x, y) función positiva y definida sobre un conjunto cerrado y aco-


tado S con área no nula, que indica la densidad en cada punto (x, y) de S.

La masa de S es la integral de la función densidad.


Z Z
M (S)= (x, y)dA
S

En el caso que la densidad es constante = k, la masa es el producto del


área por la densidad: M (S) = k · A(S).

Ejemplo.

Encuentre la masa de un cı́rculo de radio a si su densidad es veces la


distancia al centro.
Solución:

Con el uso de coordenadas polares el cálculo de la integral resultante es


más sencillo.

333
Figura 4.13:

Z Z p Z 2⇡ Z a
2 ⇡a3
M (S) = x2 + y 2 dydx = ⇢2 d⇢d✓ =
S 0 0 3

4.2.2. Momentos y centroide de una región plana

Para un conjunto S acotado y de área positiva , y una función densidad


definida en S, tenemos las siguientes definiciones.

Primer momento con respecto al eje y:


Z Z
My = (x, y)xdA
S

Primer momento con respecto al eje x:


Z Z
Mx = (x, y)ydA
S

Segundo momento con respecto al eje y:


Z Z
Iy = (x, y)x2 dA
S

334
Segundo momento con respecto al eje x:
Z Z
Ix = (x, y)y 2 dA
S

Segundo momento Polar con respecto al origen:


Z Z
I0 = (x, y)(x2 + y 2 )dA
S

Centroide: ✓ ◆
My M x
(x, y) = ,
M M
Cuando la función densidad es variable y está asociada con la distribución
de la masa, los segundos momentos se llaman también momentos de inercia
y el centroide se le llama también centro de masas.
En una forma más general, el primer y segundo momento de un conjunto
S se puede definir con respecto a una linea recta cualquiera L.
Z Z
ML = (x, y)D(x, y)dA
Z ZS
IL = (x, y) [D(x, y)]2 dA
S

Siendo D(x, y) la distancia de la recta L al punto (x,y).


Ejemplo.

Una lámina triangular tiene los vértices (0, 0), (1, 0) y (1, 2), y tiene
densidad (x, y) = x2 y. Halle su centro de masa.

Solución:
En este caso R = {(x, y) 2 R2 / 0  x  1, 0  y  2x}

Debemos calcular M = M (S), My y Mx :


Z Z Z 1 Z 2x Z 1  2x
2 2 x2 y 2
M = x ydA = x ydydx = dx
S 0 0 0 2 0
Z 1  1
4 2x5 2
=) M = 2x dx = =
0 5 0 5

335
Figura 4.14:

Calculemos ahora
Z Z Z 1 Z 2x
2
My = x yxdA = x3 ydydx
S 0 0
Z 1  6 1
5 2x
= 2x dx =
0 6 0
1
=) My =
3

Por último

Z Z Z 1 Z 2x
2
Mx = x yydA x2 y 2 dydx
S 0 0
Z 1  2 3 2x Z 1
xy 8x5
= dx = dx
0 3 0 0 3
 6 1
8x 4
=) Mx = =
18 0 9

Ası́ tenemos que


✓ ◆
My M x 5 10
(x, y) = , =( , )
M M 6 9

336
4.3. Integrales triples
4.3.1. Ideas preliminares
Los conceptos a desarrollar en lo referente a la Integral en su génesis, su
significado y su cálculo en el caso de tres variables es similar al mismo tema
en funciones de una variable y de dos variables. El tratamiento hecho en el
caso de la integral doble se extiende en forma natural a las integrales triples
en sus mecanismos conceptuales y métodos de cálculo involucrados solo hay
un cambio en el escenario, el espacio R3 .
Atendiendo a la declaración anterior no haremos el detalle de la gen-
eración del concepto porque como ya lo dijimos se trata de una generalización.
Para mantener el marco de referencia, pensemos en sumas superiores, sumas
inferiores, integral superior e integral inferior para funciones definidas sobre
una caja rectangular de tipo:

h (x, y, z) : {a1  x  b1 , a2  y  b2 , a3  z  b3 }

Con este trasfondo se plantea el teorema de Fubini que permitirá el cálculo


de la integral triple.

4.3.2. Teorema de Fubini


Si f (x, y, z) esta definida sobre una región R = {a1  x  b1 , a2  y  b2 , a3  z  b3 } ,
entonces:

Z Z Z Z b1 Z b2 Z b3
f (x, y, z)dv = f (x, y, z)dzdydx
R a1 a2 a3

siempre que estas integrales existan.


Observación: Hay otras cinco formas de calcular la integral triple de-
pendiendo del orden de integración en la integral iterada.
Ejemplo:
RR R
Calcule R
(x2 + yz) dv, donde R = {0  x  2, 1  y  2, 1  z  3}

Solución:

337
Z Z Z Z 2 Z 2 Z 3
2
x + yz dv = x2 + yz dzdydx
R 0 1 1
Z 2 Z 2 ✓
◆3
z2 2
= x z+y dydx
0 1 2 1
Z 2 Z 2 ✓ ◆ ⇣
2 9 y⌘
= 3x + y x2 + dydx
0 1 2 2
Z 2Z 2
= 4x2 + 4y dydx
0 1
Z 2
2
= 4x2 y + 2y 2 1 dx
Z0 2
= 12x2 + 6 dx
0
2
= 4x3 + 6x 0
= 44

4.3.3. Teorema de la integral triple (Para dominios


más generales)
Si f (x, y, z) esta definida sobre un conjunto acotado R formado por todos
los puntos tales que a1  x  b1 , y1 (x)  y  y2 (x) y z1 (x, y)  z 
z2 (x, y)entonces:
Z Z Z Z b1 Z y2 (x) Z z2 (x)
f (x, y, z)dv = f (x, y, z)dzdydx
R a1 y1 (x) z1 (x)

siempre que ambas integrales existan.

Importante.
? Hay otras cinco formas de calcular la integral triple dependiendo el
orden de integración para el calculo de la integral iterada.

? Si S es un conjunto acotado el cual tiene área en el plano XY y f (x, y, z)


es una función definida, acotada y no negativa sobre S, y si R es el conjunto
de todos los (x, y, z) tal que (x, y) 2 S y 0  z  f (x, y), entonces la región
R tiene volumen si y solo si f (x, y) es integrable sobre S y en tal caso

338
Z Z
V (R) = f (x, y)dA
S

? Si R es una rigión de R3 que tiene volumen, entonces

ZZ Z
V (R) = f (x, y, z)dv con f (x, y, z) = 1
R

Ejemplo

Sea R la región acotada por los paraboloides z = x2 + y 2 y


2z = 12 x2 y 2 . Usando integral triple calcule el volumen de R.
La representación gráfica de estas superficies es.

Figura 4.15: Región acotada por los paraboloides z = x2 + y 2 y 2z = 12 x2 y2

Por separado, tenemos

Solución:
La curva de intersección es el cı́rculo x2 + y 2 = 4, z = 4
Si f (x, y, z) = 1 se tiene el volumen

339
Figura 4.16:

ZZ Z
v(R) = f (x, y, z)dv
R
Z Z p Z 12 x2 y2
2 4 x2 2
= p
dzdydx
2 4 x2 x2 +y 2
Z Z p
2 4 x2 ⇥ ⇤
= 6 4 x2 + y 2 dydx
0 0
Z 2 3
= 4 4 x2 2
dx
0
= 12⇡

Ejemplo

Calcular el volumen de la región del espacio limitada por las superficies


cı́lindricas x2 + z 2 = 1, y 2 + z 2 = 1
Solución:
La representación gráfica de estas superficies es
Utilizaremos la simetrı́a del problema y proyectaremos la región al plano
xz ( también se podrı́a proyectar al plano yz ).
La proyección nos da un cı́rculo de radio 1

340
Figura 4.17: Región acotada por los cilindros x2 + z 2 = 1, y 2 + z 2 = 1

La región se puede expresar:

1  x1
p p
1 z2  y  1 z2
p p
1 x2  z  1 x2

Expresando el cálculo del volumen como una integral triple tenemos


Z Z Z
V = dxdydz
R

usando integrales iteradas


Z Z p Z p Z Z p
1 1 x2 1 z2 1 1 x2 p
V = p p
dydzdx = p
2 1 z 2 dzdx
1 1 x2 1 z2 1 1 x2

Si seguimos por este camino llegamos a una expresión dı́ficil de


resolver ( intentelo), recurriremos entonces al cambio de orden de
integración que es un recurso siempre disponible

341
Z Z p Z Z p
1 1 x2 p 1 1 z2 p
V = p
2 1 z 2 dzdx = p
2 1 z 2 dxdz
1 1 x2 1 1 z2
Z 1 3 1
z 16
= 4(1 z 2 )dxdz = (4z 4 ) =
1 3 1 3

El volumen calculado es
16
V = unidades de volumen
3

4.3.4. Cambio de variable para integrales triples


Sea T : U ✓ R3 ! R3 una transformación de clase C 1 definida por:

x = x (u, v, w)
y = y (u, v, w)
z = z (u, v, w)
Recordando el jacobiano de la transformación se tiene:
@x @x @x
@ (x, y, z) @u
@y
@v
@y
@w
@y
J= = @u @v @w
@ (u, v, w) @z @z @z
@u @v @w

Como en el caso anterior de dos variables, el jacobiano mide como la curva


la transformación distorsiona su dominio.
Formula de cambio de variable para integrales triples
Sea R una región en el espacio xyz y S una región en el espacio uvw que
corresponde a R bajo la transformación T definida por x = x (u, v, w) , y =
y (u, v, w) y z = z (u, v, w) siempre que T sea de clase C 1 y uno a uno,
@(x,y,z)
@(u,v,w)
6= 0 en S. Entonces:

ZZ Z ZZ Z
f (x, y, z)dv = f (x (u, v, w) , y (u, v, w) , z (u, v, w)) |J| dudvdw
R S

Donde
@ (x, y, z)
J=
@ (u, v, w)
Los cambios más usados en integrales triples es a coordenadas cilindricas
y coordenadas esféricas dependiendo de la naturaleza del problema.

342
Coordenadas cilı́ndricas
El cambio de variable es:
x = r cos ✓
y = r sin ✓
z=z

Figura 4.18: Cambio a coordenadas cilı́ndricas

Supongamos que: P es un punto del espacio de coordenadas xyz, P1


proyección de P en plano xy, r radio vector de O a P1 y ✓ el pángulo entre
!
eje x y OP1 , medido del lado positivo del eje x entonces r = x2 + y 2 y
✓ = arctan xy Tenemos

cos ✓ r sin ✓ 0
@ (x, y, z)
J = = sin ✓ r cos ✓ 0
@ (r, ✓, z)
0 0 1
= r cos2 ✓ + r sin2 ✓ = r

ZZ Z ZZ Z
f (x, y, z)dv = f (r cos ✓, r sin ✓, z) · r · drd✓dz
R S
Ejemplo:

Use coordenadas cilı́ndricas para calcular el volumen del sólido limitado


por el paraboloide z = x2 + y 2 y el plano z = 4.
Solución:
En el espacio xyz, la figura es al interior del elipsoide y limitado por
arriba por el plano z = 4 que es un plano paralelo al plan xy

343
Figura 4.19: Sólido limitado por el paraboloide z = x2 + y 2 y el plano z = 4

Aprovechando la simetrı́a del sólido calculamos la cuarta parte de él (por


conveniencia). En esta situación la región transformada viene descrita por:
⇡ 2
0  r  2, 0  ✓  ,r < z < 4
2
La descripción de la región en las nuevas variables es fundamental para
el acertado planteamiento de la integral doble como integral iterada, en este
caso esta circunstancia es evidente

Z ⇡ Z Z 2 Z 4
V 2
= rdzdrd✓
4 0 0 r2
Z ⇡ Z 2
2
= 4r r3 drd✓
0 0
Z ⇡
2
= 4d✓ = 2⇡
0

) V = 8⇡

Coordenadas Esféricas
El cambio de variable es:
x = ⇢ cos ✓ sin
y = ⇢ sin ✓ sin
z = ⇢ cos

344
Figura 4.20: Cambio a coordenadas esfericas

Supongamos que P es un punto del espacio de coordenadas xyz


P1 proyección de P en plano xy
!
⇢ magnitud del radio vector OP
!
✓ el ángulo entre eje x y OP1 , medido del lado positivo del eje x
!
angulo formado por OP y el eje Z, medido del lado positivo del eje z.

cos ✓ sin sin ✓ sin cos


@ (x, y, z)
J = = ⇢ sin ✓ sin ⇢ cos ✓ sin 0
@ (⇢, ✓, )
⇢ cos ✓ cos ⇢ sin ✓ cos ⇢ sin
= ⇢2 sin

4.3.5. Formula del cambio de variable

ZZ Z ZZ Z
f (x, y, z)dv = f (⇢ cos ✓ sin , ⇢ sin ✓ sin , ⇢ cos ) ⇢2 sin drd✓dz
R S

Las coordenadas esféricas se usan preferentemente en el caso en que uno


o ambas superficies que acotan la región de integración es una esfera centrada
en el origen, esto se observa en los siguientes ejemplos.
Ejemplo 1

345
Figura 4.21: Sólido limitado imitada inferiormente por el semicono z 2 = x2 +
y2, z 0 y superiormente por la esfera x2 + y 2 + z 2 = 9.

Hallar el volumen de la región sólida limitada inferiormente por el semi-


cono z 2 = x2 + y 2 , z 0 y superiormente por la esfera x2 + y 2 + z 2 = 9.
Solución:
Haciendo la intersección de

x2 + y 2 + z 2 = 9 y z = x2 + y 2

resulta que la intersección de estas superficies es una circunferencia en el


plano z = p32 definida por las ecuaciones

9 3
) x2 + y 2 = , z = p
2 2

esto permite visualizar que (0, p32 , p32 ) es un punto de la intersección por lo
que 0   ⇡4 , la esfera tiene radio 3 por lo cual 0  ⇢  3, y 0  ✓  2⇡.
Como se está calculando el volumen de una región que es simétrica respecto
del eje z, la cuarta parte de la región queda descrita por

⇡ ⇡
0  ⇢  3, 0  ✓  ,0  
2 4
346
esto implica )
ZZ Z Z ⇡ Z ⇡ Z 3
V 2 4
= f (x, y, z)dV = ⇢2 sin d⇢d d✓
4 R 0 0 0
Z ⇡ Z ⇡
V 2 4
= 9 sin d d✓
4 0 0
Z ⇡ ✓ ◆ p !
V 2 9 9 2 1
= p + 9 d✓ = p ⇡
4 0 2 2 2

Por lo tanto p !
2 1
V = 18 p ⇡
2
Ejemplo 2

Utilice coordinadas
p esféricas para hallar el volumen del sólido que está ar-
riba del cono z = x2 + y 2 y debajo de la esfera x2 + y 2 + z 2 = z

p
Figura 4.22: Sólido que está arriba del cono z = x2 + y 2 y debajo de la esfera
x2 + y 2 + z 2 = z

Solución:

1 2 1
x2 + y 2 + z 2 = z () x2 + y 2 + (z ) =
2 4
es una esfera que pasa por el origen y tiene centro en (0, 0, 12 ).
La ecuación de la esfera en coordinadas esféricas es ⇢ = cos ,a su vez de
la ecuación del cono se infiere que 0   ⇡4 .

347
Por lo que la región en coordenadas esfericas esta descrita por

, 0   , 0  ✓  2⇡, 0  ⇢  cos
4
El volumen de la región es
ZZ Z Z 2⇡ Z ⇡ Z cos
4
V = f (x, y, z)dV = ⇢2 sin d⇢d d✓
R 0 0 0
Z 2⇡ Z ⇡  3 cos
4 ⇢
= sin d d✓
0 0 3 0
Z ⇡  ⇡
2⇡ 4 3 2⇡ cos4 4

= sin cos d = . =
3 0 3 4 0 8
Por lo tanto

V =
8

4.3.6. Masa, Momentos, y Centroide de una Región


del Espacio

Como en el caso de dos dimensiones, si (x, y, z) función positiva y con-


tinua, definida sobre una region compacta (conjunto cerrado y acotado) W
con volumen, que indica la densidad en cada punto (x, y, z) de W.
La masa de W es dada por la integral de la función densidad.
Z Z Z
M (W )= (x, y, z)dV
W

El primer momento de W se define respecto de algun plano, y el segundo


momento (o momento de inercia) con respecto a algún plano, lı́nea o punto.
Daremos aqui solo las formulas tı́picas planos coordenados, ejes y el ori-
gen.
Primer momento con respecto al plano yz.
Z Z Z
Myz = (x, y, z)xdV
W

Segundo momento con repecto al plano yz.


Z Z Z
Iyz = (x, y, z)x2 dV
W

348
Segundo momento con repecto al eje x
Z Z Z
Ix = (x, y, z)(y 2 + z 2 )dV
W

Segundo momento polar con respecto del origen


Z Z Z
I0 = (x, y, z)(x2 + y 2 + z 2 )dV
W

⇣ ⌘
Myz Mzx Mxy
Centroide: ( x, y, z) = M
, M , M .
Ejemplo

Determinar el centroide de la porción de la esfera x2 + y 2 + z 2  a2 , en


el primer octante, asumiendo densidad constante.
Solución:
El problema no pierde generalidad si suponemos que = 1, y claramente
x = y = z.
Necesitamos calcular solamente
Z Z Z
Mxy = (x, y, z)zdV
W

y usando coordenadas esfericas esta integral queda


Z ⇡ Z ⇡ Z a
2 2 ⇡a4
Mxy = (⇢ cos )⇢2 sen d⇢d d✓ =
0 0 0 16
3
Como V = ⇡a6 , el centroide es 3
8
a, 38 a, 38 a
Ejemplo

Encontrar el momento de inercia IL de un cilindro circular recto co a radio


de la base, h altura y densidad proporcional a la distancia al eje del cilindro,
con respecto a una recta L paralela al eje del cilindro y a una distancia b
de él.

349
Solución:

La recta L se define por: x = b, y = 0

El cilindro es descrito por: 0 r  a, 0  z  h

La densidad es = kr

Entonces

Z Z Z
IL = (x, y, z)((x b)2 + y 2 )dV
W
Z 2⇡ Z aZ h
= kr((x b)2 + y 2 )rdzdrd✓
0 0 0
Z 2⇡ Z a Z h
= kr2 (r2 + b2 )dzdrd✓ + 0
0 0
⇣ 02 ⌘
b2
= 2⇡ka3 h a
5
+ 3

4.4. Ejercicios resueltos integrales triples y


dobles

4.4.1. Cálculo de integrales dobles en coordenadas rectángu-


lares cartesianas

Problema 1
ZZ
p
Calcular x + ydxdy si D es la región acotada por las respectivas
D
rectas y = x, y = x y x=1
Solución:
Se tiene que la región D = {(x, y) 2 IR2 / 0  x  1; x  y  x}

350
Figura 4.23: D región acotada por y = x, y = x y x = 1

ZZ Z 1 Z x
p p
x + ydxdy = x + ydydx
D 0 x
Z
2 1 x
= (x + y)3/2 dx
3 0 x
Z
2 1
= (2x)3/2 dx
3 0
25/2 2 1
= (x)5/2
3 5
p 0
8 2
=
15

Problema 2
ZZ p
Calcular x2 y 2 dxdy si D es el dominio limitado por el triángulo
D
de vértices A (0, 0) , B(1, 1), C (1, 1) . Véase figura 4.23
Solución
Entonces se tiene que el dominio está delimitado por las rectas y = x,
y = x y x = 1.
Luego el dominio de integración es:

D = (x, y) 2 IR2 / 0  x  1; x  y  x

.
Integrando a franjas verticales, resulta

351
ZZ p Z p 1 Z x
x2 y 2 dxdy = x2 y 2 dydx
D 0 x
Z 1Z x r ⇣ y ⌘2
= x 1 dydx
0 x x
y
Hacemos el cambio de variables = sent =) dy = x cos tdt y
x
determinemos los limites.⇣ ⌘
x ⇡
Para y = x =) arcsen = arcsen (1) = .
x✓ ◆ 2
x ⇡
Para y = x =) arcsen = arcsen ( 1) =
x 2
Por tanto

Z Z r ⇣ y ⌘2 Z Z ⇡
1 x 1 2 p
x 1 dydx = x2 1 sen2 tdtdx
0 x x 0 ⇡
2
Z 1 Z ⇡
2
= x2 cos2 tdtdx

0 2
Z 1 Z ⇡
2 1 + cos 2t
= x2 ( )dtdx
0 ⇡
2
2
Z 1  ⇡

2 t sen2t 2
= x + dx
0 2 4 ⇡
2
Z 1

= x2 dx
2 0
 1
⇡ x3 ⇡
= =
2 3 0 6

352
Problema 3

ZZ
Calcular y 2x2 dxdy si D es la región acotada por |x| + |y| = 2
D

Figura 4.24: D región acotada por |x| + |y| = 2

Solución:
Se tiene que la región D = {(x, y) 2 IR2 / |x| + |y|  2}
Si escogemos la región con una partición de tipo I, es necesario utilizar
dos integrales iterativas porque para 2  x  0 , la frontera inferior de la
región es la gráfica de y = x 2, y la superior es y = x + 2;y para 0  x  2
la frontera inferior de la región es la gráfica de y = x 2, y la superior es
y = x+2
Entonces se tiene D = D1 [ D2 tal que D1 [ D2 = .
donde D1 = {(x, y) 2 IR2 / 2  x  0, x 2  y  x + 2}
D2 = {(x, y) 2 IR2 / 0 < x  2, x 2y x + 2}
Por otra parte la funcion del integrando f (x, y) = y 2x2 es simétrica con
respecto al eje y, es decir 8 (x, y, z) 2 D existe ( x, y, z) tal que f ( x, y) =
y 2( x)2 = f (x, y) .
Por lo tanto

353
ZZ Z 2 Z x+2
2
y 2x dxdy = 2 y 2x2 dydx
D 0 x 2
Z 2 ✓ 2
◆ x+2
y
= 2 + 2x2 y dx
0 2 x 2
Z 1
= 2 4x3 8x2 dx
0
✓ ◆2
4 8 3
= x x
3
✓ ◆0
64 32
= 2 16 =
3 3

Problema 4
ZZ
Calcular x2 + y 2 dxdy si D = {(x, y) 2 IR2 / x2 + y 2  1} .Usando
D
coordenadas cartesianas

Figura 4.25: D región acotada por x2 + y 2  1

Solución:
Usando coordenadas cartesianas, la región de integración es un cı́rculo
centrado en el origen de radio uno
Por lo tanto
p p
D = (x, y) 2 IR2 / 1  x  1, 1 x2  y  1 x2

354
ZZ Z Z p
1 1 x2
2 2
x +y dxdy = p
(x2 + y 2 )dydx
D 1 1 x2
p
Z 1 1 x2
y3
= (x2 y + ) p dx
1 3 1 x2
Z 1 p
1p
= 2 (x2 1 x2 + (1 x2 )3 )dx
1 3
Z 1 p Z 1p
2 2
= 2 x 1 x2 dx + (1 x2 )3 dx
1 3 1

Con ayuda de una tabla de integrales obtenemos que:

Z p 1
1
xp 1 p
x2 1 x2 dx = ( 1 x2 + (x 1 x2 + arcsenx)
1 4 8 1
1 1 ⇡ ⇡ ⇡
= (arcsen(1) arcsen ( 1) = ( + ) =
8 8 2 2 8

Z p 1
1
xp 3x p 3
(1 x2 )3 dx = ( (1 x2 ) 3 + (1 x2 ) + arcsenx)
1 4 8 8 1
3⇡
=
8
Por lo tanto:
ZZ
2⇡ 2 3⇡ ⇡
x2 + y 2 dxdy = + =
D 8 3 8 2
Notese que la solución del problema usando coordenadas cartesianas es
bastante compleja

Problema 5
ZZ
p
Calcular xydxdy si D es la región acotada por y = x, y =
p D
3x 18, y 0.Usando coordenadas cartesianas.
Solución.
Si escogemos la región con una partición de tipo I, es necesario utilizar
dos integrales iterativas porque para 0  x  6 , la p frontera inferior de la
región es la gráfica de y = 0, y la superior es y = x;y para 6  x  9 la

355
p p
Figura 4.26: D y = x, y = 3x 18, y 0. Región de tipo I

p
frontera
p inferior de la región es la gráfica de y = 3x 18, y la superior es
y= x
Luego tenemos que D = D1 [ D2 tal que D1 [ D2 = .
p
Entonces D1 = {(x, y) 2 IR2 / 0  x  6, 0  y  x}
p p
D2 = (x, y) 2 IR2 / 6 < x  9, 3x 18  y  x
Por lo tanto

ZZ ZZ ZZ
xydxdy = xydxdy + xydxdy
D D1 D2
Z 6Z
p
x Z 9 Z p
x
= xydydx + p
xydydx
0 0 6 3x 18
Z  p Z  2 px
6 x 9
y2 y
= x dx + x dx
0 2 0 6 2 p
3x 18
Z Z
1 6 2 1 9
= x dx + ( 2x2 + 18x)dx
2 0 2 6
 6  3 9
1 3 x x2 185
= x + +9 =
6 0 3 2 6 2

Si escogemos la región con una partición de tipo II, es necesario utilizar


sólo una integral iterativa porque para 0  y  3 , la frontera izquierda
de la región es la gráfica de x = y 2 mentras que la frontera derecha queda
y2
determinada por la gráfica x = + 6, obteniendo ası́ la región
⇢ 3
y2
D1 = (x, y) 2 IR2 / y 2  x  + 6, 0  y  3
3

356
p p
Figura 4.27: D y = x, y = 3x 18, y 0. Región de tipo II

la integral iterativa queda


ZZ Z 3 Z (y 2 /3)+6
xydxdy = xydxdy
D 0 y2
Z 3  (y 2 /3)+6
x2
= ydy
0 2 y2
Z "✓ 2 ◆2 #(y2 /3)+6
1 3 y + 18
= y4 ydy
2 0 3 2
y
Z 3
1 ⇥ ⇤
= 8y 5 + 36y 3 + 324y dy
18 0
 3
1 4 6
= y + 9y 4 + 162y 2
18 3 0

1 4 6 185
= 3 + 36 + 2 · 36 =
18 3 2

357
Problema 6

Encontrar el área de la región determinada por las desigualdades: xy 4,


y  x, 27y 4x2 .

Figura 4.28: Región determinada por xy 4, y  x, 27y 4x2

Solución.
Sabemos que xy = 4 tiene por gráfica una hipérbola equilátera, y = x
es la recta bisectriz del primer cuadrante y 27y = 4x2 corresponde a una
parábola. Veamos cuale son los puntos de intersección de estas curvas con el
proprosito de configurar el dominio de integración
xy = 4
=) x2 = 4 =) x = ±2 =) y = ±2
y=x
)
27y = 4x 2 x = 0 27
=) 27x = 4x2 =) 27 =) y = 0, y =
y=x x= 4
4
xy = 4 4
2 =) x = 3, y =
27y = 4x 3
ZZ
Para calcular el área A(R) = dxdy, podemos escoger una partición
D
del
dominio de tipo I ó de tipo II.
Consideremos
⇢ dos subregiones de tipo I
4
D1 = (x, y) 2 IR2 / 2  x  3,  y  x
⇢ x
27 4
D2 = (x, y) 2 IR2 / 3  x  , x2  y  x
4 27

358
Si proyectamos sobre eje x
ZZ ZZ ZZ
A(R) = dxdy = dxdy + dxdy
D D1 D2

Z 3 Z x Z 27/4 Z x
A(R) = dydx + dydx
4 4 2
2 x
3 27
x
Z 3 Z 27/4
x
= y| 4 dx + y|x4 x2 dx
x 27
2 3
Z 3 Z 27/4 
4 4 2
= x dx + x x dx
2 x 3 27
 2 3  2 27/4
x x 4 3
= 4 ln x + x
2 2 2 81 3
5 3 729 9 4 273 4
= 4 ln + 3
+ 33
2 2 32 2 81 4 81
3 729 243 4
= 2 4 ln + +
2 32 16 3
665 3
= 4 ln
96 2
Si proyectamos
⇢ sobre eje y
4 3p 4
DI = (x, y) 2 IR2 / x 3y,  y  2
⇢ y 2 3
3 p 27
DI = (x, y) 2 IR2 / y  x  3y, 2  y 
2 4
ZZ ZZ ZZ
A(R) = dxdy = dxdy + dxdy
D D1 D2

Z 2 Z 3p
3y Z 27/4 Z 3p
3y
2 2
A(R) = dxdy + dxdy
4 4
3 y
2 y
Z hp i Z 
2 27/4
3p
= 3y 4 ln y dy + 3y y dy
4
3
2 2
 p 2 p 27/4
3 4 y2
= 3y 3 + 3y 3
2 y 4 2 2
3
8 3 9 · 27 729
= 4 ln + +2
3 2 8 32
665 3
= 4 ln
96 2

359
Problema 7

Encontrar el volumen de la región acotada por los tres planos coordenados


y el plano x + 2y + 3z = 6 Solución.

Figura 4.29: Región determinada por los tres planos coordenados y el plano x +
2y + 3z = 6

Usando integrales dobles y proyectando la región sobre el plano xy ten-


emos: ⇢
RR 6 x 2y 6 x
V = D dxdy , D = (x, y) 2 IR2 / 0  x  6, 0  y 
3 2

Z Z 6 x
1 6 2
V = (6 x 2y) dydx
3 0 0
Z
1 6⇥ ⇤6 x
= (6 x)y y 2 0 2 dx
3 0
Z 
1 6 (6 x)2 (6 x)2
= dx
3 0 2 4
Z 6
1
= (6 x)2 dx
12 0
 6
1 3
= (6 x) =6
36 0

Usando integrales dobles y proyectando la región sobre el plano yz ten-


emos: ⇢
RR 6 2y
V = R (6 3z 2y) dzdy , R = (y, z) 2 IR2 / 0  y  3, 0  z 
3

360
Z 3 Z 6 2y
3
V = (6 2y 3z) dzdy
0 0
Z 3  6 2y
3 2 3
= (6 2y)z z dy
0 2 0
Z 3 
(6 2y)2 (6 2y)2
= dy
0 3 6
Z 3
1
= (6 2y)2 dy
6 0
 3
1 (6 x)3
= =6
12 3 0

4.4.2. Cambios de orden de Integración


Problema 1
Invierta el orden de integración y evalúe la integral resultante .
Z 1 Z 2
2
I= ey dydx
0 2x

Solución.
El dominio de integracion dado es D = {(x, y) 2 IR2 / 0  x  1, 2x  y  2} .
Si se invierte el orden de integración tenemos que modificar la partición
del n o
y
dominio. D = (x, y) 2 IR2 / 0  x  , 0  y  2 ,entonces la inte-
2
gral
se puede escribir.

Z 1 Z 2 Z 2 Z y
2
y2 2
I = e dydx = ey dxdy
Z0 2 2x
y
0 0

y2 2
= xe dy
0 0
Z 2
y y2 1 2 4
= e dy = ey
0 2 4 0
1 16
= e 1
4

361
Problema 2
Invierta el orden de integración y evalúe la integral resultante .
Z 2Z 4
p
I= y cos ydydx
0 x2

Solución.
El dominio de integración dado es D = {(x, y) 2 IR2 / 0  x  2, x2  y  4} .
Si se invierte el orden de integración tenemos que modificar la partición
del
p
dominio, D = (x, y) 2 IR2 / 0  x  y, 0  y  4 ,entonces la inte-
gral
se puede escribir

Z 2 Z 4 Z 4 Z p
y
p p
y cos ydydx = y cos ydxdy
0 x2 0 0
Z 4
p p
= y cos(y)x|0 y dy
Z0 4
= y cos(y)dy
0

Integrando esta última integral por partes se tiene:

Z 4 Z 4
y cos(y)dy = ysen(y)|40 sen(y)dy
0 0
= ysen(y)|40 + cos(y)|40
= 4sen(4) + cos(4) 1

Problema 3
Invierta el orden de integración y evalúe la integral resultante .
Z eZ ln x
I= ydydx
1 0

Solución.
El dominio de integración dado es D = {(x, y) 2 IR2 / 1  x  e, 0  y  ln x} .
Si se invierte el orden de integración tenemos que el dominio,
D = {(x, y) 2 IR2 / ey  x  e, 0  y  1} ,entonces la integral
se puede escribir

362
Z eZ ln x Z 1 Z e
ydydx = ydxdy
1 0 0 ey
Z 4 e
= y x dy
0 ey
Z 4
= y(e ey )dy
0
 4
y2
= e ey [y ey ]40
2 0
= 8e 4e4 1

4.4.3. Cambios de variables: Coordenadas polares


Problema 1
ZZ
Calcular x2 + y 2 dxdy si D = {(x, y) 2 IR2 / x2 + y 2  1} ,usando
D
coordenadas polares. Véase figura 4.25
Solución.
A partir de la coordenadas polares tenemos:
x = rcos✓, y = rsen✓ =) x2 + y 2 = r2
El valor absoluto del Jacobiano de transformación a polares es:
@ (x, y)
=r
@ (r, ✓)
Reemplazando términos en la integral, produce
ZZ ZZ
2 2 @ (x, y)
x + y dxdy = r2 drd✓
D D @ (r, ✓)

Z 1 Z 2⇡ Z 1 Z 2⇡
= 3
r d✓dr = r3 ✓|2⇡
0 dr
0 0 0 0
Z 1 4 1
r ⇡
= 2⇡ r3 dr = 2⇡ =
0 4 0 2

Las coordenadas polares dieron una solucion más simple del problema.
La simplicidad depende de la naturaleza del problema y de la simetria que
presenta el dominio.

363
Problema 2

Calcular el área de la región interior a la circunferencia x2 + y 2 = 8y y


exterior a la circunferencia x2 + y 2 = 9.
Solución.
Determinemos el centro y radio de la circunsferencia
x2 + y 2 = 8y =) x2 + y 2 8y = 0 =) x2 + (y 4)2 = 16
ZZ
El área de la región D es: A (D) dxdy Véase figura 4.30
D

Figura 4.30: Región interior a la circunferencia x2 + y 2 = 8y y exterior a la


circunferencia x2 + y 2 = 9

Por simetrı́a, podemos calcular el área de la región D en el primer cuad-


rante
y multiplicar por 2.
A fin de conocer los lı́mites de integración en coordenadas polares
necesitamos conocer el ángulo que forma la recta OT con el eje x.
x2 + y 2 = 8y =) r2 = 8rsen✓ =) r = 8sen✓
x2 + y 2 = 9 =) r = 3
Como T pertenece a ambas circunferencias se cumple
3
8sen✓ = 3 =) ✓ = arcsen
8 ⇢
⇤ 3 ⇡
Luego, la mitad de la región D = (r, ✓) /3  r  8sen✓; arcsen  ✓ 
8 2
ZZ ZZ
@ (x, y)
dxdy = drd✓
D D ⇤ @ (r, ✓)

364
Z ⇡/2 Z 8sen✓ Z ⇡/2 8sen✓
r2
2 rdrd✓ = 2 d✓
arcsen 38 3 arcsen 38 2 3
Z ⇡/2  ✓ ◆ ⇡/2
2 ✓ sen2✓ 9
64sen ✓ 9 d✓ = 64 ✓
arcsen 38 2 4 2 arcsen 38
 ⇡/2
55
= ✓ 16sen2✓
2 arcsen 38

55 55 3 3
= ⇡ arcsen + 16sen(2arcsen )
4 2 8 8
⇡ 38, 42

Problema 3
ZZ
x2 + y 2
Calcular p dxdy , si D es el interior del cardioide r =
D x + x2 + y 2
a (1 + cos ✓)

Figura 4.31: D es la región interior del cardioide r = a (1 + cos ✓)

Solución.
Cambiando a cordenadas polares, tenemos:
ZZ ZZ
x2 + y 2 r2 @ (x, y)
p dxdy = drd✓
D x + x2 + y 2 D⇤ r cos ✓ + r @ (r, ✓)
ZZ
r2
= rdrd✓
D⇤ r cos ✓ + r

365
Z 2⇡ Z a(1+cos ✓)
r2
= drd✓
0 0 1 + cos ✓
Z 2⇡ a(1+cos ✓)
1 r3
= d✓
0 1 + cos ✓ 3 0
Z 2⇡
3
a
= (1 + cos ✓)2 d✓
3 0
Z
a3 2⇡
= 1 + 2 cos ✓ + cos2 ✓ d✓
3 0
 2⇡
a3 ✓ sen2✓
= ✓ + 2sen✓ + +
3 2 4 0
3
= ⇡a

Observacion si deseamos se rigurosos debemos hacer notar que la integral


es
impropia cuando x  0, e y = 0, pues en tal caso el denominador es cero.
Luego:

Z ↵ Z a(1+cos ✓) Z 2⇡ Z a(1+cos ✓)
r2 r2
I = lı́m drd✓ + lı́m drd✓
↵!⇡ 0 " 1 + cos ✓ !⇡+ " 1 + cos ✓
"!0 "!0
3 Z ↵ 3 Z 2⇡
a a
= lı́m (1 + cos ✓)2 d✓ + lı́m (1 + cos ✓)2 d✓
↵!⇡ 3 0 !⇡+ 3
3
 
a 3 sen2↵ a3 3 sen2
= lı́m ↵ + 2sen↵ + + lı́m 3⇡ 2sen
↵!⇡ 3 2 4 !⇡+ 3 2 4
3
= ⇡a

Problema 4
Calcular el volumen V del sólido acotado por las gráficas z = 9 x2 y 2
y z = 5.
Solución.
Como el sólido es simétrico, basta encontrar su volumen en el primer
octante y multiplicar su resultado por cuatro.
Usando integrales dobles y proyectando la región sobre el plano xy ten-
emos: Z Z
⇥ ⇤
V =4 9 x2 y 2 5 dxdy
D

366
Figura 4.32: sólido acotado por las gráficas z = 9 x2 y2 y z = 5

D = {(x, y) 2 IR2 / x 0, y 0, 0  x2 + y 2  4}
A partir de la coordenadas polares, obtenemos:
x = rcos✓
=) f (x, y) = 4 x2 y 2 = 4 r2
y = rsen✓

0  x2 + y 2 = r2  4 () 0  r  2 y 0  ✓ 
n 2
⇤ ⇡o
D = (r, ✓) / 0  r  2, 0  ✓ 
2
El valor absoluto del Jacobiano de transformación a polares es:
@ (x, y)
=r
@ (r, ✓)
Reemplazando términos en la integral, produce:
Z Z
⇥ ⇤
V = 4 4 r2 rdrd✓
D⇤
Z ⇡/2 Z 2 ⇥ ⇤
= 4 4 r2 rdrd✓
0 0
Z ⇡/2  2
4 2 1 4
= 4 r r d✓
0 2 4 0
= 8⇡

4.4.4. Cambios de variables. Coordenadas curvilı́neas


Problema 1
ZZ
Calcular I = 3xydxdy, donde D es la región acotada por las rectas
D
x 2y = 0, x 2y = 4
(1)
x + y = 4, x+y =1
Solución.
Podemos usar el cambio de variables

367
Figura 4.33: D región acotada por x 2y = 0, x 2y = 4, x + y = 4,
x+y =1

1
u = x 2y x= (2u + v)
(1) =) 3 (2)
v =x+y 1
y = (u v)
3
Asi,x 2y = 4 se transforma en u = 4

x 2y = 0 se transforma en u = 0

x + y = 1 se transforma en v = 1

x + y = 4 se transforma en v = 4

@ (x, y)
Para calcular el Jacobiano tenemos dos posibilidades.
@ (u, v)

La primera, es usar la transformación inversa (2) x e y en términos de u


yv.

@ (u, v)
La segunda, mucho más simple, es calcular a partir de (1) y
@ (x, y)
luego
 1
@ (x, y) @ (u, v)
usar la propiedad = .
@ (u, v) @ (x, y)

@ (u, v) 1 2 @ (x, y) 1
En efecto = = 1 + 2 = 3 =) =
@ (x, y) 1 1 @ (u, v) 3

Por lo tanto, del teorema del cambio e variables se deduce que:

368
ZZ ZZ ✓ ◆
1 1 @ (x, y)
I = 3xydxdy = 3 (2u + v) (u v) dudv
D D⇤ 3 3 @ (u, v)
Z 4Z 0
1
= 2u2 uv v 2 dvdu
1 4 9
Z 4 0
1 uv 2 v 3
= 2u2 v du
9 1 2 3 4
Z 
1 4 64
= 8u2 + 8u du
9 1 3
 4
1 8u3 2 64 164
= + 4u u du =
9 3 3 1 9

Problema 2
Calcular el área de la región D, que esta acotada por las curvas
x2 y 2 = 1, x2 y 2 = 9
(1)
x + y = 4, x+y =6

Figura 4.34: D región acotada por x2 y 2 = 1, x2 y 2 = 9, x+y = 4, x+y = 6

Solución.
Teniendo en cuenta el cambio de variables que transforma la región D en
la región D⇤
u = x2 y 2
(1) =)
v =x+y
La imagen D⇤ de la región D está acotada por la rectas verticales;
x2 y 2 = 1 se transforma en u = 1
x2 y 2 = 9 se transforma en u = 9
y las rectas horizontales
x + y = 4 se transforma en v = 4

369
x + y = 6 se transforma en v = 6
Es decir, D⇤ = {(u, v) /1  u  9, 4  v  6}
@ (x, y) @ (u, v)
Vamos a calcular a partir de (1) y usar la propiedad
@ (u, v) @ (x, y)
 1
@ (x, y) @ (u, v)
= .
@ (u, v) @ (x, y)
@ (u, v) 2x 2y @ (x, y) 1
En efecto = = 2 (x + y) = 2v =) =
@ (x, y) 1 1 @ (u, v) 2v
El teorema del cambio variables afirma que:

ZZ ZZ
@ (x, y)
A (D) = dxdy = dudv
D D ⇤ @ (u, v)
Z 9Z 6
1
= dvdu
1 4 3v
Z
1 9
= [ln v]64 du
2 1
✓ ◆Z 9
1 6
= ln du
2 4 1
1 3 9 3
= ln [u]1 = 4 ln
2 2 2

Problema 3
ZZ
x3 + y 3
Calcular I = dxdy, donde D es la región del primer cuad-
D xy
rante acotada por:
y = x2 , y = 4x2
(1)
x = y 2 , x = 4y 2
Solución.
El cálculo de I serı́a bastante complejo si usamos coordenadas cartesianas
por la simetrı́a que tiene el dominio.Sin embargo, una cambio de variables
simplifica la región D y la transforma en D⇤ .
x2 y2
Sean u = , v =
y x
Luego D⇤ esta acotada por la rectas verticales;
y = x2 se transforma en u = 1.
1
y = 4x2 se transforma en u = .
4
y las rectas horizontales
x = y 2 se transforma en v = 1.

370
Figura 4.35: D región acotada por y = x2 , y = 4x2 , x = y 2 , x = 4y 2

1
x = 4y 2 se transforma en v = .
4


⇤ 1 1
Es decir, D = (u, v) /1  u  , 1  v 
4 4

@ (x, y)
Para calcular tenemos dos posibilidades, la primera es despejar
@ (u, v)
x

e y en términos de u y v a partir de (1) .

 1
@ (u, v) @ (x, y) @ (u, v)
La segunda, es calcular y usar la propiedad = .
@ (x, y) @ (u, v) @ (x, y)

2x x2
@ (u, v) y y2 @ (x, y) 1
En efecto = =4 1 = 3 =) =
@ (x, y) y2 2y @ (u, v) 3
x2 x

Calculemos ahora la integral

371
ZZ ZZ ✓ 2 ◆
x3 + y 3 x y2
I = dxdy = + dxdy
D xy D y x
Z 1 Z 1
1
= (u + v) dvdu
1/4 1/4 3
Z 1  1
1 v2
= uv + du
3 1/4 2 1/4
Z 
1 1 3 15
= u+ du
3 1/4 4 32
 1 
1 3 2 15 1 3 15 15 3
= u + u = +
3 8 32 1/4 3 8 16 32 4
15
=
64

Problema 4
ZZ
Evaluar la integral I = [x + y]2 dxdy, donde D es la región del plano
D
xy acotado por las curvas
x + y = 2, x + y = 4,
(1)
y = x, x2 y 2 = 4,

Figura 4.36: D región acotada por x + y = 2, x + y = 4, y = x y x2 y2 = 4

Solución. Observese que las ecuaciones de la curvas de la frontera de


D sólo incluyen a x e y en las combinaciones de x ± y,y el integrando incluye
solamentenlas mismas combinaciones. Aprovechando estas simetrı́as, sean las
coordenadas
u = x + y, v = x y
Luego, la imagen D⇤ de la región D está acotada por las curvas;

372
x + y = 2 se transforma en u = 2.
x + y = 4 se transforma en u = 4.
A su vez
x y = 0 se transforma en v = 0.
x2 y 2 = (x + y) ⇢ (x y) = 4 se transforma en uv = 4.
4
Es decir, D⇤ = (u, v) / 2  u  4, 0  v 
u
 1
@ (x, y) @ (u, v)
El jacobiano de la transformación es = .
@ (u, v) @ (x, y)
@ (u, v) 1 1 @ (x, y) 1
En efecto = = 2 =) =
@ (x, y) 1 1 @ (u, v) 2
Entonces:
ZZ ZZ
2 1
[x + y] dxdy = u2 dudv
D 2 D⇤
Z Z
1 4 4/u 2
= u dvdu
2 2 0
Z
1 4 2 4/u
= u v|0 du
2 2
Z
1 4
= 4udu
2 2
4
4 u2
= = 12
2 2 2

4.4.5. Cálculo de integrales triples en coordenadas rectángu-


lares cartesianas
Problema 1
1
Sea R la región en IR3 acotada por: z = 0, z = y, x = 0; x = 1, y =
2
0, y = 2 ZZZ
Calcular (x + y z) dxdydz.
R

Solución.
Del gráfico de la región (Véase figura 4.37) , tenemos que 0  z 
1
y.Proyectando la región R
2
sobre el plano xy. Ası́ D = {(x, y) 2 IR2 / 0  x  1, 0  y  2} .
Por lo tanto;

373
1
Figura 4.37: R región acotada por z = 0, z = y, x = 0; x = 1, y = 0, y = 2
2

ZZZ ZZ Z 1
y
2
(x + y z) dxdydz = ( (x + y z) dz)dxdy
R D 0

Z 1 Z 2 Z 1
y Z 1 Z 2  1
y
2 z2 2
( (x + y z) dz)dydx = xz + yz dydx
0 0 0 0 0 2 0
Z 1Z 2 Z 1Z 2
1 y2 1 3
(x + y)y dydx = xy + y 2 dydx
0 0 2 8 0 0 2 8
Z 1 2 Z 1  1
1 2 1 3 1
xy + y dx = [(x + 1)] dx = x2 + x
0 4 8 0 0 2 0
3
=
2

También es posible resolver el problema anterior proyectando la región R


sobre el plano xz.En tal caso, 2z  y  2 y
D = {(x, z) 2 IR2 / 0  x  1, 0  z  1}

ZZZ Z 1 Z 1 Z 2
(x + y z) dxdydz = ( (x + y z) dy)dzdx
R 0 0 2z

374
Z 1 Z 1  2 Z 1 Z 1
y2
xy + zy dzdx = 2 [x + 1 z xz] dzdx
0 0 2 2z 0 0
Z 1  1 Z 1 
z2 z 2
1 x
2 xz + z x dx = 2 x+1 dx
0 2 2 0 0 2 2
Z 1  1
1 2 3
[(x + 1)] dx = x +x =
0 2 0 2

Una tercera posibilidad de solución consiste en proyectar la región R


sobre el plano yz.
Esta se deja como ejercicio.

Problema 2
ZZZ
Calcular x2 dxdydz si D es la región acotada por y 2 + z 2 = 4ax,
D
y 2 = ax, x = 3a

Figura 4.38: D región acotada por y 2 + z 2 = 4ax, y 2 = ax, x = 3a

Solución.
La superficie y 2 + z 2 = 4ax corresponde a un paraboloide de revolución
como el bosquejado en la figura.
En dos variables el gráfico de y 2 = ax es una parábola, pero es tres
variables es la superficie de un manto parabólico.
Finalmente, el gráfico x = 3 es un plano paralelo al plano xz a la distancia
3a.
Luego el gráfico de la región es
La proyección
n de la region sobre el plano xy es: o
p p
3 2 2
D = (x, y, z) 2 IR /D1 [ D2 , 4ax y  z  4ax y

375
Por simetrı́a se tiene:

ZZZ ZZ Z p4ax y2
2
I = x dxdydz = 2 p x2 dzdxdy
D D1 4ax y2
Z p
3a Z ax Z
p
2 4ax y2
= 2 p p x2 dzdydx
0 ax 4ax y2
Z Z p
3a 2 ax ⇥ 2
⇤p4ax y2
= 2 p
xz p dydx
0 ax 4ax y 2
Z 3a Z p
2 ax p
= 4 p
x2 4ax y 2 dydx
0 ax

De una tabla de integrales obtenemos


Z p
1 p u
a2 u2 du = (u a2 u2 + a2 arcsen )
2 a
Ası́ al integrar la expresión:
Z 2pax p  ✓ p ◆ 2pax
1 y
p
4ax y 2 dy = y 4ax y 2 + 4ax arcsen p
ax 2 2 ax pax

1 p p 1
= 2ax arcsen (1) ax 3ax + 4ax arcsen
2 2
⇡ 1 p ⇡
= 2ax + ax 3 2ax
2 2p 6
2⇡ 3
= ax + ax
3 2
Por lo tanto al sustituir en la integral anterior, queda

Z " p # " p ! #3a


3a
2⇡ 3 2⇡ 3
4 + ax3 dx = + ax4
0 3 2 3 2
0
p !
3 3
= 27a5 2⇡ +
2

Problema 3

Calcular el volumen del sólido ⌦ acotado por la superficie y = x2 y los


planos y + z = 4 ; z = 0.

376
Figura 4.39: ⌦ acotado por la superficie y = x2 y los planos y + z = 4, z = 0

Solución.
Consideremos que la región ⌦ está acotada inferiormente por la frontera
z = 0 y superiomente por z = 4 y.
Si Proyectamos la región ⌦ sobre el plano xy, se tiene:
⌦ = {(x, y, z) 2 IR3 / (x, y) 2 D, 0  z  4 y}
D = {(x, y) 2 IR2 / 2  x  2, x2  y  4}
Luego el volumen de la región es

ZZZ Z 2 Z 4 Z 4 y
V (⌦) = dxdydz = dzdydx
⌦ 2 x2 0
Z 2 Z 4 Z 2  4
y2
= (4 y) dydx = 4y dx
2 x2 2 2 x2
Z 2 
x4
= 8 4x2 + dx
2 2
 2
4 3 x4 256
= 8x x + =
3 10 2 15

4.4.6. Coordenadas esféricas


Problema 1
ZZZ p
x2 + y 2 + z 2 e (x +y +z ) dxdydz si D es la región de
2 2 2
Resolver I =
D
IR3 limitada por las superficies x2 + y 2 + z 2 = a2
x2 + y 2 + z 2 = b2 con 0 < b < a anillo esférico.
Solución.
Por la simetrı́a del dominio y la forma del integrando

377
Figura 4.40: D es la región de IR3 limitada por las superficies x2 + y 2 + z 2 = a2
x2 + y 2 + z 2 = b2 con 0 < b < a

usaremos coordenadas esféricas:


9 b 2  x 2 + y 2 + z 2  a2 =) bra
x = rsen✓ cos = y
y = rsen✓sen =) tg✓ = = 0 =) 0✓⇡
; zy
z = r cos ✓ tg = = 0 =) 0  2⇡
x
Recordando que el valor absoluto del Jacobiano a esféricas es :
@ (x, y, z)
= r2 sen✓ se tiene:
@ (r, ✓, )

Z 2⇡ Z ⇡ Z a
r2 @ (x, y, z)
I = re drd✓d
0 0 b @ (r, ✓, )
Z 2⇡ Z ⇡ Z a
r2
= r3 e sen✓ drd✓d
0 0 b
Z 2⇡ Z ⇡  a
1 2 r2 2
= r e e r sen✓ d✓d
0 0 2 b
✓ ◆ Z 2⇡ Z ⇡
1 2 b2 1 b2 1 2 a2 a2
= be + e ae e sen✓ d✓d
2 2 2 0 0
✓ ◆ Z 2⇡
1 2 b2 1 b2 1 2 a2 a2
= be + e ae e cos ✓|⇡0 d
2 2 2
✓ ◆ 0Z 2⇡
1 2 b 2 1 b 2 1 2 a2 2
= 2 be + e ae e a d
2 2 2 0
✓ ◆
1 2 b2 1 b 2 1 2 a2 a2
= 4⇡ be + e ae e
2 2 2

378
Problema 2

Encontrar el volumen de la región determinada por x2 + y 2 + z 2  16 , z 2


x2 + y 2 .

Figura 4.41: Región determinada por x2 + y 2 + z 2  16 , z 2 x2 + y 2

Solución.
x2 + y 2 + z 2 = 16 es una esfera con centro en el origen y radio 4
z 2 = x2 +y 2 es un cono con vértice en el origen y eje de simetrı́a coincidente
con el eje z.
Como z 0 , sólo debemos considerar sólo la región sobre el plano xy.
La intersección de la esfera con el cono
p se obtiene mediante el sistema:
x2 + y 2 + z 2 = 16 z= 8
=)
x2 + y 2 = z 2 x2 + y 2 = 8

Usaremos coordenadas esféricas:


9 0  x2 + y 2 +pz 2  16 =) 0r4
x = rsen✓ cos = y 8 ⇡
y = rsen✓sen =) tg✓ = = p = 1 =) 0✓
; z 8 4
z = r cos ✓ y
tg = = 0 =) 0   2⇡
x
Recordando que el valor absoluto del Jacobiano a esféricas es :
@ (x, y, z)
= r2 sen✓ se tiene:
@ (r, ✓, )

379
ZZZ Z 2⇡ Z ⇡ Z 4
4
V = dxdydz = r2 sen✓ drd✓d
D 0 0 0
Z 2⇡ Z ⇡
3 4
4 r
V = sen✓ d✓d
0 0 3 0
3 Z 2⇡
4 ⇡
V = cos ✓|04 d
3 0
3 Z 2⇡
p ! p !
4 2 43 2
V = 1 d = 1 2⇡
3 0 2 3 2

Otra opción para resolver este problema es usar coordenadas cilı́ndricas,en


tal caso 9
x = r cos ✓ = x2 + y 2 + z 2 = 16 =) z = 16 r2 .
2 2 2
y = rsen✓ =) x +y =z =) z = r2
;
z=z
Tenı́amos que el Jacobiano de transformación a cilı́ndricas es:
@ (x, y, z)
= r luego:
@ (r, ✓, z)
ZZZ Z Z p Z p
2⇡ 8 16 r 2
V = dxdydz = rdzdrd✓
D 0 0 r2
Z Z p
2⇡ 8 p
r2
= rz|r216 drd✓
0 0
Z Z p
2⇡ 8 ⇣ p ⌘
2
= r 16 r2 r drd✓
0 0
Z ✓ ◆p 8
2⇡
1p r 3
= (16 r2 )3 d✓
0 3 3 0
2⇡ ⇣ p 3 p 3 ⌘ 2⇡ ⇣ p ⌘
= 2 8 16 = 64 32 2
3 3

4.4.7. Coordenadas Cilı́ndricas


Problema 1
Usando integrales triples calcular el volumen de la región acotada por
z = x2 + y 2 y z = 27 2x2 2y 2 .
Solución.
Por la simetrı́a del volumen los resolveremos usando coordenadas cilı́ndri-
cas.

380
Figura 4.42: Región acotada por z = x2 + y 2 y z = 27 2x2 2y 2

9
x = r cos ✓ = z = x2 + y 2 =) z = r2 .
y = rsen✓ =) z = 27 2x2 2y 2 =) z = 27 2r2
;
z=z x2 + y 2 = 9 =) r = 3.
Como el Jacobiano de transformación a cilı́ndricas es:
@ (x, y, z)
= r se tiene:
@ (r, ✓, z)
ZZZ Z 2⇡ Z 3 Z 27 2r2
V = dxdydz = rdzdrd✓
D 0 0 r2
Z 2⇡ Z 3
2r 2
= r z|27
r2 drd✓
0 0
Z 2⇡ Z 3
= r 27 3r2 drd✓
0 0
Z 2⇡  3
3 4 27 2
= r d✓ r
0 4 0 2
Z
243 2⇡ 243 243
= d✓ = 2⇡ = ⇡
4 0 4 2

Problema 2
Calcular el volumen de la región acotada por la esfera x2 + y 2 + z 2 = 13
y el cono (z 1)2 = x2 + y 2 , z 1
Solución.
El volumen pedido es
ZZZ
V = dxdydz
R

donde la región R está dada por

381
Figura 4.43: Región acotada por la esfera x2 + y 2 + z 2 = 13 y el cono (z 1)2 =
x2 + y 2 , z 1

n p p o
3
R = (x, y, z) 2 IR / (x, y) 2 D; 1 + x2 + y 2  z  4 x2 y2

D corresponde a la proyección de R sobre el plano xy.


D = {(x, y, z) 2 IR2 /x2 + y 2  13}
Por la simetrı́a del volumen conviene usar coordenadas cilı́ndricas.
9
x = r cos ✓ =
y = rsen✓ =) x2 + y 2 + z 2  r2 + z 2  13 ,
;
z=z
Determinemos la imagen R⇤ de R
p
(z 1)2 = x2 + y 2 () z 1 + r =) 1 + r  z  13 r2
Luego
p
R⇤ = (r, ✓, z) 2 IR3 / (r, ✓) 2 D; 1 + r  z  13 r2
La región R al ser proyectada sobre el plano xy. produce
z = 0 =) x2 + y 2 = 13
n ⇡ ⇡o
D1⇤ = (r, ✓) 2 IR3 /  r  2 ; ✓
2 2
Como el Jacobiano de transformación a cilı́ndricas es:
@ (x, y, z)
= r se tiene:
@ (r, ✓, z)

382
ZZZ Z Z Z p
2 2⇡ 13 r 2
V = dxdydz = rdzd✓dr
R 0 0 1+r
Z 2 Z 2⇡ p
13 r2
= rz1+r d✓dr
0 0
Z 2 Z 2⇡ ⇣p ⌘
= r 13 r2 (1 + r) d✓dr
0 0
Z 2 ⇣ p ⌘
= 2⇡ r 13 r2 r + r2 dr
0
 ✓ 2 ◆ 2
1 2 3/2 r r3
= 2⇡ 13 r +
3 2 3 0
 ✓ ◆
1 4 8
= 2⇡ 133/2 73/2 +
3 2 3

Problema 3

Calcular utilizando coordenadas cilı́ndricas el volumen de la región R ,


donde R es el interior a la esfera x2 + y 2 + z 2 = 4, z 0,y exterior al cilindro
(x 1)2 + y 2 = 1.

Figura 4.44: R, región interior a la esfera x2 + y 2 + z 2 = 4, z 0,y exterior al


cilindro (x 1)2 + y2 =1

Solución.
La región
n R se describe en coordenadas cartesianas mediante
o
p
R = (x, y, z) 2 IR3 / (x, y) 2 D; 0  z  4 x2 y 2
donde D es la proyección de R sobre el plano xy.

383
D = {(x, y) 2 IR3 /x2 + y 2  4 ; (x 1)2 + y 2 1}

Transformemos la región R a coordenadas cilindricas definidas por


9
x = r cos ✓ =
y = rsen✓ =) x2 + y 2 + z 2 = r2 (cos2 ✓ + sen2 ✓) + z 2  4
;
z=z

p
() 0  z  4 r2

La región R al ser proyectada sobre el plano xy da origen a dos subregiones

⇡ 3⇡
x2 + y 2  r2  4 () 0  r  2 si ✓
2 2
⇡ ⇡
(x 1)2 + y 2 1 () r 2 cos ✓ y r  2 si - ✓
2 2

Entonces, la región R⇤ puede describirse mediante


p
R⇤ = (r, ✓, z) / (r, ✓) 2 D⇤ = D1⇤ [ D1⇤ ; 0  z  4 r2
n ⇡ ⇡o
D1⇤ = (r, ✓) 2 IR3 /2 cos ✓  r  2 ; ✓
2 2

⇡ 3⇡
D2⇤ = (r, ✓) 2 IR3 /0  r  2 ; ✓
2 2

Ademas, el Jacobiano de la transformación a cilı́ndricas es:

@ (x, y, z)
=r
@ (r, ✓, z)

384
En consecuencia la integral puede describirse por
ZZZ
I = (r) drd✓dz
R
Z Z Z p Z Z Z p
⇡/2 2 4 r2 3⇡/2 2 4 r2
= rdzdrd✓ + rdzdrd✓
⇡/2 2 cos ✓ 0 ⇡/2 0 0
Z ⇡/2 Z 2 p Z 3⇡/2 Z 2 p
4 r2 r2
= r [z]0 drd✓ + r [z]0 4 drd✓
⇡/2 2 cos ✓ ⇡/2 0
Z ⇡/2 Z 2 p Z 3⇡/2 Z 2 p
= r 4 r2 drd✓ + r 4 r2 drd✓
⇡/2 2 cos ✓ ⇡/2 0
Z ⇡/2  2 Z 3⇡/2  2
1 2 3/2 1 2 3/2
= 4 r d✓ + 4 r d✓
⇡/2 3 2 cos ✓ ⇡/2 3 0
Z ⇡/2 Z 3⇡/2
8 2 3/2 8
= 1 cos ✓ d✓ + d✓
3 ⇡/2 3 ⇡/2
Z ⇡/2 Z 3⇡/2
8 3 8
= sen ✓d✓ + d✓
3 ⇡/2 3 ⇡/2
 ⇡/2
8 cos3 ✓ 8 8
= cos ✓ + + ⇡= ⇡
3 3 ⇡/2 3 3

Problema 4
ZZZ ✓ ◆
x2 y 2 z 2
Calcular I = 2
+ 2 + 2 dxdydz.
⇢ a
D b c
x2 y 2 z 2
En la región D = (x, y, z) 2 IR3 / 2 + 2 + 2  1 a > 0, b > 0, c >
a b c
0

x2 y 2 z 2
Figura 4.45: D es la región interior al elipsoide + 2 + 2 =1
a2 b c

385
Solución.
La región de integración es un elipsoide de semieejes a,b,c.
Efectuemos un primer cambio de variables:
x = au, y = bv, z = cw.
Con ello, D se transforma en la bola.
D⇤ = {(u, v, w) /u2 + v 2 + w2  1} y el valor absoluto del Jacobiano que-
da :
a 0 0
@ (x, y, z)
= 0 b 0 = abc
@ (u, v, w)
0 0 c
Luego, aplicando el teorema del cambio de variables y obtenemos la inte-
gral

ZZZ ✓ ◆
x2 y 2 z 2
I = + 2 + 2 dxdydz.
D a2 b c
ZZZ
@ (x, y, z)
= u2 + v 2 + w 2 dudvdw
⇤ @ (u, v, w)
Z Z ZD
@ (x, y, z)
= u2 + v 2 + w 2 dudvdw
D⇤ @ (u, v, w)
ZZZ
= (u2 + v 2 + w2 ) (abc) dudvdw
D⇤

Ahora, transformamos a coordenadas esféricas.


9 0  u2 + v 2 + w2  1 =) 0  r  1
u = rsen✓ cos = v
v = rsen✓sen =) tg✓ = =) 0  ✓  ⇡
; wv
w = r cos ✓ tg = =) 0   2⇡
⇤⇤
u
Quedando, la region D = {(r, ✓, ) /0  r  1, 0  ✓  ⇡, 0   2⇡}

ZZZ Z 2⇡ Z ⇡ Z 1
2 2 2
abc (u + v + w )dudvdw = abc r2 r2 sen✓ drd✓d
D⇤ 0 0 0
Z 2⇡ Z ⇡ 1
r5
= abc sen✓ d✓d
0 0 5 0
Z
abc 2⇡
= cos ✓|⇡0 d
5 0
Z
2abc 2⇡ 4⇡abc
= d =
5 0 5

Observación

386
Es claro que la integración se podrı́a haber efectuado usando directamente
la trasformación compuesta.
9
x = arsen✓ cos =
@ (x, y, z)
y = brsen✓sen =) = abcr2 sen✓
; @ (r, ✓, )
z = cr cos ✓

Problema 5
ZZZ
dxdydz.
Calcular I = q ,
D 2 2 2
(x a) + (y b) + (z c)
en la región D = {(x, y, z) 2 IR3 /x2 + y 2 + z 2  R2 } , (a, b, c) es un
punto fijo
no peteneciente a la esfera x2 + y 2 + z 2  R2 .

Figura 4.46: D es la región interior a la esfera x2 + y 2 + z 2 = R2

Solución.

Si usamos coordenadas cartesianas los lı́mites de integración son dificul-


tosos, pues en tal caso tendrı́amos.

ZZZ
dxdydz.
I = q
D (x a)2 + (y b)2 + (z c)2
Z r Z p
r 2 x2 Z p r 2 x2 y2
dzdydx.
I = p p q
r r 2 x2 r 2 x2 y 2 (x a)2 + (y b)2 + (z c)2

Es claro que si usamos este camino las cosas no serán fáciles.


Sin embargo , dada la simetria esférica del dominio y observando que el
integrando no es nada más que el reciproco de la distancia desde (a, b, c) 2
/D
hasta (x, y, z) 2 D,nos damos cuenta que el resultado no puede depender
más que de la distancia d entre dichos puntos.Por ello, el resultado no puede

387
variar si ubicamos el eje z pasando por el punto (a, b, c). Si (0, 0, d) son las
nuevas coordenadas del punto fijo tenemos.
ZZZ
dxdydz.
I= q
D x2 + y 2 + (z d)2

Observación
El razonamiento anterior es muy usado el cálculo de integrales que apare-
cen aplicaciones a la Fı́sica pues en dicha Ciencia son comunes las leyes en
que aparece una distacia o el cuadrado de una distancia en el denominador
del integrando.
Para calcular I en (*) usamos coordenadas esféricas. Obtenemos:

Z R Z ⇡ Z 2⇡
r2 sen✓ d d✓dr
I = p
0 0 0 r2 + d2 2dr cos ✓
Z R Z ⇡
r2 sen✓ d✓dr
= 2⇡ p
0 0 r2 + d2 2dr cos ✓

Para calcular
Z ⇡
r2 sen✓ d✓dr
J= p
0 r2 + d2 2dr cos ✓
podemos hacer
s = r2 + d2 2dr cos ✓
ds = 2drsen✓d✓
Además, ✓ = 0 =) s = r2 + d2 2dr = (d r)2
✓ = ⇡ =) s = r2 + d2 + 2dr = (d + r)2
Reemplazando en la integral anterior produce

Z (d+r)2
r r (d+r)2
J = s 1/2 ds = 2s1/2 (d r)2
2d (d r)2 2d
r
= [2 (d + r) 2 (d r)]
2d
r 2r2
= [4r] =
2d d

Por lo tanto

388
Z R
2r2
I = 2⇡ dr
0 d
3 R
4⇡ r
I =
d 3 0
4⇡ 3
I = R
3d

4.5. Ejercicios propuestos integrales dobles y


triples
4.5.1. Integrales dobles
Integrales dobles en coordenadas cartesianas
1.- Resolver las integrales:
Z 1Z 1
a) p
2x3 dxdy
0 y
Z 4 Z y+2
9
b) p
dxdy
5 2 4 y
Z Z p
2 4 x2
c) p
xydydx
2 4 x2

Respuestas
Z 1Z 1
3 1
a) 2x dxdy =
0
p
y 3
Z 4 Z y+2
3 9
b) p
dxdy =
5 2 4 y 2
Z 2 Z p4 x2
c) p
xydydx = 0
2 4 x2
ZZ
2.- Calcular la integral xydxdy, donde D es la región acotada
D
por y 2 = x, y 2 = 3x 18, y 0
ZZ
135
Respuesta: xydxdy = Véase Figura 4.47
D 2

389
Figura 4.47: Región acotada por y 2 = x, y 2 = 3x 18, y 0

ZZ ⇢
x x2
3.- Calcular la integral dxdy, donde D = (x, y) 2 IR2 /1  x  2,  y  x2
D x + y2
2 2

Figura 4.48: Región acotada por y 2 = x, y 2 = 3x 18, y 0

ZZ
x 3 1 7
Respuesta: I = dxdy = arctan 2 ln 5 ⇡ + ln 2
D x2 + y2 2 4 2
Véase Figura 4.48

Z 8 Z 2
y
4.- Evalúe la integral resultante: p dxdy
p
3 16 + x7
0 y

Z 2 Z x3
y 8
Respuesta p dydx = Véase Figura 4.49
0 0 16 + x 7 7

390
Figura 4.49: Región acotada por y 2 = x, y 2 = 3x 18, y 0

5.- Calcule el volumen del sólido acotado por las gráficas de:
x2 + y 2 = 9, y 2 + z 2 = 2.

Figura 4.50: Sólido acotado por las gráficas de x2 + y 2 = 9, y 2 + z 2 = 2.

Z 3 Z p9 y2 p
Respuesta V (R) = 8 9 y 2 dxdy
0 0

6.- Calcule el volumen del sólido acotado por las gráficas de las
ecuaciones dadas por z = x2 + 4, y = 4 x2 , x + y = 2; z = 0.
423
Respuesta V (R) =
20

391
Figura 4.51: Sólido acotado por las gráficas de las ecuaciones dadas por z = x2 +4,
y=4 x2 , x + y = 2 z = 0.

4.5.2. Cálculo de Integrales dobles usando transforma-


ción de coordenadas
ZZ
y
1.- Calcular dxdy,donde R es a región plana determinada
R x2
+ y2
por los puntos del primer cuadrante que son interiores a la circun-
fererencia x2 4x + y 2 = 0 y exteriores a la circunferencia x2 + y 2 = 4.

Figura 4.52: Región plana determinada por los puntos del primer cuadrante que
son interiores a la circunfererencia x2 4x + y 2 = 0 y exteriores a la circunferencia
x2 + y 2 = 4

ZZ
y 1
Respuesta: I = dxdy =
R x2 +y 2 2

392
ZZ
⇥ ⇤ 1/2
2.- Calcular la integral (x y)2 + 2 (x + 2) + 1 dxdy, donde R
R
es la región plana acotada por y = 0, x = 2, y = x.
a) Utilice la transformación u = x y, v = x + y.

Figura 4.53: R es la región plana acotada por y = 0, x = 2, y = x y las transfor-


maciones u = x y, v = x + y. y x = u (1 + v) , y = v (1 + u)

b) Utilice la sustitución x = u (1 + v) , y = v (1 + u) .
ZZ
⇥ ⇤ 1/2 1
Respuesta. I = (x y)2 + 2 (x + 2) + 1 dxdy = 2 ln 2
D 2

3.- Resolver las integrales usando las coordenadas más convenientes

Z Z p
2a 2ax x2
a) x2 + y 2 dydx
0 0
Z 2 Z p4 y2 p
b) x2 + y 2 dxdy
2 0
Z p Z p
⇡ ⇡ y2
c) p p sen x2 + y 2 dxdy
⇡ ⇡ y2
Z Z p
2 2+ 4 x2 p
d) p
x2 + y 2 dxdy
4 x2
Z Z2 2
y
e) e x dxdy,
R

si R esta acotado por x = a, x = b, y = px, y = qx, con a < b, p < q.

393
Respuestas.
Z 2a Z p2ax x2
3
a) x2 + y 2 dydx = ⇡a2
0 0 4
Z Z p 2 4 y2 p 4
b) x2 + y 2 dxdy = ⇡
2 0 3
Z p
⇡ Z p⇡ y2
c) p p sen x2 + y 2 dxdy = 2⇡
⇡ ⇡ y2
Z Z p
2 p 2+ 4 x2
64
d) x 2 + y 2 dxdy = (3⇡ 4)
p
2 9
Z Z2 2 4 x
y 1 2
e) e x dxdy = b a2 (eq ep )
R 2

4.5.3. Integrales triples


4.5.4. Integrales triples iteradas
1.- Resolver las integrales triples por iteraciones:

p
Z 2 Z p
2 x Z 4x y 2
2
a) xdzdydx
0 0 0
Z a Z p
a 2 x2 Z pa2 x2 y 2 p
b) a2 x2 y2 z 2 dzdydx
0 0 0
Z 1 Z p
1 x2 Z p 12 x2 y 2
c) p
x2 + y 2 dzdydx
1 1 x2 0

Respuestas
p
Z Z p Z 4x y 2
2 2 x 2 4 p
a) xdzdydx = ⇡ 2
0 0 0 3
Z aZ
p
a2 x2 Z p a2 x2 y 2 p ⇡2 2
b) a2 x2 y2 z 2 dzdydx = a
0 0 0 8
Z 1 Z p
1 x2 Z p12 x2 y 2
4
c) p
x2 + y 2 dxdy = ⇡
1 1 x2 0 15

394
2.- Calcular las áreas de las regiones D⇢ R2 acotadas por:

a) xy  4, y  x, 27y 4x2 .

Figura 4.54: D es la región plana acotada por xy  4, y  x, 27y 4x2

b) x2 + y 2  9, y  x + 3, y  x.

Figura 4.55: D es la región plana acotada por x2 + y 2  9, y  x + 3, y  x

Respuestas

2 3
a) A (D) = + 4 ln .
3 2
9
b) A (D) = (3⇡ + 2) .
8
395
4.5.5. Integrales triples en coordenadas rectángulares
cartesianas.
ZZZ
1.- Calcular la integral triple xyzdxdydz, siendo el dominio de
D
integración D el tetaedro delimitado por los planos coordenados y
y z
el plano de ecuación x + + = 1.
2 3
ZZ
1
Respuesta. I = xyzdxdydz = .
D 20
ZZZ
2.- Calcular la integral triple x2 dxdydz, siendo el dominio de
D
integración D acotado por y 2 + z 2 = 4ay, y 2 = ax, x = 3a,con a > 0
ZZZ p !
2 2 3 3
Respuesta. I = x dxdydz = 27a 2⇡
D 2

ZZZ
3.- Calcular la integral triple xdxdydz, si D es la región de IR3
D
acotada por las superficies 2x + 3y = 6, 2x + 3z = 6, y = 3 + 2x x2 ,
z=0

Figura 4.56: D es la región de IR3 acotada por las superficies 2x+3y = 6, 2x+3z =
6, y = 3 + 2x x2 , z = 0

 3
1 6 5 34 4 42 3
Respuesta. I = x x + x + 9x2
9 5 4 3 1/3

396
4.5.6. Calcular las integrales dadas usando las coorde-
nadas adecuadas:
ZZZ
a) (x + y + z) 3 dxdydz,si D esta acotado por x = 0, y = 0, z = 0,
D
x + y + z = 0.

Figura 4.57: D acotado por x = 0, y = 0, z = 0, x + y + z = 0


ZZZ
b) (x+y+z)2 dxdydz,si D esta acotado por 2 z = x2 +y 2 , x2 +y 2 +z 2 = 3
D

Figura 4.58: D acotado por 2 z = x2 + y 2 , x2 + y 2 + z 2 = 3

397
ZZZ
c) z 2 dxdydz,si D esta limitado por x2 + y 2 + z 2 = a2 , x2 + y 2 + z 2 = 2az
D

Figura 4.59: D esta limitado por x2 + y 2 + z 2 = a2 , x2 + y 2 + z 2 = 2az


ZZZ
x2 y 2 z 2
d) zdxdydz,si D esta acotado por z = 0, + 2 + 2 =1
D a2 b c

x2 y 2 z 2
Figura 4.60: D esta acotado por z = 0, + 2 + 2 =1
a2 b c

398
ZZZ
e) zdxdydz,si D esta limitado por z = h, z 2 = h2 (x2 + y 2 )
D

Figura 4.61: D esta limitado por z = h, z 2 = h2 x2 + y 2

Respuestas
ZZZ
1 5
a) (x + y + z) 3 dxdydz = ln 2
Z Z ZD 2✓ 16 ◆
⇡ p 97
b) (x + y + z)2 dxdydz = 18 3
ZZZD 5 6
59
c) z 2 dxdydz = ⇡a5
Z Z ZD 480
⇡abc2
d) zdxdydz =
ZZZD 4
⇡h2
e) zdxdydz =
D 4

4.5.7. Resolver las integrales usando coordenadas esféri-


cas:
Z a Z p
a 2 x2 Z p a2 x2 y 2
a) p
x2 + y 2 dzdydx.
Z aZ Z a2 x2 0

b) x + y 2 dxdydz, si D es la región limitada por


2
D

r  x + y 2 + z 2  R2 , z > 0.
2 2

Respuestas:

399
Z a Z p
a 2 x2 Z p a2 x2 y 2
4 5
a) p
x2 + y 2 dzdydx = ⇡a .
Z aZ Z a 2 x2 0 15
4
b) x + y 2 dxdydz =
2
R5 r5 .
D 15

Calcular los volumenes de las regiones D⇢ R3 acotado por:


a) z = x, z = x2 + y 2

Figura 4.62: Región limitada por z = x, z = x2 + y 2

b) x2 + y 2 = 4, x2 + y 2 z2 = 1

Figura 4.63: Región limitada por x2 + y 2 = 4, x2 + y 2 z2 = 1

c) z 2 = x2 + y 2 , y = 0, y = x, x = a.

400
Figura 4.64: Región limitada por z 2 = x2 + y 2 , y = 0, y = x, x = a

d) z = x + y, z = 0, x2 + y 2 = 1, x = 1, y = 1.
Respuestas.

Figura 4.65: Región limitada por z = x + y, z = 0, x2 + y 2 = 1, x = 1, y = 1


a) V (D) =
32 p
b) V (D) = 4⇡ 3
2 p p
c) V (D) = a 3 2 + ln 1 + 2
1
d) V (D) =
3

4.6. Aplicaciones integrales dobles y triples


4.6.1. Volumenes de cuerpos en el espacio
Si la función z = f (x, y) es continua tal que f (x, y) 0, entonces la
integral doble: Z Z
V (⌦) = f (x, y) dA
R

401
define el volumen ⌦ bajo la superficie de la gráfica de la función f (x, y) sobre
la región R ⇢ IR2 Problema 1
Calcular el volumen acotado por el paraboloide z = x2 + y 2 entre los
planos z = 1 y z = 4.

Figura 4.66: Volumen acotado por el paraboloide z = x2 + y 2 y los planos z = 1


yz=4

Solución. Es claro que el volumen a calcular esta dado por


Z Z
V (⌦) = (x2 + y 2 )dA
R
donde R = {(x, y) 2 IR /1  x + y 2  1}
2 2

Cambiando a coordenadas polares, queda


x = r cos ✓
=) 1  x2 + y 2  4 =) 1  r  2, 0  ✓  2⇡
y = rsen✓
Luego , el dominio de integración se transforma en:
R⇤ = {(r, ✓) /1  r  2, 0  ✓  2⇡}

@(x, y)
Además, el jacobiano de transformacion es =r
@(r, ✓)
Reemplazando términos en la última integral se tiene

Z 2⇡ Z 2
V = r2 (r) drd✓
0 1
Z 2⇡  2 Z 2⇡ 
r4 24 1
= d✓ = d✓
0 4 1 0 4 4
15 15
= (2⇡) = ⇡
4 2
402
Problema 2
Calcular el volumen acotado por z  36 3x2 y2 , y z x2 + 3y 2 .

Figura 4.67: Volumen acotado por el paraboloide z  36 3x2 y 2 , y z x2 +3y 2

Solución. Ambas ecuaciones z = 36 3x2 y 2 , y z = x2 + 3y 2


representan paraboloides de revolución.
La región de integración está definida por

R = (x, y, z) 2 IR3 / x2 + 3y 2  z  36 3x2 y2

.
Determinemos la proyección de la intersección de ambas superficies sobre el
plano xy.
z = 36 3x2 y 2
=) 36 3x2 y 2 = x2 +3y 2 =) 32 = 4x2 +4y 2
z= x2 + 3y 2
Por tanto, x2 + y 2 = 9, corresponde a una circunferencia centrada en el
origen de radio 3.
En consecuencia, el volumen de la región es

Z Z
⇥ ⇤
V = 36 3x2 y2 (x2 + 3y 2 ) dxdy
Z ZD
⇥ ⇤
= 36 4x2 4y 2 dxdy
D

donde D = {(x, y) 2 IR2 / 0  x2 + y 2  9}


Como la función del integrando es simétrica con respecto al plano xy,
podemos calcular el volumen en el primer octante y multiplicar por cuatro.

403
Z Z
⇥ ⇤
V =4 36 4x2 4y 2 dxdy
D+
p
donde D⇤ = (x, y) 2 IR2 / 0  x  3, 0  y  9 x2 .

Z Z p
3 9 x2 ⇥ ⇤
V = 4 36 4x2 4y 2 dydx
0 0
Z Z p
3 9 x2 ⇥ ⇤
= 4 4 9 x2 y 2 dydx
0 0

Para simplificar los calculos usemos coordenadas polares


x = r cos ✓ ⇡
=) 0  x2 + y 2  9 =) 0  r  3, 0  ✓ 
y = rsen✓ 2
Luego , el n
dominio de integración se transforma en:
⇡o
D⇤⇤ = (r, ✓) /0  r  3, 0  ✓ 
2
@(x, y)
Además, el jacobiano de transformación es =r
@(r, ✓)
Reemplazando términos en la última integral se tiene

Z ⇡ Z 3
2 ⇥ ⇤
V = 16 9 r2 (r) drd✓
0 0
Z ⇡  3 Z ⇡ 
2 9 2 r4 2 34 34
= 16 r d✓ = 16 d✓
2 4 0 2 4
0 4 0
3 ⇡
= 16 = 162⇡
4 2

4.6.2. Área de figuras planas.


Sea D una región acotada en el plano xy. El área de D se define por:
ZZ
A (D) = dxdy
D

Si se tiene una región D de tipo I,

D = (x, y) 2 IR2 / a  x  b, 1 (x)  y  2 (x)

404
obtendrı́amos que
ZZ Z bZ 2 (x)

A (D) = dxdy = dydx


D a 1 (x)
Z b
= ( 2 (x) 1 (x))dx
a

Problema 1

Calcular el área D de la región situada en el primer cuadrante, acotada


por:
y = x2 , y = 4x2
(1)
x = y 2 , x = 4y 2

Figura 4.68: área D de la región situada en el primer cuadrante, acotada por


y = x2 , y = 4x2 , x = y 2 y x = 4y 2

Solución. El cálculo del área serı́a bastante engorroso si usamos una


región de tipo I en coordenadas cartesianas por la simetrı́a que tiene la región.
Sin embargo, un cambio de variables simplifica la región D y la transforma
en D⇤ .
x2 y2
Sean u = , v =
y x
Entonces las curvas que acotan la región se tranforman en
y = x2 =) u = 1.
1
y = 4x2 =) u = .
4
Luego D⇤ está acotada por la rectas verticales; y las rectas horizontales
x = y 2 =) v = 1.

405
1
x = 4y 2 =) transforma en v = .
4

1 1
Es decir, D⇤ = (u, v) /1  u  , 1  v 
4 4
@ (x, y)
Para calcular tenemos dos posibilidades, la primera es despejar
@ (u, v)
@ (u, v)
x e y en términos de u y v a partir de (1) . La segunda, es calcular
@ (x, y)
 1
@ (x, y) @ (u, v)
y usar la propiedad = .
@ (u, v) @ (x, y)
2x x2
@ (u, v) y y2 @ (x, y) 1
En efecto = =4 1 = 3 =) =
@ (x, y) y2 2y @ (u, v) 3
x2 x
Calculemos ahora el área pedida:

ZZ Z Z
1
1
A (D) = dxdy = dvdu
D D 3
Z Z Z
1 1 1 1 1
= dvdu = [v]1 du
3 1/4 1/4 3 1/4 1/4
1 1 3
= [u]1/4 =
4 16

4.6.3. Momentos y centros de masa para placas planas


delgadas

Definición: Sea L una lámina con la forma de una región R del plano xy.
Si la densidad en (x, y) es (x, y) , y es continua en R, entonces la masa M y
los primeros momentos de la lámina con respecto al eje x e y respectivamente
son Mx , My y los centros de masa (x, y) se definen:

406
Z Z
M = (x, y) dA
Z ZR
Mx = y (x, y) dA
Z ZR
My = x (x, y) dydx
R RR
My x (x, y) dA
x = = R RR
M (x, y) dA
R RR
Mx y (x, y) dA
y = = R RR
M R
(x, y) dA
Si una partı́cula de masa m se encuentra en el punto (x, y, z) , entonces su
distancia al eje z es
Sea L una lámina con la forma de una región R del plano xy. Si la densidad
en (x, y) es (x, y) , y es continua en R, entonces los segundos momentos
de la lámina o momentos de inercia con respecto al eje x e y respectivamente
son Ix , Iy y los momentos de inercia con respecto a una recta L y respecto al
origen se definen:

Z Z
Ix = y 2 (x, y) dA
Z ZR
Iy = x2 (x, y) dydx
Z ZR
IL = r2 (x, y) (x, y) dA
Z ZR
I0 = (x2 + y 2 ) (x, y) dA = Ix + Iy
R

4.6.4. Centroide de figuras geométricas


Cuando (x, y) es igual a uno, la posición del centro de masa x, y se
convierte en una caraterı́stica de la forma del objeto y no del material que
está hecho y se define como el centroide de una placa.
Problema 1
Dada una placa delgada que cubre la región triangular acotada por el eje
x y las rectas x = 1, y = 2x en el primer cuadrante si la densidad de la placa
en en cada punto de la región esta dada por (x, y) = 6x + 6y + 6.Calcular:

407
a) la masa de la placa.
b) los primeros momentos
c) el centro de masa.
d) momentos de inercia(segundos momentos) con respecto a los ejes x e
y
e) radios de giro con respecto a los ejes x e y respectivamente
f) radio de giro con respecto al origen

Figura 4.69: Región triangular acotada por el eje x y las rectas x = 1, y = 2x en


el primer cuadrante

Solución.
Consideremos una partición de la región que delimita la placa como una
región de tipo I: R = {(x, y) / 0  x  1, 0  y  2x} .
a) La masa de la placa es

Z 1 Z 2x
M = (x, y) dydx
0 0
Z 1 Z 2x
= (6x + 6y + 6) dydx
0 0
Z 1 ⇥ ⇤2x
= 6xy + 3y 2 + 6y 0
dx
0
Z 1 ⇥

= 24x2 + 12x dx
0
⇥ 3 ⇤1
= 8x + 6x2 0 = 14

b) El primer momento con respecto al eje x es

408
Z 1 Z 2x
Mx = y (x, y) dydx
0 0
Z 1 Z 2x
= 6xy + 6y 2 + 6y dydx
Z0 1 0
⇥ ⇤2x
= 3xy 2 + 2y3 + 6y 2 0
dx
0
Z 1 ⇥⇤
= 28x3 + 12x2 dx
0
⇥ 4 ⇤1
= 7x + 4x1 0 = 11

Por otra parte, el primer momento con respecto al eje y es:

Z 1 Z 2x
My = x (x, y) dydx
0 0
Z 1 Z 2x
= 6x2 + 6xy + 6x dydx
Z0 1 0
⇥ ⇤2x
= 6x2 y + 3xy 2 + 6xy 0
dx
0
Z 1 ⇥ ⇤
= 12x3 + 12x3 + 12x2 dx
0
⇥ 4 ⇤1
= 6x + 4x3 0 = 10

c) Ası́, las coordenadas del centro de masa son:

My 10 5
x = = =
M 14 7
Mx 11
y = =
M 14

d) El momento de inercia con respecto al eje x es

409
Z 1 Z 2x
Ix = y 2 (x, y) dydx
Z0 1 Z0 2x
= 6xy 2 + 6y 3 + 6y 2 dydx
0 0
Z 1  2x
3
= 6xy + y 4 + 2y 3 3
dx
0 2 0
Z 1
⇥ ⇤
= 40x4 + 116x3 dx
0
⇥ 5 ⇤1
= 8x + 4x4 0 = 12

Por otra parte, el momento de inercia con respecto al eje y es

Z 1 Z 2x
Iy = x2 (x, y) dydx
Z0 1 Z0 2x
= 6x3 + 6x2 y + 6x2 dydx
Z0 1 0
⇥ ⇤2x
= 6x3 y + 3x2 y 2 + 6x2 y 0
dx
0
Z 1 ⇥ ⇤
= 12x4 + 12x4 + 12x3 dx
0
 1
24 5 39
= x + 3x4 =
5 0 5
Los radios de giros con respecto a los ejes x e y son
r r r
Ix 12 6
Rx = = =
M 14 7
r r
Iy 39
Ry = =
M 70
El radio de giro con respecto al origen es
r
I0
R0 =
M
donde el momento de inercia con respecto al origen esta dado por:
Z Z
I0 = (x2 + y 2 ) (x, y) dxdy = Ix + Iy

410
s
99
5
R0 =
14
Problema 2

Calcular el momento de inercia con respecto al eje x de la lámina plana


R situada en el primer cuadrante, acotada por las curvas
y 2 = x, y 2 = 2x, x2 = y, x2 = 3y, si la densidad en cada punto de la
región es (x, y) = x3 y

Figura 4.70:

Solución: El momento de inercia con respecto al eje x en coordenadas


cartesianas queda
ZZ
Ix = y 2 (x, y) dxdy
Z ZR
= y 2 (x3 y)dxdy
Z ZR
= (xy)3 dxdy
R
Sin embargo, conviene hacer un cambio de variables para simplificar la
geometrı́a de la
9 región
y2 > 
u= = @ (x, y) @ (u, v)
1
x2 =) J = = =
x > @ (u, v) @ (x, y)
v= ;
y
2 1
y 2y
x2 x2 1
= 2x x =
3
y y2

411
Examinemos ahora como se transforma la región
y2 y2
y 2 = x, y 2 = 2x =) = 1; =2
x x
x2 x2
x2 = y, x2 = 3y =) = 1; =3
y y
Luego, R⇤ = {(u, v) / 1  u  2, 1  v  3} .
y 2 x2
Por otra parte, el integrando se puede expresar uv = = xy
x y

ZZ
Ix = (xy)3 dxdy
Z 2RZ 3 ✓ ◆
3 1
= (uv) dvdu
1 1 3
Z Z 3
1 2 3
= u du v 3 dv
3 1 1
 4 2 4 3
1 u v
=
3 4 1 4 1
 
1 1 81 1
= 4
3 4 4 4
1 15
= 20 = 25
3 4

4.6.5. Momentos y Centros de masa de un sólido

Sea Q un sólido una región del espacio en IR3 . Si la densidad en (x, y, z)


es (x, y, z) , y es continua en Q, entonces la masa M y los primeros mo-
mentos del sólido con respecto a los planos xy, xz e yz respectivamente son
Mxy , Mxz , Myz y las coordenadas del centro de masa (x, y, z) se definen:

412
Z Z Z
M = (x, y, z) dV
Q
Z Z Z
Mxy = y (x, y) dV
Q
Z Z Z
Mxz = y (x, y, z) dV
Q
Z Z Z
Myz = x (x, y, z) dV
Q
RRR
Mxy Q
z (x, y, z) dV
x = = RRR
M (x, y, z) dV
R R RQ
Mxz Q
y (x, y, z) dV
y = = RRR
M (x, y, z) dV
R R RQ
Mxy Q
x (x, y, z) dV
z = = RRR
M Q
(x, y, z) dV

Sea Q un sólido una región del espacio en R3 . Si la densidad en (x, y, z) es


(x, y, z) , y es continua en Q, entonces los segundos momentos del sólido o
momentos de inercia con respecto al eje x, al eje y,y al eje z respectivamente
son Ix , Iy , Iz se definen:

Z Z Z
Ix = (y 2 + z 2 ) (x, y, z) dV
Z Z ZQ
Iy = (x2 + z 2 ) (x, y, z) dV
Z Z ZQ
Iz = (x2 + y 2 ) (x, y, z) dV
Q

4.6.6. Masa de un sólido


Problema 1

La densidad en un cubo de lado a es proporcional al cuadrado de la dis-


tancia respecto a uno de los vértices ( al que denotaremos por 0). Demostrar
que la masa del cubo es igual a su volumen multiplicada por la densidad en
un vértice adyacente a 0.

413
Solución. Si R es una región cerrada y acotada en IR3 y la densidad
en el punto (x, y, z) 2 R es (x, y, z) , entonces la masa de R está dada por
Z Z
M= (x, y, z) dV
R

Situando el cubo con el vértice en el origen del sistema de coordenadas y


tres de sus caras coincidiendo con los planos coordenados, se tiene:
(x, y, z) = x2 + y 2 + z 2 y por lo tanto
Z Z
M= x2 + y 2 + z 2 dxdydz
R

donde R = {(x, y, z) 2 IR3 /0  x  a; 0  y  a; 0  z  a} . Luego

Z a Z a Z a Z a Z a  a
2 2 2 z3
M = x +y +z
dzdydx = (x2 + y 2 )z + dydx
0 0 0 0 0 3 0
Z aZ a Z a a
2 2 a3 2 y 3 a3
= x a+y a+ dydx = ax y + a + y dx
0 0 3 0 3 3 0
Z a 4
 3 4 a
2a x 2a
= a2 x 2 + dx = a3 + x = a5
0 3 3 3 0

Por otra parte la distancia entre 0 y cualquier vértice adyacente a él es a.


Entonces la densidad de un vértice adyacente es (a, 0, 0) = a2 , (0, a, 0) =
a2 , (0, 0, a) = a2 .
Por lo tanto, queda V (R) · (a, 0, 0) = a3 · a2 = a5 = M
Problema 2

Un sólido tiene forma de cilindro circular recto con radio de la base R


y altura h. Si la densidad en un punto es directamente proporcional a la
distancia a una de las bases. Encontrar:
a) las coordenadas del centro de masa
b) el momento de inercia con respecto al eje axial de simetrı́a del
sólido cilı́ndrico
Soluciones
La región del sólido cilı́ndrico está acotado por
Q = {(x, y, z) 2 IR3 / 0  x2 + y 2  R2 , 0  z  h}
Calculemos en primer lugar la masa total del cilı́ndro, usando coorde-
nadas cilı́ndricas , la región del sólido se transforma en
Q⇤ = {r, ✓, z)/ 0  r  R, 0  ✓  2⇡, 0  z  h}

414
ZZZ
M = (x, y, z) dV
Q
ZZZ Z R Z 2⇡ Z h
= kzrdrd✓dz = kzrdzd✓dr
Q⇤ 0 0 0
Z R Z 2⇡  h Z R Z 2⇡ 
z2 h2
= k d✓dr = k rd✓dr
0 0 2 0 0 0 2
2 Z R  2 R
h r
= k [✓]2⇡
0 rdr = ⇡kh
2
2 0 2 0
1
= ⇡kh2 R2
2

El momento de inercia del sólido con respecto al plano xy es

ZZZ ZZZ
Mxy = z (x, y, z) dV = z (kz) dV
Q Q
ZZZ Z R Z 2⇡ Z h
2
= kz (r) drd✓dz = kz 2 rdzd✓dr
Q⇤ 0 0 0
Z R Z 2⇡  3 h 3 Z R
z h
= k d✓dr = k [✓]2⇡
0 rdr
0 0 3 0 3 0
 2 R
2 r 1
= ⇡kh3 = ⇡kh3 R2
3 2 0 3

Finalmente, las coordenadas del centro de masa son:

My
x = =0
M
Mx
y = =0
M
1
Mxy ⇡kh3 R2 2
z = = 3 = h
M 1 3
⇡kh2 R2
2
415
b) El momento de inercia con respecta al eje de simetrı́a z es:
ZZZ ZZZ
2 2
Iz = (x + y ) (x, y, z) dV = (r2 )kz (r) drd✓dz
Q Q⇤
Z R Z 2⇡ Z h Z R Z 2⇡
h2
= (r3 )kzdzd✓dr = k (r3 )d✓dr
0 0 0 2 0 0
Z R  4 R
r
= ⇡kh2 (r3 )dr = ⇡kh2
0 4 0
1
= ⇡kh2 R4
4
Problema 3
Calcular el momento respecto al eje de simetrı́a de un cono homogéneo aco-
tado por las gráficas x2 y 2 + z 2 = 0 e y = 3, si tiene una densidad
(x, y, z) constante

Figura 4.71:

Solución Sea la región del cono circular recto :


n p p o
Q = (x, y, z) 2 IR3 / y  x  y, 0  y  3, y2 x2  z  y2 x2

Sea (x, y, z) = k, la densidad es constante. Luego, el momento con


respecto al eje y es:
ZZZ
Iz = (x2 + y 2 ) (x, y, z) dV
Q

416
Usando coordenadas cilı́ndricas tenemos:
x = r cos ✓
y= y ) Q⇤ = {(r, ✓, y) / 0  r  3, 0  ✓  2⇡, r  y  3}
z = rsen✓
El Jacobiano de transformación es

@ (x, y, z)
=r
@ (r, ✓, z)
Reemplazando en la expresión anterior se tiene

ZZZ Z 3 Z 2⇡ Z 3
2
Iz = (r )k (r) drd✓dy = k (r3 )dyd✓dr
Q⇤ 0 0 r
Z 3 Z 2⇡ Z 3
= k (r3 ) [y]3r d✓dr = k (r3 ) [3 r] [✓]2⇡
0 dr
0 0 0
Z 3  3
3 4 1 5
= 2⇡k (3r3 r4 )dr = 2⇡k r r
0 4 5 0
11 5
= 3 ⇡k
10

Problema 4

Calcular el volumen y el centroide de la región acotada arriba por la esfera



⇢ = a, y abajo por el cono ✓ = ✓0 , donde 0 < ✓0 < , que tiene una densidad
2
(x, y, z) = 1
Solución.
Sea la⇢ región
3 2 2 a2 p 2 p
Q = (x, y, z) 2 IR / 0  x + y  , y + x2  z  a2 y 2 x2
2
El volumen de esta región en coordenadas esféricas es

Z 2⇡ Z ✓0 Z a Z 2⇡ Z ✓0
2 1 3
V = ⇢ sen✓d⇢d✓d = a sen✓d⇢d✓d
0 3
Z 02⇡ 0 0 0
1 3 2
= a [ cos ✓]✓00 d = ⇡a3 [1 cos ✓0 ]
3 0 3

Por simetrı́a, el centroide está en el eje z, luego determinemos el momento


del sólido respecto al plano xy

417
ZZZ
Mxy = z (x, y, z) dV
Q
ZZZ Z 2⇡ Z ✓0 Z a
2
Mxy = (⇢ cos ✓) ⇢ sen✓d⇢d✓d = ⇢3 cos ✓sen✓d⇢d✓d
Q⇤ 0 0 0
Z 2⇡ Z ✓0 Z 2⇡  ✓0
1 4 1 sen2 ✓
= a cos ✓sen✓d✓d = a4 d
0 0 4 4 0 2 0
Z 2⇡
1 4 1
= a sen2 ✓0 d = ⇡a4 sen2 ✓0
8 0 4
Ası́, las coordenadas del centro de masa son:
Myz
x = =0
M
Mxz
y = =0
M
Mxy 3
z = = a (1 + cos ✓0 )
M 8
Problema 5

Se tiene una región sólida que se encuentra al exterior de la esfera x2 +


y 2 +z 2 = a2 y dentro de la esfera x2 +y 2 +z 2 = b2 ,suponiendo que la densidad
de un punto es directamente proporcional al cuadrado de la distancia de P
al centro de las esferas.
a) Calcule la masa del sólido
b) Calcule su momento de inercia con respecto a una recta que pasa por
el
centro
Solución.
a) La región sólida esta acotada por Q = {(x, y, z) 2 IR3 / a2  x2 + y 2 + z 2  b2 }
y la densidad del punto es (x, y, z) = k (x2 + y 2 + z 2 ) .
La masa total del casquete esférico en coordenadas cartesianas es

ZZZ
M = (x, y, z) dV
Q
ZZZ
= k x2 + y 2 + z 2 dxdydz
Q

Para calcular la masa usaremos coordenadas esféricas:

418
Figura 4.72: Región sólida que se encuentra al exterior de la esfera x2 + y 2 + z 2 =
a2 y dentro de la esfera x2 + y 2 + z 2 = b2

9
x = ⇢sen✓ cos = @ (x, y, z)
y = ⇢sen✓sen =) = ⇢2 sen✓
; @ (⇢, ✓, )
z= ⇢ cos ✓
La región de integración queda:
Q⇤ = {(⇢, ✓, ) / a  ⇢  b, 0  ✓  ⇡, 0   2⇡}
Al sustituir en la expresión anterior tenemos que la masa total es

Z 2⇡ Z ⇡ Z b
M = (k⇢2 )⇢2 sen✓d⇢d✓d
0 0 a
Z 2⇡ Z ⇡  b
⇢5
= k sen✓d✓d
0 0 5 a
Z 2⇡
1
= k b5 a5
[ cos ✓]⇡0 d
5
Z0 2⇡
2
= k b5 a5 d
5 0
4
= ⇡k b4 a4
5

Por la simetrı́a, el momento de inercia con respecto al eje z es

419
ZZZ
Iz = x2 + y 2 (x, y, z) dV
Q
Z 2⇡ Z ⇡ Z b
= ⇢2 sen2 ✓ (k⇢2 )⇢2 sen✓d⇢d✓d
0 0 a
Z 2⇡ Z ⇡ Z b
= k⇢6 sen3 ✓d⇢d✓d
0 0 a
Z 2⇡ Z ⇡  b
⇢7
= k sen3 ✓d✓d
0 0 a 7
2⇡ Z ⇡Z
1
= k b7 a7 sen✓ 1 cos2 ✓ d✓d
7 0 0
Z 2⇡  ⇡
1 1
= k b7 a 7 3
cos ✓ + cos ✓ d
7 0 3 0
Z 2⇡
4
= k b7 a7 d
21 0
8⇡ 7
= k(b a7 )
21
Problema 6

Calcular el momento de inercia con respecto al eje z del sólido acotado


entre las esferas x2 + y 2 + z 2 a2 , x2 + y 2 + z 2  9a2 . Si la densidad en
x2 + y 2 + z 2
cada punto de la región es (x, y, z) = .
x2 + y 2

Figura 4.73: Sólido acotado entre las esferas x2 +y 2 +z 2 a2 , x2 +y 2 +z 2  9a2 .

Solución
Por la simetrı́a, el momento de inercia con respecto al eje z es

420
ZZZ
Iz = x2 + y 2 (x, y, z) dV
Q
ZZZ
2 2 x2 + y 2 + z 2
= x +y dV
Q x2 + y 2
ZZZ
= = x2 + y 2 + z 2 dxdydz
Q

donde la región sólida está acotada por


Q = {(x, y, z) 2 IR3 / a2  x2 + y 2 + z 2  9a2 }.
Para calcular la última
9 integral usaremos coordenadas esféricas:
x = ⇢sen✓ cos = @ (x, y, z)
y = ⇢sen✓sen =) = ⇢2 sen✓
; @ (⇢, ✓, )
z= ⇢ cos ✓
La región de integración queda:
Q⇤ = {(⇢, ✓, ) / a  ⇢  3a, 0  ✓  ⇡, 0   2⇡}
Al sustituir en la expresión anterior tenemos que Iz queda:

Z 2⇡ Z ⇡ Z 3a
Iz = (⇢2 )(⇢2 sen✓)d⇢d✓d
Z0 2⇡ Z0 ⇡ Za 3a
= ⇢4 sen✓d⇢d✓d
0 0 a
Z 2⇡ Z ⇡  3a
⇢5
= sen✓d✓d
0 0 5 a
Z 2⇡ Z ⇡
1
= (3a)5 a5 sen✓d✓d
5 0 0
Z 2⇡
8 5
= a [ cos ✓]⇡0 d
5
Z0 2⇡
16 5
= a d
5 0
32 5
= ⇡a
5
Problema 7

Encontrar el centro de masa del sólido homogéneo , cuya densidad es


(x, y, z) = 1, que está acotado por arriba por z 2 = 64 + x2 + y 2 y por
abajo z 2 = 2( x2 + y 2 ), z 0.

421
Figura 4.74:

Solución.
Como la densidad es constante (x, y, z) = 1,y de acuerdo a la simetrı́a
,que presenta el problema las coordenadas del centro de masa son

Myz
x = =0
M
Mxz
y = =0
M RRR
Mxy Q
zdV
z = =
M V (Q)
Para representar9el problema, usaremos coordenadas cilı́ndricas.
x = r cos ✓ =
@ (x, y, x)
y = rsen✓ =) = r es el Jacobiano de transforma-
; @ (r, ✓, z)
z= z
ción.
Primero determinemos la función intersección de las superficies:
z 2 = 64 + x2 + y 2
=) x2 + y 2 = 64
z 2 = 2(x2 + y 2 )
Pero en cilı́ndricas describen: x2 + y 2 = r2 =) 0  r2  64 =) 0 
r8
Por otra parte 0  ✓  2⇡
Ahora determinemos las cotas para z p
z 2 = 64 + x2 + y 2 = 64 r2 =) z = 64 2
p + r ,con z 0
2 2 2 2
z = 2(x + y ) = 2r =) z = 2r
Luego la región de integración queda: p p
Q⇤ = (r, ✓, z) / 0  r  8, 0  ✓  2⇡, 2r  z  64 + r2
Calculemos el volumen de la región:

422
ZZZ

V (Q ) = rdrd✓dz
Q⇤
Z Z Z p Z Z
2⇡ 8 64+r 2 2⇡ 8 p
2
= p
r dzdrd✓ = [z]p64+r
2r
rdrd✓
0 0 2r 0 0
Z 2⇡ Z 8 hp
p i
= 64 + r2 2r rdrd✓
0 0
Z 2⇡ Z 8 p p Z 8 2
= 2
r 64 + r dr 2 r dr d✓
0 0 0
Z 2⇡ "  8 p
 3 8! #
1 2 r
= (64 + r2 )3/2 2 d✓
0 2 3 0 3 0
 21/2
2 218/2 p 83
= 2⇡ 2
3 3 3
2 h p i
) V (Q⇤ ) = ⇡ 29 23/2 1 229
3
Por otro lado, el momento de inercia es

ZZZ
M = (x, y, z) zdV
Q
ZZZ
= z rdrd✓dz
Q⇤
Z Z Z p
2⇡ 8 64+r2
= p
r zdzd✓dr
0 0 2r
p
Z 2⇡ Z 8  64+r2
z2
= r drd✓
0 0 2 p2r
Z Z
1 2⇡ 8 ⇥ ⇤
= 64 + r2 2r2 rdrd✓
2 0
Z 2⇡ Z0 8
1 ⇥ ⇤
= 64r r3 drd✓
2 0 0
Z 2⇡  8
1 r4
= 32r2 d✓
2 0 4 0
1 ⇥ 2 ⇤
= 2⇡ 8 (32 16) = 1024⇡
2
Por tanto, la tercera coordenada del centro de masa del sólido es:

423
RRR
zdV 1024⇡ ⇣p ⌘
Q
z= = p =3 2+1
V (Q) 1024⇡ 2 1
3

4.6.7. Determinación del centroide dee un sólido


Problema 1

Encontrar el centroide del sólido encerrado por el cilindro x2 + y 2 = 4


, acotado por arriba por el paraboloide z = x2 + y 2 , y por abajo por el
plano XY. Para efectos de cálculo suponga que la densidad es (x, y, z) = 1
constante.

Figura 4.75: Sólido encerrado por el cilindro x2 + y 2 = 4 , acotado por arriba por
el paraboloide z = x2 + y 2 , y por abajo por el plano XY

Solución
El centroide del sólido (x, y, z) ,está en su eje de simetrı́a,en este caso
el eje z,. Esto hace que x = y = 0.
Para hallar z, dividimos el primer momento Mxy por la masa M del sólido,
es decir

Mxy
z=
M
RRR
Donde el primer momento RRRse define M xy = ⌦
z (x, y, z) dxdydz y la
masa del sólido es M = ⌦
(x, y, z) dxdydz siendo
⌦ = {(x, y, z) 2 IR3 / x2 + y 2  4, 0  z  x2 + y 2 }

424
Por la simetrı́a del dominio, es necesario hacer un cambio de variables a
coordenadas cilı́ndricas
9
x = r cos ✓ =
y = rsen✓ =) el jacobiano de transformación es
;
z= z
cos ✓ rsen✓ 0
@ (x, y, z)
= sen✓ r cos ✓ 0 = r
@ (r, ✓, z)
0 0 1
Entonces la región ⌦ se tranforma en
⌦⇤ = {(r, ✓, z) / 0  r  2, 0  ✓  2⇡, 0  z  r2 }
El valor de Mxy es:

ZZZ
Mxy = z (x, y, z) dxdydz

ZZZ Z 2⇡ Z 2 Z r2
= zrdrd✓dz = zrdzdrd✓
⌦⇤ 0 0 0
Z 2⇡ Z 2 r2 Z 2⇡ Z 2
z2 r5
= rdrd✓ = drd✓
0 0 2 0 0 0 2
Z 2⇡ 6 2 Z 2⇡
r 16
= d✓ = d✓
0 12 0 0 3
32
= ⇡
3
El valor de la masa M es

ZZZ
M = (x, y, z) dxdydz

ZZZ Z 2⇡ Z 2 Z r2
= rdrd✓dz = rdzdrd✓
⌦⇤ 0 0 0
Z 2⇡ Z 2 Z 2⇡ Z 2
2
= z|r0 rdrd✓ = r3 drd✓
0 0 0 0
Z 2⇡ 2 Z 2⇡
r4
= d✓ = 4d✓
0 4 0 0
= 8⇡

Por lo tanto
Mxy 4
z= =
M 3
425
✓ ◆
4
En consecuencia, el centroide se localiza en 0, 0, .
3
Problema 2

Calcular el momento de inercia del sólido con respecto al eje z , que


está acotado por arriba por la esfera sólida x2 + y 2 + z 2 = a2 y por abajo
por el cono ✓ = ⇡3 ,y cuya densidad es constante = 1

Solución:
En coordenadas rectangulares, el momento de inercia con respecto al eje
z es
ZZZ
Iz = x2 + y 2 (x, y, z) dV
Q

En coordenadas esféricas, queda


Z 2⇡ Z ⇡/3 Z 1
Iz = ⇢2 sen2 ✓ ⇢2 sen✓d⇢d✓d
0 0 0

Z 2⇡ Z ⇡/3 Z 1 Z 2⇡ Z ⇡/3  1
4 3 ⇢5
⇢ sen ✓d⇢d✓d = sen3 ✓d✓d
0 0 0 0 0 5 0
Z 2⇡ Z ⇡/3 Z 2⇡  ⇡/3
1 2 1 1
sen✓ 1 cos ✓ d✓d = cos ✓ + cos3 ✓ d
5 0 0 5 0 3 0
Z 2⇡
1 5 1
= d = (2⇡)
5 0 24 24

Iz =
12

4.7. Autoevaluación Integrales dobles y triples


Tiempo 2 horas
Pregunta 1
R2R4 x2 xe2y
a) Dada la integral 0 0
dydx, dibuje la región de integración y
4 y
evalúe la integral.
b) Calcular el volumen del sólido limitado por las superficies

x2 + y 2 + z 2 = 1, x2 + y 2 + z 2 = 4, z 2 = x2 + y 2 , con z 0

Pregunta 2

426
RRR
Calcular R
(x2 + y 2 )dxdydz, siendo R un cono recto de revolución de
altura h, base situada en el plano XY, eje de simetrı́a en z, dado por
la ecuación a2 (h z)2 = h2 (x2 + y 2 ).
Pregunta 3 RRR
Calcular la integral I = R
xyzdxdydz, en la región del primer octante
limitada por los planos coordenados y el plano x + y + z = 1.

Pauta de Corrección
Pregunta 1
a) Por la forma del integrando es conveniente cambiar el orden de inte-
gración,
en efecto

Z 2 Z 4 x2 Z 4 Z p
4 y
xe2y xe2y
dydx = dxdy
0 0 4 y 0 0 4 y
Z  p
4 4 y
x2 e2y
= dy
0 4 (4 y) 0
Z 4
e2y
= dy
0 2
 4
e2y e8 1
= =
4 0 4

b) El volumen del sólido está dado por


Z Z Z
V (D) = dxdydz
R

Usando coordenadas esféricas, tenemos

x = r sin ✓ cos
y = r sin ✓ sin
z = r cos ✓
@ (x, y, z)
que tiene por Jacobiano de la transformación = r2 sin ✓.
@ (r, ✓, )
Entonces ésta región R corresponde en el espacio r✓ a la región R⇤ dada
por R⇤ = (r, ✓, ) /1  r  2; 0  ✓  ⇡4 ; 0   2⇡ .

427
Des esta manera

Z 2⇡ Z ⇡ Z 2
4
V (D) = r sin ✓drd✓d
0 0 1
Z 2⇡ Z ⇡  2
4 r3
= sin ✓d✓d
0 0 3 1
Z 2⇡
7 ⇡
= [ cos ✓]04 d
3 0
p !
7 2
= 1 2⇡
3 2
p !
14 2
= 1 ⇡
3 2

Pregunta 2
Queremos calcular la integral la integral de la función f (x, y) = y 2 + z 2
sobre
la región R limitada por el cono a2 (h z)2 = h2 (x2 + y 2 ).
Este es un caso tı́pico que por la simetrı́a de la región y del integrando,
bajo cualquier punto de vista conviene hacer el cambio a coordenadas
cilı́ndricas para evaluar la integral. Sea

x = ⇢ cos ✓
y = ⇢ sin ✓
z = z
@ (x, y, z)
que tiene por Jacobiano de la transformación =⇢ .
@ (⇢, ✓, z)
Obsérvese que la región de integración R se describe, proyectando sobre
el
plano XY z = 0 =) (x2 + y 2 )  a2 =) 0  ⇢  a y 0  ✓  2⇡. De
modo que
la proyeción de la región de integración R⇤ se ve en el plano (⇢, ✓) como
el
rectángulo 0  ⇢  a y 0  ✓  2⇡.
Por otra parte, sustituyendo en a2 (h z)2  h2 (x2 + y 2 ) =)
h
0  z  (a ⇢) . De esta manera, la región de integración en el nuevo
a
428
sistema queda como

⇤ h
R = (⇢, ✓, z) /0  ⇢  a ; 0  ✓  2⇡; 0  z  (a ⇢)
a
La función a integrar f (y, z) = x2 + y 2 se reduce a f (⇢ sin ✓, z) = ⇢2 .Se
tiene
entonces que

Z Z Z Z Z Z
2 2
(x + y )dxdydz = ⇢2 (⇢) d⇢d✓dz
R R⇤
Z a Z 2⇡ Z h
(a ⇢)
a
= ⇢3 dzd✓d⇢
Z0 a Z0 2⇡ 0
h
= ⇢3 (a ⇢) d✓d⇢
0 a
Z0
2⇡h a
= a⇢3 ⇢4 d⇢
a 0
 a
2⇡h ⇢4 ⇢5
= a
a 4 5 0
⇡h 4
= a
10

Pregunta 3.
El plano corta a los ejes coordenados en los puntos (1, 0, 0) , (0, 1, 0) y
(0, 0, 1) .
Si se proyecta la región sobre el plano XY, se tiene z = 0 =) x + y = 1,
luego Rxy = {(x, y) /0  x  1, 0  y  1 x} . Por consiguiente, una
partición
de la región está dada por

R = {(x, y, z) /0  x  1, 0  y  1 x, 0  x  1 x y}

429
Luego la integral se puede expresar:
Z Z Z Z 1Z 1 xZ 1 x y
xyzdxdydz = xyzdzdydx
R 0 0 0
Z 1Z 1 x
1
= xy (1 x y)2 dydx
2
Z0 1 Z0 1 x
1⇥ ⇤
= xy y(1 x)2 2(1 x)y 2 + y 3 dydx
0 0 2
Z 1 " #
1 (1 x)4 (1 x)4 (1 x)4
= x 2 + dx
2 0 2 3 4
Z
1 1 1
= x(1 x)4 dx =
2 0 720

Autoevaluación No 2
Tiempo 2 horas
Pregunta 1
Encontrar el volumen del sólido, en el primer octante, acotado por los
planos
coordenados, el cilı́ndro x2 + y 2 = 4 y el plano z + y = 3.
Pregunta R R2.R 1
Calcular R 1+(x2 +y 2 +z 2 )3/2 3/2
dxdydz, siendo R la esfera
[ ]
x 2 + y 2 + z 2 = a2 .
Pregunta 3.
y
Utilice la sustitución u = , v = y + x2 para evaluar la integral
x
Z Z
y + 2x2
dydx
x2 + xy
Rxy

en donde Rxy es la región del primer cuadrante acotada por las curvas
y = 3 x2 , y = 8 x2 , e y = 2x. 
1
@ (x, y) @ (u, v)
Nota: Usar la propiedad = en un dominio en que
@ (u, v) @ (x, y)
ambos Jacobianos son distintos de cero.

Pauta de Corrección.

430
Pregunta 1
El volumen de la región R está dado por
Z Z
V = f (x, y)dxdy
R

En nuestro caso f (x, y) = z = 3 y queda definida en


p la región del primer
octante dada por R = (x, y) /0  x  2, 0  y  4 x2 . Entonces,
al sustituir términos en el integrando, obtenemos

Z Z p
2 4 x2
V = (3 y) dydx
0 0
p
Z 2  4 x2
y2
= 3y dx
0 2 0
Z  ✓ ◆
2 p 4 x2
= 3 4 x2 dx
0 2
 ⇣x⌘ 2
3p x3
= 4 x2 + 6arcsen 2x +
2 2 6 0
⇣⇡ ⌘ 8 9⇡ 8
= 6 4+ =
2 6 3
Pregunta 2.
1
Calculemos la integral triple de la función f (x, y, z) = 3/2
[1+(x2 +y2 +z2 )3/2 ]
sobre la región R limitada por la esfera x + y + z = a2 .
2 2 2

Aunque el problema se puede plantear en coordenadas directamente en


coordenadas cartesianas , por la forma de la región y del integrando, será
dificı́l resolverlo directamente. En consecuencia, lo resolveremos en
coordenadas esféricas. Sea

x = r sin ✓ cos
y = r sin ✓ sin
z = r cos ✓
@ (x, y, z)
que tiene por Jacobiano de la transformación = r2 sin ✓.
@ (r, ✓, )
Entonces esta región R corresponde en el espacio r✓ a la región R⇤
dada por R⇤ = {(r, ✓, ) /0  r  a; 0  ✓  ⇡; 0   2⇡}
1
El integrando f (x, y, z) = 3/2 en las nuevas coordenadas
[1+(x +y +z2 )3/2 ]
2 2

431
1
queda f (r sin ✓ cos , r sin ✓ sin , r cos ✓) = ,de modo que la
(1 + r3 )3/2
integral por calcular toma la forma:

Z Z Z Z Z Z
1 (r2 sin ✓)
h i3/2 dxdydz = drd✓d
R
1 + (x2 + y 2 + z 2 )3/2 R⇤ [1 + r3 ]3/2

Z 2⇡ Z ⇡ Z a Z 2⇡ Z ⇡
a
r2 sin ✓ 2
drd✓d = sin ✓d✓d
0 0 0 [1 + r3 ]3/2 0 0 (1 + r3 )1/2 0

!Z !
2⇡
1 1
2 1 1/2
cos ✓|⇡0 d = 4 1 1/2
|2⇡
0
(1 + a) 0 (1 + a)
!
1
= 8⇡ 1
(1 + a)1/2
Pregunta 3.
R R y + 2x2
Vamos a evaluar la integral 2
dydx, en la región Rxy del primer
Rxy x + xy
cuadrante acotada por las curvas y = 3 x2 , y = 8 x2 , e y = 2x.
y
Usando el cambio de variables u = , v = y + x2 , la región Rxy se
x
transforma en la región del primer cuadrante acotada por las curvas:
y + x2 = 3 =) v = 3
y + x2 = 8 =) x = 8,
y
= 2 =) u = 2;
x
y
= 0 =) u = 0
x
Luego, Ruv = {(u, v) /0  u  2, 3  v  8} .
Calculemos ahora el Jacobiano
 de la transformación usando la propiedad
1 y 1
@ (x, y) @ (u, v) x2
1
x
@ (x, y) x2
= = =) =
@ (u, v) @ (x, y) 2x 1 @ (u, v) y + 2x2
Sustituyendo términos en la integral, obtenemos
Z Z Z Z
y + 2x2 1
2
dydx = dudv
x + xy 1+u
Rxy Ruv
Z 2 Z 8 Z 2
1 5
= dudv = du
0 3 1+u 0 1+u
= 5 ln |1 + u|20 = 5 ln 3

432
Autoevaluación No 3

Tiempo 2 horas
Pregunta 1
R1R1 2
Considere la integral I = 0 y e x dxdy, dibuje la región de integración,
cambie el orden de integración y calcúlela.
Pregunta 2
RR x y
Utilice la sustitución u = x+y, y = x y para evaluar I = R
e ( x+y ) dA,
donde R es la región triángular acotada por las rectas x = 0, y = 0 y
x = 1 y.

Pregunta 3 p
En la esfera x2 + y 2 + z 2 = a2 los planos y = x , y = 3x determinan
un
sector esférico D, con x 0, y 0, z 0. Calcular el volumen de D

Pauta Autoevaluación
Pregunta 1 R1R1 2
La integral I = 0 y e x dxdy está definida sobre la región de tipo II
dada
por RII = {(x, y) 2 R2 /y  x  1, 0  y  1} . Invirtiendo la partición
de la región a una de tipo I, tenemos:

RI = (x, y) 2 R2 /0  x  1, 0  y  x

entonces
Z 1 Z 1 Z 1 Z x
x2 x2
e dxdy = e dydx
0 y 0 0
Z h ix
1
x2 1 h x2 i 1
ye dx = e
0 0 2 0
✓ ◆
1 1
= 1
2 e
Pregunta 2

Usando la sustitución
u= x+y
(1)resolviendo x, y en términos de u, v produce
v= x y

433
u+v
x= 2
u v (2) determinemos como tranforma la región triángular
y= 2

R = (x, y) 2 R2 /0  x  1, 0  y  1 x

Reemplazando valores en (2)


9 9
x+y =1 = u =1 =
x =0 =) u + v = 0
; ;
y =0 v =u
Luego, la proyección de la región en el plano (u, v) es
R⇤ = {(u, v) 2 R2 /0  u  1, u  v  u}
Calculemos el Jacobiano de la transformación, nos queda
@ (x, y) 1 1
= 21 2 1 = 12 = 12
@ (u, v) 2 2
La función a integrar es, en términos de las nuevas variables

Z Z Z Z ✓ ◆
( x y
) ( v
) 1
e x+y dA = e u dudv
R R⇤ 2
Z 1Z u ✓ ◆ Z
( ) 1 1 1h iu
ue ( u )
v v
e u dudv = du
0 u 2 2 0 u
Z  1
(e 1 e) 1 (e e 1 ) u2
udu =
2 0 2 2 0
1
(e e )
=
4
Pregunta 3
El volumen de la región R está dado por
Z Z Z
V (D) = dxdydz
R

Usando coordenadas esféricas, tenemos

x = r sin ✓ cos
y = r sin ✓ sin
z = r cos ✓
@ (x, y, z)
que tiene por Jacobiano de la transformación = r2 sin ✓.
@ (r, ✓, )
Entonces ésta región R corresponde en el espacio r✓ a la región R⇤

434
dada por R⇤ = (r, ✓, ) /0  r  a; 0  ✓  ⇡2 ; ⇡4   ⇡
3
.
De ésta manera

Z ⇡ Z ⇡ Z a
3 2
V (D) = r2 sin ✓drd✓d

4
0 0
Z ⇡ Z ⇡
3 2 a3
= sin ✓d✓d

4
0 3
3 Z ⇡
a 3
= d
3 ⇡
4

a3 ⇣ ⇡ ⇡ ⌘ a3
= = ⇡
3 3 4 36

435
Capı́tulo 5

Integral de Linea

Es una forma de generalizar la integral de Riemann en una variable,


conocida en el curso anterior, a una integral definida sobre una curva del
espacio bi o tridimensional.
Planteamiento

Supongamos que tenemos:


a) f : U ⇢ IR3 ! IR función escalar continua en U y
b) !c : [a, b] ! IR3 , !
c (t) = (x(t), y(t), z(t)) una trayectoria de clase
C 1,
es decir una curva en IR3 .
La integral de la función f calculada sobre la trayectoria descrita por la
curva C se define de la forma dada a continuación.
La integral de trayectoria de f (x, y, z) a lo largo de la trayectoria !c,
está definida cuando ! c : [a, b] ! IR3 es de clase C 1 y además la función
compuesta f (t) = f (x(t), y(t), z(t)) es continua en I = [a, b]; por:
Z Z b
f ds = f (x(t), y(t), z(t)) k!
c 0 (t)k dt
!
c a

Alternativamente se denota como:


Z Z Z b
f ds = f (x, y, z)ds = f (!
c (t)) k!
c 0 (t)k dt
!
c !
c a

Observaciones

i) Si f (x, y, z) = 1, tenemos la longitud del arco.

436
En efecto, como f (x, y, z) = 1,
Z Z Z b
f (x, y, z) ds = ds = k!c 0 (t)k dt
!
c !
c a
Z bq
= (x0 (t))2 + (y 0 (t))2 + (z 0 (t))2 dt
a

ii) Si C la curva definida por ! c (t) es de clase 1 !


R C por tramos o f ( c (t))
es continua por tramos, entonces calculamos ! c
f ds en segmentos sobre los
Rb ! ! 0
cuales a f ( c (t)) k c (t)k dt es continua y sumamos las integrales resultantes.
Por ejemplo si ! c =! c 1 [! c 2 [! c 3 con a lo sumo sus extremos en común

Z Z Z Z
f (x, y, z)ds = f (x, y, z)ds + f (x, y, z)ds + f (x, y, z)ds
!
c !
c1 !
c2 !
c2

Algunos ejemplos donde aplicamos esta definición; son los siguientes.


Ejemplo 1
Sea la función escalar f (x, y, z) = x2 + y 2 + z 2 continua en R y
!c : [0, 2⇡] ! IR3R, definida por ! c (t) = (cos t, sin t, t) una trayectoria de
clase C 1 . Calcular ! c
f (x, y, z)ds

Solución:
En este caso tenemos la ecuaciones paramétricas :
x(t) = cos t, y(t) = sin t, z(t) = t
Luego la función compuesta es f (x(t), y(t), z(t)) = cos2 t + sin2 t + t2 =
1 + t2 p
Además ! c 0 (t) = ( sin t, cos t, 1) =) k!
c 0 (t)k = 2
Por lo tanto, sutituyendo términos en la integral, queda

Z Z 2⇡
f (x, y, z)ds = f (x(t), y(t), z(t)) k!
c 0 (t)k dt
!
c
Z0 2⇡ p
= 1 + t2 2dt
0
 2⇡
p t3
2 t+
3 0
p
2 2⇡
= 3 + 4⇡ 2
3

Ejemplo 2

437
R
Calcular la siguiente integral de trayectoria ! c
f (x, y, z)ds donde
a) f (x, y, z) = x + y + z y ! c (t) = (cos t, sin t, t), t 2 [0, 2⇡]
Solución:
En este caso se tiene que x(t) = cos t, y(t) = sin t, z(t) = t
Que produce la función compuesta f (! c (t)) = cos t + sent + t,
!0 ! p
Además c (t) = ( sin t, cos t, 1) y k c 0 (t)k = 2. Entonces
Z Z 2⇡ p
f (x, y, z)ds = 2(cos t + sent + t)dt
c 0

 2⇡
p t2
= 2 sent cos t +
2
Z p 0
4 2 2
=) f (x, y, z)ds = ⇡
c 2

R Como taller adicional, evaluar las siguientes integrales de trayectoria


!
c
f (x, y, z)ds
donde:
b) f (x, y, z) = cos z y ! c (t) = (cos t, sin t, t), t 2 [0, 2⇡]
c) f (x, y, z) = x cos z y ! c (t) = (t, t2 , 0), t 2 [0, 1]

Ejemplo 3
a) Demostrar que la integral de lı́nea de f (x, y), a lo largo de una trayec-
toria dada en coordenadas polares por r = r(✓), con ✓1  ✓  ✓2 es:
s ✓ ◆2
Z ✓2
2
dr
I= f (r cos ✓, r sin ✓) r + d✓
✓1 d✓

b) Calcular la longitud de arco de r = 1 + cos ✓, con 0  ✓  2⇡


Solución:

a) En primer lugar, determinemos la ecuaciones paramétricas de la


trayectoria en coordenadas polares

x = r cos ✓ =) x = r(✓) cos ✓


=) !
c (✓) = (r(✓) cos ✓, r (✓) sin ✓)
y = r sin ✓ =) y = r (✓) sin ✓

Derivando la ecuación de la trayectoria, queda

438
!
c 0 (✓) = (r0 (✓) cos ✓ r (✓) sin ✓, r0 (✓) sin ✓ + r(✓) cos ✓)
Lo que implica

q
k!
c 0 (✓)k = (r0 (✓) cos ✓ r (✓) sin ✓)2 + (r0 (✓) sin ✓ + r(✓) cos ✓)2
q
= r (✓)2 + r0 (✓)2

Por tanto
Z Z ✓2 q
I= f (x, y) ds = f (r cos ✓, r sin ✓) r (✓)2 + r0 (✓)2 d✓
!
c ✓1

b) En este caso se tiene f (x, y) = 1, y como r = 1 + cos ✓, con


0  ✓  2⇡ q q
Obtenemos r (✓) + r (✓) = (1 + cos ✓)2 + ( sin ✓)2
2 0 2

Entonces:
Z 2⇡ q
s = (1 + cos ✓)2 + ( sin ✓)2 d✓
Z0 2⇡ p
= 2 + 2 cos ✓d✓
0
Z ⇡

= 2 | cos |d✓
0 2
Z ⇡ Z ⇡✓ ◆
✓ ✓
= 2 cos d✓ + cos d✓ =
0 2 ⇡ 2
8

Es útil tener en cuenta que la integral de trayectoria se puede aplicar a


problemas de fı́sica e ingenierı́a
1. Si f (x, y, z) representa la masaR por unidad de longitud de un alambre
delgado con la forma de C, entonces ! c
f ds es la masa total del alambre.
2. Si f (x,
R y, z) es la componente tangencial de una fuerza en (x, y, z) de
C, entonces ! c
f ds representa el trabajo realizado por la fuerza al mover una
partı́cula a lo largo de la curva C.
3.- Si f : I ⇢ IR2 ! IR función escalar continua en I = [a, b] y
R Rb
f (x, y) 0,entonces ! c
f (x, y)ds = a f (x(t), y(t)) k!
c 0 (t)k dt representa el
área de una pared cuya base es la imagen de ! c y la altura f (x, y).

439
Figura 5.1: Pared zigzagueante

Ejemplo 4

a) Calcular analı́ticamente la masa M de un resorte que tiene la forma


de una hélice dada por la ecuación: !
r (t) = (5cost; 5sent; 4t); t 2 [0; 2⇡] si la
densidad en cada punto (x, y, z) es el cuadrado de la distancia en cada punto
al origen de coordenadas.

Figura 5.2: Hélice circular

b) Si el centro de masa de un resorte con forma de alambre delgado de


masa M, se define por (XM ; YM ; ZM ) con

440
Z
1
XM = x (x, y, z)ds
M !
Zr
1
YM = y (x, y, z)ds
M !
Zr
1
ZM = z (x, y, z)ds
M ! r

Calcular la componente ZM del resorte helicoidal


Solución.
a) La fórmula para el cálculo de la masa del resorte es:
Z Z b
M= (x, y, z)ds = (!
r (t)) k!
r 0 (t)k dt
!
r a

donde se tiene que:


!
r (t) = (5cost; 5sent; 4t); t 2 [0; 2⇡] =) ! r 0 (t)
! 0
p 2
p = ( 5 sin t; 5 cos t; 4)
2
=) k r (t)k = 25 sin t + 25 cos t + 16 = 41 y la densidad
(x; y; z) = x2 + y 2 + z 2 =) (! r (t)) = (25 + 16t2 )
Reemplazando en el integrando, queda
Z Z 2⇡ p
M = (x, y, z)ds = (25 + 16t2 ) 41dt
!
r 0
 2⇡ 
p 16 3 p 128 3
= 41 25t + t = 41 50⇡ + ⇡
3 0 3

b) La componente ZM del centro de masa es


Z Z b
1 1
ZM = z (x, y, z)ds = z(t) (! r (t)) k!
r 0 (t)k dt
M r ! M a
Z 2⇡ p  2⇡
1 2
p 41 25 2
= t(25 + 16t ) 41dt = t + 4t4
M 0 M 2 0
p
41 ⇥ ⇤ ⇡ [50 + 64⇡ ]2
= 50⇡ 2 + 64⇡ 4 = 
M 128 2
50 + ⇡
3

5.1. Campos vectoriales


!
Sea F = (F1 , F2 , F3 ) un campo vectorial en R3 continuo definida sobre
la trayectoria !
c : [a, b] ! R3 con !
c perteneciente a C 1 .

441
!
Definimos la integral de lı́nea de F a lo largo de ! c por
Z Z b
! ! !!
F · ds = F ( c (t)) · !
c 0 (t)dt
!
c a

Esta definición se puede generalizar para campos vectoriales en Rn

Figura 5.3: Campo vectorial radial

Figura 5.4: Campo circular tangencial

Otras
R observaciones.
! ! !
1) ! c
F · d s también se puede definir si F (! c (t)) · !
c 0 (t) es continua
por tramos, expresando en este caso la integral como una suma de integrales
del tipo anterior.
!
2) Si C es trayectoria tal que ! c 0 (t) 6= 0 , podemos usar el vector
tangente unitario Tb para calcular la integral de lı́nea como sigue.

442
Figura 5.5: Flujo de un fluido en un tubo cilı́ndrico largo

!c 0 (t)
Tb =
k!c 0 (t)k
Z Z b !
! !! c 0 (t)
F · d!
s = F ( c (t)) · !0 k!c 0 (t)k dt
!
c a k c (t)k
Z bh i
= F ( c (t)) · T k!
! b c 0 (t)k dt
Za
! b
= F · T ds
!
c

Por lo tanto Z Z
! ! ! b
F ·ds = F · T ds
!
c !
c
!
Si ' es el ángulo entre F y Tb

Z Z
! b ! ! ! ! ! !
F ·T = F T cos ' = F cos ' =) F ·ds = F cos 'ds
!
c !
c

!
Si F es un campo de fuerza , entonces esta última integral es precisa-
mente la definición de trabajo realizado por el campo de fuerza sobre una
partı́cula de masa unitaria que se mueve desde el punto inicial hasta un
punto terminal a lo largo de la curva C descrita por la trayectoria ! c (t).
3) Otra manera usual de escribir la integral de lı́nea es:
Z Z
! ! !
F ·ds = F1 dx + F2 dy + F3 dz donde F = (F1 , F2 , F3 )
!
c !
c

443
Ası́:
Z Z Z b
! dx dy dz
F · d!
s = F1 dx + F2 dy + F3 dz = F1 + F2 + F3 dt
!
c !
c a dt dt dt
Z b
dx dy dz
= (F1 , F2 , F3 ) · ( , , )dt
a dt dt dt
Z b
!!
= F ( c (t)) · !
c 0 (t) dt
a

Ejemplo 1:
R
Evaluar I = c x2 ydx + (x2 y 2 ) dy , si C es el arco de la parábola
y = 3x2 desde (0, 0) hasta (1, 3) .

Figura 5.6: Gráfico de la curva y = 3x2

Solución:
Al parametrizar la trayectoria se tiene C : x = t, y = 3t2 con 0  t  1.
luego !c (t) = (t, 3t2 ) =) !c 0 (t) = (1, 6t)
Sustituyendo términos en el integrando, obtenemos
Z 1 ⇥ ⇤
I = (3t4 , t2 9t4 ) · (1, 6t) dt
Z0 1
⇥ ⇤
= 3t4 + t2 9t4 6t dt
0
 1
3 5 6 4 54 6 69
= t + t t =
5 4 6 0 10

444
Ejemplo 2
R
Evaluar c yzdx + xzdy + xydz donde C está formada por los segmentos
de rectas que unen A (1, 0, 0) a B (0, 1, 0) a C (0, 0, 1)
Solución.
! ! ! !
En este caso C = AB + BC + CA por lo cual
Z Z Z
yzdx + xzdy + xydz = !
yzdx + xzdy + xydz + ! yzdx + xzdy + xydz
!
c AB BC
Z
+ ! yzdx + xzdy + xydz
CA

Pametrizando cada segmento de trayectoria, produce


!
AB : x = 1 t =) dx = dt
y=t =) dy = dt
z=0 =) dz = 0
Z Z 1

!
yzdx + xzdy + xydz = 0dt = 0
AB 0

!
BC : x = 0 =) dx = 0
y=1 t =) dy = dt
z=t =) dz = dt
Z Z 1

!
yzdx + xzdy + xydz = 0dt = 0
BC 0

!
CA : x = t =) dx = dt
y=0 =) dy = 0
z=1 t =) dz = dt
Z Z 1

!
yzdx + xzdy + xydz = 0dt = 0
CA 0

Por tanto Z
yzdx + xzdy + xydz = 0
!
c

Ejemplo 3
R ! ! !
Evaluar I = !c
F · d s , donde F (x, y) = (xy, x2 y) y
!
c : [ 1, 1] ! IR2 tal que !c (t) = (t, |t|). ! / C1 .
c 2
Solución

445
La curva C es continua por tramos.
Ası́ !
c 1 : [ 1, 0] ! IR2 , !
c 1 (t) = (t, t) =) dx = dt, dy = dt
! 2 !
c2 : [0, 1] ! IR , c 2 (t) = (t, t) =) dx = dt, dy = dt
! !
De modo que: c = c 1 + c 2 !

Z Z Z
! ! !
F · d!
s = F · d! F · d!
s + s
!
c !
c1 !
c2
Z Z
2
= xydx + x ydy + xydx + x2 ydy
!
c1 !
c2
Z 0 Z 1
= ( t2 + t3 )dt + (t2 + t3 )dt = 0
1 0

Por tanto Z
!
F · d!
s =0
!
c

5.2. Cambio de parametrización


La pregunta crucial que surge al pensar en una curva es ¿cambiará el
valor de la integral si se cambia la parametrización de la curva sobre la que
se integra?.
En primer lugar, ilustremos esto con el siguiente ejemplo.

Ejemplo 1
Si C es la mitad inferior del cı́rculoR unitario en el plano xy que une los
puntos ( 1, 0) con (1, 0) . Calcular C (1 + xy)ds parametrizando de dos
maneras diferentes esta curva.
Solución. ✓ ◆
! p ! 0 t
Sea c (t) = (t, 1 t ) =) c (t) = 1, p
2 =)
1 t2
1
k!c 0 (t)k = p
1 t2
entonces
Z Z 1 p 1
(1 + xy)ds = (1 t 1 t2 ) p dt
!
c 1 1 t2
Z 1✓ ◆
1
= p t =⇡
1 1 t2

446
Por otro lado, si ponemos ! c (t) = (cos t, sin t) con ⇡  t  2⇡
!c 0 (t) = ( sin t, cos t) =) k! c 0 (t)k = 1
entonces, al reemplazar los términos del integrando queda
Z Z 1  2⇡
sin2 t
(1 + xy)ds = (1 + sin t cos t) dt = t + =⇡
!
c 1 2 ⇡
Por lo tanto, hemos obtenido el mismo valor para la integral a
lo
largo de estas dos trayectorias diferentes que tienen la misma
traza.
En segundo lugar, enunciemos un resultado general con el siguiente teo-
rema.
Teorema 5.2.1. Si !c (t) y !
p (t) son dos parametrizaciones distintas suaves
o suaves por tramos, que conecta los puntos P0 y P1 , que tienen la misma
traza y dirección, y si f (x, y, z) está definida y es continua sobre la traza,
entonces Z Z
f (x, y, z)ds = f (x, y, z)ds
!
c !
p

5.2.1. Reparametrización
Sea h : I1 ! I una función de clase C 1 con valores reales que sea biyectiva
entre I = [a, b] y I1 = [a1 , b1 ]. Si !
c : [a, b] ! IR3 una trayectoria C 1 por
tramos. Entonces a la composición
!p =! c h : [a , b ] ! IR3
1 1

La llamamos reparametrización de ! c
El valor de la integral de lı́nea no varı́a por cambios de parametrización
de la curva ! c , excepto por el signo si hay cambios de orientación, el teorema
correspondiente es:
!
Teorema 5.2.2. Sea F un campo vectorial continuo y ! c : [a, b] ! IR3 ,
1 ! 3
trayectoria de clase C , y sea p : [a1 , b1 ] ! IR una reparametrización de
!c . Entonces
i) Si !p conserva la orientación:
Z Z
! ! ! !
F ·ds = F ·ds
!
c !
p

ii) Si !
p invierte la orientación
Z Z
! ! ! !
F ·ds = F ·ds
!
p !
c

447
Figura 5.7:

Ejemplo 1:
!
Sea F (x, y, z) = (yz, xz, xy) campo vectorial y ! c : [ 5, 10] ! IR3
! 2 3
trayectoria dada por c (t) = (t, t , t ) .
Una trayectoria opuesta: ! c op : [ 5, 10] ! R3 se puede construir:
!
c op (t) = c(a + b t)

en este caso queda !


c (t) = !c (5 t) = ((5 t), (5 t)2 , (5 t)3 ) .
R ! ! opR ! !
Evaluar ! c
F ·ds y ! c op
F ·ds.

Solución:
Z Z 10 
! ! dx dy dz
F ·ds = F1 + F2 + F3 dt
!
c 5 dt dt dt
Z 10
= yzdx + xzdy + xydz
5
Z 10 ⇥ ⇤
= t5 + 2t5 + 3t5 dt
5
= 984,375

por otro lado


Z Z 10
! !
F ·ds = yzdx + xzdy + xydz
!
c op 5

448
Z 10 ⇥ ⇤
= (5 t)5 2(5 t)5 3(5 t)5 dt
5
Z 10 ⇥ ⇤10
= 6(5 t)5 dt = (5 t)6 5
5
= 984,375

5.3. Independencia de trayectoria


R ! !
Cuando el valor de una integral !c
F · d s sobre una trayectoria contin-
ua por tramos que conecta dos puntos de una región R depende solamente
de los puntos extremos, se dice que la integral es independiente de la
trayectoria.

Ejemplo. Independencia de trayectoria en una integral de lı́nea


✓ 2 ◆
! y 2y
Si F (x, y) = , ; A = (1, 1); B = (4, 2)
x2 x
!
Calcular el trabajo realizado por el campo de fuerza F al llevar un objeto
desde A hasta B, por:
a) un camino compuesto de un tramo horizontal seguido de un tramo
vertical;
b) un camino compuesto de un tramo vertical seguido de un tramo hori-
zontal.
Solución.
a) Si llamamos C a la curva, la podemos subdividir en las curvas C1 y
C2
Tendremos
Z Z Z
! ! ! ! ! !
F ·ds = F ·ds + F ·ds
!
c !
c1 !
c1

Calculamos
⇢ ambas integrales por separado
x= 1+t
C1 = , 0t3
y= 1
Z Z
! ! y2 2y
=) F ·ds = dx dy
!
c1 !
c1 x2 x
Z 3  3
1 1 3
= dt = =
0 (1 + t)2 (1 + t) 0 4

449

x= 4
C2 = , 0t3
y= 1 t
Z Z Z 3
! ! y2 2y 2(1 t)
=) F ·ds = dx dy = dt
!
c2 !
c2 x2 x 0 4
 3
1 1 2 3 9 3
= t t = =
2 4 0 2 4 4
Con lo que resulta
Z ✓ ◆
y2 2y 3 3
dx dy = + =0
!
c x2 x 4 4

b) Llamemos C * a este otro camino, también lo podemos separar en dos


tramos C3 y C4 . Tendremos
Z Z Z
! ! ! ! ! !
F ·ds = F ·ds + F ·ds
!
c⇤ !
c3 !
c4

Calculamos
⇢ ambas integrales por separado haciendo parametrizaciones:
x= 1
C3 = , 0t3
y= 1 t
Z Z
! ! y2 2y
=) F ·ds = dx dy
!
c3 c3 x2 x
Z 3
2(1 t) ⇥ ⇤3
= ( 1)dt = 2t t2 0 = 3
0 1

x= 1+t
C4 = , 0t3
y= 2
Z Z
! ! y2 2y
=) F ·ds = x 2 dx dy
!
c4 !
c4 x
Z 3  3
( 2)2 4
= dt = =3
0 (1 + t)2 (1 + t) 0

Sumando se obtiene
Z
y2 2y
dx dy = 3+3=0
!
c⇤ x2 x
Por ambos caminos dio el mismo resultado.
Intente otro camino para ésta integral que lleve desde A hasta B.

450
5.4. Campos Conservativos
Ciertos campos de fuerza importantes en la fı́sica provienen de un poten-
cial escalar. Si existe una función definida en una región R y si al campo
!
de fuerza F tiene la propiedad de que
!
F =r = grad
!
entonces se dice que es un potencial de F .

5.4.1. Campo gradiente


Un campo vectorial continuo el cual se obtiene como el gradiente de una
función escalar se llamará campo gradiente y una función de la cual se
obtiene, la función potencial
!
F =r
!
Teorema 5.4.1. (Teorema fundamental) Sea F un campo gradiente con
potencial definida en una región R y sean P0 y P1 puntos cualesquiera de
R, entonces:
Z
! !
F · d s = (P2 ) (P1 )
!
c
donde ! c : [a, b] ! IR3 tal que !
c (a) = P1 , y !
c (b) = P2 , y !
c es una
1
trayectoria de clase C .
Demostración

Z Z
! ! @ @ @
F ·ds = dx + dy + dz
!
c !c @x @y @z
Z b✓ ◆
@ dx @ dy @ dz
= + + dt
a @x dt @y dt @z dt
Z b
@ !
( c (t)) dt = [ (!
b
= c (t))]a
a @t
= (P2 ) (P1 )
Por tanto Z
! !
F ·ds = (P2 ) (P1 )
!
c

451
Adicionalmente podemos afirmar que
!
Teorema 5.4.2. Si F es un campo gradiente en una región R, entonces
R ! !
!
c
F · d s es independiente de la trayectoria.
!
Comentario: Si F es campo gradiente existe definida en R tal que
!
F =r , y el teorema fundamental garantiza la independencia de trayectoria.
Observación: No es dificı́l ver que el recı́proco de este teorema también
es cierto.
!
Teorema 5.4.3. Si F es un campo vectorial continuo sobre una region R y
R ! ! !
si ! c
F · d s es independiente de la trayectoria, entonces F es un campo
gradiente.

Curva simple C, se define como la imagen de una aplicación ! c : I ! IR3


1
de C que sea uno a uno en el intervalo I. Una curva simple es aquella que
no se intersecta a si misma, ! c (a) y !c (b) se llaman punto inicial y punto
final de la curva.
Curva cerrada simple, se define como ! c : [a, b] ! IR3 de C 1 tal que
1) es uno a uno en [a, b]
2) ! c (a) = !c (b) Observación:
i) Si la curva satisface sólo (2) es curva cerrada.
ii) Las curvas simples cerradas tienen dos direcciones de movimiento posi-
ble.
Integrales de lı́nea sobre curvas simples orientadas y curvas cerradas sim-
ples C.
Sea C una curva simple orientada imagen de ! c : [a, b] ! IR3 entonces
definimos:
Z Z Z Z
! ! ! !
F ·ds = F ·ds y f ds = f ds
C !
c C !
c
De las curvas cerradas y la independencia de trayectoria se tiene el siguiente
teorema.
R !
Teorema 5.4.4. C F · d! s es independiente de la trayectoria en R si y
R ! !
solo si C F · d s = 0 para toda curva cerrada C contenida en R.

Un dominio D es conexo si dos puntos cualesquiera de D se pueden unir


con un segmento de recta y todos los puntos de la recta pertenecen a D
Una región es simplemente conexa y abierta si ella es un conjunto abierto
y tal que toda curva simple cerrada en D encierra puntos que solo están en
el interior de D. Si un campo de fuerza tiene la propiedad de que el trabajo

452
realizado sobre una partı́cula en movimiento conforme se mueve de un punto
a otro es independiente de la trayectoria, se llama campo conservativo.
De los teoremas anteriores se puede formular lo siguiente:Üna condición
necesaria y suficiente para que un campo de fuerzas sea consevativo es que
sea un campo gradiente”

Teorema 5.4.5. Sean M (x, y) y N (x, y) funciones con derivadas parciales


continuas en un conjunto abierto y conexo D.
!
Entonces el campo vectorial F (x, y) = (M (x, y), N (x, y))es conservativo
@M @N
si y solo si = en D
@y @x

Ejemplo 1
!
Sea F (x, y) = (2xy, (x2 y))
a) Probar que es campo conservativo.
b) Hallar función potencial.
Solución. 9
@N >
>
= 2x >
=
@x
a) =) Campo conservativo en IR2
@M >
>
= 2x >
;
@x
b) Existe Potencial (x, y) tal que
Z
@ (x, y)
= 2xy ) (x, y) = 2xydx + h1 (y)
@x
Z
@ (x, y) 2
=x y ) (x, y) = x2 y dx + h2 (x)
@y
(x, y) = x2 y + h1 (y) y2
) 2 y2 ) (x, y) = x2 y +C
(x, y) = x y 2
+ h2 (x) 2
!
(Rotacional) Sea F (x, y, z) = (P (x, y, z), Q(x, y, z), R(x, y, z)) función
! !
vectorial, se define el rotacional de F ,denotado por rot F (x, y, z) o r ⇥
!
F (x, y, z) por:
bi b
j b
k
! @ @ @
r ⇥ F (x, y, z). =
@x @y @z
P Q R

453
!
Teorema 5.4.6. Si F (x, y, z) = (P (x, y, z), Q(x, y, z), R(x, y, z)) función
vectorial donde P, Q, R son funciones con derivadas parciales continuas en
un conjunto abierto y conexo D. Entonces
! ! !
F es un campo conservativo () r ⇥ F (x, y, z) = 0 en D
@P @Q @P @R @Q @R
es decir si y sólo si = , = , =
@y @x @z @x @z @y

Ejemplo 2
!
Verifique que F (x, y, z) = (yz, xz, xy) es un campo conservativo en IR3
Solución.

bi b
j b
k
! @ @ @
r ⇥ F (x, y, z). =
@x @y @z
yz xz xy
= ((x x) , (y y) , (z z))
= (0, 0, 0)
! !
o sea r ⇥ F (x, y, z) = 0 en IR3

Ejemplo 3
!
Dado el campo vectorial F (x, y, z) = (3y 2 z+yex , 6xyz+ex , 3xy 2 ).Verifique
que es campo conservativo y calcular potencial.

Solución:
@R @Q
= ) 6yz + ex = 6yz + ex
@y @z
@R @P
= ) 3y 2 = 3y 2
@x @z
@Q @P
= ) 6yz + ex = 6yz + ex
@x @y
! !
Luego r ⇥ F (x, y, z) = 0 en IR3 ,es entonces claramente un campo
vectorial
conservativo, entonces existe (x, y, z) tal que
Z
@ (x, y, x) 2 x
= 3y z + ye ) (x, y, z) = 3y 2 z + yex dx + h1 (y, z)
@x
= 3xy 2 z + yex + h1 (y, z)

454
Z
@ (x, y, x)
= 6xyz + ex ) (x, y, z) = (6xyz + ex ) dx + h1 (x, z)
@y
= 3xy 2 z + yex + h1 (x, z)

Z
@ (x, y, x)
= 3xy 2 ) (x, y, z) = 3xy 2 dx + h1 (x, z)
@z
= 3xy 2 z + h1 (x, y)

De aquı́ se deduce que

) (x, y, z) = 3xy 2 z + yex

R ! !
Además, si se pide calcular I = C
F ·ds y C es la curva descrita
por
!
c (t) = (cos t, sent, t), t 2 [0, 2⇡] , entonces

Z
! ! ⇥ ⇤3
I = F · d s = 3xy 2 z + yex 0
C
= (1, 0, 2⇡) (1, 0, 0) = 0 0=0

(Divergencia de un campo vectorial)


!
Si F (x, y, z) = (P (x, y, z), Q(x, y, z), R(x, y, z)) tal que P, Q y R tienen
!
derivadas parciales en alguna región D. La divergencia de F se denota por:

! ! @P (x, y, z) @Q(x, y, z) @R(x, y, z)


div F = r · F = + +
@x @y @z
! !
Si F es un campo vectorial, entonces la div F da información acerca del

flujo en un punto P(x,y,z):


! !
i) Si div F < 0 en un punto P(x,y,z) , entonces el flujo del campo F se
orienta hacia el punto y se dice que hay un sumidero en P.
! !
ii) Si div F > 0 en un punto P(x,y,z) , entonces el flujo del campo F se
orienta desde el punto y se dice que hay una fuente en P.
! !
iii) Si div F = 0 en un punto P(x,y,z) , entonces el flujo del campo F
es nulo.

455
Introducción Teorema de Green
El teorema de Green, que enunciaremos a continuación, dice que bajo
ciertas condiciones las integrales de lı́nea pueden expresarse y calcularse con
integrales dobles.
Comencemos por definir una región compacta R descrita simultaneamente
por las desigualdades

1 (x) y 2 (x), axb

denominada región orientada en dirección del eje y

1 (y) x 2 (y), cyd


denominada región orientada en dirección del eje x donde las funciones
1 y 2 son continuas y seccionalmente suaves en [a, b] y las funciones
1 y 2 son continuas y seccionalmente suaves en [c, d] .
En esta región cerrada R con una frontera continua seccionalmente suave
consideramos definidas las funciones M y N continuamente
diferenciables ( o de clase C1 ) definidas en R.
La curva C frontera de la región R, se dice que esta orientada
positivamente ,si al caminar sobre ella la región R estará a su izquierda.
En este marco de ideas y condiciones se plantea el teorema de Green.

5.4.2. Teorema de Green


Sea R una región cerrada y acotada con frontera C,orientada en sentido
@M @N
positivo. Si M, N, y son continuas en R. Entonces
@y @x
Z Z Z ✓ ◆
@N @M
M dx + N dy = dA
C R @x @y

456
Figura 5.8: Región cerrada de tipo a, frontera orientada positiva

Demostración.
Supongamos que R está descrita por

1 (x) y 2 (x), axb

Z Z Z
M dx = M dx + M dx
C C1 C2
Z b Z a
= M (x, 1 (x))dx + M (x, 2 (x))dx
a b

Z Z b
=) M dx = [M (x, 1 (x)) M (x, 2 (x))] dx
C a
Por otra parte
Z Z Z bZ 2 (x)
Z b
@M @M
dA = dA = [M (x, 2 (x)) M (x, 1 (x))] dx
R @y a 1 (x)
@y a

Luego Z Z Z
@M
M dx = dA
C R @y
De manera simlar y considerando ahora

1 (y) x 2 (y), cyd

Se establece que Z Z Z
@N
N dy = dA
C R @x

457
Figura 5.9: Región cerrada de tipo b, frontera orientada positiva

Por lo tanto
Z Z Z ✓ ◆
@N @M
M dx + N dy = dA
C R @x @y

Ejemplo 1

Evaluar la integral
Z
I= y 2 + senx2 dx + cos y 2 x dy
C

donde C es la frontera de la región cuadrada 0  x  1, 0  y  1

Figura 5.10: Frontera de la región cuadrada 0  x  1, 0  y  1

458
Solución.
Usando el Teorema de Green podemos afirmar que
@M
M (x, y) = y 2 + senx2 =) = 2y
@y
@N
N (x, y) = cos y 2 x =) = 1
@x
Por lo tanto aplicando el teorema
Z 1Z 1 Z 1
⇥ ⇤1
I= ( 1 2y)dxdy = ( 1 2y)dy = y y2 0
= 2
0 0 0

Comentario. Sin usar el teorema de Green este ejercicio resulta muy


complicado de resolver, vale la pena que lo intente.

459
Ejemplo 2

I
Calcular I = ey dx + (xey + 2y)dy con D = (x; y) 2 IR2 /x2 + y 2  1
C

Figura 5.11: D = (x; y) 2 IR2 /x2 + y 2  1

Solución
Por el Teorema de Green podemos afirmar que

@M
M (x, y) = ey =) = ey
@y
@N
N (x, y) = xey + 2y =) = xey + 2y
@x
Por lo tanto aplicando el teorema
Z Z ✓ ◆ Z Z
@N @M
I= dxdy = (ey ey )dxdy = 0
D @x @y

Corolario 5.4.1. Aplicando el teorema de Green a M = y y N = x se


deduce que I
1
A(D) = xdy ydx
2 C
donde A(D) es el área de la región D contenida en el plano XY , y C es el
borde de D recorrrido en sentido positivo. Esta fórmula nos permite calcular
un área en términos de una integral de lı́nea.

460
x2 y 2
Figura 5.12: Región + 2 1
a2 b

Ejemplo 3
x2 y 2
Consideremos la región 2 + 2  1, cuyo borde es una elipse de semi-ejes
a b
dados por a y b.Calcular el área de la region D
Solución
Podemos parametrizar la elipse usando x(t) = acost , y(t) = bsent,
t 2 [0, 2⇡], y luego se obtiene
!
r (t) = (acost, bsent) =) !r 0 (t) = ( asent, b cos t)
Z
1 2⇡
A(D) = (xy 0 yx0 )dt
2 0
Z
1 2⇡
= (ab cos2 t ( absen2 t))dt
2 0
= ⇡ab

5.5. Aplicaciones de la integral de trayectoria


A continuacion se resumen las formulas para el cálculo de la masa y el
momento de resortes, varillas delgadas y alambres a lo largo de una curva
suave C en el espacio.
R
Masa: M = C (x, y, z) ds, donde = (x, y, z) es la densidad y ds un
elemento diferencial de arco.
MomentosR con respecto a los planosR coordenados: R
Myz = C x (x, y, z) ds; Mxz = C y (x, y, z) ds; Mxy = C z (x, y, z) ds
Coordenadas del centro de Masa
Myz Mxz Mxy
x= ; y= ; z=
M M M
Momentos
R 2 de inercia con respecto a los
R ejes2 y otras rectas
2 2
Ix = R C (y + z ) (x, y, z) ds, Iy = C (x + z ) (x, y, z) ds;
Iz = C (y 2 + z 2 ) (x, y, z) ds;

461
R
IL = C r2 (x, y, z) ds donde r (x, y, z) es la distancia del punto (x, y, z)
a la recta L.
Radio r de giro con respecto a una recta L.
IL
RL =
M

Problema 1
Dado el resorte de densidad constante = 1 con forma helicoidal
! j + tb
r (t) = cos 4tbi + sen4tb k, 0  t  2⇡.

Encontrar la masa del resorte y el momento de inercia con respecto al eje z


Solución
La masa del resorte esta dada por
Z
M = (x, y, z) ds
Z C

= (1)ds
C
Z 2⇡
= |!
r 0 (t)| dt
0
q p
Encontremos primero |! r 0 (t)| = ( 4sen4t)2 + (4 cos 4t)2 + 1 = 17.
Luego evaluemos la integral
Z 2⇡
M = |!
r 0 (t)| dt
Z0 2⇡ p
= 17dt
0
p
M = 2⇡ 17

El momento de inercia con respecto al eje z se define por:

Z
Iz = x2 + y 2 ds
ZC2⇡ p
= cos2 4t + sen2 4t (1) 17dt
0
Z 2⇡ p p
= 17dt = 2⇡ 17
0

462
Problema 2
Sea un arco de metal con forma de semicircunferencia y 2 +z 2 = 1, z 0.
Si la densidad en el punto (x, y, z) del arco es (x, y, z) = 2 z , determinar
las coordenadas del centro de masa del arco.
Solución.
Por simetrı́a sabemos que x = 0; y = 0; dado que el arco está el el plano
yz con la masa distribuida simetricamente con respecto al eje z.
Para encontrar z , parametizamos el arco de semicircunferencia
q como:
! j+sentb
r (t) = cos tb !
k, 0  t  ⇡ =) | r (t)| = ( sent) + (cos t)2 =
0 2

1
Por otra parte, (x, y, z) = 2 z =) (! r (t)) = (2 sent)
Ahora, determinemos la masa utilizando la definición de integral de trayec-
toria

Z Z ⇡
M = (x, y, z) ds = (!
r (t)) |!
r 0 (t)| dt
ZC⇡ 0

= (2 sent) (1) dt = [2t + cos t]⇡0


0
M = 2⇡ 2

A continuación calculemos el momento de inercia Mxy


Mxy 8 ⇡
Luego, z = = ⇡ 0, 57.
M 2 (2⇡ 2)
Finalmente, la coordenada del centro de masa es (0; 0; 0, 57) .

Problema 3
Un alambre tiene la forma de una semicircunferencia de radio a. La densi-
dad lineal de masa en un punto P es directamente proporcional a la distancia
de P a la recta que pasa por los extremos del alambre, ¿cuál es la masa total
del alambre?.
Solución
En primer lugar situar el alambre en el plano xy de sistema coordenado,
como en la figura:
Ahora, la densidad puede interpretarse como un campo escalar, ya que,
en
cada punto P = (x, y) de la curva que describe el alambre, la densidad es
(x, y) = ky.
Calcular la masa total equivale a sumar la densidad sobre todos los puntos

463
de la curva, y ası́ lo que queremos determinar es
Z Z b
M= (x, y, z) ds = (!
r (t)) |!
r 0 (t)| dt
C a

donde C es la curva que describe el alambre, la semicircunferencia de


radio a.
Una parametrización para C es: ! r (t) = (a cos t, asent) , t 2 [0, ⇡]
entonces r (t) = ( asent, a cos t) =) |!
! 0
r 0 (t)| = a
Lo que implica que la masa del alambre es

Z ⇡ Z ⇡
2
M = (kasent) (a) dt = ka sentdt
0 0
= ka2 [ cos t]⇡0 = 2ka2

Problema 4

Un alambre tiene forma de circunferencia, x2 + y 2 = a2 . Determine su


masa y su momento de inercia respecto de un diámetro si la densidad en un
punto (x, y) del alambre está dada por la función (x, y) = |x| + |y| .
Solución
La masa del alambre viene dada por la expresión:

Z Z b
M= (x, y, z) ds = (!
r (t)) |!
r 0 (t)| dt
C a

siendo C la curva cuya trayectoria representa la forma del alambre, en


este
caso una circunferencia que parametrizamos por:

!
r (t) = (acost, asent); t 2 [0, 2⇡]

que es de clase C1 .
!r 0 (t) = ( asent, a cos t) =) k!
r 0 (t)k = 1
Por lo tanto la masa es igual a

464
Z 2⇡ Z 2⇡
M = (!
r (t)) |!
r 0 (t)| dt = (|a cos t| + |asent|)adt
0 0
Z ⇡/2 Z ⇡
2 2
= a (cos t + sent) dt + a ( cos t + sent) dt +
0 ⇡/2
Z 3⇡/2 Z 2⇡
2 2
+a ( cos t sent) dt + a (cos t sent) dt
⇡ 3⇡/2
⇡/2
= a2 [sent cost]0 + a2 [ sent cost]⇡⇡/2 +
+a2 [ sent + cost]3⇡/2
⇡ + a2 [sent + cost]2⇡
3⇡/

= 8a2

Para calcular el momento de inercia respecto de un diámetro necesitamos


la distancia de un punto cualquiera (x, y) a dicho diámetro. Para simplificar,
tomaremos como eje el eje OX, por tanto, la función que da la distancia de
un punto al eje es r(x, y) = |y|. Teniendo en cuenta la definición del momento
de inercia respecto de un eje se tiene:
Z Z
2
IL = r (x, y, z) ds = y 2 (|x| + |y|) ds
C C
Z 2⇡
= a4 sen2 t (|sent| + |cost|) dt
0
Z ⇡/2 Z ⇡
4 2 4
= a sen t (sent + cost) dt + a sen2 t (sent cost) dt
0 ⇡/2
Z 3⇡/2 Z 2⇡
4 2 4
+a sen t ( sent cost) dt + a sen2 t ( sent + cost) dt
⇡ 3⇡/2
4
= 4a

5.5.1. Área de una pared


Problema 1
h ⇡i
!
La base de una pared en el primer cuadrante es la trayectoria c : 0, !
! 2
2 3 3
IR ,dada por c (t) = (3 cos t, 3sen t) y su altura es en cada punto (x, y) es
y
f (x, y) = 1 + .Calcular el área de la pared.
3
Solución

465
El área de una pared esta dada por la integral de trayectoria
Z Z b
f (x, y) ds = f (!
c (t)) |!
c 0 (t)| d
C a

donde f (x, y) 0, 8 (x, y) 2 C


En este caso tenemos que h ⇡i
!c (t) = (3 cos3 t, 3sen3 t) , t 2 0,
2
!c 0 (t) = ( 9 cos2 tsent, 9sen2 t cos t) =) k!
c (t)k = 9sent cos t
!
f ( c (t)) = 1 + sen t3

Ası́, la integral es
Z h Z
yi ⇡/2
1+ ds = 1 + sen3 t (9sent cos t) dt
C 3 0
Z ⇡/2
= 9 sent + sen4 t cos tdt
0
 ⇡/2
sen2 t sen5 t 63
= 9 + =
2 5 0 10

Problema 2

La base de una pared en el primer cuadrante es la trayectoria !c :[0, 1] !


IR 2 ! 2
p ,dada por c (t) = (t, t ) y su altura es en cada punto (x, y) es f (x, y) =
1 + 4y.
a) Determinar la longitud de la base de la pared.
b) Calcular el área de la pared.
Solución
Tenemos que
! p
c (t) = (t, t2 ) =) !c 0 (t) = (1, 2t) =) k!
c 0 (t)k = 1 + 4t4
La longitud de la pared se determina por
Z 1 Z 1 p
l = k!
c 0 (t)k dt = 1 + 4t2 dt
0 0

1 ⇣ ⌘ 1
1 p 2
p
2
= t 1 + 4t + ln 2t + 1 + 4t
2 2 0
 ⇣
1 p 1 p ⌘
= 5 + ln 2 + 5
2 2

466
El área de una pared esta dada por la integral de trayectoria
Z Z 1
f (x, y) ds = f (!
c (t)) k!
c 0 (t)k dt
C
Z 0 1 ⇣p ⌘ ⇣p ⌘
= 1+ 4t4 1+ 4t4
dt
0
Z 1  1
4
= 1 + 4t4 dt = t + t5
0 5 0
9
=
5

5.6. Aplicaciones de la integral de lı́nea


!
Definición . Sea C una curva simple y regular, y sea F :✓ R3 ! R3
un campo vectorial continuo. Definimos la integral de trabajo (o integral de
!
lı́nea) de F sobre la curva C ✓ D por

Z Z b
! ! ! !
W = F ·dr = F ( r (t)) · !
r 0 (t) dt
C a

donde !
r : [a, b] ! IR3 es una parametrización regular de C
Cuando la curva C es cerrada, entonces se puede escribir
I
! !
W = F ·dr
C

! !
y esta integral recibe el nombre de circulación de F a lo largo de C. Si F
representa el campo de velocidades de un fluido, la circulación es la integral
de la componente tangencial de la velocidad a lo largo de la curva cerrada
C, proporcionando la cantidad neta de giro del fluido alrededor de C.

(Campo Conservativo)
!
En general, se dice que un campo vectorial F : D ✓ R3 ! R3 es conservativo
en D si existe un potencial
!
: D ✓ R3 ! R tal que F = r sobre D. En efecto, si C es una curva

467
regular parametrizada por !
r : [a, b] ! IR3 , entonces tenemos que

Z Z b
! ! ! !
F ·dr = F ( r (t)) · !
r 0 (t) dt
C a
Z b
= r (!
r (t)) · !
r 0 (t) dt
a
Z b
d
= [r (!
r (t))] dt
a dt
= (!
r (b)) (!r (a))

! !
Proposición 5.6.1. Sea F = F (x, y, z) un campo vectorial continuo sobre
!
un abierto conexo de IR3 . Entonces las propiedades sobre F siguientes, son
equivalentes, para que sea campo vectorial conservativo:
! !
i) r⇥ F = 0
ii) Para toda curva C ✓ D cerrada y regular por pedazos se tiene

I
! !
F ·dr =0
C

iii) Para cualquier par de curvas regulares, C1 ✓ D y C2 ✓ D, con iguales


puntos inicial y final, se tiene

Z Z
! ! ! !
F ·dr = F ·dr
C1 C2

Problema 1
!
Calcular el trabajo ejercido por el campo vectorial F (x, y, z) = (3x
2y, y + 2z, x2 )sobre una partı́cula que se mueve según la curva C dada por
x = z 2 , z = y 2 desde el punto (0, 0, 0) hasta el (1, 1, 1).
Solución
En primer lugar parametricemos la curva C:! r (t) = (t4 , t, t2 ), t 2 [0, 1]
!
De modo que ! r 0 (t) = (4t3 , 1, 2t) y F (!
r (t)) = (3t4 2t, t + 2t2 , t8 )

468
Calculando directamente tenemos
Z Z b
! ! ! !
W = F ·dr = F ( r (t)) · ! r 0 (t) dt
ZC1 a

= (3t4 2t, t + 2t2 , t8 ) · (4t3 , 1, 2t)dt


Z0 1
= (12t7 8t4 + t + 2t2 2t9 )dt
0
 1
3 8 8 5 1 2 2 3 1 10
= t t + t + t t
4 5 2 3 5 0
13
=
15

Problema 2
! !
Un campo de fuerza gravitacional F (x, y, z) esta dado por F (x, y, z) =
k !
! 3 r . Calcular el trabajo que realiza la fuerza gravitacional que actúa
krk
sobre una partı́cula que se mueve a lo largo del eje x desde el P0 (1, 0, 0)
hasta P(2, 0, 0) .
Solución.
Si parametrizamos la trayectoria tenemos:! r (t) = (t, 0, 0) 1  t  2
! k
Ahora, determinemos la función compuesta: F (! r (t)) = 3 (t, 0, 0).
t
Aplicando la definición de trabajo tenemos

Z Z 2
! ! ! !
W = F ·dr = F ( r (t)) · !
r 0 (t) dt
ZC2 1
k
= (t, 0, 0) · (1, 0, 0) dt
1 t3
Z 2  2
k k k
= dt = =
1 t2 t 1 2

Problema 3
Si una partı́cula es atraı́da hacia el origen por una fuerza cuya magnitud
es proporcional a la distancia k! r k de la partı́cula al origen, ¿que trabajo se
hace cuando se mueve la partı́cula desde el punto (0, 1) hasta el (1, 2) a lo
largo de y = 1 + x2 suponiendo un coeficiente de roce µ entre la partı́cula y
la trayectoria?. Desprecie la fuerza de gravedad.

469
Solución.
Formulemos el problema usando el parametro longitud de arco s
Z Z s2
! ! ! !
W = F ·dr = F ( r (s)) · !
r 0 (s) ds
ZCs2 s1
Z s2
! !
= F ( r (s)) · b
t (s) ds = F T (!r (s)) ds
s1 s1

La fuerza que trabaja es la componente tangencial a la curva. Entonces,


determinemos esta componente a partir de la informacion.
Sea ✓ el ángulo que forma la tangente a la curva en un punto P (x, y)
cualquiera con el eje x.
Sea el ángulo que forma el radio vector P con el eje y.
Sea ↵ el ángulo entre la tangente y el radio vector en P.
Al mover la partı́cula a lo largo de la trayectoria, hay que realizar trabajo
contra dos fuerzas que tienen componente tangencial, a saber, la componente
tangencial de la fuerza central:
Ft = F cos ↵ = kr cos ↵ y la fuerza de rozamiento

Fr = µFn = µF sen↵ = µkrsen↵.

Donde Fn es la componente de la fuerza central que es perpendicular a la


trayectoria y que actúa para mantener la partı́cula en su trayectoria.
Por tanto la fuerza tangencial resultante es:
FT = kr cos ↵ + µkrsen↵
Aplicando, el teorema del ángulo externo de la geometrı́a plana se tiene:
↵= ✓
Reemplazando en la expresión anterior, queda
FT = kr cos ( ✓) + µkrsen ( ✓) .
Luego, el trabajo queda
Z s2
W = F T (!
r (s)) ds
s1
Z L
= (kr cos ( ✓) + µkrsen ( ✓)) ds
0
Z L Z L
= k r cos ( ✓) ds + kµ rsen ( ✓) ds
0 0
Z L
= k r [cos cos ✓ + sen sen✓] ds +
0
Z L
+kµ r [sen cos ✓ds sen✓ cos ] ds
0

470
Ahora bien, el trabajo expresemos el trabajo en coordenadas cartesianas
x = r cos dx = cos ✓ds
y = rsen dy = sen✓ds
Por lo tanto, sustituyendo éstas en la última expresión para W, se tiene:
Z Z
W =k xdx + ydy + µk ydx xdy
C C

Parametizando la curva C, queda: ! r (t) = (t, t2 + 1), 0  t  1 =)


!
r 0 (t) = (1, 2t), 0  t  1
Sustituyendo en la integral anterior, obtenemos

Z 1 Z 1
2
W = k (t, t + 1) · (1, 2t)dt + kµ (t2 + 1, t) · (1, 2t)dt
Z0 1 Z 1 0

= k (2t3 + 3t)dt + kµ (1 t2 )dt


0 0
 1  1
1 4 3 2 1 3
= k t + t + kµ t t
2 2 0 3 0
3
W = 2k + µk
2
Es el trabajo total efectuado durante el movimiento.

Problema 4
!
Sea F (x, y) el campo de fuerzas definido en IR2 por
!
F (x, y) = (2x + ycos(xy), xcos(xy))
!
Calcular el trabajo realizado por F sobre cualquier curva cerrada con-
tenida en IR2
Solución
Si el campo es conservativo, el trabajo realizado por el campo sobre
cualquier curva cerrada será nulo. Por tanto, supongamos que existe : IR2
!
! IR diferenciable tal que F = r . Entonces, se debe satisfacer
@
@x
= 2x + ycos(xy)
@ (1)
@y
= xcos(xy)
Integrando la primera ecuación respecto de x obtenemos,

(x, y) = x2 + sen(xy) + h(y).

471
Si derivamos respecto de y , luego comparamos con la segunda ecuación
de (1) obtenemos que

xcos(xy) + h0 (y) = xcos(xy)

lo que implica
h0 (y) = 0 =) h (y) = c
!
Por lo tanto, la función (x, y) = x2 + sen(xy) + c verifica que F = r
Finalmente I
! !
W = F ·dr =0
C

Problema 5
¿Para qué valores de a2 IR el campo vectorial
!
F (x, y, z) = (axy z 3 , (a 2)x2 , (1 a)xz 2 )

es conservativo? Para esos valores, calcular la función potencial.


Solución
!
Para cualquier valor de a el campo F es de clase C1 en R3 y será
conservativo si su rotacional es cero 8(x; y; z) 2 R3 . Calculemos
el rotacional

i j k
! @ @ @
r⇥ F = @x @y @z
axy z 3 (a 2)x2 (1 a)xz 2
= (0, 3z 2 (1 a)z 2 , 2x(a 2) ax)

que se anula si se cumplen las ecuaciones:

(1 a)z 2 + 3z 2 = 0
2x(a 2) ax = 0

!
Por tanto, para a = 4 el campo F es conservativo y 9 : IR3 ! IR tal
que
!
F = r = (4xy z 3 , 2x2 , 3xz 2 ),entonces

472
@
= 4xy z 3
@x
@
= 2x2
@y
@
= 3xz 2
@z
Integrando la primera ecuación respecto de x, queda
Z
(x, y, z) = (4xy z 3 )dx = 2x2 y xz 3 + h(y, z)

Por tanto
(x, y, z) = 2x2 y xz 3 + g(z)
derivando la función con respecto a z y luego comparamos con la tercera
ecuación tenemos

@
= 3xz 2 + g 0 (z) = 3xz 2 =) g 0 (z) = 0 =) g (z) = c
@z
!
En consecuencia, la función potencial del campo F es

(x, y, z) = 2x2 y xz 3 + c

Problema 6
Una masa M en el origen en IR3 ejerce una fuerza sobre una masa m lo-
mM
calizada en ! r = (x, y, z) con magnitud G 2 y dirigida hacia el origen.
r
Aquı́ G es la constante gravitacional, que depende de las unidades de medi-
p !r
! 2 2 2
ción y r = | r | = x + y + z ,si recordamos que es un vector unitario
r
dirigido hacia el origen, entonces podemos escribir el campo de fuerza como
! mM
F = G 3 ! r
r
a) Mostrar que el campo vectorial es conservativo.
!
b) Hallar un potencial escalar para F .
!
c) Hallar el trabajo realizado por F al trasladar la partı́cula de masa m
desde un punto P1 hasta P2
!
d) ¿Cual es el trabajo realizado por la fuerza F al mover la partı́cula
de masa m desde el infinito hasta una distancia r del origen?.

473
Solución
a) En primer lugar calculemos
✓ ◆
! mM !
r⇥ F = r⇥ G 3 r
r
 ✓ ◆
1 1
= GmM r 3 ⇥ ! r + 3r ⇥ !
r
r r
Puesto ⇣que de las identidades básicas del análisis vectorial, tenemos
!⌘ ! !
i) r ⇥ F = r ( ) ⇥ F + (r ⇥ F )
ii)r (rn ) = nrn 2 !r
Entonces, queda:

! 3! ! 1
r⇥ F = GmM r ⇥ r + 3r ⇥ ! r
r r
De modo que el primer término se anula pues ! r ⇥!
r =0
Luego queda aún por calcular el segundo término
i j k
r⇥!
r = @
@x
@
@y
@
@z = (0, 0, 0)
x y z
Por tanto
! ! !
r ⇥ F = 0 para ! r 6= 0
!
b) Luego existe tal que F = r .(En fı́sica se define que el campo de
fuerza proviene del valor negativo del gradiente de la función escalar).
Aprovechando la simetrı́a radial del campo de fuerza expresemos el
gradiente en coordenadas cilı́ndricas
Z
@ mM ! mM
= G 2 =) ( r ) = G 2 dr
@r r r
GmM
(!
r) = +C
r
Donde c es una constante de integración
c) Como el campo es conservativo, el trabajo depende de la posición de
los
puntos inicial y final y el trabajo es igual a
Z
! !
F ·dr = (!
r (2)) (! r (1))
C

1 1
= GmM
r 1 r2

474
donde r1 y r2 son las distancias radiales de los puntos P1 y P2
respectivamente al origen.

d) Si r1 ! 1 y r2 = r se tiene que

GmM
W =
r

Problema 7
!
Dado el campo de fuerzas F (x, y) = (y 3 + 1, 3xy 2 + 1).
!
a) ¿Es F conservativo? en caso que lo sea, hallar la función potencial de
!
F.
b) Hallar el trabajo realizado al mover un objeto desde el punto (0, 0) al
(2, 0),a lo largo de la semicircunferencia (x 1)2 + y 2 = 1 con y 0.
c) Hallar el trabajo realizado al mover el objeto a lo largo de la circun-
ferencia
completa.
Solución
!
Examinemos el rotor de F
i k j
! @ @
r⇥ F = @x
0 @y = 0, 0, 3y 2 3y 2
3 2
y + 1 3xy + 1 0
= (0, 0, 0)

Por tanto, estamos en presencia de un campo conservativo, luego existe


una
función escalar (x, y) tal que

@
= y3 + 1
@x
@
= 3xy 2 + 1
@y

Integrando la primera ecuación respecto de x, queda


Z
(x, y) = (y 3 + 1)dx = xy 3 + x + h(y)

Para calcular la funciónh(y),calculamos la derivada parcial de respecto


de

475
y luego comparamos con la segunda ecuación:
@
= 3xy 2 + h0 (y) = 3xy 2 + 1 =) h0 (y) = 1 =) h(y) = y + c
@y
!
En consecuencia, la función potencial del campo F es

(x, y, z) = xy 3 + x + y + c
!
b) Como F es conservativo la integral es independiente del camino,
únicamente depende de los puntos inicial y final. El trabajo realizado al
mover el objeto desde (0, 0) hasta (2, 0) será:

Z
! !
F ·dr = (2, 0) (0, 0)
C
= (2 + c) c=2

c) A lo largo de la circunferencia tenemos una curva cerrada,lo que


implica que el trabajo será nulo:
I
! !
W = F ·dr =0
C

Problemas propuestos para reflexionar


1.- Considere dos curvas que parten del punto P = (0, 2) y finalizan en
Q = (⇡, 0); la braquistócrona dada por la ecuación:
!
r (t) = (t + sen(t + ⇡); 1 cos(t + ⇡)); t 2 [0; ⇡];
y el segmento rectitlı́neo que va de P a Q. Pruebe que si se suelta una bola
que se desliza por cada curva, bajo el efecto de la gravedad, entonces llega
antes la bola de la braquistócrona que la del segmento rectilı́neo. Téngase
en cuenta que el tiempo total, para una curva cualquiera, es la integral a lo
largo de dicha curva del campo escalar
1
(x, y) = p
2g (h0 y)
donde g es la gravedad y h0 la altura inicial de la bola.
2.- Calcular el trabajo realizado por una fuerza proporcional al vector
dirigido hacia el origen, sobre el recorrido del primer cuadrante de la elipse
x2 y 2
+ 2 = 1, desde el punto (a, 0) hasta el (0, b).
a2 b
476
k 2
Solución: (a b2 ) con k > 0.
2
3.- Dado h > 0, sea C la curva que se encuentra sobre la superficie defini-
z2
da por x2 + y 2 = 2 , de forma tal que la altura z = z() satisface la ecuación
h
dz
diferencial = z; z(0) = h donde z y representan las coordenadas cilı́ndri-
d
cas.
(a) Parametrice la ecuación de la curva C ! r (✓) .Calcule la curvatura 
y la torsión ⌧ ✓ ◆
! 1 1 1
(b) Considere el campo vectorial F (x, y, z) = , ,
h⇡ ⇡ i x y z2
Sea C0 la restricción de C a 2 , .Calcule el trabajo realizado por el
! 6 3
campo F al desplazar una partı́cula a través de C0 .
Solución p
2
a) !r (✓) = e✓ cos ✓, e✓ sen✓, he✓ , curvatura  = ✓ ; torsión⌧ =
e (2 + h2 )
h
e (2 + h2 )


R ! ! ! ⇡/3 ⇡ 1 1 1
b) C F · d r = ( r (✓))|⇡/6 = + ⇡/3 ⇡/6
3 h e e

5.7. Problemas Resueltos


Problemas 1
R x3
Calcular la integral de trayectoria !
r
ds, donde !
r es la trayectoria
y
x2
y= entre los puntos (0, 0) y (2, 2) .
2
Solución
Primero, determinemos
9 la ecuación paramétrica de la trayectoria
x= t = ✓ 2◆
2 ! t
t t 2 [0, 2] () r (t) = t,
y= ; 2
2
Derivando la expresión anterior,pqueda
!r 0 (t) = (1, t) =) k! r 0 (t)k = 1 + t2
Además
x3
f (x, y) = =) f (! r (t)) = 2t
y
A partir de la definición de integral de trayectoria tenemos

477
Z Z 2 p
x3
ds = 2t 1 + t2 dt
!
r y 0
 2
2 3/2
= 1 + t2
3 0
2 ⇣p ⌘
= 75 1
3

Problema 2
Dada la función escalar f (x, y) = 2xy, calcular la integral de trayectoria
x2 y 2
a lo largo de la curva elipse + = 1 desde el punto (3, 0) hasta (0, 2).
9 4
Solución
Observemos que ! r es el segmento de elipse que está en el primer
cuadrante.Entonces al parametrizar la curva queda
x = 3 cos t ⇥ ⇤
t 2 0, ⇡2 () ! r (t) = (3 cos t, 2sent)
y = 2sent
Derivando la trayectoria p
!r 0 (t) = ( 3 sin t, 2 cos t) () k! r 0 (t)k = 5sen2 t + 4
Calculemos la función escalar f sobre la trayectoria
f (x, y) = 2xy =) f (x (t) , y (t)) = 6 cos tsent
Calculemos la integral

Z Z ⇡/2 p
2xyds = 12 cos tsent 5sen2 t + 4dt
!
r 0
 ⇡/2
4 3/2
= 5sen2 t + 4
5 0
76
=
5

Problema 3
R
Calcular la integral de lı́nea ! r
xydx + x2 dy, donde ! r es la trayectoria
2 2
x + 4y = 4, x > 0.
Solución
Primero, escribamos la ecuación paramétrica de la trayectoria orientada
positivamente
x = 2 cos t ⇥ ⇡ ⇡⇤
t2 2
, 2 =) ! r (t) = (2 cos t, sent)
y = sent

478
! !
F (x, y) = xy, x2 =) F (x (t) , y (t)) = 2 cos tsent, (2 cos t)2

Determinemos el vector !
r 0 = ( 2sent, cos t)
Calculemos la integral

Z Z ⇡/2
2
xydx + x dy = 2 cos tsent, (2 cos t)2 · ( 2sent, cos t) dt
!
r ⇡/2
Z ⇡/2
= ( 4sen2 t cos t + 4 cos3 t)dt
⇡/2
Z ⇡/2
= ( 8sen2 t cos t + 4 cos t)dt
⇡/2
 ⇡
8 2 8
= sen3 t + 4sent =
3 ⇡ 3
2

Problema 4
R
Calcular la integral de lı́nea !
r
y 2 dx + xdy, donde !
r es la trayectoria
y 2 = 2x x2 , tal que x > 1, y > 0.
Solución
Observemos que !
r es el segmento de circunferencia:
y 2 = 2x x2 () (x 1)2 + y 2 = 1 tal que x > 1, y > 0.
Entonces:
x = 1 + cos t ⇥ ⇤
t 2 0, ⇡2 =) ! r (t) = (1 + cos t, sent)
y= sent
=) ! r 0 (t) = ( sent, cos t)
!
Calculemos el campo vectorial F sobre la trayectoria
! !
F (x, y) = ( y 2 , x) =) F (x (t) , y (t)) = ( sen2 t, 1 + cos t)

479
Calculemos la integral

Z Z ⇡/2
2
y dx + xdy = sen3 t + cos2 t + cos t dt
!
r 0
Z ⇡/2 ✓ ◆
2 1 + cos 2t
= 1 cos t sent + ( ) + cos t dt
0 2
 ⇡/2
cos3 t t sen2t
= cos t + + + + sent
3 2 4 0
1 ⇡
= 1 + +1
3 4
5 ⇡
= +
3 4

Problema 5
R
Calcular la integral de lı́nea ! r
(8x + z)dx + 2xz 2 dy 4y 2 dz, siendo !
r la
curva definida por las ecuaciones: z = 9 2x2 4y 2 , z = 1.
Solución
Observemos, que la curva contenida en el plano z = 1, es la elipse
p
2x2 + 4y 2 = 8,con semi ejes a = 2 y b = 2, que se parametriza
mediante. 9
x = p2 cos t = p
y= 2sent t 2 [0, 2⇡] =) ! r (t) = 2 cos t, 2sent, 1 t 2
;
z= 1
[0, 2⇡]
!
Calculemos el campo vectorial F sobre la trayectoria
! !
F (x, y, z) = (8x + z, 2xz 2 , 4y 2 ) =) F (x (t) , y (t)) = (16 cos t + 1, 4 cos 1t, 1)
Evaluemos el vector
! p
r 0 (t) = 2sent, 2 cos t, 0 luego, obtenemos
! p
F (x (t) , y (t)) · !
r 0 (t) = (16 cos t + 1, 4 cos t, 1) · 2sent, 2 cos t, 0
Entonces la integral de lı́nea es

Z Z 2⇡ ⇣ p ⌘
2 2
(8x+z)dx+2xz dy 4y dz = 32sent cos t 2sent + 4 2 cos2 t dt
!
r 0

480
✓ ◆
⇥ 2
⇤2⇡ p Z 2⇡
1 + cos 2t
= 16sen t + 2 cos t 0 +4 2 dt
0 2

p t sen2t 2⇡
= 4 2 +
2 4 0
p
= 4 2⇡

Problema 6
!
Calcular el trabajo producido por campo de fuerzas dado por F = (3x +
4y, 2x+3y 2 ), a lo largo de la circunferencia C de radio 2 centrada en el origen
y recorrida con orientación positiva.

Solución
Definimos el trabajo mediante la integral de lı́nea
Z Z b
! ! ! !
F ·dr = F ( r (t)) · !
r 0 (t) dt
!
r a

Luego, parametrizando la trayectoria tenemos:


!
r (t) = (2cost, 2sent), t 2 [0, 2⇡].
=) ! r 0 (t) = ( 2sent, 2 cos t)
Reemplazando el integrando, queda

I
W = (3x + 4y, 2x + 3y 2 , 0) · (dx, dy, dz)
Z 2⇡
= (6cost + 8sent, 4cost + 12sen2 t, 0) · ( 2sent, 2cost, 0)dt
Z0 2⇡
= [ 16sen2t + 8cos2t]dt
0
= 16⇡ + 8⇡ = 8⇡.

Trabajo negativo significa que el campo de fuerza disipa energı́a.

5.7.1. Campo conservativo


Problemas 1
! !
Sea el campo vectorial F : IR3 ! IR3 dado por F (x, y, z) = (x, y, z).
R ! !
Calcular la integral !
r
F ·dr

481
a) Si C es la circunferencia x2 + y 2 = 4 recorrida en el sentido positivo.
b) Si C es la recta que une P = (1, 0, 0) con Q = (1, 0, 4).
c) Si C es la helicoide !
r (t) = (cos(4⇡t), sen(4⇡t), 4t), t 2 [0, 1] que une
P = (1, 0, 0) con Q = (1, 0, 4).
Solución
a) Las ecuación paramétrica de la circunferencia de radio 2 centrada en
el
origen y recorrida en sentido positivo, es
!r (t) = (2 cos t, 2sent, 0) t 2 [0, 2⇡] =) !
r 0 (t) = ( 2sent, 2 cos t, 0)
Entonces, la integral de linea queda
Z Z
! !
F ·dr = (x, y, z) · (dx, dy, dz)
!
r !
Z r2⇡
= (2 cos t, 2sent, 0) · ( 2sent, 2 cot s, 0) dt
0
Z 2⇡
= 8 cos tsentdt = 0
0

b) La ecuación paramétrica de la recta que une P = (1, 0, 0) con Q =


(1, 0, 4)
es:
! ! ! !
r (t) = P + ( Q P )t = t(1, 0, 4) t 2 [0, 1] =) !r 0 (t) = (1, 0, 4) .
Entonces
Z Z 1
! !
F ·dr = (t, 0, 4t) · (1, 0, 4) dt
!
r 0
Z 1 ⇥ ⇤1
= 16tdt = 8t2 0 = 8
0

c) A partir de la ecuación de la helicoide se obtiene


!r 0 (t) = ( 4⇡sen(4⇡t), 4⇡ cos(4⇡t), 4)
Sustituyendo términos en el integrando, queda
Z Z 1
! !
F ·dr = (cos(4⇡t), sen(4⇡t), 4t) · ( 4⇡sen(4⇡t), 4⇡ cos(4⇡t), 4)dt
!
r 0
Z 1
= ( 8⇡sen(4⇡t) cos(4⇡t) + 4t)dt
0
Z 1 ⇥ ⇤1
= 16tdt = 8t2 0 = 8
0

482
El valor de la integral de lı́nea es el mismo por ambas trayectorias.

Problema 2
R
Calcular la integral ⇣ ! r
2x cos⌘ydx x2 senydy, donde ! r : [1, 2] ! IR2

definida por ! r (t) = et 1 , sen .
t
Solución.
Determinemos si el campo vectorial es conservativo, de modo que calcu-
lamos
i j k
! @ @ @
r⇥ F = = (0, 0, 0)
@x @y @z
2x cos y x2 seny 0
! ! !
Como hallamos que r ⇥ F = 0 , entonces F tiene una función potencial
(x, y) tal que
9
@
(x, y) = 2x cos y > =
@x
@
(x, y) = x2 seny > ;
@y
Integrando la primera ecuación parcialmente con respecto a x, se tiene
@ 0
(x, y) = x2 cos y + h (y) =) (x, y) = x2 seny + h (y) = x2 seny
@y
0
h (y) = 0 () h (y) = c
!
En consecuencia, la función potencial (x, y) para F (x, y) es
(x, y) = x2 cos y + c
Entonces , podemos afirmar que
Z Z
2x cos ydx 2
x senydy = r · d!
r
!
r !
r
= (!
r (2)) (!
r (1))
donde
⇣ ⇡⌘
(!
r (2)) = e, sen = e2 + c
2
(!
r (1)) = (1, sen⇡) = 1 + c
Por tanto. obtenemos:
Z
2x cos ydx x2 senydy = e2 1
!
r

483
Problema 3
!
Considere el campo vectorial F (x, y, z) en IR3 definido por :
✓ ◆
! yz xz xy
F (x, y, z) = , ,
1 + x y z 1 + x y z 1 + x2 y 2 z 2
2 2 2 2 2 2

R yzdx + xzdy + xydx


Evaluar !
r
,donde !
r es:
1 + x2 y 2 z 2
a) el segmento rectı́lineo entre (0, 0, 0) y (1, 1, 1) .
b) la intersección de x2 + y 2 + (z 1)2 = 1, con x2 + y 2 + z 2 = 1.
Solución
Se tiene que las componentes del campo vectorial son continuas
8 (x, y, z) 2 IR3
Primero, verifiquemos si el campo vectorial es conservativo o no.
i j k
! @ @ @
r⇥ F = @x @y @z = (0, 0, 0)
yz xz xy
1 + x2 y 2 z 2 1 + x2 y 2 z 2 1 + x2 y 2 z 2
Puesto que
@ xy @ xz @ yz @ xy
2 2 2
= 2 2 2
, 2 2 2
= , etc.
@y 1 + x y z @z 1 + x y z @z 1 + x y z @x 1 + x2 y 2 z 2
= (0, 0, 0)
! ! !
Como hallamos que r ⇥ F = 0 , entonces F tiene una función potencial
(x, y) tal que:

9
@ yz >
(x, y) = >
@x 1 + x2 y 2 z 2 >
>
>
=
@ xz
(x, y) =
@y 1 + x2 y 2 z 2 >
>
@ xy >
>
(x, y) = >
;
@z 1 + x2 y 2 z 2
Integrando la primera ecuación parcialmente con respecto a x, se tiene

(x, y) = arctg (xyx) + h (y, z) =)


@ xz 0 xz
(x, y) = 2 2 2
+ h (y, z) =
@y 1+x y z 1 + x2 y 2 z 2
0
h (y, z) = 0 () h (y, z) = g (x)

484
!
En consecuencia, la función potencial (x, y) para F (x, y)es

(x, y) = arctg (xyx) + g (z) =)


xy 0 xy
(x, y) = 2 2 2
+ g (x) =
1+x y z 1 + x2 y 2 z 2
0
g (x) = 0 () g (x) = c

Entonces , podemos concluir que

(x, y) = arctg (xyx) + c

En este caso hallamos , que el valor de la integral

Z Z
yzdx + xzdy + xydx
= r · d!r = (1, 1, 1) (0, 0, 0)
!
r 1 + x2 y 2 z 2 !
r
= arctg (1) arctg (0)

=
4
Si !
r es la intersección de dos esferas la curva resultante es cerrada, en

consecuencia
Z Z
yzdx + xzdy + xydx
= r · d!
r =0
!
r 1 + x2 y 2 z 2 !
r

5.7.2. Teorema de Green


Problema 1
!
Verificar el teorema de Green para el campo vectorial F (x, y, z) = (2(x2 + y 2 ), (x + y)2 ) ,donde
las curvas frontera de la región D corresponden al contorno del triángulo con
vértices en los puntos (1, 1) , (2, 2) ,y (1, 3) orientado positivamente.
Solución
!
Como el campo vectorial F (x, y) es de clase C 1 , y la región D conexa,
entonces el teorema de Green afima que:
Z Z Z 
@ @
P dx + Qdy = Q P dxdy
C D @x @y
Identificando términos, tenemos que
@P
P (x, y) = 2(x2 + y 2 ) =) = 4y
@y

485
@Q
Q (x, y) = (x + y)2 =) = 2(x + y)
@x
Entonces calculemos
ZZ ✓ ◆ ZZ
@Q @P
dxdy = 2 (x y) dxdy
D @x @y D

donde D = {(x, y) 2 IR2 : 1  x  2, x  y  4 x} . Luego

ZZ Z 2 Z 4 x
2 (x y) dxdy = 2 (x y) dydx
D 1 x
Z 2  4 x
y2
= xy dx
1 2 x
Z 2
= 2x(4 x) x)2 2x2 + x2 dx
(4
1
Z " #
2 3 2
(x 2)
= 4 (x 2)2 dx = 4
1 3
1
4
=
3

Calculemos directamente la integral de lı́nea, segmentando la frontera


en tres curvas:

Z Z Z
P dx + Qdy = P dx + Qdy + P dx + Qdy
C C1 C2
Z
+ P dx + Qdy
C3

Parametricemos los segmentos de curvas que unen los puntos (1, 1) y


(2, 2) ;
(2, 2) y (1, 3) ; (1, 3) y (1, 1)

Acá debe ir gráfico

Sea C1 la recta y = x, 1  x  2 =) ! r (t) = (t, t) , t 2 [1, 2]


! 0
=) r (t) = (1, 1) , t 2 [1, 2] entonces:

486
Z Z 2 ⇥ ⇤
2 2
2(x + y )dx + (x + y) dy = 2
2 t2 + t2 + (2t)2 dt
C1 1
Z 2  2
2 t3
= 8t dt = 8
1 3 1
56
=
3

Sea C2 la recta y = 4 x, 1  x  2 =) ! r (t) = (4 t, t) , t 2 [2, 3]


! 0
=) r (t) = ( 1, 1) , t 2 [2, 3] , entonces:

Z Z 3 ⇥ ⇤
2 2
2(x + y )dx + (x + y) dy = 2
2 (4 t)2 + t2 ( 1) + (4)2 dt
C2 2
Z 3 ⇥ ⇤
= 8t + 2t2 + 16 dt
2 16
2
Z 3 Z 3
⇥2 ⇤
= 4 t 4t + 4 dt = 4 [t 2]2 dt
2 2
" #3
(t 2)3 4
= 4 =
3 3
2

Sea C3 la recta x = 1, 1  y  3 =) ! r (t) = (1, 3 t) , t 2 [0, 2]


! 0
=) r (t) = (0, 1) , t 2 [0, 2] , entonces:

Z Z 2
2 2 2
2(x + y )dx + (x + y) dy = (4 t)2 ( 1)dt
C3 0
" #2
(4 t)3 8 64 56
= = =
3 3 3 3
0

Por lo tanto, al sumar los tres términos tenemos:


Z
56 4 56 4
2(x2 + y 2 )dx + (x + y)2 dy = =
C 3 3 3 3

Lo que muestra la validez de la formula del teorema de Green.

487
Problema 2
H
Verificar el teorema de Green para C x2 ydx + xy 2 dy, donde C es la
frontera de la región R en el primer cuadrante, limitada por las gráficas de
y = x, y 3 = x2 .
Solución.
Primero, calculemos la integral de lı́nea considerando la orientación pos-
itiva
de la frontera, dividiendola en dos segmentos C1 y = x y C2 y 3 = x2 .
Determinemos los puntos que se intersectan ambas curvas:
y= x
=) x3 = x2 =) x2 (x 1) = 0 () x = 0 y x = 1
y 3 = x2
Luego, ambas curvas se intersectan en los puntos (0, 0) y (1, 1) .En
consecuencia la región R queda delimitada por
R = (x, y) 2 IR2 /0  x  1, x  y  x2/3
Parametrizando el segmento de curva C1 tenemos:
C1 : ! r 1 (t) = (t, t) , t 2 [0, 1] =) ! r 01 (t) = (1, 1)
Calculemos el campo vectorial sobre la curva C1
!!
F ( r 1 (t)) = t3 , t3 =)
!!
F ( r 1 (t)) · !
r 01 (t) = t3 , t3 · (1, 1) = t3 + t3 = 2t3
Para C2 encontramos ✓ ◆
2
C :!
2 r (t) = 1 t, (1
2 t)2/3 , t 2 [0, 1] =) !
r 01 (t) = 1, (1 t) 1/3
3
Luego, la función compuesta para el campo sobre C2 es:
!!
F ( r 2 (t)) = (1 t)8/3 , (1 t)7/3 =)
✓ ◆
!! ! 0 8/3 7/3 2 1/3
F ( r 1 (t)) · r 1 (t) = (1 t) , (1 t) · 1, (1 t)
3
2
= (1 t)8/3 (1 t)2
3
Entonces la integral de lı́nea queda
I Z 1 Z 1
2 2 3 2
x ydx + xy dy = 2t dt ((1 t)8/3 + (1 t)2 )dt
C 0 0 3
 4 11
t 3(1 t)11/3 2(1 t)3
= + +
4 11 9 0
1 3 2
=
4 11 9
1
=
198
488
!
Por otra parte el campo vectorial F (x, y) de clase C 1 , es decir campo con-
tinuo con primera derivada continua,definido en la región R conexa,acotado
por una frontera cerrada, entonces podemos aplicar el teorema de Green que
afima:
I Z Z 
2 2 @ 2 @
x ydx + xy dy = (x y) (xy 2 ) dxdy
C R @x @y
donde R = (x, y) 2 IR2 /0  x  1, x  y  x2/3 ,entonces:

Z Z  Z 1 Z x2/3
@ 2 @ ⇥ ⇤
(x y) (xy 2 ) dxdy = y2 x2 dydx
R @x @y 0 x
Z 1 x2/3
y3
= x2 y dx
0 3 x
Z 1 2
x 2
= 8x8/3 + x3 dx
0 3 3
 3 1
x 3 11/3 x4
= x +
9 11 6 0
1 3 1 1
= + =
9 11 6 198
Con esto,verificamos el teorema de Green en este caso particular.

Problema 3
H y x
Calcule la integral C
dx + 2 dy a lo largo de la curva C
2x2
+ 3y 2 2x + 3y 2
formada por los lados del cuadrado con vértices en (1, 1) , ( 1, 1) , ( 1, 1) , (1, 1) .
Solución.
!
Claramente vemos que el campo vectorial F (x, y) en la región acotada
por
C no es continuo, con primeras derivadas parciales continuas en el origen
(0, 0) .
Luego, vamos a excluir el origen de la región . Dado que la región envuelta
por la curva C, que excluye la singularidad, no es simplemente conexa, se
tiene que: ✓ ◆
R R RR @Q @P
C
P dx + Qdy + C1 P dx + Qdy = D dxdy
@x @y
donde la curva C1 es la elipse con ecuación 2x2 + 3y 2 = r2 , orientada en
el sentido horario, con normal apuntando hacia fuera de la región D.

489
Por otra parte. 9
@Q 3y 2 2x2 >
= >
= ✓ ◆
@x (2x2 + 3y 2 )2 @Q @P
=) =0
@P 3y 2 2x2 >
> @x @y
= ;
@y (2x2 + 3y 2 )2
Entonces, tenemos
Z Z ZZ ✓ ◆
@Q @P
P dx + Qdy + P dx + Qdy = dxdy = 0
C C1 D @x @y

Z Z
=) P dx + Qdy = P dx + Qdy
C C1
Z Z
=) P dx + Qdy = P dx + Qdy
C C1

✓ ◆
r r
Parametrizando C1 como !
r (t) = p cos t, p sent con 0  t  2⇡
2 3
Se obtiene
Z Z 2⇡  ✓ ◆ ✓ ◆
rsent r r r
P dx + Qdy = p p sent + p cos t p cos t dt
C1 0 3r2 2 2r2 3
1
= p (2⇡)
6
r
2
= ⇡
3

Por lo tanto, la integral


Z r
2
P dx + Qdy = ⇡
C 3

Problema 4
Sea C una curva cerrada simple que encierra una región

x2 y 2
D = (x, y) 2 IR2 / + =1
4 5

Calcular el área del interior de la elipse usando el teorema de Green.


Solución.

490
A partir del teorema de Green tenemos
I
1
A (D) = xdy ydx
2
C

Parametricemos la p ecuación de la elipse , mediante


!r (t) = (2cos(t), 5sen(t)), 0  tp 2⇡.
Entonces x(t) = 2cos(t), py(t) = 5sen(t), luego
dx = 2sen(t), dy(t) = 5 cos(t)
Reemplazando términos en el integrando

I Z 2⇡ p p
1 1
xdy ydx = (2 5cos2 (t) + 2 5sen2 (t))dt
2 2 0
C
p Z 2⇡
= 5 dt
0
p
= 2⇡ 5

Problema 5
Considere la región R del plano x2 + (y a)2  a2 ; x2 + y 2 2a2 y usando
el teorema de Green, verifique que el área de dicha región coincide con el área
de un cuadrado de lado a.
Solución
La curva C1 descrita por la ecuación x2 + (y a)2 = a2 ,corresponde a
la circunferencia con centro en (0, a) y radio a y la curva C2 es la

ecuación
p x2 + y 2 = 2a2 de la circunferencia con centro en (0, 0) y radio
a 2.
Calculemos los puntos de intersección de ambas curvas, igualando

ambas ecuaciones, produce 2a2 2ay = 0 =) y = a


Sustituyendo este resultado en la segunda ecuación, obtenemos
x2 = a2 () x = ±a
Por lo tanto, los puntos de intersección de ambaspcurvas son P1 =
p( a, a)
y P2 = (a, a) que tienen coordenadas polares (a 2, 3⇡/4) y (a 2, ⇡/4)
respectivamente.

En consecuencia, la curva cerrada C que forma la frontera de R es la


unión

491
de la curva C1 parametrizada por:

x(t) = acost, y(t) = a + asent, donde t 2 [0, ⇡]

y de la curva C2 parametrizada por


p p
x(t) = a 2cost, y(t) = a 2sent, dondet 2 [⇡/4, 3⇡/4].

El teorema de Green afirma que


I
1
A (R) = xdy ydx
2
C

donde la orientación de C es positiva.


I I
1 1
A (R) = xdy ydx + xdy ydx
2 2
C1 C2

Las orientaciones de C1 y C2 , van en sentido opuesto a los punteros del


reloj,
para que C tenga orientación positiva. Entonces
Z Z
1 ⇡ 2 2 1 ⇡/4 2
A (R) = (a sent + a )dt + (2a )dt
2 0 2 3⇡/4
1⇣ 2 ⇡/4

= a [ cos t + t]⇡0 + 2a2 [t]3⇡/4
2
1
= 2a2 + ⇡a2 ⇡a2 = a2
2
Resultado que verifica que el área de la región R es igual a la de un
cuadrado de lado a.

5.8. Problemas propuestos


5.8.1. Integral de trayectoria
1.- Calcular el área de la superficie lateral cuya base está sobre la curva
!
r del plano XY y la altura en cada (x,y) es z = f (x, y) ,donde:
a) f (x, y) = xy , !r : x2 + y 2 = 1 desde (1, 0) hasta (0, 1) .
b) f (x, y) = h , !r : y = 1 x2 desde (1, 0) hasta (0, 1) .
!
c) f (x, y) = xy, r : y = 1 x2 desde (1, 0) hasta (0, 1) .
Respuestas

492
1
a) A(S) = ,
2
h⇥ p p ⇤
b)A(S) = 2 5 + ln 2 + 5 ,
4
1 ⇥ p ⇤
c)A(S) = 25 5 11
120 R
2.- Evaluar las siguientes
p
integrales de trayectoria ! r
f (x, y, z) ds
z ! !
a) f (x, y, z) = e , r : [0, 1] ! IR dada por r (t) = (1, 2, t2 ) .
3

b) f (x, y, z) = yz, ! r : [1, 3] ! IR3 dada por ! r (t) = (t, 3t,


✓ 2t) . ◆
x+y ! 3 ! 2 3/2
c) f (x, y, z) = , r : [1, 2] ! IR dada por r (t) = t, t , t .
y+z 3
d) f (x, y, z) = z, ! r : [0, 2⇡] ! IR3 dada por ! r (t) = (tcost, tsent, t) .
Respuestas
R pz
a) ! e ds = 2,
Rr p
b) ! r
yzds = 52 14,
R x+y 16 p
c) ! r
ds = 2 3
y+z h 3
R 1 p p i
d) ! zds = (2 + 4⇡ )2 3 2 2
r
3 R
3.- Evaluar las siguientes integrales de trayectoria ! r
f (x, y) ds
2 2/3 2/3
a) f (x, y) = 8y 5x, la curva C dada por x + y = 1.
b) f (x, y) = x + y, siendo C un triángulo de vértices (0, 0), (1, 0) y (0,
1), recorrido en sentido antihorario.
Respuestas
R
a) C (8y 2 5x)ds = 12
R p
b) C (x + y)ds = 1 + 2

5.8.2. Integral de lı́nea


4.-R Evaluar las siguientes integrales de lı́nea:
a) ! r
xdx + ydy + zdz, donde ! r : [ 1, 2] ! IR3 dada por ! r (t) =
2 3
(t , 3t, R2t ) .
b) ! r
2xyzdx + x2 zdy + x2 ydz, donde ! r es una curva simple orientada
que
uneR (1, 1, 1) con (1, 2, 4).
c) ! r
x2 zdx + 6ydy + yz 2 dz, donde ! r : [1, 3] ! IR3 dada por
!r R(t) = (t, t2 , ln t) .
d) C xydx + (x y)dy, donde C esta formado por los segmentos de recta
que van
desde
R (0, 0) a (2, 0) y de (2, 0) a (3, 2) .
2
e) C (x+2)dx+3zdy+y dz,siendo C la curva intersección de las superficies
x2 + y 2 + z 2 = 1; z = x 1

493
Respuestas.
R
a) R!r
xdx + ydy + zdz = 147
b)R ! r
2xyzdx + x2 zdy + x2 ydz = 7
c) ! r
x2 zdx + 6ydy + yz 2 dz = 249, 49
R 17
d) C xydx + (x y)dy =
3 p
R 3 2
e) C (x + 2)dx + 3zdy + y 2 dz = ⇡
4
!
5.- Evaluar la integral de lı́nea de F a lo largo de cada una de las siguientes
trayectorias.
!
a) F (x, y) = (x2 2xy, y 2 2xy),a lo largo de la parábola y = x2 desde
( 1, 1) a (1, 1)
!
b) F (x, y, z) = (x, y, xz y) sobre el segmento de recta desde el punto
(0, 0, 0) hasta el punto
⇣ (1, 2, 4).⌘
!
c) F (x, y) = xx+y x+y 2
2 +y 2 , x2 +y 2 a lo largo de la circunferencia x + y
2
= a2
recorrida en sentido ✓ positivo. ◆
! x y x2 y 2
d) F (x, y) = p 2 2 , p 2 2 sobre el cuarto de elipse 2 + 2 = 1
1+x +y 1+x +y a b
situado en el primer cuadrante, desde el punto (a, 0) hasta el punto (0, b).
!
e) F (x, y, z) = (yz, xz, xy) a lo largo de la hélice ! r (t) = (a cos t, asent, bt)
entre los puntos (a, 0, 0) y (a, 0, 2b⇡).
Respuestas
R ! ! 14
a) ! C
F ·dr =
15
R ! ! 23
b) ! C
F ·dr =
R ! ! 6
c) ! F ·dr =0
RC! ! p p
d) ! C
F · d r = 1 + b2 1 + a2
R ! !
e) ! F · d r = 0
C

5.8.3. Campos conservativos


R
6.- Pruebe que la integral ! C
(6xy 2 y 3 )dx + (6x2 y 3xy 2 )dy ; es inde-
pendiente del camino que une los puntos (1, 2) con (3, 4).
a) Calcule el valor de la integral parametrizando el segmento.
b) Verifique el resultado anterior utilizando la función potencial.
Respuestas
R1!
a) 0 F (r (t)) .!
r 0 (t) dt = 236
!
b) F = r (x, y, z) = 3x2 y 2 xy 3 + C

494
!
7.- ¿Para qué valores de a 2 IR el campo vectorial F (x, y, z) = (axy
z 3 ; (a 2)x2 ; (1 a)xz 2 ) es conservativo? Para esos valores, calcule la función
potencial.
Respuesta
!
Si a=4 la función potencial del campo F es (x; y; z) = 2x2 y xz 3 + C.
!
8.- En los casos en los que F sea conservativo, halle la correspondiente
función potencial.
!
a) F (x, y, z) = (10xz 3 + 1, 6y 2 , 15x2 z 2 ).
!
b) F (x, y, z) = (2xy 3 , x2 z 3 , 3x2 yz 2 ) .
Respuestas
!
a) F es conservativo, función potencial: (x, y, z) = 5x2 z 3 + x 2y 3 + C
!
b) F no es conservativo.
!
9.- Sea F (x, y, z) = (z 3 + 2xy, x2 , 3xz 2 ).
!
a) Muestre que F es un campo conservativo.
!
b) Calcule directamente la integral de F alrededor del contorno del cuadra-
do convértices (±1, ±1) .
c) Determine la función potencial.

5.8.4. Teorema de Green


10.-R Usar el teorema de Green para hallar el valor de la integral de lı́nea:
a) ! r
xy 2 dx + x2 ydy, donde ! r : [0, 2⇡] ! IR2 dada por ! r (t) =
(4 cos Rt, 2sent)
b) ! r
xydx + x2 dy, donde ! r es la frontera que limita la región
2
yR x ,yx
c) C (arctgx + y 2 )dx + (ey x2 )dy, donde C es la frontera que encierra
la
2 2
R 1  x + y2  9, y 0
región
d) C 2xydx + (x + 2x)dy, donde Ces la frontera que limita la región
interior
x2 y 2
a la elipse + = 1 y exterior a la circunferencia x2 + y 2 = 1
H 9 4
e) C (x2 + y 2 ) dx + (2x + y 2 ) dy, donde C es la frontera del cuadrado
con vértices (0, 0) , (k, 0) , (0, k) , (k, k) .

Respuestas
R
a) !r
xy 2 dx + x2 ydy = 0
R 1
b) !r
xydx + x2 dy =
12
495
R 104
c) !
r
(arctgx + y 2 )dx + (ey x2 )dy =
R 3
d)H !
r
2xydx + (x2 + 2x)dy = 10⇡
e) C (x2 + y 2 ) dx + (2x + y 2 ) dy = k 2 (2 k)
y H x
10.- Demuestre que dx + 2
C
dy = 2⇡, donde C es cualquier
+y 2 x2
x + y2
curva cerrada , simple , que incluya el origen

11.- Calcular el área de una elipse con semi ejes a, b usando el teorema de
Green.
Respuesta
A (R) = ⇡ab.
12.- Hallar el área acotada por el arco de hipocicloide x = a (t sent) , y =
a (1 cos t) ,donde a > 0, y 0  t  2⇡, y el eje x.

Respuesta
A (R) = 3⇡a2

5.9. Autoevaluaciones
Autoevaluación No 1
Tiempo 2 horas
Pregunta 1
! !
Sea F (x, y) el campo de fuerzas definido por F (x, y) = (x2 + 7y, x + y sin y 2 ) .Calcular
H! !
F · d r sobre la frontera del triángulo de vértices (0, 2) , (0, 0) y (1, 0) us-
C
ando la definición de integral de lı́nea.
Pregunta 2 H p
Evalue la integral 1 + e x dx + (cos y 2 + x2 ) dy, siendo C una curva
C
cerrada, ubicada en el primer cuadrante, formada por los arcos de
circunferencia de radios 1 y 2 respectivamente y por los segmentos
rectos 1  x  2, 1  y  2.
Pregunta 3
R! !
Calcular la integral I = F · d! r ,si F = (x, y, z) es un campo vectorial
C
definido en R3 C es dada por y 2 + z 2 = 1, x = z entre A = (1, 0, 1) y
B = (0, 1, 0) .

Pauta de Corrección

496
Pregunta 1
La circulación a lo largo de la frontera del triángulo es la suma de las
circulaciones en cada uno de los lados
I Z Z Z
! ! ! ! ! ! ! !
F ·dr = F ·dr + F ·dr + F ·dr
C C1 C2 C3

Parametrizando cada uno de los lados del triángulo se tiene


C1 : y = 0, !r 1 (t) = (t, 0) , t 2 [0, 1] =)
! !
r 1 (t) = (1, 0) y F (!
0
r 1 (t)) = t2 , t

C2 : x) , !
r 2 (t) = ( t, 2 (1 + t)) , t 2 [ 1, 0] =)
y = 2 (1
! ! !
r 2 (t) = ( 1, 2) y F ( r 1 (t)) = t2 + 14 (1 + t) , t + 2 (1 + t) sin (2 (1 + t))2
0

C3x = 0, !
: r 3 (t) = (0, t) , t 2 [ 2, 0] =)
! !
r 03 (t) = (0, 1) y F (! r 1 (t)) = 7t, t sin ( t)2
Sustituyendo términos en los integrandos , obtenemos
I Z 1 Z 0
! ! ⇥ 2 ⇤
F ·dr = 2
t dt + t 14 (1 + t) + 2t + 4 (1 + t) sin (2 (1 + t))2 dt
0 1
C
Z 0
+ t sin t2 dt
2
 " #0
1
t 3
t3 cos (2 (1 + t))2
= + 14t 7t2 + t2
3 0 3 2
1
 2 0
cos t
+ = 8
2 2

Pregunta 2
De acuerdo con el teorema de Green tenemos
I ⇣ p ⌘
ZZ 
x 2 2 @ @ ⇣ p ⌘
1+e dx + cos y + x dy = cos y 2 + x2 1 + e x dxdy
@x @y
C
ZDZ
= 2xdxdy
D

497
Cambiando a coordenadas polares ,queda
ZZ ZZ
2xdxdy = 2 (r cos ✓) rdrd✓
D D⇤
Z ⇡/2 Z 2
= 2 r2 cos ✓ drd✓
0 1
Z ⇡/2  2
r3
= 2 cos ✓d✓
0 3 1
14 14
= [sin ✓]⇡/2
0 =
3 3

Pregunta 3
!
Como el rotacional del campo F :
i j k
! @ @ @
r ⇥ F = @x @y @z = (0, 0, 0) es nulo el campo es conservativo =)
x y z
!
existe (x, y, z) tal que F = r (x, y, z)
@ @ @
Es decir = x, = y, = z. Integrando parcialmente la primera
@x @y @z
x2
componente con respecto a x, obtenemos (x, y, z) = + h (y, z) =)
2
2
@ @h y
(x, y, z) = = y =) h (y, z) = + g (z)
@y @y 2
x2 y 2 @
De modo que (x, y, z) = + + g (z) =) (x, y, z) = g 0 (z) = z
2 2 @z
x2 y 2 z 2
Por tanto, el potencial es (x, y, z) = + +
2 2 2
y la integral de lı́nea queda

Z Z
! !
F ·dr = xdx + ydy + zdz = (B) (A)
C C
1 1
= 1=
2 2

Autoevaluación No 2

Tiempo 2 horas
Pregunta 1.

498
!
Calcular el trabajo realizado por el campo de fuerza F (x, y, z) = (x, y, z)
al mover una partı́cula a lo largo de la parabóla y = x2 , z = x,desde x = 0
hasta x = 2.

Pregunta 2.
!
Verifique el teorema de Green para el campo F : IR2 ! IR2 dado por
!
F (x, y) = ( x2 y, xy 2 ) sobre la región R x2 + y 2  a2 .
Pregunta 3.
R 2 x2 y2
Calcular 2xydx + x dy si C es la curva = 1 entre los puntos
4 9
p C
A = 4, 3 3 y B = (2, 0) .

Pauta de Corrección

Pregunta 1.
!
El trabajo realizado por el campo de fuerza F se define por la
integral de trayectoria
Z
! !
W = F ·dr
C

donde C es la trayectoria, que en este caso se describe por la


ecuación paramétrica
!r (t) = (t, t2 , t) t 2 [0, 2] =) !
r 0 (t) = (1, 2t, 1). Además al evaluar
!
el campo sobre la trayectoria queda F (! r (t)) = (t, t2 , t)
Entonces, el valor del trabajo es
Z 2 Z 2
! !
F ( r (t)) · !
r 0 (t)dt = t + 2t3 + t dt
0 0
= 12

Pregunta 2.
La frontera de R orientada positivamente es la curva C, dada por la
circunferencia x2 + y 2 = a2 , que puede parametrizar como:
x = a cos t, y = a sin t, 0  t  2⇡
Entonces, por una parte se tiene

499
I I
! !
F ·dr = x2 ydx + xy 2 dy
C C
Z 2⇡
= (a4 cos2 t sin2 t + a4 cos2 t sin2 t)dt
Z0 2⇡
= (2a4 cos2 t sin2 t)dt
Z0 2⇡
a4
= sin2 2tdt
0 2
⇡a4
=
2
Por otra parte
Z Z ✓ ◆ Z Z
@(xy 2 ) @ ( x2 y)
dxdy = (y 2 + x2 )dxdy
R @x @y R
Z 2⇡ Z a
= r2 rdrd✓
Z0 2⇡ 0 4 a
r
= d✓
0 4 0
⇡a4
=
2
Pregunta 3
Método 1
Como f (x, y) = 2xy y g (x, y) = x2 verifican condiciones de continuidad
@g @f
y derivabilidad , y = 2x = , entonces existe potencial (x, y) = x2 y+C
@x @y
tal que:
Z
2xydx + x2 dy = (B) (A)
C
⇣ p ⌘
= (0 + C) 48 3 + C
p
= 48 3

Método 2
Parametrizando la curva como x (t) = 2 cosh t, y (t) = 3 sinh t

500
Autoevaluación No 3
Tiempo 2 horas
PreguntaR 1 3
Calcular (2xy + yz)dx + (3x2 y 2 + xz)dy +xydz si C es determinada
C
por
a) !r (t) = (t, t2 , t3 ) desde A = (1, 1, 1) hasta B = (2, 4, 8) .
b) x2 + y 2 + z 2 = 2, z = x2 + y 2 .
Pregunta 2
a) Probar que el área de una región D limitada H por curva regular ( o
curvas regulares por tramos) es dada por A (D) = xdy.
C
b) Aplicar la fórmula anterior para calcular el área de D limitada por las
curvas y = x +2, y = x2
Pregunta H 3
Calcular zdx + xdy + ydz si C es la curva resultante de la intersección
C
de las superficies x2 + y 2 + z 2 = 4, z = 1.

Pauta de Autoevaluación

Pregunta 1
!
Las componentes del campo vectorial F son:
f (x, y, z) = 2xy 3 + yz; g (x, y, z) = 3x2 y 2 + xz; h (x, y, z) = xy
continuas con primeras derivadas continuas
!
Como el rotacional del campo F , tiene derivadas parciales continuas y
i j k
! @ @ @
r⇥ F = @x @y @z = (0, 0, 0) es nulo entonces es
2xy 3 + yz 3x2 y 2 + xz xy
conservativo
!
=) existe (x, y, z) tal que F = r (x, y, z)
@ @ @
Es decir = 2xy 3 + yz, = 3x2 y 2 + xz, = xz.
@x @y @z
De la primera R de estas expresiones se obtiene que
3
(x, y, z) = (2xy + yz)dx + h (y, z)
(x, y, z) = x2 y 3 + xyz + h (y, z) =)
Derivando esta expresión con respecto a y y usando las dos expresiones
restantes para , tenemos
@ @h
(x, y, z) = 3x2 y 3 + xz + = 3x2 y 3 + xz =) h (y, z) = g (z)
@y @y
De modo que (x, y, z) = x2 y 3 + xyz + g (z) =)

501
Derivando esta expresión con respecto a z
@
(x, y, z) = xy + g 0 (z) = xy =) g (z) = C
@z !
Entonces una función potencial para F es (x, y, z) = x2 y 3 + xyz + C
Finalmente la integral
Z
(2xy 3 + yz)dx + (3x2 y 2 + xz)dy + xydz = (B) (A)
C
= 318

b) En este caso
I
(2xy 3 + yz)dx + (3x2 y 2 + xz)dy + xydz = 0
C

pues el campo es conservativo y la curva C es cerrada


Pregunta 2
a) El teorema de Green asegura que
I ZZ 
@Q (x, y) @P (x, y)
P (x, y) dx + Q (x, y) dy = dxdy
@x @y
C D

Sea P (x, y) = 0 y Q (x, y) = x en D ,usando la expresión anterior se


obtiene
I ZZ
xdy = (1) dxdy = A (D)
C D

a) Calculemos la circulación , siendo C = C1 [ C2 la curva que limita a


región D.
Parametrizando las curvas se tiene

C1 : !
r 1 (t) = t, t2 , 1  t  2 =)
2 0
y (t) = t y y (t) = 2t

C2 : ! r 2 (t) = (t, t + 2) , 1  t  2 =)
0
y (t) = t + 2 y y (t) = 1

502
Aplicando la fórmula anterior

I
A (D) = xdy
C
Z 2 Z 1
= t (2t) dt + tdt
1 2
 2  2 1
2 3 t
= t +
3 1 2 2
9
=
2

Pregunta 3
Si C es la curva resultante de la intersección de las superficies x2 +y 2 +z 2 =
4, z = 1 =) x2 + y 2 = 3.
Determinemos la ecuación paramétrica de la trayectoria
⇣p p ⌘
C : ! r (t) = 3 cos t, 3 sin t, 1 , 0  t  2⇡
⇣ p p ⌘
! 0
r (t) = 3 sin t, 3 cos t, 0

Calculemos la integral de lı́nea

I Z 2⇡ p
zdx + xdy + ydz = ( 3 sin t + 3 cos2 t + 0)dt
0
C
Z 2⇡
= 3 cos2 tdt = 3⇡
0

503
Capı́tulo 6

Integrales de superficie

La integral de superficie generaliza la integral doble, tal como la integral


de lı́nea generaliza la integral de Riemann vista en el primer curso de Cálculo.
Para ver el significado de esta integral sigamos el siguiente razonamiento: S
es una superficie definida por

z = g(x, y)

y R la región proyección de S en el plano xy.


Supongamos que g, gx , gy son continuas sobre R. Sea P una partición de
R tal que cada subrectángulo Ri generado por la partición es la proyección
un Si porción de la superficie S generada por la partición P. Sea 4Si el
área de Si
Sea además f : S ! IR una función continua. Definimos en términos
de las sumas intermedias de Riemann la integral de superficie de f sobre S
como ZZ n
X
f (x, y, z)dS = lı́m f (xi , yi , zi )4Si
kpk!0
S i,j=1

Para el cálculo de ésta integral:


Sea S superficie definida por z = g(x, y) y R región proyección de S en
el plano xy.
Suponiendo g, gx , gy son continuas sobre R y f función continua sobre
S, entonces
ZZ ZZ q
f (x, y, z)dS = f (x, y, g(x, y)) 1 + [gx ]2 + [gy ]2 dA
S Rxy

De manera similar.
Si S se expresa de la forma x = g(y, z), (y, z) 2 Ryz entonces

504
ZZ ZZ q
f (x, y, z)dS = f (g(y, z), y, z) 1 + [gy ]2 + [gz ]2 dA
S Ryz

Si S se expresa de la forma y = g(x, z), (x, z) 2 Rxz entonces


ZZ ZZ q
f (x, y, z)dS = f (x, g(x, z), z) 1 + [gx ]2 + [gz ]2 dA
S Rxz

Ejemplo 1:
ZZ
Evalúe xydS S:z=9 x2 , 0  x  2, 0  y  x
S

Figura 6.1:

Solución:

ZZ ZZ p
xydS = xy 1 + 4x2 dA
S Rxy
Z 2 Z x p
= xy 1 + 4x2 dydx
0 0
¡No por aquı́!

505
ZZ ZZ p
xydS = xy 1 + 4x2 dA
S Rxy
Z 2 Z 2 p
= xy 1 + 4x2 dxdy
0 y
p
391 17 + 1
=
240

Observación. ZZ ZZ q
1.- Si f (x, y, z) = 1 =) Área de S = dS. = 1 + [gx ]2 + [gy ]2 dA
S R
2.- Si f (x, y, z) = ⇢(x,
Z Z y, z) densidad de una lámina ,entonces
Masa de lámina = ⇢(x, y, z)dS
S

Ejemplo 2:
Calcule el área lateral del cono z 2 = x2 + y 2 entre los planos z = 1 y
z = 4 usando integral de superficie.

Figura 6.2: Cono z 2 = x2 + y 2 acotado por los planos z = 1 y z = 4

Solución. q
En primer lugar calculemos el integrando 1 + [gx ]2 + [gy ]2 donde
x2 y2
1 + [gx ]2 + [gy ]2 = 1 + 2 + =2
x + y 2 x2 + y 2
entonces ZZ ZZ p
Área de S = dS = 2dA
S R

506
Cambiando a coordenadas polares, se tiene:
ZZ p 
p Z 2⇡ Z 4 p Z 2⇡
r2
4
2dA = 2 rdrd✓ = 2 d✓
0 1 0 2 0
R
p Z 2⇡ p
= 8 2 d✓ = 16 2⇡
0

6.1. Superficie orientada


Decimos que una superficie es orientada si se puede definir un vector
!
normal unitario N en todo punto de la superficie que no pertenezca a la
frontera de forma tal que lo vectores normales varı́an de forma continua
sobre la superficie S.

Figura 6.3: Superficie orientada

Sea S una superficie descrita por z = g(x, y) . Hay dos vectores normales
unitarios a S en (xo , yo , g(xo , y0 )) a saber, si ponemos G(x, y, z) = z g(x, y)
entonces

! rG ! gxbi gyb j+bk


N = o sea N = q vector normal unitario
krGk 2 2
1 + [gx ] + [gy ]

Podemos orientar todas estas superficies tomando el lado positivo de S


!
como el lado desde el cual apunta N . Ası́, el lado positivo de dicha superficie
!
esta determinado por la normal unitaria N con componente positiva b k.
Por otra parte, si ponemos G(x, y, z) = g(x, y) z

507
! rG gxbi + gybj b k
N = =q
krGk
1 + [gx ]2 + [gy ]2
!
Corresponde, al lado negativo de la superficie con normal unitaria N con
componente b
k hacia abajo.

6.1.1. Integral de flujo.


!
Si F es un campo vectorial perteneciente a C 1 definido sobre una super-
b , entonces la integral de superficie de
ficie orientada S, con vector unitario N
!
F sobre S es.
ZZ ZZ
! ! ! b
F · dS = F · N dS
S S

La que se llama integral de flujo. El nombre es debido a su aplicación a un


problema fı́sico de un fluı́do a través de una superficie.
Supongamos que una superficie S está inmersa en un fluido que tiene
un campo de velocidades continuo F.
Entonces
ZZ
! b
Volumen de fluido a través de S = F · N dS
(por unidad de tiempo)
S

S es superficie orientada mediante N b, y !F (x, y, z) = (P (x, y, z), Q(x, y, z), R(x, y, z))
campo vectorial tal que P, Q, R son funciones escalares con primeras derivadas
parciales continuas en S.
Además si ⇢ = ⇢(x, y, z) representa la densidad del fluido en cada (x, y, z)de
S,entonces
ZZ
! b
Masa del fluı́do a través de S = ⇢F · N dS
(por unidad de tiempo)
S

Ejemplo
Sea S la parte del paraboloide z = 4 (x2 + y 2 ) Sobre el plano XY,
orientado hacia arriba. Un fluı́do de densidad constante ⇢ = k pasa a través
de S según el campo de velocidades.
!
F (x, y, z) = (x, y, z)

508
Hallar la razón de flujo de masa a través de S
Solución:

b=p j+b
2xbi + 2yb k
G(x, y, z) = z 4 + x2 + y 2 =) rG = (2x, 2y, 1) =) N
1 + 4x2 + 4y 2
ZZ
! b
Masa del fluı́do a través de S = kF · N dS
S

ZZ ZZ
! b (2x, 2y, 1) p
k F ·N dS = k x, y, 4 x2 + y 2 · p 1 + 4x2 + 4y 2 dA
1 + 4x2 + 4y 2
S Rxy

ZZ ZZ
⇥ 2 2 2 2
⇤ ⇥ ⇤
= k 2x + 2y + 4 x +y dA = k x2 + y 2 + 4 dxdy
Rxy Rxy
Z 2⇡ Z 2 Z 2⇡
= k 4 + r2 rdrd✓ = k 12d✓ = 24k⇡
0 y 0

Por lo tanto

Masa del fluido a través de S = 24k⇡


(por unidad de tiempo)

En base a lo observado en el desarrollo de este ejercicio se tiene el siguiente


planteamiento:
”Si S es una superficie definida por z = g(x, y) y Rxy denota la región
proyección de S en el plano xy, entonces
ZZ ZZ
! b !
F · N dS = F · ( gxbi gyb j+b k)dA
S Rxy

orientada hacia arriba, y


ZZ ZZ
! b ! b
F · N dS = F · (gxbi + gyb
j k)dA
S Rxy

orientada hacia abajo.

509
6.1.2. Superficies Parametrizadas.
En el tratamiento anterior de este tema hemos supuesto que una superficie
viene definida por la ecuación z = g(x, y) , sin embargo se debe aclarar que
hay superficies que no se pueden definir de esta forma, extenderemos la idea
definiendo la forma paramétrica de dar una superficie.
!
Una superficie parametrizada es una función : D ✓ IR2 ! IR3 donde
!
D es alguna dominio de IR2 La superficie S que corresponde a es su
!
imagen , es decir S = (D) Podemos escribir
!
(u, v) = (x (u, v) , y (u, v) , z (u, v)) , (u, v) 2 D

Véase figura 6.4 Si cada una de las funciones componentes son diferenciables
de clase C 1 , entonces llamamos a S una superficie diferenciable o de clase
C 1.

Figura 6.4: Superficie parametrizada

6.1.3. Vector normal a S :


!
Definimos en cada punto (uo , vo )

! @x(u0 , v0) b @y(u0 , v0) b @z(u0 , v0) b


Tu= i+ j+ k
@u @u @u
es vector tangente a la curva t ! (t, v0 ) en (uo , vo )

! @x(u0 , v0) b @y(u0 , v0) b @z(u0 , v0) b


Tv= i+ j+ k
@v @v @v
es vector tangente a curva t ! (uo , t) en (uo , vo )
! !
Tu⇥ Tv es vector normal a la superficie S.

510
Ahora el concepto de superficie suave se da en la siguiente definición.
! ! !
Decimos que la superficie S es suave en (uo , vo ) si T u ⇥ Tv 6= 0 en
(uo , vo ) 2 D. Diremos que una superficie es suave si es suave en todos los
!
puntos (uo , vo ) 2 S.

Ejemplo 1
Sea S la superficie descrita por x2 + y 2 + z 2 = r2 , z 0
a) Parametrizar usando coordenadas esféricas.
b) Determine el vector normal a la superficie S.

Figura 6.5: semisfera de ecuación x2 + y 2 + z 2 = r2 , z 0

Solución:
a) La superficie x2 + y 2 + z 2 = r2 , z 0 corresponde a una semiesfera con
centro el el origen (0, 0, 0) radio r.
Usando coordenadas9esféricas tenemos:
x = r sin u cos v =
y = r sin u sin v
;
z = r cos u
Luego la superficie parametrizada es
!
(u, v) = (r sin u cos v, r sin u sin v, r cos u) , (u, v) 2 D
n ⇡ o
donde D = (u, v) /0  u  , 0  v  2⇡ .
2
b) Determinemos los vectores tangentes a la superficie S
! !
T u = u = (r cos u cos v, r cos u sin v, r sin u)
! !
T v = v = ( r sin u sin v, r sin u cos v, 0)
Calculemos el producto vectorial
i j k
! !
T u ⇥ T v = r cos u cos v r cos u sin v r sin u
r sin u sin v r sin u cos v 0

511
Por lo tanto, el vector normal a la esfera es
! !
T u ⇥ T v = r2 sin2 u cos v, sin2 u sin v, sin u cos v .

Ejemplo 2
Sea S la superficie descrita por z 2 = x2 + y 2 , z 0.
a) Parametrizar usando coordenadas cilı́ndricas.
b)¿Es diferenciable ésta superficie?
c) Probar que no es suave en (0, 0, 0).

Figura 6.6: cono con vértice en el origen

Solución:
a) La superficie z 2 = x2 + y 2 , z 0 corresponde a un cono su vértice en
(0, 0, 0) .
Usando coordenadas
9 cilı́ndricas tenemos:
x = u cos v =
y = u sin v =) z 2 = x2 + y 2 = u2 cos2 u + sin2 v
;
z= u
() z = u, u 0
Luego la superficie parametrizada queda

!
(u, v) = (u cos v, u sin v, u) , u 0

b) Como cada una de las componentes es diferenciable como función de


u, v
entonces S es una superficie diferenciable.
c) Veamos si es o no una superficie suave

512
! !
T u = (cos v, sin v, 1) =) T u (0, 0) = (cos 0, sin 0, 1)
!
) T u (0, 0) = (1, 0, 1)

! !
T v = ( u sin v, u cos v, 0) =) T v (0, 0) = (0 ( sin 0) , 0 cos 0, 0)
! !
) T v = (0, 0, 0) = 0
! ! !
Ası́ Tu ⇥ T v = 0 , de modo que la superficie no es suave en (0, 0, 0).
Para efectos de cálculo, consideraremos sólo superficies suaves por
!
pedazos que sean uniones de imágenes parametrizadas i : Di ✓ IR2 !
IR3 ,para las cuales:
i) Di es una región elemetal del plano.
!
ii) i es de clase C1 y uno a uno, excepto quizás en la frontera D.
!
iii) Si es la imagen de i es suave , excepto quizás en un número finito
de
puntos.

6.1.4. Área de una superficie parametrizada


Definimos el área de una superficie parametrizada S , denotada por A (S) ,mediante:
ZZ
! !
A (S) = Tu ⇥ T v dudv
D

! ! ! !
donde Tu ⇥ T v es la norma del vector normal T u ⇥ T v .

Propiedades 6.1.1. i) Si S es la unión de superficies parametrizadas S =


[ Si entonces
X
A (S) = A (Si )
i=1
s  
2 2 2
! ! @ (x, y) @ (y, z) @ (z, x)
ii) Tu ⇥ T v = + +
@ (u, v) @ (u, v) @ (u, v)

513
Figura 6.7: Área de una superficie parametrizada

Figura 6.8: manto del paraboloide z = x2 + y 2 , 0  z  a2

Ejemplo 1
Determine el área del manto del paraboloide z = x2 + y 2 , 0  z  a2 .
Solución:
Usando coordenadas cilı́ndricas tenemos
!
(u, v) = u cos v, u sin v, u2 , (u, v) 2 D
donde D = {(u, v) /0  u  a, 0  v  2⇡}

Como el área del manto está dado por


ZZ
! !
A (S) = Tu ⇥ T v dudv
D

514
Calculemos el integrando
@ (x, y) cos v u sin v
= = u,
@ (u, v) sin v u cos v
@ (y, z) sin v u cos v
= = 2u2 cos v
@ (u, v) 2u 0
@ (z, x) 2u 0
= = 2u2 sin v
@ (u, v) cos v u sin v
Ası́ el integrando del área es
! ! p p
Tu ⇥ T v = u2 + 4u4 cos2 v + 4u4 sin2 v = u 1 + 4u2
! ! !
Es evidente que Tu ⇥ T v se anula para u = 0 y (0, v) = (0, 0, 0)
para
cualquier v. Luego (0, 0, 0) es el único punto donde la superficie no es
suave.
Entonces
ZZ Z 2⇡ Z a p
! !
Tu ⇥ T v dudv = u 1 + 4u2 dudv
D 0 0
Z 2⇡ h i
1 3/2 a
= 1 + 4u2 dv
12 0 0
⇡h 3/2
i
= 1 + 4a2 1
6

Área de una superficie en coordenadas cartesianas.


Una susperficie S dada de la forma z = g (x, y) donde (x, y) 2 D, admite
la parametrización
!
(x, y) = (x, y, g (x, y)) , (x, y) 2 D
C 1 , esta parametrización
Si f es✓de clase ◆ ✓ ◆ es suave, entonces
✓ ◆
! @g ! @g ! ! @g @g
T x = 1, 0, y T y = 0, 1, =) T x ⇥ T y = , ,1
@x @y @x @y
Entonces: s✓ ◆
2 ✓ ◆2
! ! @g @g
Tx⇥ Ty = + + 1.
@x @y
De este modo la fórmula para el área de la superficie se reduce a
ZZ
! !
A (S) = Tx ⇥ T y dxdy
D
s✓ ◆ ✓ ◆2
ZZ 2
@g @g
= + + 1 dxdy
D @x @y

515
Resultado que coincide con lo propuesto en la primera parte.

6.1.5. Integral de una función escalar sobre una su-


perficie.
Si f : D ⇢ IR3 ! IR,dada por f (x, y, z) es una función continua definida
sobre una superficie parametrizada S, definimos, la integral de f sobre S como:
Z Z ZZ ⇣! ⌘ ! !
f dS = f (u, v) Tu ⇥ T v dudv
S D

Si S es la unión de superficies parametrizadas S = [ Si entonces


Z Z XZ Z
f dS = f dS
S i=1 Si

Ejemplo
Z Z
Evaluar z dS, donde S es el hemisferio superior de radio r de la
p S
esfera z = r2 x2 y 2 .
Solución.
Para este problema es conveniente representar el hemisferio
parametricamente por:
!
(u, v) = (r sin u cos v, r sin u sin v, r cos u) , (u, v) 2 D

sobrenla región D del plano uv dada omediante



D = (u, v) /0  u  , 0  v  2⇡
2
De las ecuaciones paramétricas obtenemos, los vectores tangentes a la
superficie S.
! !
T u = u = (r cos u cos v, r cos u sin v, r sin u)
! !
T v = v = ( r sin u sin v, r sin u cos v, 0)
Calculemos el producto vectorial
i j k
! !
T u ⇥ T v = r cos u cos v r cos u sin v r sin u
r sin u sin v r sin u cos v 0
Luego, el vector normal a la esfera es

516
! !
Tu⇥ Tv = r2 sin2 u cos v, sin2 u sin v, sin u cos v
! !
=) T u ⇥ T v = r2 sin u

De modo que

Z Z Z 2⇡ Z ⇡/2
z dS = (r cos u) r2 sin u dudv
S 0 0
Z 2⇡ Z ⇡/2
= r3 cos u sin ududv
0 0
Z 2⇡  ⇡/2
3 sin2 u
= r dv
0 2 0
= ⇡r3

6.1.6. Integral de Superficie de campos vectoriales


Sean S una superficie regular orientable, N b : S ! IR3 un campo de
!
normales continuo sobre S, y F : ⌦ ✓ IR ! IR3 un campo vectorial
3

continuo definido sobre un abierto que contiene a S. Definimos la integral


! b mediante
de F sobre la superficie S orientada según N
ZZ ZZ
! b ! ⇣! ! ⌘
F · N dS = F · u⇥ v dudv
! D

!
donde : D ✓ IR2 ! IR3 es una parametrización regular de S compatible
con la orientación, esto es, tal que
! !
b u⇥ v
N= ! !
u⇥ v

Para interpretar correctamente el valor de la integral de flujo es necesario


especificar el campo de normales N b . Por ejemplo, si orientamos un casquete
esférico usando la normal exterior N b = rb, la integral de flujo corresponde
al flujo neto que sale de la esfera a través del casquete. Si este valor fuese
negativo, significa que lo que el flujo entra a la esfera.

517
Ejemplo 1:

Sea S es la esfera unitaria x2 + y 2 + z 2 = 1 parametrizada en coordenadas


!
esféricas por ( , ✓) = (sin cos ✓, sin sin ✓, cos ) , 0   ⇡, 0  ✓  2⇡.
! RR !
Sea F (x, y, z) = (x, y, z) radio vector del origen. Calcular F ·Nb dS
!

Solución:
En este caso D es región del plano ✓ definida por: 0   ⇡, 0  ✓ 
2⇡.
!
T = (cos cos ✓, cos sen✓, sin )

!
T ✓ = ( sin sin ✓, sin cos ✓, 0)

! !
T ⇥ T✓= cos sin2 ✓ bi j + (cos sin ✓) b
sin sin2 ✓ b k

Notese que el vector normal apunta hacia el lado positivo de la superficie

! ⇣! !⌘
F · T ⇥ T ✓ = sin

ZZ ZZ
! b
F · N dS = sin d d✓
! D
Z 2⇡ Z ⇡
= sin d d✓
0 0
= 4⇡

Observación
b
Si S es una superficie regular orientada según un campo de normales N
b
y si S es la misma superficie pero con la orientación opuesta N , entonces
se tiene que
ZZ ZZ
! ! ! !
F · dS = F · dS
S S

518
6.1.7. Aplicación al campo de la fı́sica:
Calcular el flujo del campo eléctrico producido por una carga Q en el
origen, a través del manto de la esfera S de radio R orientado según la normal
exterior.
El campo eléctrico producido por la carga Q viene dado por

! Q rb
E =
4⇡"o r2
donde "o una constante universal . De esta forma se obtiene el siguiente
flujo
eléctrico
ZZ ZZ
! ! Q rb
E · dS = · rbdS
4⇡"o R2
S S

Parametrizando la superficie en coordenadas esféricas se tiene


!
( , ✓) = (R sin cos ✓, R sin sin ✓, R cos ) , 0   ⇡, 0  ✓  2⇡
! !
=) ⇥ ✓ = R2 sin

ZZ Z 2⇡ Z ⇡
Q rb Q 1 2
· rbdS = R sin d d✓
4⇡"o R2 0 0 4⇡"o R2
S
Q
=
"o

6.2. Teoremas de Gauss y de Stokes

Ahora dos teoremas de gran importancia en el campo de las aplicaciones,


particularmente en la fı́sica, el primero de ellos es el teorema de Gauss o
teorema de la divergencia.

6.2.1. Divergencia
!
En IR3 ,sea F un campo vectorial,anteriormente hemos definimos

! ! @P @Q @R
div F = r · F = + +
@x @y @z

519
Ejemplo.
! 2
Sea F (x, y, z) = (xy, y 2 +exz , sen(xy)), calcule la divergencia del campo
vectorial 2
@xy @(y 2 + exz ) @sen(xy)
divF = + + = 3y
@x @y @z
El foco de atención lo ponemos ahora en regiones R sólidas simples (
por ejemplo cajas rectángulares, esferas, elipsoides etc). La frontera S es
una superficie cerrada y convenimos que la orientación positiva es hacia
afuera, representada por el vector normal N b hacia afuera de la superficie.
En este contexto enunciamos el teorema.

6.2.2. Teorema de la divergencia de Gauss.


Sea R una región simple sólida limitada por una superficie cerrada S
!
orientada por un vector normal unitario dirigido al exterior de R. Si F es
un campo vectorial cuyas funciones componentes P, Q y R tienen derivadas
parciales continuas en la región R, entonces
ZZ ZZZ
! b !
F · N dS = div F dV
S R

Demostración.
Sólo daremos algunas ideas generales para indicar un procedimiento para
la
demostración
!
Sea F (x, y, z) = (P (x, y, z), Q(x, y, z), R(x, y, z))
! ! @P @Q @R
=) div F = r · F = + +
@x @y @z
Integrando
ZZZ ZZZ ZZZ ZZZ
! @P @Q @R
div F dV = dV + dV + dV
@x @y @z
R R R R

b es el normal unitario de S hacia afuera la integral de


Por otro lado, si N
superficie es
ZZ ZZ ZZ ZZ ZZ
! ! ! b
F · dS = F · N dS = b b
P i · N dS + b b
Qj · N dS + Rbk·Nb dS
S S S S S

520
Siguiendo el procedimiento de la demostración del teorema de Green basta
probar las siguientes tres igualdades:

ZZZ ZZ
@P b dS
dV = Pbi · N
@x
Z ZRZ Z SZ
@Q b dS
dV = Qb
j·N
@y
Z Z ZR Z SZ
@R
dV = Rb b dS
k·N
@z
R S

Detalles y comentarios los puede consultar en el texto Çálculo de James


Stewart, Tercera edición (Thomson), página 935”

Ejemplo 1:
ZZ
2 2 2
Sea S esfera unitaria x +y +z = 1. Demostrar que (xy + yz + xz) dS =
S
0
Solución:
j + zb
Claramente xbi + yb k es vector normal a la esfera en todo (x, y, z) .

b = p j + zb
xbi + yb k
N
x2 + y 2 + z 2
= xbi + ybj + zbk, ya que x2 + y 2 + z 2 = 1

consideramos ahora el siguiente arreglo para identificar

xy + yz + xz = (y, z, x) · (x, y, z)
!
= (y, z, x) · N
!
=) F = (y, z, x)

! @P @Q @R
r· F = + + =0+0+0=0
@x @y @z
Aplicando Teorema de Divergencia

521
ZZ ZZZ
!
(xy + yz + xz) dS = r · F dV
S
Z ZRZ
= 0dV = 0
R

Ejemplo 2
ZZ ZZZ
! b !
Verificar el teorema de la divergencia F · N dS = div F dV para
S R
!
F (x, y, z) = (2x y)bi (2y j + zb
z)b k S: limitado por los planos
coordenados
y x + 2y + z = 6

Figura 6.9: S limitado por los planos coordenados y x + 2y + z = 6

Solución.
En este caso S = S1 + S2 + S3 +S4 donde S1 será el lado de la figura en
el plano x + 2y + z = 6, S2 el lado de la figura en el plano coordenado
yz, S3 el lado de la figura en el plano coordenado xz, S4 el lado
correspondiente al plano coordenado xy.
Tenemos
ZZ ZZ ZZ ZZ ZZ
! b ! b ! b ! b ! b
F · N dS = F · N1 dS + F · N2 dS + F · N3 dS + + F · N4 dS
S S1 S2 S3 S4

522
ZZ ZZ
! b !
F · N1 dS = F (x, y, z) · ( gx (x, y), gy (x, y), 1)dA
S1 Rxy
ZZ ZZ
!
= F (x, y, z) · (1, 2, 1)dA = (2x y, (2y z), z) · (1, 2, 1)dA
Rxy Rxy
ZZ ZZ
= ((2x y) 2(2y z) + z)dA = (2x 5y + 3(6 x 2y)dxdy
Rxy Rxy
Z 6Z 1
x+3 Z 6
2 7 2 9 9
= ( x 11y + 18)dydx = ( x + x + )dx = 45
0 0 0 8 2 2

ZZ ZZ ZZ
! b !
F · N2 dS = F · ( bi)dS = (y 2x)dS
S2 S S2
Z Z2
= (y 2(6 2y z)dydz
Ryz
Z 3 Z 6 2y
= (5y + 2z 12) dzdy = 27
0 0

ZZ ZZ
! b !
F · NdS = F ·( b
j)dS
S3 S
Z Z3 ZZ
6 x z
= (2y z)dS = (2( ) z)dxdz
2
S3 Rxz
Z 6 Z 6 x
= (6 x 2z) dzdx = 0
0 0

ZZ ZZ
! b !
F · N4 dS = F ·( b
k)dS
S4 S
Z Z4
= 0dxdy = 0
Rxy

De estos cálculos se tiene que


ZZ
! b
F · N dS = 45 27 + 0 + 0 = 18
S

523
!
Por el otro lado div F = 2 2 + 1 = 1, la integral triple es

ZZZ ZZZ Z 6 Z x
+3 Z 6 x 2y
! 2
div F dV = dV = dzdydx
0 0 0
R R
Z 6Z x
+3 Z 6 ✓ ◆
2 x2
= (6 x 2y)dydx = 3x + 9 dx = 18
0 0 0 4

6.2.3. Teorema de Stokes.


Este teorema da la relación entre una integral de superficie sobre una
superficie orientada S y una integral de lı́nea a lo largo e una curva cerrada
siempre seccionalmente suave C con orientación positiva que acota a S.

Teorema 6.2.1. Sea S una superficie limitada por una curva C ,orienta-
!
da positivamente, seccionalmente suave. Si F es un campo vectorial cuyas
componentes tienen derivadas parciales continuas en una región abierta que
contiene a S y C, entonces.
I ZZ ⇣ ZZ ⇣
! ! !⌘ ! !⌘ !
F ·ds = rot F · d S = r ⇥ F · dS
C S S

Recuerde que la integral de lı́nea se puede expresar:


Z Z
!
P dx + Qdy + Rdz o F · Tbds
C C

Ejemplo 1
I
! ! !
Use el teorema de Stokes para evaluar F · d s , donde F (x, y, z) =
C
C
(3z, 5x, 2y) y C es la intersección del plano z = y + 3 con el cilindro
x2 + y 2 = 1. Oriente la elipse en sentido contrario al movimiento de las
agujas del reloj, vista desde arriba.
Solución.
La intersección del plano con el cilindro permite visualizar que S en este
caso es la porción del plano que queda al interior del cilindro y que C es la
elipse intersección

524
I ZZ ⇣
! ! !⌘ b
F ·ds = r⇥ F ·N dS
C S

bi b
j b
k
!
r⇥ F = @
@x
@
@y
@
@z
= ( 2bi + 3bi + 5b
k)
3z 5x 2y

z y+3=0 b = p1 ( b
=) N j + k)
2
Calculando
⇣ !⌘ b 1
r⇥ F ·N = ( 2bi + 3bi + 5b
k) · p ( b
j + k)
2
2 p
= p = 2
2
y entonces la integral será
I ZZ p p ZZ
! !
F ·ds = 2dS = 2 dS
C S S
p p p
= 2 · Area(S) =
2 · (1 · 2⇡) = 2⇡
p
S es el interior de una elipse de semi ejes 1 y 2.

6.3. Problemas Resueltos


Problema 1
Sea la función !r : D ! IR2 , definida por !
r (r, ✓) = (r cos ✓, r senv, r)
donde D = {(r, ✓) /0  r  1, 0  ✓  2⇡} una parametrización de un cono
S. Hallar su área de superficie.

Solución Z Z
El área de una superficie parametrizada se define A(S) = k!
r u⇥!
r v k dudv
D
Calculemos el integrando
s de área usando la propiedad:
2  2  2
! ! @ (x, y) @ (y, z) @ (z, x)
k r r ⇥ r ✓k = + +
@ (r, ✓) @ (r, ✓) @ (r, ✓)
siendo los Jacobianos
@ (x, y) cos ✓ rsen✓
= =r
@ (r, ✓) sen✓ r cos ✓

525
Figura 6.10: Representación paramétrica de un cono

@ (y, z) sen✓ r cos ✓


= = r cos ✓
@ (r, ✓) 1 0
@ (z, x) 1 0
= = rsen✓
@ (r, ✓) cos ✓ rsen✓
Ası́ que el integrando
p de área es p
! !
k r r ⇥ r ✓ k = r + r2 cos2 ✓ + r2 sen2 ✓ = r 2
2

Por tanto,el área de la superficie S es

Z Z Z 2⇡ Z 1 p
A(S) = k!r r⇥!r ✓k dudv = 2rdrd✓
D 0 0
Z 2⇡ p  2 1
r
= 2 d✓
0 2 0
p Z 2⇡
2
= d✓
2
p 0
= 2⇡

Observación, k! r r⇥! r ✓kse anula para r = 0,pero !r (0, ✓) = (0, 0, 0) 8✓.


Ası́, (0, 0, 0) es el único punto donde la superficie no es suave.

Problema 2

Calcular el área S de la región del cono definido por x2 = y 2 + z 2 , interior


al cilindro x2 + y 2 = a2 , acotado en el octante x 0, y 0, z 0.
Solución
Parametrizando la ecuación del cono, usando coordenadas cartesianas
(y, z),se
tiene que

526
Figura 6.11: región del cono definido por x2 = y 2 +z 2 interior al cilindro x2 +y 2 =
a2 , acotado en el octante el primer octante

⇣p ⌘
!
r (y, z) = 2 2
y + z , y, z , (y, z) 2 D
La superficie S está al interior del cilindro , por lo que

x2 + y 2 = 2y 2 + z 2  a2 .

Entonces, la región D esta definida sobre el plano yz por


D = {(y, z) 2 IR2 /2y 2 + z 2  a2 , y 0, z 0} .
El área de la superficie es:
Z Z
A(S) = k!
r y⇥! r z k dydz
D

luego ,si

! !
! y z
ry = p , 1, 0 , !p
rz= , 0, 1 =)
y2 + z2 y2 + z2
! y z
r y⇥!
rz = (1, p , p )
2
y +z 2 y + z2
2
p
=) k!r ⇥!
y r k= 2
z

Z Z p
A(S) = 2dydz
D

Observemos que: 2y 2 + z 2  a2 , y 0, z 0 ()

527
y2 z2
✓ ◆2 +  1, y
0, z 0 es el área de la cuarta parte de la elipse
a a2
p
2
Usando cambio de variables tenemos:
a a a
x = p r cos ✓ @(x,y) p cos ✓ p rsen✓ a2
2 =) @(r,✓) = 2 2 =p r
y = arsen✓ asen✓ ar cos ✓ 2

y2 z2
✓ ◆2 +  1 () 0  r2  1 =) 0  r  1 y 0  ✓  ⇡
a a2
p
2
Entonces la región D⇤ = {(r, ✓) /0  r  1 y 0  ✓  ⇡} .Luego, si

Z Z p p Z Z a2
A(S) = 2dydz = 2 p rdrd✓
D D⇤ 2
Z ⇡/2 Z 1 Z ⇡/2 2 1
r
= a2 rdrd✓ = a2 d✓
0 0 0 2 0
⇡a2
=
4
Problema 3
Calcular el área S de la región del manto del cilindro x2 + y 2 = 2y
comprendida entre y + z = 2 y z = 0.

Figura 6.12: Región del manto del cilindro x2 + y 2 = 2y comprendida entre


y + z = 2 y z = 0. D, parametrización de esta región

Solución
El área de la superficie es:
Z Z
A(S) = k!
r u⇥!
r v k dudv
D

528
Parametrizando la superficie del cilindro, x2 +y 2 = 2y () x2 +(y 1)2 =
1
Usando coordenadas cilindricas (u, v),se tiene que

!
r (u, v) = (cos u, 1 + senu, v) , (u, v) 2 D

donde D = {(u, v) /0  u  2⇡, 0  v  2 (1 + senu)}


Determinemos el integrando de área

i j k
!
r u⇥!
rv = senu cos u 0 = (cos u, senu, 0)
0 0 1
=) ! !
k r u ⇥ r vk = 1

Por tanto,el área de la superficie S es

Z Z
A(S) = dudv
D
Z 2⇡ Z 1 senu
= dvdu
0 0
Z 2⇡
= (1 senu) du
0
= 2⇡

6.3.1. Integrales de superficie


Problema 1
Sea S el octante positivo de la superficie x2 + y 2 + z 2 = 1.Calcular la
RR 1
integral de superficie S
p dS
x2 + y 2 + (z 1)2
Solución
Parametricemos la Superficie S usando coordenadas esféricas, es decir

!
r (u, v) = (senu cos v, senu senv, cos u)

onde D⇤ = {(u, v) /0  u  ⇡/2, 0  v  ⇡/2}

Calculemos el valor del integrando sobre S

529
1 1
p = p
x2 + y 2 + (z 1)2 sen2 u cos2 v + sen2 u sen2 v + (cos2 u 1)2
1
= p
sen2 u + cos2 u + 1 2 cos u
1
= p
2 2 cos u

Calculemos el producto vectorial

i j k
!
r u⇥!
rv = cos u cos v cos u senv senu
senu senv senu cos v 0
= sen2 u cos v, sen2 u senv, senu cos v

Luego , obtenemos la norma del producto :


k!r u⇥!
1/2
r v k = (sen4 u + sen2 u cos2 u) = |senu| = senu porque 0 
u  ⇡/2
Finalmente, apliquemos la definición de integral de superficie y calculemos

Z Z Z Z
1 ⇡/2 ⇡/2
k!
r ⇥! r vk
p dS = p u dudv
S x2 + y 2 + (z 1)2 0 0 2 2 cos u
Z ⇡/2 Z ⇡/2
senu
= dudv p
0 0 2 2 cos u
Z ⇡/2
1 ⇥ p ⇤⇡/2
= p 2 1 cos u 0 dv
2 0
Z ⇡/2
2
= p dv
2 0

= p
2
ProblemaZZ 2
Evaluar z 2 ds, donde S es el paraboloide z = x2 + y 2 , comprendido
S
entre z = 1 y z = 4.
Solución
Para este problema tenemos dos opciones para representar parametrica-
mente la corona del paraboloide.

530
Figura 6.13:

Usando coordenadas cartesianas rectángulares


!
r (x, y) = (x, y, x2 + y 2 ) , donde D = {(x, y) 2 IR2 /1  x2 + y 2  4}
Calculemos el vector normal a la superficie S, dado por
i j k
!
r x⇥!
ry= 1 0 2x = (2x, 2y, 1)
0 1 2y
Luego , obtenemos la norma del producto :
p
k!
r ⇥!
x r k = 4x2 + 4y 2 + 1
y

Aplicando la definición de integral de superficie

ZZ Z Z
2
z ds = z 2 k!
r y⇥!
r z k dxdy
S Z ZD p
= (x2 + y 2 ) 4x2 + 4y 2 + 1dxdy
D

Usando coordenadas polares para evaluar la integral doble,

531
Z Z p Z 2⇡ Z 2 p
2 2
(x + y ) 4x2 + 4y 2 + 1dxdy = r2 4r2 + 1rdrd✓
D 0
Z "1 Z #
2⇡ 2 2
1 3/2 2 3/2
= r2 4r2 + 1 r 4r2 + 1 dr d✓
0 12 1 12 1
Z " #
2⇡ 2 2
1 3/2 1 5/2
= r2 4r2 + 1 4r2 + 1 d✓
0 12 1 120 1

⇡ 1 5/2 1
= 173/2 53/2 17 + 53/2
6 10 10

6.3.2. Integral de Flujo de un campo vectorial


Problema 1
!
Calcular el flujo para F (x, y, z) = (3x, 3y, z) en la superficie z = 9
x2 y 2 tal que z 0

Figura 6.14:

Solución
Usando coordenadas cartesianas rectangulares, la parametrización de la
superficie S es
!r (x, y) = (x, y, 9 x2 y 2 ) , donde D = {(x, y) 2 IR2 /x2 + y 2  9, z 0}
Calculemos el vector normal a la superficie S, dado por
i j k
!
r x⇥!
ry= 1 0 2x = (2x, 2y, 1)
0 1 2y
Aplicando la definición de integral de superficie al flujo a través de S, se
obtiene

532
ZZ ZZ
! ! !
F ·n
bds = F ( r (x, y)) · (!
r x⇥!
r y ) dxdy
S Z ZD
(3x, 3y, z) · (2x, 2y, 1)dxdy
Z ZD
= 6x2 + 6y 2 + z dxdy
Z ZD
= 6x2 + 6y 2 + 9 x2 y 2 dxdy
Z ZD
= 5x2 + 5y 2 + 9 dxdy
D

Cambiando a coordenadas polares

ZZ Z 2⇡ Z 3
2 2
5x + 5y + 9 dxdy = 5r2 + 9 rdrd✓
D 0 0
567⇡
=
2

Alternativamente, podemos parametrizar directamente la superficie S en


coordenadas cilı́ndricas.

!
r (r, ✓) = r cos ✓, rsen✓, 9 r2 , 0  r  3, 0  ✓  2⇡

Calculemos el vector normal a la superficie y el campo vectorial sobre la


superficie:

i j k
!
n = !
r r⇥!
r✓= cos ✓ sen✓ 2r
rsen✓ r cos ✓ 0
= = 2r2 cos ✓, 2r2 sen✓, r

Aplicando la definición de integral de superficie al flujo sobre S, se obtiene

533
ZZ ZZ
! ! !
F ·n
bds = F ( r (r, ✓)) · (!
r r⇥!
r ✓ ) drd✓
S Z ZD
= 3r cos ✓, 3 rsen✓, 9 r2 · 2r2 cos ✓, 2r2 sen✓, r drd✓
Z ZD
= 6r3 (cos2 ✓ + sen2 ✓) + 9r r3 drd✓
Z ZD
= 5r3 + 9r drd✓
D
Z 2⇡ Z 3
567⇡
= 5r3 + 9r drd✓ =
0 0 2

Problema 2
! !
Sea F el campo vectorial F (x, y, z) = (y, x, xz) . Calcular la integral de
RR !
flujo (r ⇥ F ) · n
bdS, donde nb el campo normal exterior al paraboloide, y S
S
el sector de la superficie del paraboloide z = x2 + (y 1)2 , interior al cilindro
x2 + (y 2)2 = 3.

Figura 6.15: S, sector de la superficie del paraboloide z = x2 + (y 1)2 , interior


al cilindro x2 + (y 2)2 =3

Solución
Tenemos que calcular la integral de flujo
ZZ ZZ
! !
(r ⇥ F ) · n
bdS = (r ⇥ F ) · !
n dS
S D

534
!
En primer lugar, calculamos el rotor del campo F ,

i j k
! @ @ @
r⇥ F = = (0, z, 0),
@x @y @z
y x xz

Parametricemos el sector de la superficie del paraboloide z = x2 + (y 1)2


interior al cilindro x2 + (y p2)2 = 3 usando coodenadas cilindricas con centro
en el punto (0,2,0) y radio 3.Entonces
! p
(u, v) = (ucosv, 2 + usenv, u2 + 2usenv + 1) , 0  u  3, 0  v  2⇡.
Determinemos los vectores tangentes a la superficie del paraboloide
!
u (u, v) = (cosv, senv, 2u + 2senv)
!
v (u, v) = ( usenv, ucosv, 2ucosv)
Dado que en el vértice (0, 1, 0) del paraboloide el vector normal apunta
hacia el exterior , tenemos que la normal exterior tiene componentes (0,0,-1),
entonces
i j k
! ! !
n = v (u, v) ⇥ u (u, v) = usenv ucosv 2ucosv
cosv senv 2u + 2senv
= 2u2 cosv, 2u2 senv + 2u, u

En consecuencia, la integral queda

ZZ Z Z
!
r⇥ F ·!
n dS = (0, u2 + 2usenv + 1, 0) · (2u2 cosv, 2u2 senv + 2u, u)dudv
D
D
Z p Z
3 2⇡
= (2u4 senv + 4u3 sen2 v + 6u2 senv + 2u3 + 2u)dvdu
0 0
Z p
3  ✓ ◆ 2⇡
3 v sen2v
= 4u + (2u3 + 2u)v du
0 2 4 0
Z p
3 ⇥ p
⇤ 3
= 8⇡u3 + 4⇡u du = 2⇡u4 + 2⇡u2 0
0
= 24⇡

Problema 3
!
Calcular el flujo de salida del campo vectorial F (x, y, z) = (2y, zy, 3z) a
través de la superficie cilı́ndrica x2 + y 2 = 4 acotada en el primer octante por
los planos x = 0, y = 0, z = 0, z = 5.

535
Figura 6.16: Superficie cilindrica x2 + y 2 = 4, acotada en el primer octante por
los planos coordenados

Solución
Tenemos que calcular

ZZ ZZ
! ! ⇣! ⌘ ⇣! !⌘
F ·n
bdS = F (✓, z) · ✓ ⇥ z d✓dz
S D

Parametricemos la superficie cilı́ndrica

!
(✓, z) = (2 cos ✓, 2 sin ✓, z)

donde D = {(✓, z) /0  ✓  ⇡/2, 0  z  5}.


Determinemos los vectores tangentes a la superficie
!
✓ (✓, z) = ( 2 sin ✓, 2 cos ✓, 0)
!
z (✓, z) = (0, 0, 1)

El vector normal que apunta al exterior de la superficie es


i j k
! !
✓ (✓, z) ⇥ z (✓, z) = 2sen✓ 2 cos ✓ 0 = (2cos✓, 2sen✓, 0)
0 0 1

536
!
Entonces, el flujo de F a través de esta superficie es
ZZ
! ⇣! ⌘ ⇣! !⌘
F (✓, z) · ✓ ⇥ z drdz
Z Z D

= (4 sin ✓, 2z sin ✓, 3z) · (2 cos ✓, 2 sin ✓, 0) d✓dz


D
Z ⇡/2 Z 5
= 4 5 sin 2✓ + z sin2 ✓ dzd✓
0 0
Z ⇡/2 ✓ ✓ ◆◆
25 1 cos 2✓
= 4 sin 2✓ + d✓
0 2 2
 ⇡/2  ⇡/2
cos 2✓ ✓ sin 2✓
= 20 + 50
2 0 2 4 0
25
= 20 + ⇡
2

6.3.3. Teorema de la divergencia de Gauss


Problema 1 Z Z
! !
Calcular el flujo ndS del campo vectorial F (x, y, z) = (x2 + senz, xy + cos z, ey )
F ·b
S
a través de la frontera S limitada por la superficie cilı́ndrica x2 + y 2 = 2y y
los planos z = 0, y + z = 2,

Figura 6.17: S limitada por la superficie cilı́ndrica x2 + y 2 = 2y y los planos z = 0,


y + z = 2,

Solución

537
!
Puesto que el campo vectorial F es continuo, con primeras derivadas
parciales continuas en IR3 y la región V✓ IR3 encerrada es conexa y S es una
superficie cerrada suave, entonces es aplicable el teorema de la divergencia
de Gauss.
ZZ ZZZ
! !
F ·nbdS = r · F dV
S V

Para resolver la ultima integral necesitamos describir el sólido V. Comple-


tando el cuadrado, la ecuación de la superficie cilı́ndrica dada puede escribirse
como:
x2 + y 2 = 2y () x2 + (y 1)2 = 2y.
Luego, la proyección del sólido sobre el plano XY es la circunsferencia
de centro (0, 1) y radio 1. Por otra parte z esta acotado entre los dos planos
dados 0  z  2 y.
Por tanto,
n el sólido esta dado por o
p p
V = (x, y, z) 2 IR3 / 2y y 2  x  2y y 2 , 0  y  2, 0  z  2 y
Calculemos la divergencia del campo vectorial:
! @ @ @ y
r· F = (x2 + senz) + (xy + cos z) + (e ) = 3x
@x @y @z
Entonces:

ZZZ Z 2 Z p2y y2 Z 2 y
3xdxdydz = p 3xdzdxdy
V 0 2y y2 0
Z 2 Z p2y y2
= p 3x (2 y) dxdy
0 2y y 2
Z 2 ✓ ◆ p2y y2
3 2
= (2 y) x p dy = 0
0 2 2y y 2

Problema 2
Verificar el teorema de la divergencia para el campo vectorial
!
F (x, y, z) = (x, y, z) sobre la esfera x2 + y 2 + z 2 = a2 .
Solución
El teorema de la divergencia afirma que.
ZZ ZZZ
! !
F ·n bdS = r · F dV
S V
!
La divergencia de F es

538
! @ @ @
r· F = (x) + (y) + (z) = 3.
@x @y @z
de modo que.
ZZZ ZZZ
!
r · F dV = 3 dV
V V
✓ ◆
4 3
= 3 ⇡a
3
= 4⇡a3

Calculemos ahora el Flujo sobre la superficie.


ZZ ZZ
! ! !
F ·nbdS = F ( r (u, v)) · (!
r u⇥!
r v ) dudv
S D

Parametricemos la superficie usando coordenadas esféricas.


!
r (u, v) = a (sin u cos v, sin u sin v, cos u) , con D = {(u, v) /0  u  ⇡, 0  v  2⇡}
Calculemos el producto vectorial

i j k
!
r u⇥!
r v = a2 cos u cos v cos u sin v sin u
sin u sin v sin u cos v 0
= a2 sin2 u cos v, sin2 u sin v, sin u cos u
!
Por consiguiente F (! r (u, v)) · (!
r u⇥!r v ) = a3 sin u
entonces la integral de flujo queda
ZZ ZZ
!
F ·nbdS = a3 sin ududv
S D
Z 2⇡ Z ⇡
= a3 sin ududv
0
Z 2⇡0
= a3 [ cos u]⇡0 dv
Z0 2⇡
= 2a3 dv
0
= 4⇡a3

Por tanto, comparando estos resultados hemos verificado el teorema de


Gauss.

Problema 3

539
Figura 6.18: Región acotada por la frontera x2 + y 2 = 4, z = 0, y z = 3

!
Verificar el teorema de la divergencia para F (x, y, z) = (x3 , y 3 , z 3 ) en
R, la región acotada por la frontera x2 + y 2 = 4, z = 0,y z = 3.
Solución RR ! RRR !
El teorema de la divergencia afirma que S
F · b
n ds = R
r · F dV
Calculemos el flujo que produce el campo sobre la frontera:
ZZ ZZ ZZ ZZ
! ! ! !
F ·n
bds = F ·n
bds + F ·n
bds + F ·n
bds
S S1 S2 S3

Si definimos parametrizacion de las superficies inferior y superior que


limitan el cilindro tenemos:
S1 : !r (u, v) = (u, v, 0) D⇤ = {(u, v) /0  u  2, 0  v  2⇡, 0}
El vector normal con orientacion positiva es
i j k
! !
r u ⇥ r v = 1 0 0 = (0, 0, 1)
0 1 0
Ası́ , obtenemos:

ZZ ZZ
! ! !
F ·n
bds = F ( r (u, v)) · (!
r u⇥!
r v ) dudv
S1 ⇤
Z ZD
= u3 , v 3 , 0 · (0, 0, 1) dudv = 0
D⇤

Analogamente para:

540
S3 : !
r (u, v) = (u, v, 0) donde D⇤ = {(u, v) /0  u  2, 0  v  2⇡}
i j k
!
r u⇥!
rv= 1 0 0 = (0, 0, 1)
0 1 0
Luego:
ZZ ZZ
! ! !
F ·n
bds = F ( r (u, v)) · (!
r u⇥!
r v ) dudv
S3 ⇤
Z ZD
= u3 , v 3 , 33 · (0, 0, 1) dudv
D⇤
Z 2 Z 2⇡
= 33 dudv
0 0
= 108⇡
Si definimos la parametrización para el manto del cilı́ndro se tiene:
S : ! r (u, v) = (2 cos u, 2senu , v)
2

donde D = {(u, v) /0  u  2⇡, 0  v  3}

El vector normal con orientacion positiva es,


i j k
!
r u⇥!
rv= 2senu 2 cos u 0 = (2 cos u, 2senu, 0)
0 0 1
En consecuencia,
ZZ ZZ
! ! !
F ·n
bds = F ( r (u, v)) · (!
r u⇥!
r v ) dudv
S2 ⇤
Z ZD
= (2 cos u)3 , (2senu)3 , v 3 · (2 cos u, 2senu, 0) dudv
D⇤
Z 3 Z 2⇡
= 16 cos4 u + sen4 u dudv
Z0 3 Z0 2⇡
1
= 16 (6 + 2 cos 4u) dudv
0 0 8
Z 3  2⇡
1 2
= 16 6u + sen4u dv
0 8 4 0
Z 3
= 2 (12⇡) dv
0
= 72⇡

541
Por lo tanto, sumando las integrales de flujo se tiene:
ZZ
!
F ·nbds = 102⇡ + 72⇡ = 180⇡
S

! @x3 @y 3 @z 3
Por otra parte, como r · F = + + = 3 (x2 + y 2 + z 2 ) ,
@x @y @z
entonces
ZZZ ZZZ
!
r · F dV = 3 x2 + y 2 + z 2 dV
R R
Usando coordenadas cilı́ndricas para evaluar la integral triple, tenemos
9
x = r cos ✓ =
y = rsen✓ =) 0  x2 + y 2  4 () 0  r2  4 () 0  r  2
;
z= z

Entonces la región R⇤ = {(r, ✓, z) /0  r  2 , 0  ✓  2⇡, 0  z  3}

ZZZ Z 2⇡ Z 2 Z 3
2 2 2
3 x +y +z dV = 3 r2 + z 2 rdzdrd✓
R 0 0 0
Z 2⇡ Z 2  3
r3 2
= 3 r z+ rdrd✓
0 0 3 0
Z 2⇡ Z 2
⇥ 2 ⇤
= 3 3r + 9 rdrd✓
0 0
Z 2⇡  2
3 4 9 2
= 3 r + r d✓
0 4 2 0
Z 2⇡
= 3 30d✓
0
= 90✓|2⇡
0 = 180⇡

Problema 4
!
Calcular el flujo del campo F (x, y, z) = (x + y 3 , 2y ez , 3z 1) en la
R la región acotada por la frontera S x2 + y 2 + 3z 2 = 1.
Solución:
Usemos el teorema de la divergencia de Gauss para calcular el flujo, que
afirma que, ZZ ZZZ
! !
F ·nbds = r · F dV
S R
Calculemos la divergencia del campo vectorial en la región R.

542
! @ @ @
r· F = (x + y 3 ) + (2y ez ) + ( 3z 1) = 1 + 2 3 = 0
@x @y @z
!
Entonces el flujo del campo vectorial F a través de S verifica
ZZ ZZZ
!
F ·n
bds = 0 dV = 0
S R

6.3.4. Teorema de Stokes


Problema 1
!
Verificar el teorema de Stokes para F (x, y, z) = ( y 3 , x3 , z 3 ) donde S
es la porción del plano x + y + z = 1 al interior del cilindro x2 + y 2 = 1.

Figura 6.19: S es la porción del plano x + y + z = 1 al interior del cilindro


x2 + y 2 = 1

Solución.
H ! RR !
El teorema de Stokes afirma que C F · d!
r = Sr⇥ F ·n
bdS
Calculemos

i j k
! @ @ @
r⇥ F = = (0, 0, 3x2 + 3y 2 ),
@x @y @z
y 3 x3 z3
!
y N = r(x + y + z 1) = (1, 1, 1)

543
ZZ ZZ
! ! !
r⇥ F ·n
bdS = r ⇥ F · N dxdy
S Z ZD
= 3(x2 + y 2 ) dxdy
D

donde D = {x, y) 2 IR2 /x2 + y 2  1} .


Usando coordenadas polares :

x = r cos ✓ cos ✓ rsen✓


=) |@ (x, y)@ (r, ✓)| = =r
y = rsen✓ sen✓ r cos ✓
x2 + y 2  1 () 0  r2  1 =) 0  r  1 y 0  ✓  2⇡

Entonces la región D⇤ = {(r, ✓) /0  r  1 y 0  ✓  2⇡} .Luego, si

ZZ Z 1 Z 2⇡
2 2
3(x + y ) dxdy = 3 r3 d✓dr
D 0 0
Z 1
= 6⇡ r3 dr
0
3⇡
=
2
Problema 2

Verificar el teorema de Stokes para evaluar la integral de lı́nea


Z
xdx + yz 2 dy + xzdz
C

donde C es la intersección de la semiesfera x2 + y 2 + z 2 = 1, z 0 y el


cilindro x2 + y 2 = y.

Solución
El teorema de Stokes afirma que
I ZZ
! ! !
F ·dr = r⇥ F ·n
bdS
C S

donde S es la región de la superficie de la semiesfera cuya frontera es la curva


C.
Calculemos directamente la integral de lı́nea, parametrizando la curva C
mediante coordenadas cilı́ndricas x = r cost, y = r sent, z = z.

544
Figura 6.20:

Al sustituir enpla ecuaciones de las superficies que definen la curva C,


tenemos que z = 1 r2 , r = sent donde 0  t  ⇡.
Luego, la ecuación de la curva es
⇣ p ⌘
!
r (t) = sin t cos t, sin2 t, 1 sin2 t , 0  t  ⇡

Usando identidades trigonométricas sin2 t = 1


2
(1 cos 2t), cos2 t = 1
2
(1 +
cos 2t) se tiene:

✓ ◆
! 1 1
r (t) = sin 2t, (1 cos 2t), cos t , 0  t  ⇡
2 2
! 0
r (t) = (cos 2t, sin 2t, sin t) , 0  t  ⇡

Evaluemos la integral de linea:

Z Z ⇡ 
2 1 1 1
xdx + yz dy + xzdz = sin 2t cos 2t (1 cos 2t) cos2 tsen2t sin 2t cos t sin t
C 0 2 2 2
Z ⇡
1 1 1 2
= sin 2t cos 2t (1 cos2 2t) sin 2t sin 2t dt
0 2 4 4
Z ⇡ ✓ ◆
1 1 2 1 1 cos 4t
= sin 2t cos 2t (1 cos 2t) sin 2t dt
0 2 4 4 2
 ⇡
1 2 1 1 1 sin 4t
= sin 2t + cos 2t cos3 2t t+
4 8 24 8 32 0

=
8
545
Por otra parte, debemos evaluar
ZZ ZZ
! ! !
r⇥ F ·n bdS = r ⇥ F · N dA
S D
Como S es la región de la superficie de la semiesfera cuya frontera es la
curva C, una parametrizacion de S viene dada por

! ⇣ p ⌘
T (r, ✓) = r cos ✓, rsen✓, 1 r2 , 0  r  sen✓, 0  ✓  ⇡
A continuación calculemos el vector normal a la superficie:
i j k
! ! ! r
N = Tr⇥ T✓= cos ✓ sin ✓ p
1 r2
r sin ✓ r cos ✓ 0
✓ 2
◆ 2
r cos ✓ r sin ✓
= p ,p ,r
1 r2 1 r2
La orientación del vector normal a la superficie es compatible con la
orientación de su frontera C.
Determinemos el rotor del campo vectorial
i j k
! @ @ @
r⇥ F = = ( 2yz, z, 0),
@x @y @z
x yz xz

Por lo que la función compuesta queda


! ⇣! ⌘ ⇣ p p ⌘
r ⇥ F T (r, ✓) = 2r sin ✓ 1 r2 , 1 r2 , 0
Entonces, estamos en condiciones de calcular la integral de superficie,
ZZ Z ⇡ Z sin ✓
! !
r ⇥ F · N dA = 2r3 sin ✓ cos ✓ r2 sin ✓ drd✓
D 0 0
Z ⇡ 4 sin ✓
r r3
= sin ✓ cos ✓ + sin ✓ d✓
0 2 3 0
Z ⇡ 5
sin ✓ sen4 ✓
= cos ✓ + d✓
0 2 3
 6 ⇡ Z
sin ✓ 1 ⇡ 4
= sin ✓d✓
12 0 3 0
Z
1 ⇡ 4
= sin ✓d✓
3 0

546
Puesto que el integrando queda

✓ ◆2
4 1 cos 2✓ 1
sin ✓ = = 1 2 cos 2✓ + cos2 2✓
2 4
✓ ◆
1 1 + cos 4✓
= 1 2 cos 2✓ +
4 2
✓ ◆
1 3 cos 4✓
= 2 cos 2✓ +
4 2 2
Podemos calcular

Z ⇡ Z ✓ ◆
1 4 1 ⇡1 3 cos 4✓
sin ✓d✓ = 2 cos 2✓ + d✓
3 0 3 0 4 2 2
 ⇡
1 3✓ sin 4✓
= sin 2✓ +
3 2 2 0

=
8
Problema 3 I
! !
Sea F (x, y, z) = (x, y, z), calcular la integral de lı́nea F · dr, siendo C
C
la curva intersección de las superficies S1 dada por ecuación z = x2 + 2y 2 y
S2 por la ecuación z = 4 x2 .

Figura 6.21: C la curva intersección de las superficies S1 dada por ecuación z =


x2 + 2y 2 y S2 por la ecuación z = 4 x2

Solución
El teorema de Stokes afirma que
I ZZ
! ! !
F ·dr = r⇥ F ·n
bdS
C S

547
donde la curva C es la frontera de la superficie S dada por

S = {(x, y, z) 2 IR3 : x2 + y 2  2, z = 4 x2 }
Calculemos

i j k
! @ @ @
r⇥ F = = (0, 0, 0).
@x @y @z
x y z
Por lo tanto
ZZ ZZ
!
r⇥ F ·n
bdS = (0, 0, 0) · n
bdS
S S
Lo que indica que estamos frente a un campo conservativo.
Problema Z Z4
! !
Calcular r⇥ F ·n bdS, siendo F = (xz + yz 2 + x, xyz 3 + y, x2 z 4 ) y
S
S = S1 [ S2 donde S1 es la superficie cilı́ndrica x2 + y 2 = 1, 0  z  1 y S2
es la semiesfera x2 + y 2 + z 2 = 1, z 0.

Figura 6.22: Superficie S = S1 [S2 donde S1 es la superficie cilı́ndrica x2 +y 2 = 1,


0  z  1 y S2 es la semiesfera x2 + y 2 + z 2 = 1, z 0

Solución.

El teorema de Stokes afirma que


ZZ I
! ! !
r⇥ F ·n bdS = F ·dr
S C

donde la curva C es la frontera de la superficie S dada por x2 + y 2 = 1


Parametizando la circunferencia, queda:
!
r (t) = (cos t, sin t, 0) , t 2 [0, 2⇡]
!
r 0 (t) = ( sin t, cos t, 0) , t 2 [0, 2⇡] .

548
! !
F ( r (t)) = (cos t, sin t, 0)
Calculemos la integral
I Z 2⇡
! !
F ·dr = (cos t, sent, 0) · ( sin t, cos t, 0) dt
C 0
= 0
Problema 5
!
Determine la circulación del campo F = (x2 y, 4z, x2 ) alrededor de la
curva Cpdada por la intesección del plano z = 4 y el cono
z = x2 + y 2 ,recorrida en sentido positivo.

p
Figura 6.23: Plano z = 4 y cono z = x2 + y 2

Solución
El teorema de Stokes afirma que
I ZZ
! ! !
F ·dr = r⇥ F ·n
bdS
C S

donde la curva C es la frontera de la superficie S dada por

S = {(x, y, z) 2 IR3 : x2 + y 2  z 2 , z  4}
Parametrizando el cono se tiene
!
T (r, ✓) = (r cos ✓, r sin ✓, r) ,
donde D = {(r, ✓) /0  r  2, 0  ✓  2⇡}
A continuación calculemos el vector normal a la superficie:
i j k
! ! !
N = Tr⇥ T✓= cos ✓ sin ✓ 1
r sin ✓ r cos ✓ 0
= ( r cos ✓, r sin ✓, r)

549
La orientación del vector normal a la superficie es hacia el exterior del
cono
Además

i kj
! @ @
@
r⇥ F = = ( 4, 2x, 1).
@x @y
@z
x2 x2
y 4z
! ⇣! ⌘
Luego la función compuesta r ⇥ F T (r, ✓) es

! ⇣! ⌘
r ⇥ F T (r, ✓) = ( 4, 2r cos ✓, 1)

Por lo tanto
ZZ ZZ
! ! !
r⇥ F ·nbdS = r ⇥ F · N drd✓
S Z ZD
= ( 4, 2r cos ✓, 1) · ( r cos ✓, r sin ✓, r) drd✓
D
Z 2 Z 2⇡
= (4r cos ✓ + r sin ✓ + r) d✓dr
0 0
= 4⇡

6.4. Ejercicios Propuestos


6.4.1. Área de una superficie
1. Problema
Calcule el área de la superficie de x2 + y 2 + z 2 = a2 , a > 0, z 0.
Respuesta
A (R) = 2⇡a2

2. Problema
Calcule el área de la superficie del sector del plano ax + by + cz + d = 0
que queda dentro del cilindro x2 + y 2 = r2 (c 6= 0, r > 0) .
Respuesta
p
⇡r2 a2 + b2 + c2
A (R) =
c

550
Figura 6.24: Superficie del sector del plano ax + by + cz + d = 0 que queda
dentro del cilindro x2 + y 2 = r2 (c 6= 0, r > 0)

Figura 6.25:

3. Problema
Calcule el área del helicoide !
r (u, v) = (u cos v, u sin v, v) para 0  u 
1, 0  v  2⇡
Respuesta
p p
A (R) = ⇡(ln( 2 + 1) + 2

4. Problema
Hallar el área de la superficie S descrita por la parametrización: ! r :D⇢
R 2 ! R3 ; ! r (u; v) = (u cos v; u sin v; u2 ) ; donde D = {(u, v) 2 IR2 /0  u  4, 0  v  2⇡} .
Respuesta
1 h 3/2
i
I= (65) 1
12
551
Figura 6.26:

5. Problema
Obtener el área de la superficie S , frontera de
= {(x, y, z) 2 IR3 : (z 1)2  x2 + y 2 y x2 + y 2 + z 2  1}

Figura 6.27:

Respuesta
p
Área de la superficie S = Sesf [ Scono = 2⇡ + ⇡ 2
6. Problema
Calcule el área de la porción de superficie cónica z 2 = x2 + y 2 situada
por encima del plano z = 0 y limitada por la esfera x2 + y 2 + z 2 = 2ax
Respuesta
p
2 2
I= ⇡a
4

552
Figura 6.28:

7. Problema
Calcule el área de la superficie S de una pelota de rugby formada por la
⇡ ⇡
rotación de la curva x = cos z, y = 0,  z  alrededor del eje z.
2 2
Respuesta
Ecuacion paramétrica de la superficie ! r (u, v) = (cos u cos v, cos u sin v, u)
⇥p p ⇤
A = 2⇡ 2 + ln 1 + 2

6.4.2. Integrales de funciones escalares sobre superficie


1. Problema Z
Calcule la integral x2 zdS ; siendo S la superficie externa de
S
x2 + y 2 = a2 comprendida entre z = 2y
z = 2.

Figura 6.1:

Respuesta

553
Z
x2 zdS = 0
S
2. Problema Z
Calcule la integral de superficie (x4 y4 + y2z2 x2 z 2 + 1)dS . Si
S
la superficie S es la hoja superior del cono z 2 = x2 + y 2 que está en el
interior del cilindro (x 1)2 + y 2 = 1.

Figura 6.2: Superficie S, hoja superior del cono z 2 = x2 +y 2 que está en el interior
del cilindro (x 1)2 + y 2 = 1

Respuesta
p
A = 2⇡

554
3. Problema Z
Calcule la integral de superficie I = zdS, donde S es la superficie
S
parametrizada por ! r (x, y) = (x, y, 2x + 3y + 2a), (x, y) 2 D tal que
D = {(x, y) 2 IR2 /x2 + y 2  a2 } .

Figura 6.3: Aca voy

Respuesta
p
I = 2 14⇡a3
4. Problema Z p
Calcule la integral de superficie I = 1 x2 y 2 dS, donde S es una
S
pelota de rugby formada por la rotación de la curva x = cos z, y =
⇡ ⇡
0,  z  alrededor del eje z.
2 2
Respuesta
Superficie parametrizada !
r (u, v) = (cos u cos v, cos u sin v, u) ,
n ⇡ ⇡ o
D = (u, v) /  u  , 0  v  2⇡
2 2
4⇡ p
I= 2 2 1
3

6.4.3. Integral de Flujo


1. Problema
!
Calcule la integral del campo vectorial F (x, y, z) = (x, y, z) a través de
la superficie lateral del paraboloide x2 + y 2 = 2az , con 0  z  2a.
Respuesta
I = 4⇡a3
2. Problema
!
Calcule la integral del campo vectorial F (x, y, z) = (yz, x, z 2 ) a través
de la superficie del cilindro parabólico y = x2 , con 0  x  1, 0  z 
4,donde la primera componente de ! n es positiva.

555
(a) Campo vectorial (b) paraboloide

Figura 6.1:

(a) Campo vectorial (b) sector de cilindro


parabólico

Figura 6.2:

Respuesta
Se tiene que !
n = (2x, 1, 0)
I=2

3. Problema

!
Calcule la integral del campo vectorial F (x, y, z) = (0, yz, z 2 ) a través
de la superficie S cortada del cilindro y 2 + z 2 = 1, por los planos
x = 0;  x = 1.
Respuesta
Se tiene que !
n = (0, y, z)
I=2

556
(a) Campo vectorial (b) Cilindro cortado por
dos planos

Figura 6.3:

6.4.4. Teorema de la divergencia de Gauss


1. Problema ZZ
! 2 !
Sea F (x, y, z) = (xz, yz, z ). Calcule F ·n
bds ; siendo S la cara
S
externa del paraboloide x2 + y 2 = 3z; entre
z = 0 y z = 1.

Figura 6.1: Paraboloide x2 + y 2 = 3z limitado por los planos z = 0 y z = 1

a) Directamente.
b) Aplicando el teorema de Gauss.
Respuesta.
I = 3⇡
2. Problema ZZ
! !
Calcular el flujo F ·n
bds, del campo vectorial F = (x, y, z) a través
S
de la superficie S del elipsoide ax2 + by 2 + cz 2 = 1

557
Respuesta.
ZZ
! 2⇡
bds = p
F ·n
S abc
3. Problema ZZ
! !
Calcular el flujo bds,del campo vectorial F = (2xy 2 , z 3 , x2 y)
F ·n
S
a través la superficie S formada por el hemisferio superior de la esfera
x2 + y 2 + z 2 = a2 y el cilindro x2 + y 2 = a2

(a) Campo vectorial (b) Superficie formada por el hemisfe-


rio superior de la esfera y el cilindro

Figura 6.2:

Respuesta.
ZZ
! 4
bds = ⇡a5
F ·n
S 15

558
4. Problema ZZ
! !
Calcular el flujo nds, del campo vectorial F = 0, tan z + esin xz , y 2
F ·b
S
a través de la superficie S del semi-elipsoide 2x2 + 3y 2 + z 2 = 6, z 0,
con la normal apuntando
hacia el exterior.

(a) Campo vectorial (b) Semi-elipsoide

Figura 6.3:

Respuesta.
ZZ r
! 3
F ·n
bds = ⇡
S 2

559
5. Problema ZZ
! ! (x, y, z)
Calcular el flujo F ·b
nds, del campo vectorial F =
S (x2 + y 2 + z 2 )3/2
a través de la superficie S dada por el paraboloide z = 2 x2 y 2 ,tales
z 0, con la normal apuntando hacia el exterior.
Pendiente
Respuesta.
ZZ ⇣ p ⌘
!
F ·n bds = ⇡ 2 2
S
6. Problema

Sea S la superficie del paraboloide z = x2 + (y 1)2 , interior al cilindro


! !
x2 + (y 2)2 = 3. Sea Z Z F el campo vectorial F (x, y, z) = (y, x, xz)
!
. Calcular la integral r⇥ F ·n bds,utilizando el teorema de Gauss,
S
b es la normal exterior al paraboloide.
donde n

(a) Campo vectorial (b) Paraboloide interior al


cilindro

Figura 6.4:

Respuesta
S
Colocando una tapa T a la superficie S, de modo que S ⇤ = S T sea
una superficie cerrada, podemos aplicar el teorema de Gauss
ZZ ZZ
! !
r⇥ F ·n bds = r⇥ F ·nbds = 2⇡
S T
7. Problema
Sea S la porción del paraboloide z + 1 = x2 + y 2 , situada
ZZ debajo del
! !
plano z = 1, y sea F (x, y, z) = (0, x 2yz, x2 ). Hallar r⇥ F ·nbds,
S

560
donde nb es la normal exterior al paraboloide, utilizando el teorema de
Gauss.

(a) Campo vectorial (b) Paraboloide situada de-


bajo del plano

Figura 6.5:

Respuesta
S
Colocando una tapa T a la superficie S, de modo que S ⇤ = S T sea
una superficie cerrada, podemos aplicar el teorema de Gauss
ZZ ZZ
! !
r⇥ F ·n bds = r⇥ F ·nbds = 2⇡
S T

6.4.5. Teorema de Stokes


1. Problema
Sea S el sector del paraboloide z = x2 + y 2 , situado en el primer
octante , limitado por el plano z = 1 y los planos principales,
I y sea
! ! !
F = (y z, z x, x y) . Calcular la integral de lı́nea F ·dr .
C
Respuesta
I
! ! 8 ⇡
F ·dr .=
C 3 2
2. Problema
!
Para el campo vectorial F = (x2 + yez , y 2 + zex , z 2 + xey ) definida so-
!
bre S : x2 + y 2 + z 2 = a2 con z 0.Calcular el flujo del rotacional de F
a través de S.
Respuesta
I ZZ
! ! !
F ·dr = r⇥ F ·n bds = ⇡a2
C S

561
(a) Campo vectorial (b) Sector de
paraboloide, limita-
do por planos

Figura 6.1:

(a) Campo vectorial (b) hemisferio superior de la


esfera

Figura 6.2:

3. Problema
!
Dado el campo vectorial F (x, y, z) = (y + sin x, z 2 + cos y, x3 ) y la curva
!
r (t) = (sin t, cos t, sin 2t); t 2 [0; 2⇡]; halle
I
! !
F · d r utilizando el teorema de Stokes.
C
Respuesta
I = ⇡2

562
(a) Campo vectorial (b) paraboloide hiperbólico

Figura 6.3:

4. Problema
!
Calcule el flujo del campo F (x, y, z) = (x, y, z) a través de la superficie
del sólido limitado por x2 + y 2 = 9, z = 0, y z = 0.
a) Directamente.
b) Aplicando el Teorema de Stokes
Respuesta
I = 81⇡

5. Problema I
Calcule la integral y 2 dx + xydy + xzdz, siendo C la curva intersección
C
del cilindro x2 + y 2 = 2y y el plano y = z.
a) Directamente.
b) Aplicando el teorema de Stokes.

Figura 6.4:

Respuesta

563
I=0
6. Problema I
Calcule la integral I = (y 1)dx + z 2 dy + ydz, aplicando el teorema
C
z2
de Stokes, siendo C la curva intersección de las superficies x2 + y 2 = ,
2
z =y+1

Figura 6.5:

Respuesta
p
I= 2⇡

6.5. Aplicaciones
En esta sección presentamos algunas aplicaciones simples que ilustran la
utilización de las, integrales de lineas, integral de flujo y de los teoremas de
Gauss , Stokes y Green .

6.5.1. Aplicaciones Integral de Flujo


Problema 1
Calcular el flujo del campo eléctrico producido por una carga Q en el
origen, a través del manto de la esfera S de radio R orientado según la normal
exterior.

Solución
El campo eléctrico producido por la carga Q viene dado por

564
! Q rb
E =
4⇡"o r2
donde "o una constante universal . De esta forma se obtiene el siguiente flujo
eléctrico
ZZ ZZ
! ! Q rb
E · dS = · rbdS
4⇡"o R2
S S
Parametrizando la superficie en coordenadas esféricas se tiene
!
( , ✓) = (R sin cos ✓, R sin sin ✓, R cos ) , 0   ⇡, 0  ✓  2⇡
! !
=) ⇥ ✓ = R2 sin
ZZ Z 2⇡ Z ⇡
Q rb Q 1 2
· rbdS = R sin d d✓
4⇡"o R2 0 0 4⇡"o R2
S
Q
=
"o
Por tanto ZZ
! ! Q
E · dS =
"o
S
Corresponde a la formulación integral de la Ley de Gauss, que se aplica
también a distribuciones de carga más generales.

Problema 2
Calcular el flujo del campo eléctrico generado por una carga Q en el
origen, sobre el plano infinito x = 1.
Solución
En este caso la normal es constante Nb = bi . El campo eléctrico producido
por una carga Q en coordenadas cartesianas, esta dado por

! Q rb Q (x, y, z)
E = =
4⇡"o r 2 4⇡"o [x2 + y 2 + z 2 ]3/2
El flujo del campo viene dado por
ZZ ZZ
! b Q (1, y, z)
E · idS = · (1, 0, 0) dS
4⇡"o [x2 + y 2 + z 2 ]3/2
S S
Z 1Z 1
Q dxdy
=
4⇡"o 1 1 [1 + y 2 + z 2 ]3/2

565
Utilizando el cambio de variables a coordenadas polares, tenemos
y = rcos✓, z = rsen✓ =) y 2 + z 2 = r2
@ (y, z)
y el jacobiano de transformación es =r
@ (r, ✓)
Reemplazando el integrando, obtenemos

Z 1 Z 2⇡ Z 1
Q rd✓dr Q rdr
=
4⇡"o 0 0 [1 + r2 ]3/2 2"o 0 [1 + r2 ]3/2
Qh 2
i
1/2 1
= 1+r
"o 0
Q
=
"o

Problema 3
!
Si E (x, y, z) = (2x, 2y, 2z) es un campo eléctrico, encontrar el flujo de
!
E que sale a través de la superficie cerrada S que consta de la semiesfera
x2 + y 2 + z 2 = 1, z 0, y su base.
Solución
Como los puntos de la circunferencia x2 + y 2 = 1, z = 0, son singulares,
debemos descomponer la superficie en dos partes, de modo que llamaremos
S1 a la semiesfera y S2 al cı́rculo que forma la tapa inferior.
Una parametrización de S1 viene dada por la función
!
r 1 (u, v) = ( sin u cos v, sin u sin v, cos u)
n ⇡ o
donde D1 = (u, v) 2 IR3 /0  u  , 0  v  2⇡
2
El vector normal exterior a la superficie es
! !
T u ⇥ T v = (sin2 u cos v, sin2 u sin v, cos u sin u)
De este modo, el flujo a través de S1 es
ZZ ZZ
!
E ·nbdS = 2(cos2 v sin3 u + sin2 v sin3 u + sin u cos2 u)dudv
S1 D
Z 2⇡ Z ⇡/2
= 2 sin ududv
0 0
= 4⇡
Análogamente, si parametrizamos la superficie S2 por
!r (u, v) = (ucosv, u sin v, 0)
2

566
donde D2 = {(u, v) 2 IR2 /0  u  1, 0  v  2⇡}

i j k
!
r u⇥!
rv= cos v sin v 0 = (0, 0, u)
u sin v u cos v 0
El flujo a través de la superficie S2 es

ZZ ZZ
!
E ·n
bdS = 2(u cos v, u sin v, 0) · (0, 0, u) dudv
S2 D
ZZ
= 0 dudv = 0
D

Por lo tanto, la suma de ambos flujos da como resultado


ZZ
!
E ·nbdS = 4⇡
S

Problema 4
! p
Si F (x, y, z) = (0, y, 0) representa la velocidad de un fluido. Calcular
la razón del flujo de dicho fluido que atraviesa la superficie
S = {(x, y, z) : x2 + z 2 = y, 0  y  1}.

Figura 6.6: Superficie S = {(x, y, z) : x2 + z 2 = y, 0  y  1}

Solución

567
En primer lugar, consideremos la siguiente parametrización de la superficie:
r(u, v) = (u cos v, u2 , u sin v), 0  u  1, 0  v  2⇡.
El vector normal a la superficie viene dado por:

i j k
! !
Tu⇥ Tv = cos v 2u sin v =
u sin v 0 u cos v
= (2u2 cos v, u, 2u2 sin v)

Entonces el flujo esta dado por,

Z Z Z 2⇡ Z 1
!
F ·n
bdS = (0, u, 0) · (2u2 cos v, u, 2u2 sin v)dudv
S 0 0
Z 2⇡ Z 1 Z 2⇡  3 1
2 u
= ( u )dudv = dv
0 0 0 3 0
2⇡
=
3
El signo negativo del resultado es debido a que el vector normal tiene sentido
!
opuesto al del movimiento del fluido (la componente b j de F es positiva pero
la de nb es negativa). Ası́ pues, la razón de flujo que entra a la superficie es
2⇡
de .
3

6.5.2. Aplicación del teorema de Gauss


Problema 1
Suponga que la temperatura de un punto de una esfera S es proporcional
al cuadrado de la distancia de dicho punto al centro de la esfera. Se sabe que
!
el campo estacionario de temperaturas J asociado a este potencial queda
!
definido por la expresión J = rT , donde k > 0 representa la conductivi-
dad térmica del medio y T la temperatura.
Calcular directamente la razón total del flujo de calor que atraviesa la
esfera, suponiendo que tiene centro el origen y radio a.
Solución
Debemos calcular
Z Z Z Z ⇣!
! ! ! ! ⌘
J ·n
bdS = J ( r (u, v)) · T u ⇥ T v dudv
S D

Consideremos la parametrización de S dada por:

568
!
r (u, v) = (a sin u cos v, a sin u sin v, a cos u)
donde D = {(u, v) 2 IR3 /0  u  ⇡, 0  v  2⇡}
Determinemos el flujo de calor. Como T (x, y, z) = C·(x2 + y 2 + z 2 ), entonces
!
J (x, y, z) = krT (x, y, z) = kC(2x, 2y, 2z)
El vector normal exterior a la superficie es
! !
T u ⇥ T v = (a2 sin2 u cos v, a2 sin2 u sin v, a cos u sin u)
y la razón de flujo a través de la esfera viene dada por la integral
Z Z Z Z ⇣!
! 2⇡ ⇡
! ⌘
J ·n
bdS = kCrT · T u ⇥ T v dudv
S
Z0 2⇡ Z0 ⇡
= 2kC·a3 sin ududv
0 0
3
= 8kC⇡a

Problema 2
Suponga que la temperatura en una región de IR3 está dada por
T (x, y, z) = C(x2 + y 2 + z 2 ).
!
Se sabe que el campo estacionario de temperaturas J asociado a este
!
potencial queda definido por la expresión J = rT .Suponga que la región
contiene a la esfera S centrada en el origen y de radio a.
Calcular el flujo de calor que sale a través del casquete esférico S,usando
el teorema de Gauss , dada la conductividad térmica > 0 del material.
Solución
Sea ⌦ un abierto de IR3 de frontera regular S orientada según la normal
exterior N b
El flujo de calor producido por el campo estacionario de temperaturas es

ZZ ZZZ
! b !
J · N dS = r · J dV
S V
ZZZ
=  r · (rT ) dV
ZV Z Z
=  r2 T dV
V

569
Calculemos el Laplaciano de temperaturas

2 @ 2 x2 @ 2 y 2 @ 2 z 2
rT = + + = C(2 + 2 + 2) = 6C
@x2 @y 2 @z 2
Reemplazando el integrando, queda
ZZ ZZZ
! !
J · dS = 6C dV
S
✓⌦ ◆
4 3
= 6C ⇡R
3
= 8C⇡R3

Problema 3
El hecho de que la temperatura decrece hacia el origen hace que el gra-
diente apunte en esa dirección de modo que el flujo neto de calor que entra
a la esfera es negativo.
El filtro de una n máquina de lavar ropa tiene la forma
o cónica descrita
p
3 2 2
por el conjunto ⌦ = (x, y, z) 2 IR / x + y  z  3 , por el circula una
corriente de agua, cuyo campo de velocidades esta dado por la expresión
!
F (x, y, z) = (2yz cos (y 2 ) , 2x cos (x2 ) , 1) .
a) Muestre que la cantidad de agua en el interior del filtro se mantiene
constante, suponiendo que la densidad el agua es ⇢ = 1 gr/cm3 .
b) Usando el teorema de Stokes, calcule el flujo total de agua través de
las paredes del filtro

Solución
Por el teorema de la divergencia de Gauss, se tiene que el flujo total de
agua a través de las paredes del filtro es
ZZ ZZZ
! !
F ·nbds = r · F dV
S ⌦

Calculemos la divergencia del campo de velocidades


! @ @ @
r· F = 2yz cos y 2 + 2x cos x2 + (1) =)
@x @x @x
!
r· F = 0

570
n p o
Figura 6.7: ⌦ = (x, y, z) 2 IR3 / x2 + y 2  z  3

!
Como r · F = 0,el flujo total de agua a través de las paredes del filtro es
nulo. Por lo tanto la cantidad de agua que entra al filtro es igual a la que sale,
entonces la cantidad de agua en el interior del filtro se mantiene constante.
!
b) Sea S la pared del manto de ⌦ .Como F es un campo de divergencia
!
nula en IR3 , entonces F es un campo rotacional, es decir, existe un campo
! ! !
vectorial R tal que F = r ⇥ R .
!
En consecuencia, se busca R que satisfaga la condicion anterior.
!
Sea R = (R1 , R2 , R2 ) tal que

@R3 @R2
= 2yz cos y 2
@y @z
@R1 @R3
= 2xz cos x2
@z @x
@R2 @R1
= 1
@x @y

Este problema tiene muchas soluciones.Formulemos un modelo en que


R1 = 0, entonces

@R3 @R2
= 2yz cos y 2
@y @z
@R3
= 2xz cos x2
@x
@R2
= 1
@x
Integrando parcialmente las dos últimas ecuaciones se obtiene

571
@R3 @R2
= 2yz cos y 2
@y @z
R3 = zsen x2 + f (x, y)
R2 = x + g (y, z)

Se tiene más de una solución, consideremos g (x, y) = 0, luego queda


@R3
= 2yz cos y 2
@y
R3 = zsen x2 + f (x, y)
R2 = x

Derivando la segunda ecuación parcialmente con respecto a y y se iguala con


la primera
@
f (x, y) = 2yz cos y 2
@y
Integrando parcialmente con respecto a y esta última expresión produce

f (x, y) = zsen y 2 + h (z)


Para simplificar consideremos h (z) = 0,por consiguiente

R3 = zsen x2 + zsen y 2
!
Finalmente, el campo R queda:
!
R = 0, x, zsen x2 + zsen y 2

El teorema de Stokes afirma que


ZZ I
! ! !
r⇥ R ·n
bdS = R ·dr
S C
!
El flujo que produce el campo R a través del manto es igual a la integral
de lı́nea sobre la frontera C que limita la supercicie S.
C = {(x, y, z) 2 IR3 / x2 + y 2 = 9, z = 3}
Parametrizando la curva se tiene
!
r (t) = (3 cos t, 3set, 3) , t 2 [0, 2⇡]
I Z 2⇡
! ! ! !
R ·dr = R ( r (t)) · !
r 0 (t) dt
C 0

572
Z 2⇡
= 0, 3 cos t, 27sen3 t 27sent cos2 t · ( 3sent, 3 cos t, 0) dt
0
Z 2⇡ Z 2⇡   2⇡
2 1 + cos 2t 9 sen (2t)
= 9 cos tdt = 9 dt = +t
0 0 2 2 2 0
= 9⇡

6.5.3. Aplicación teorema de Stokes


Problema 1
!
Calcular el trabajo producido por la fuerza F (x, y, z) = ( y 3 , x3 , z 3 )
,sobre la trayectoria recorrida en el sentido positivo, dada por la intersección
de las superficies x + y + z = a, x2 + y 2 = a2 .

Figura 6.8:

Solución
Como el campo es continuo y la trayectoria es cerrada, podemos calcular
el trabajo aplicando el teorema de Stokes.

I ZZ ⇣
! ! !⌘
W = F ·dr = r⇥ F ·n
b dS
ZCZ ⇣ S
!⌘ !n
= r ⇥ F · ! k!
n k dA
D knk

Al parametrizar en coordenadas cartesianas el plano que contiene a la


curva obtenemos
!
(x, y) = (x, y, a x y), donde (x, y) 2 D tal que x2 + y 2  a2 .

573
Calculemos el integrando del lado derecho de la formula anterior
i j k
! @ @ @
r⇥ F = = (0, 0, 3x2 + 3y 2 )
@x @y @z
y 3 x3 z3
El vector normal al plano esta dado por
!
n = r = (1, 1, 1) , donde = x + y + z a = 0
En consecuencia, al sustituir términos en al última integral queda

ZZ ⇣ ZZ
!⌘
r⇥ F ·!
n dxdy = 0, 0, 3x2 + 3y 2 · (1, 1, 1) dxdy
D ZDZ
= 3 (x2 + y 2 )dxdy
D

Si hacemos cambio a coordenadas polares obtenemos

Z 2⇡ Z a Z 2⇡  a
2 ⇢4
3 ⇢ (⇢) d⇢d✓ = 3 d✓
0 0 0 4 0
Z 2⇡
4
a
= 3 d✓
4 0
6⇡a4
=
4
Corresponde al trabajo total realizado por el campo de fuerzas.

Problema 2
Calcular la circulación del campo de velocidades de un fluı́do dado por
!
F (x, y, z) = (arctan(x2 ), 3x, e3z tan (z)) a lo largo de la intersección de la
esfera x2 + y 2 + z 2 = 4, con el cilindro x2 + y 2 = 1, con z > 0.
Solución.
La circulación de un campo vectorial es su integral de lı́nea a lo largo de
una curva cerrada.
Vemos que el campo vectorial es bastante complejo, por lo que calcular
directamente la integral de linea puede resultar engorroso.
Se espera que al ocupar el teorema de Stokes se simplifique el calculo.
Entonces
I ZZ ⇣
! ! !⌘
F ·dr = r⇥ F ·n b dS
C S

574
Figura 6.9:

Calculemos el integrando del lado derecho de la formula anterior


i j k
! @ @ @
r⇥ F = = (0, 0, 3)
@x @y @z
arctan(x2 ) 3x e3z tan(z)
Luego, queda

I ZZ
! !
F ·dr = (0, 0, 3) · !
n dA
C D

Para determinar el vector normal parametricemos la superficie en coor-


denadas cilı́ndricas 9
x = ⇢ cos ✓ = ⇣ p ⌘
y = p⇢ sin ✓ ! 2
=) r (⇢, ✓) = ⇢ cos ✓, ⇢ sin ✓, 4 ⇢ donde
;
z= 4 ⇢2
(⇢, ✓) 2 D tal que 0 ⇢  1, 0  ✓  2⇡
Luego

i j k !
⇢ ⇢2 cos ✓ ⇢2 sin ✓
!
n = (!
r ⇢⇥!
r ✓) = cos ✓ sin ✓ p = p ,p ,⇢
4 ⇢4 4 ⇢4 4 ⇢4
⇢ sin ✓ ⇢ cos ✓ 0
Reemplazando términos en el integrando se tiene

575
I ZZ !
! ! ⇢2 cos ✓ ⇢2 sen✓
F ·dr = (0, 0, 3) · p ,p , ⇢ d⇢d✓
C D 4 ⇢4 4 ⇢4
Z 2⇡ Z 1 Z 2⇡  2 1

= 3⇢d⇢d✓ = 3 d✓
0 0 0 2 0
Z
3 2⇡
= d✓
2 0
= 3⇡

6.5.4. Aplicacion teorema de Green


Problema 1
Usando el teorema de Green determinar el momento de inercia de una
arandela homogénea de densidad superficial ,que tiene masa M, radio inter-
no a y radio externo b; respecto de uno de sus diámetros.

Figura 6.10: región R = (x, y) 2 IR2 / a2  x2 + y 2  b2

Solución.
Determinaremos el momento de inercia con respecto al diámetro que es
colineal con el eje x.
El momento de inercia con respecto a éste eje esta dado por
ZZ
Ix = y 2 dA
R

donde es la densidad superficial.

576
La región R = {(x, y) 2 IR2 / a2  x2 + y 2  b2 } , no es simplemente
conexa, sin embargo, se puede extender el teorema de Green, sumando inte-
grales de lı́neas sobre fronteras separadas, de un número finito de subregiones
(simplemente conexas), para construir una integral sobre una sola frontera.

ZZ  Z Z
@Q @P
dxdy = P dx + Qdy P dx + Qdy
R @x @y C1 C2
Z Z
= P dx + Qdy + P dx + Qdy
C1 C2

Buscamos P y Q, tales que


@Q @P
= y2
@x @y
@P y3
Consideremos en este caso Q = 0 =) = y 2 =) P =
@y 3
Asi, el momento de inercia queda:
ZZ Z Z
2 y3 y3
Ix = y dA = dx dx
R C1 3 C2 3
Z Z
y3 y3
= dx + dx
C1 3 C2 3

Parametrizamos
⇢ ambas curvas:

x = b cos t dx = b sin tdt
C1 : =) con 0  t  2⇡
y = b sin t dy = b cos tdt
⇢ ⇢
x = a cos t dx = a sin tdt
C2 : =) con 0  t  2⇡
y = a sin t dy = a cos tdt
Reemplazando términos en la última integral, queda:

Z 2⇡ Z 2⇡ 3 3
b3 sin3 t a sin t
Ix = ( b sin t)dt + ( a sin t)dt
0 3 0 3
Z
(b4 a4 ) 2⇡ 4
= sin tdt
3 0
Z
(b4 a4 ) 2⇡ 2
= sin t 1 cos2 t dt
3 0

Aplicando las identidades trigonométricas:


(1 + cos 2t) (1 cos 2t)
cos2 t = , sen2 t = , obtenemos
2 2
577
Z 2⇡
(b4 a4 ) 1
Ix = [(1 cos 2t)(1 cos 2t)] dt
3 0 4
4 4
 2⇡
(b a ) 3t sen2t sen4t
= +
3 8 4 32 0
4 4

(b a ) 6⇡ (b2 + a2 )
= = ⇡ (b2 a2 )
3 8 4
2 2
M (b + a )
=
4

Problema 2
Un fluido de densidad constante gira alrededor del eje z con velocidad
!
v = w( ybi+xb j) donde w es una constante positiva llamada rapidez angular,
muestre que la circulación del campo de velocidades es
I
v · d!
r = 2⇡R2 w
c

Solución
La circulación de un campo vectorial es su integral de lı́nea a lo largo de
una curva cerrada. Por el teorema de Stokes, la circulación de ! v alrededor
de la circunferencia C de radio R que acota a un disco S en el plano xy , esta
dado por
Calculemos
i j k
@ @ @
r⇥! v = = (0, 0, (w ( w)) = (0, 0, 2w)
@x @y @z
wy wx 0
Luego, (r ⇥ ! v)·bkdS = 2wbk·bkdS = 2wdxdy.Entonces
I ZZ
!
v · d!
r = (r ⇥ !
v)·b
k dS
C Z ZS
= 2wdxdy
S
= 2w ⇡R2

Problema 3
! x2
Sea el campo de fuerzas F (x, y) = (2xy 2 + y, 2x2 y + + x), demostrar
2
que en cualquier camino cerrado simétrico con respecto al eje y, la circulación

578
es cero.
Solución
! x2
Tenemos que F (x, y) = (2xy 2 + y, 2x2 y + + x) es un campo vectorial
2
continuo con derivadas parciales
9
@Q
= 4xy + x + 1 > =
@x , continuas definidas en la región de integración.
@P
= 4xy + 1 > ;
@y
Luego, aplicando el teorema de Green se tiene

I I ZZ 
! ! @Q @P
F ·dr = (P dx + Qdy) = dxdy
C R @x @y
ZZ
= xdxdy
R

donde R = {(x, y) 2 IR2 / a  x  a, 1 (x)  y  2 (x)}


Particionemos la Región R en dos subregiones simétricas con respecto al
eje y, tal que:
R = R1 [ R2 y R1 \ R2 = .Entonces
ZZ ZZ ZZ
xdxdy = xdxdy + xdxdy
R R1 R2

Figura 6.11: Región R = (x, y) 2 IR2 / a  x  a, 1 (x) y 2 (x) sub-


dividida en dos regiones R1 y R2

Se tiene que x es una función impar definida en un intervalo simétrico,


entonces
Z 0 Z a
xdx + xdx = 0
a 0

579
Por lo tanto ZZ ZZ
xdxdy + xdxdy = 0
R1 R2
En consecuencia I
! !
F ·dr =0
C

6.5.5. Aplicaciones al electromagnetismo


Problema 1
! !
Sean E (t, x, y, z)y B (t, x, y, z) los campos eléctrico y magnético respec-
tivamente que dependen de la posición y del tiempo, en el espacio. Sea S una
superficie con frontera C. Definimos
I
! !
Voltaje alrededor de C = E ·dr
C ZZ
!
Flujo magnetico a través de S = B ·n
bdS
S
La ley de Faraday afirma que el voltaje alrededor de C es igual a la razón
de cambio negativo del flujo magnético a través de S, es decir
I ZZ
! ! 1@ !
E ·dr = B ·nbdS (1,1)
C c @t S
Utilice el teorema de Stokes para deducir la Ley de Faraday a partir de
la ecuación diferencial de Maxwell
!
! 1@B
r⇥ E =
c @t
Solución
Supongamos que se cumple
!
! 1@B
r⇥ E =
c @t
Sea C una curva cerrada, estacionaria en las coordenadas x, y, z, y S una
!
superficie limitada por C . Si B (t, x, y, z) es el campo magnético medido en
x, y, z en el instante t, entonces tomando la integral de superficie sobre S se
tiene. ZZ ZZ
! 1 @!
(r ⇥ E ) · n
bdS = B ·nbdS
S c S @t
Transformando el lado izquierdo por medio del teorema de Stokes:
Z ZZ
! ! 1 @!
E ·dr = B ·n bdS
C c S @t

580
Aplicando linealidad al lado derecho obtenemos
Z ZZ
! ! 1@ !
E ·dr = B ·n
bdS
C c @t S
que es la ley de Faraday.

Problema 2
La ley de Ampére afirma que si la densidad de corriente eléctrica está de-
! !
scrita por un campo vectorial J ,y el campo magnetico es B , entonces la
!
circulacion B alrededor de la frontera C que limita una susperficie S es igual
!
a la integral de J sobre S es decir
Z ZZ
! ! 4⇡ !
B ·dr = J ·nbdS
C c S
Pruebe que la ley de Ampére se deduce a partir de la ecuación de Maxwell
del estado estacionario
! 4⇡ !
r⇥ B = J
c
Solución
Supongamos que se cumple
! 4⇡ !
r⇥ B = J
c
Tomando la integral de superficie sobre S se tiene.
ZZ ZZ
! 4⇡ !
(r ⇥ B) · n
bdS = J ·n
bdS
S c S
Aplicando el teorema de Stokes al lado izquierdo de la ecuación
Z ZZ
! ! 4⇡ !
B ·dr = J ·nbdS
C c S
que es la ley de Ampere.
Esta Ley también se puede expresar como
Z
! ! 4⇡
B ·dr = I
C c
donde ZZ
!
I= J ·nbdS
S
es la intensidad de la corriente eléctrica en una superficie S limitada por una
curva cerrada C.

581
Problema 3
!
Hallar el campo magnético B producido por un alambre recto infinito, a
una distancia r del alambre que lleva una corriente estacionaria I.
Solución
Considere un alambre recto extendido sobre el eje z desde -1 hasta
+1.Como hay simetrı́a cilindrica, el alambre coincide con el eje axial del
cilindro,se escoge una trayectoria circular con con un punto del eje z como
centro, con radio r
!
Por la simetrı́a, el vector B no es solamente constante azimutal sino
!
tambien tienen la disma dirección que d! r y su magnitud B es constante
alrededor de la trayectoria C. Por consiguiente

Z
! ! 4⇡
B ·dr = I
C c
Z
! 4⇡
B kd!r k cos 0 = I
C c
Z
! 4⇡
B kd!rk = I
C c
! 4⇡
B (2⇡r) = I
c
Finalmente queda
! 2
B = Ibe✓
cr

Problema 4

! !
Sean E (t, x, y, z)y B (t, x, y, z) los campos eléctrico y magnético respec-
tivamente que dependen de la posición y del tiempo, en el espacio. que sat-
isfacen la ecuaciones de Maxwell

!
! 1@B
r⇥ E =
c @t
!
! 1@E 4⇡ !
r⇥ B = + J
c @t c
!
r · E = 4⇡⇢
!
r· B = 0

582
que están escritas para los campos en el vacı́o, en presencia de una densidad
de carga eléctrica ⇢y de corriente,es decir,cargas en movimiento de densidad
!
J
a) Deducir la forma integral de la ecuación de Maxwell
Z ZZ ZZ
! ! 1@ ! 4⇡ !
B ·dr = E ·nbdS + J ·nbdS
C c @t S c S

donde C representa una espira de alambre por el que fluye corriente en el


sentido contrario al de los punteros del reloj con respecto al vector unitario
b. La integral de superficie del primer término del lado
normal a la superficie n
derecho de la ecuación es llamada flujo eléctrico y el segundo corresponde a
la intensidad de corriente I. Si S cualquier superficie orientada con frontera
C.
!
b) Suponga que B (t, x, y, z) es un campo solenoidal, es decir,
!
r · (r ⇥ B ) = 0, que satisface ecuación (2) de Maxwell. Demostrar que la
densidad de corriente satisface la ecuación de continuidad.

4⇡ @⇢
r·J + =0
c @t
Solución
b) Consideremos la ecuación de Maxwell.
!
! 1@E 4⇡ !
r⇥ B = + J
c @t c
Tomando la integral de superficie sobre S se tiene.
ZZ ZZ ! ZZ
! 1 @E 4⇡ !
(r ⇥ B) · n
bdS = ·n
bdS + J ·n
bdS
S c S @t c S

Aplicando el teorema de Stokes al lado izquierdo de la ecuación


Z ZZ ! ZZ
! ! 1 @E 4⇡ !
B ·dr = ·n
bdS + J ·n
bdS
C c S @t c S

Aplicando linealidad al primer término del lado derecho, obtenemos


Z ZZ ZZ
! ! 1@ ! 4⇡ !
B ·dr = E ·nbdS + J ·n
bdS
C c @t S c S

que es la forma integral de la ecuación de Maxwell.


c) Consideremos la ecuación

583
!
! 1@E 4⇡ !
r⇥ B = + J
c @t c
Tomemos la divergencia en los dos lados .

! 1@ ! 4⇡ !
r · (r ⇥ B ) = r· E + r· J
c @t c
!
Aplicando la condición de solenoidal y reemplazando r · E por (3)

1@ 4⇡ !
0= (4⇡⇢) + r· J
c @t c
Por lo tanto, se tiene

4⇡ ! @
r· J + ⇢ =0
c @t

Problema 5
! !
Sean E y B dos vectores con derivadas parciales continuas de segundo
!
orden con respecto a la posición y el tiempo . Supongase ádemas que E y
!
B satisfacen las ecuaciones de Maxwell en el vacı́o, en ausencia de densidad
!
de cargas ⇢ y densidad de corriente J :

!
r· E = 0 (6.5.1)
!
r· B = 0 (6.5.2)
!
! 1@B
r⇥ E = (6.5.3)
c @t
!
! 1@E
r⇥ B = (6.5.4)
c @t
donde c es una constante que corresponde a la velocidad de la luz en el vacı́o
! !
a) Demostrar que E y B satisfacen la ecuación de onda
!
2! 1 @2
r = 2 2
c @t
! !
b) Demostrar que E y B satisfacen la ecuación :

@ 1 !2 ! 2 ! !
( E + B ) + cr · ( E ⇥ B ) = 0
@t 2

584
c) Integre la expresión anterior sobre un volumen ⌦ encerrado dentro de
una superficie ⌃, y demostrar que
ZZZ  ZZ
@ 1 !2 !2 ! !
( E + B ) dV + c (E ⇥ B ) · n
bdS = 0
@t ⌦ 2 S

Solución
a) Si tomamos el rotacional de la ecuación 6.5.3 tenemos:
! 1@ !
r ⇥ (r ⇥ E ) = (r ⇥ B )
c @t
!
Eliminamos a r ⇥ B por medio de la ecuación 6.5.4, entonces
!
! 1 @ 1@E
r ⇥ (r ⇥ E ) = ( )
c @t c @t
!
! 1 @2 E
r ⇥ (r ⇥ E ) =
c2 @t2
Aplicando al rotacional la identidad:
! ! !
r ⇥ (r ⇥ E ) = r(r · E ) r2 E
se tiene: !
! 2! 1 @2 E
r(r · E ) r E =
c2 @t2
Por tanto, aplicando la ecuación 6.5.1 al primer término se tiene el campo
!
eléctrico E satisface la ecuación de onda
!
2! 1 @2 E
r E = 2 2
c @t
Por otra parte, si tomamos el rotacional de la ecuación 6.5.4 tenemos:
! 1@ !
r ⇥ (r ⇥ B ) = (r ⇥ E )
c @t
!
Eliminamos a r ⇥ E por medio de la ecuación 6.5.3, entonces
!
! 1@ 1@B
r ⇥ (r ⇥ B ) = ( )
c @t c @t
!
! 1 @2 B
r ⇥ (r ⇥ B ) =
c2 @t2
Aplicando al rotacional la identidad:

585
! ! !
r ⇥ (r ⇥ B ) = r(r · B ) r2 B
se tiene: !
! 2! 1 @2 B
r(r · B ) r B =
c2 @t2
En consecuencia, aplicando la ecuación 6.5.2, al primer término se tiene
!
que el campo eléctrico B también satisface la ecuación de onda
!
2! 1 @2 B
r B = 2 2
c @t
!
b) Si hacemos en la ecuación 6.5.3 el producto interno por H queda
! ! 1! @ !
B · (r ⇥ E ) = B· B
c @t
!
Analogamente , si hacemos en la ecuación 6.5.4 el producto interno por E
se tiene
!
! ! 1! @ E
E · (r ⇥ B ) = E ·
c @t
Restamos la segunda ecuación de la primera, entonces obtenemos

!
! ! ! ! 1! @ ! 1! @ E
B · (r ⇥ E ) E · (r ⇥ B ) = B· B E·
c " @t c @t #
!
! ! ! ! 1 ! @! ! @E
B · (r ⇥ E ) E · (r ⇥ B ) = B· B +E·
c @t @t

Ahora, reemplazando por las identidades:


! ! ! ! ! !
B · (r ⇥ E ) E · (r ⇥ B ) = r · ( E ⇥ B )
! 
! @! ! @E @ 1 !2 !
B· B +E· = ( B + E 2)
@t @t @t 2
La ecuación anterior se convierte en

! ! 1 @ 1 !2 !
r · (E ⇥ B ) = ( B + E 2)
c @t 2
por lo tanto, multiplicando por la constante c, obtenemos:

@ 1 !2 ! 2 ! !
( E + B ) + cr · ( E ⇥ B ) = 0
@t 2

586
c) Si integramos la última expresión sobre el volumen ⌦ dentro de una
superficie cerrada S, obtenemos
ZZZ ⇢ 
@ 1 !2 ! 2 ! !
( E + B ) + cr · ( E ⇥ B ) dV = 0
⌦ @t 2
Aplicando linealidad queda:
ZZZ  ZZZ
@ 1 !2 ! 2 ! !
( E + B ) dV + c r · ( E ⇥ B )dV = 0
@t ⌦ 2 ⌦

Usando el teorema de la divergencia en el segundo término de la ecuación


queda:
ZZZ  ZZ
@ 1 !2 !2 ! !
( E + B ) dV + c (E ⇥ B ) · n
bdS = 0
@t ⌦ 2 S

6.6. Auto evaluaciones


Autoevaluación No 1
Tiempo 2 horas
Problema 1
!
Dado el campo vectorial F (x, y, z) = (0, 0, z) .
RR !
a) calcular F ·n
bdS, siendo S la superficie descrita por la ecuación
S
paramétrica: !
r (u, v) = (sin u cos v, sin u sin v, cos u sin u) , donde

D = {(u, v) /0  u  ⇡, 0  v  2⇡}

b) determine el volumen de la región ⌦ acotada por la superficie S.

Problema 2

Sea !
RR !r = (x, y, z) en una superficie cerrada regular S. Obtener
r ·n
bdS en términos del volumen V que encierra S.
S
Problema RR 3.
!
Calcular F ·n
bdS, siendo el campo vectorial
S

!
F (x, y, z) = x2 + y 3 + z 4 , x2 + y 3 + z 4 , x3 + y 4 + z 4

y S la superficie formada por las seis caras de paralelepı́pedo

⌦ = (x, y, z) 2 IR3 /3  x  5, 1  y  2, 0  z  1

587
Pauta de Corrección

Problema 1
!
a) Calculemos directamente el flujo del campo F sobre la superficie S.
ZZ ZZ
! ! !
F ·n
bdS = F ( r (u, v)) · !
n dudv
S D
!
Se tiene que F (!
r (u, v)) = (0, 0, cos u sin u) y el vector normal exterior
a la
superficie es

i j k
!
n = !
r u⇥!
rv= cos u cos v cos u sin v sin2 u
sin u sin v sin u cos v 0
= sin3 u cos v, sin3 u sin v, cos u sin u
! !
F ( r (u, v)) · !
n = (0, 0, cos u sin u) · sin3 u cos v, sin3 u sin v, cos u sin u
Reemplazando en el integrando obtenemos:
Z 2⇡ Z ⇡ Z Z
2 2 1 2⇡ ⇡ 2
cos u sin ududv = sin 2ududv
0 0 4 0 0
Z Z ✓ ◆
1 2⇡ ⇡ 1 cos 4u
= dudv
4 0 0 2
Z  ⇡
1 2⇡ 2u sin 4u
= dv
4 0 4 0
Z
1 2⇡ ⇡ ⇡2
= dv =
4 0 2 4
!
el flujo del campo F sobre la superficie S.
b) Sea ⌦ la región cuyo volumen esta acotado por la superficie S. De
acuerdo
con el teorema de la divergencia de Gauss, se tiene
ZZ ZZZ
! !
F ·nbdS = r · F dV
S ⌦

!
Como al calcular r · F = 0 + 0 + 1 = 1,obtenemos

588
ZZ ZZZ
!
F ·n
bdS = dV = V (⌦)
S ⌦

Por tanto, el volumen pedido es

⇡2
V (⌦) =
4

Problema 2.

De acuerdo con el teorema de la divergencia de Gauss se tiene

ZZ ZZZ
!
r ·n
bdS = r·!
r dV
S
Z⌦
ZZ
= 3dV

= 3V

donde V es el volumen encerrado por S

Problema 3
Sea la región cuyo volumen esta acotado por la superficie S. De acuerdo
con
el teorema de la divergencia de Gauss, se tiene

ZZ ZZZ
! !
F ·n
bdS = r · F dV
S ⌦

!
Como al calcular r · F = 2x + 3y 2 + 4z 3 ,obtenemos

589
ZZZ ZZZ
!
r · F dV = 2x + 3y 2 + 4z 3 dxdydz
⌦ ⌦
Z 5 Z 2 Z 1
= 2x + 3y 2 + 4z 3 dzdydx
Z3 5 Z1 2 0
⇥ ⇤1
= 2xz + 3y 2 z + z 4 0
dydx
3 1
Z 5 Z 2
= 2x + 3y 2 + 1 dydx
Z3 5 1
⇥ ⇤2
= 2xy + y 3 + y 1
dx
3
Z 5 ⇥ ⇤
= (2x + 8)dx = x2 + 8x = 32
3

Autoevaluación No 2
Tiempo 2 horas
Problema 1. RR ! !
Calcular la integral bdS, en donde F (x, y, z) = (xy 2 , x2 y, y) , y S
F ·n
S
es la superficie del cilindro x2 + y 2 = 1 acotado por los planos:
z = 1 y z = 1.

Problema 2.
! p
Sea F (x, y, z) = 0, y, 0 el campo de velocidades de un fluido. Calcular
RR !
el flujo ndS, que atraviesa la superficie S = {(x, y, z) 2 IR3 /x2 + y 2 = y, 0  y  1} .
F ·b
S
Problema 3.
!
Verificar el teorema de Stokes para F (x, y, z) = (3y, xz, yz 2 ) , siendo S
la superficie del paraboloide 2z = x2 + y 2 , limitada por z = 2.Considere que
b es la normal exterior a la superficie S.
n

Pauta de Autocorreción

Problema 1
De acuerdo con el teorema de la divergencia de Gauss,se tiene
ZZ ZZZ
! !
F ·n
bdS = r · F dV
S ⌦

590
siendo ⌦ la región limitada por la superficie S, es decir el cilindro macizo
dado por la ecuación ⌦ = {(x, y, z) 2 IR3 /x2 + y 2  1, 1  z  1} .
Teniendo en cuenta que el integrando en cordenadas cartesianas
!
r · F = y 2 + x2 + 0,queda:
ZZZ ZZZ
!
r · F dV = x2 + y 2 dxdydz
⌦ ⌦

Por la simetria del integrando y del dominio conviene aplicar el cambio


de
coordenadas cilı́ndricas x = ⇢ cos ✓, y = ⇢ sin ✓ , cuyo jacobiano es
J (⇢, ✓, z) = ⇢. Entonces la región ⌦ transforma a
!
⌦⇤ = {(⇢, ✓, z) /⇢2  1, 1  z  1} y el integrando produce r · F =
⇢2 ,luego.

ZZZ ZZZ
2 2
x +y dxdydz = ⇢2 ⇢d⇢d✓dz
⌦ ⌦⇤
Z1 Z 2⇡ Z 1
= ⇢3 d⇢d✓dz
1 0 0
Z 1 Z 2⇡  1
⇢4
= d⇢d✓dz
1 0 4 0
= ⇡

Problema 2.
En primer lugar consideremos la siguiente parametrizacion de la Superfi-
cie
!
r (u, v) = (u cos v, u2 , u sin v) , donde ⌦ = {(u, v) , 0  u  1, 0  v  2⇡}
Determinemos el vector normal a la superficie
!
r u (u, v) = (cos v, 2u, sin v)
!
r v (u, v) = ( u sin v, 0, u cos v)
i j k
! !
ru⇥ rv = cos v 2u sin v = (2u2 cos v, u, 2u2 sin v)
u sin v 0 u cos v
Entonces

591
ZZ ZZ
! ! !
F ·n
bdS = F ( r (u, v)) · (!
r u⇥!
r v ) dudv
S ⌦
Z 2⇡ Z 1
= (0, u, 0) · 2u2 cos v, u, 2u2 sin v dudv
Z0 2⇡ Z0 1
= u2 dudv
0 0
2⇡
=
3

Problema 3.
De acuerdo con el teorema de Stokes, debemos verificar que
I ZZ
! ! !
F ·dr = r⇥ F ·n bdS
C S

H! !
En primer lugar, calculemos F ·dr
C
Determinemos la curva C que limita la Superficie abierta del paraboloide:
si z = 2 =) x2 + y 2 = 4
Parametricemos la curva C, con una orientación positiva, se tiene ! r (t) =
! !
(2 cos t, 2 sin t, 2) t 2 [2⇡, 0] y evaluemos el campo sobre la curva F ( r (t)) =
(6 sin t, 4 cos t, 8 sin t) , luego
I Z 0
! !
F ·dr = 12 sin2 t + 8 cos2 t dt
2⇡
C
Z 2⇡
= 8 + 4 sin2 t dt
Z0 2⇡ ✓ ✓ ◆◆
1 cos 2t
= 8+4 dt
0 2
 2⇡
sin 2t
= 16⇡ + 2 t
2 0
= 20⇡

592
Calculemos, ahora
ZZ ZZ
! !
r⇥ F ·nbdS = r ⇥ F · (!
r y⇥!
r x ) dxdy
S R

Las ecuación
✓ paramétrica ◆ de la superficie S en coordenadas cartesianas
2 2
x + y
es !
r (x, y) = x, y, ,y el vector normal exterior es
2
i j k
!n =! r y⇥!r x = 0 1 y = (x, y, 1)
1 0 x
Además
i j k
!
r ⇥ F = @/@x @/@y @/@z = (z + x, 0. (z + 3))
3y xz yz 2
Finalmente R es la proyección sobre el plano xy, R = {(x, y) 2 IR2 /x2 + y 2 = 4} ,entonces

Z Z "✓ ◆2 #
x2 + y 2 x2 + y 2
+ x, 0, 3 · (x, y, 1) dxdy
2 2
R
ZZ " ✓ 2 ◆2 #
x + y2 2 x2 + y 2
= x +x + + 3 dxdy
2 2
R

Haciendo un cambio a coordenadas polares x = ⇢ cos ✓, y = ⇢ sin ✓


Z 2⇡ Z 2 
1 5 1
⇢ cos ✓ + ⇢2 cos ✓ + ⇢2 + 3 ⇢d⇢d✓
0 0 4 2
Z 2⇡ Z 2⇡ Z 2⇡
128 2
= cos ✓d✓ + 4 cos ✓d✓ + 8 d✓
28 0 0 0
Z 2⇡
1 + cos 2✓
= 0+4 d✓ + 16⇡ = 4⇡ + 16⇡
0 2
= 20⇡

Autoevaluación No 3
Tiempo 2 horas
Pregunta 1
RR !
Usar el teorema de Stokes para resolver, r⇥ F ·n
bdS , si la superficie
S
!
S esta dada por z = 1 x2 y 2 con z 0 y F = (y, z, x) .

593
Pregunta 2
Determinar el área de la superficie x2 + y 2 = 4, limitada por el plano
z = 0 y por la superficie z = 2x2 + y 2

Pauta Correcciones
Pregunta 1
El teorema de Stokes asegura que
ZZ I
! ! !
r⇥ F ·n bdS = F ·dr
S C

donde C es la curva cerrada que limita S.


Sea C: 1 x2 y 2 = 0; z = 0 =) x2 + y 2 = 1 corresponde a una
circunferencia en el plano xy centrada en el origen de radio 1.
Al parametrizar la curva considerando una orientación positiva, produce
!r (t) = (cos t, sin t, 0) t 2 [2⇡, 0] =)
! 0
r (t) = ( sin t, cos t, 0) t 2 [2⇡, 0] =)
! !
F ( r (t)) = (sin t, 0, cos t)
!
Calculemos entonces la integral de lı́nea F de sobre la curva C

I Z 2⇡
! !
F ·dr = [sin t ( sin t) + 0 + 0] dt
0
C
Z 2⇡ Z 2⇡ ✓ ◆
2 1 cos 2t
= sin tdt = dt
0 0 2
 2⇡
t sin 2t
= = ⇡
2 4 0

Pregunta 2
Método 1
En primer lugar, consideremos la Superficie cilı́ndrica parametrizada por
!
r (u, v) = (2 cos u, 2 sin u, v) , donde ⌦ = {(u, v) , 0  u  2⇡, 0  v  4 cos2 u + 4}
Determinemos el vector normal a la superficie
!
r u (u, v) = ( 2 sin u, 2 cos u, 0)
!
r v (u, v) = (0, 0, 1)
i j k
! !
ru⇥ rv = 2 sin u 2 cos u 0 = (2 cos u, 2 sin u, 0)
0 0 1

594
|!
r u⇥! r v| = 2
El área de la superficie es entonces
Z Z
A (S) = |!
r u⇥!
r v | dudv

Z 2⇡ Z 4 cos2 u+4
= 2dvdu
0 0
Z 2⇡
= 2 4 cos2 u + 4 du
0
= 24⇡

Método 2
El área de la superficie se puede determinar con la integral curvilı́nea
Z
A (S) = zds
C

Sea C: x2 + y 2 = 4 es a una circunferencia en el plano xy centrada en


el origen de radio 2. Al parametrizar la curva considerando una orientación
positiva, produce
!r (t) = (2 cos t, 2 sin t) , t 2 [2⇡, 0] =)
! 0
r (t) = ( 2 sin t, 2 cos t) , t 2 [2⇡, 0] =)
z (!
r (t)) = 4 cos2 u + 4

Luego, el área de la superficie es

Z 2⇡
A (S) = 4 cos2 u + 4 2dt
0
= 24⇡

595
Bibliografı́a

[1] Tom M. Apóstol, Calculus, Vol I, 2a edición, Ed. Reverté, 1982


[2] George Arfken, Mathematical Methods for Physicists, Academic Press,
1985
[3] R.V. Churchill, Fourier Series and Boundary Value Problems, 2o ed.
Nueva York, Mac Graw Hill,1963
[4] Erwing Kreyszing, Matemáticas Avanzadas para Ingenierı́a, Vol I y II,
Limusa, 1995
[5] R. Larson ,R.Hostetler, B. Edwars, Cálculo con Geometrı́a Analı́tica,
Vol 2, Mc Graw Hill, 1999.
[6] J.E. Marsden, A.J. Tromba, Cálculo Vectorial, Addison Wesley Long-
man, 1998
[7] Peter V. O’Neil, Matemáticas Avanzadas para Ingenierı́a, Thomson,
2005
[8] Claudio Pita Ruiz, Cálculo Vectorial, Prentice-Hall Hispanoamérica, SA,
1995
[9] George B. Thomas Jr., Cálculo varias variables, Addison-Wesley, Pear-
son 2000S

596

S-ar putea să vă placă și